Está en la página 1de 243

1.

Niña de cinco años de edad, que acude a guardería, presenta una dermatosis
diseminada que afecta palmas y plantas, caracterizada por máculas, pápulas y
vesículas ovaladas rodeadas de un halo eritematoso. Además, muestra enantema
en lengua y paladar,caracterizada por vesículas sobre una base eritematosa y
exulceraciones dolorosas. Tres días previos al inicio de la dermatosis presentó
fiebre y ataque al estado general. Dos de sus compañeros de la guardería tienen
lesiones similares, informa la madre, El diagnóstico más probable es:
e} Enfermedad mano-pie-boca

2. Niña de cuatro años de edad que inicia súbitamente con cólico abdominal
intenso y vómitos; posteriormente presenta una evacuación abundante de heces
mezclada con sangre fresca y moco. En el examen físico se observa letárgica, con
debilidad, pálida, muy sensible a la exploración abdominal y con retracción de
ambas extremidades inferiores hacia el abdomen. ¿Cuál es el diagnóstico de esta
paciente?
e} lntususcepción

3. Paciente masculino de 35 años con antecedente de uso de drogas intravenosas


acude a urgencias por presentar fiebre y escalofríos de una semana de evolución
y disnea progresiva. Hace dos días presentó parestesias en el brazo izquierdo con
debilidad de inicio súbito que duró aproximadamente 3 h y se resolvió de manera
espontánea. Sus signos vitales son temperatura de 38.5 ºC, pulso de 120 latidos
por minuto, frecuencia respiratoria de 24 respiraciones por minuto, presión arterial
de 116/76 mm Hg. A la exploración física se ausculta soplo sistólico 3/6 en el
borde"
inferior esternal derecho y campos pulmonares bien ventilados. Usted diagnostica
endocarditis infecciosa y procede a tomar hemocultivos previo a iniciar tratamiento
antibiótico empírico.
¿Cuál es el agente etiológico más común en usuarios de drogas intravenosas y
que debe ser cubierto por el esquema
empírico de antibióticos?
b} Staphy/ococcus aureus

4. Paciente masculino de 75 años de edad con diagnóstico de diabetes mellitus


tipo 2, hipertensión arterial e insuficiencia renal crónica. Acude al servicio de
urgencias por presentar cefalea occipital de una semana de evolución, intensidad
5/1O. A la exploración física, la presión arterial es 200/100 mm Hg; el resto de la
exploración física no muestra datos relevantes, el fondo de ojo es normal. El
paciente niega dolor precordial. Se emite el diagnóstico de urgencia hipertensiva.
4b. El tratamiento inicial de este paciente debe incluir:
b} Captopril sublingual

4d. Una emergencia hipertensiva debe ser manejada con:


a} Nitroprusiato de sodio intravenoso
5. Paciente femenino que acude a consulta ginecológica en busca de métodos de
planificación familiar. Se sabe que cada uno de ellos tiene sus indicaciones y sus
contraindicaciones, así como su eficacia.
Siendo a = anticonceptivos orales con progestágenos, b = DIU,c = condón
masculino, d = diafragma, e = anticonceptivos orales combinados, f = condón
femenino. Ordénelos de mayor a menor eficacia según la Norma Oficial Mexicana
(NOM).
b} e > b > a > e > f > d

6. Lo siguiente es cierto sobre la erisipela:


e} Los márgenes de la lesión están bien definidos y sonindurados

7. De los siguientes microorganismos, ¿en cuál NO se ha demostrado una relación


importante con algún tipo de cáncer?
d} Virus herpes simple tipo 1

8. Femenino de 69 años de edad, se presenta al servicio de urgencias por


padecimiento de 6 h de evolución caracterizado por dolor intenso en 2° dedo del
pie izquierdo, hiperestesia y amoratamiento del mismo. Antecedentes: diabetes
mellitus de 15 años de evolución tratada con metformina + glibenclamida,
hipertensión arterial sistémica de 15 años de evolución controlada con losartán y
metoprolol,tabaquismo crónico intenso desde su juventud hasta la actualidad a
razón de 2 cajetillas por día, claudicación intermitente a más de 300 m.
Exploración física: obesa bien hidratada orientada en las tres esferas, abdomen
blando depresible no doloroso con soplo holosistólico en mesogastrio, pulsos
femorales con soplos bilaterales, buena intensidad, pulsos poplíteos, tibiales
anteriores y posteriores disminuidos, en pie izquierdo se observa 2° dedo con
cianosis intensa, hipotermia de todo el dedo e hiperalgesia.
8a. El mecanismo fisiopatológico responsable del cuadro clínico de esta paciente
es:
d} Ateroembolismo distal

8b. La medida terapéutica indicada en esta paciente es:


d} Manejo conservador e iniciar estudio de patología aórtica

8e. El mecanismo fisiopatológico responsable del cuadro clínico de esta paciente


es:
d} Ulceración y embolización de placa de ateroma

8d. Debido a los antecedentes de la paciente, se decide que es necesario


descartar la presencia de aneurisma de la aorta abdominal. ¿Cuál es el estudio de
elección para corroborar este diagnóstico?
c}Angiotomografía

8e. Según los resultados de los estudios realizados, ¿cuál sería el tratamiento
médico de elección para disminuir la claudicación de la paciente?
d} Medidas de higiene arterial + cilostazol
9. Paciente masculino de 69 años de edad que acude al servicio de urgencias por
presentar disnea. Tiene antecedente de tabaquismo intenso desde los 20 años de
edad. Al interrogatorio menciona que ha perdido alrededor de 9 kg de manera no
intencionada en el último mes y que desde hace tres meses ha tenido tos crónica.
A la exploración física la presión arterial es 88/66 mm Hg, la frecuencia cardiaca
es de 109 latidos/minuto. Existe distensión de las venas yugulares. La
auscultación de los campos pulmonares revela disminución de los sonidos
respiratorios en el pulmón derecho. Los ruidos cardiacos se encuentran
notablemente disminuidos. La radiografía de tórax muestra crecimiento de la
silueta cardiaca y una tumoración en el hilio derecho. ¿Cuál de los siguientes
hallazgos electrocardiográficos se esperaría observar en este paciente?
c} Alternancia eléctrica

1O. ¿Cuál es la producción normal de cortisol al día?


c} 8 a 25 mg en 24 horas

11. ¿Por qué los pacientes con insuficiencia suprarrenal de origen central no
cursan con desequilibrio hidroelectrolítico?
c} No se observa hipopotasemia ni deshidratación

12. A los 1O días de vida un recién nacido presenta caída del cordón umbilicaly
posterior a esto, se observa a nivel de la cicatriz umbilical una masa rojiza, blanda
y granular que en ocasiones produce una secreción blanquecina espesa.
¿Cuál es el manejo más adecuado en este caso?
b} Cauterización con nitrato de plata

13. Una mujer de 65 años de edad acude a consulta para revisión médica general.
En su historia clínica refiere hipotiroidismo, tratado con levotiroxina, y asma leve
intermitente, que maneja con albuterol inhalado. Es viuda y actualmente vive sola.
Niega cualquier síntoma al IPAS. A la exploración física sus signos vitales son FC
88/min, FR 16/min, T 37.0 C y PA 142/80 mm Hg. En las extremidades superiores
se observan nódulos de Heberden y de Bouchard. A la auscultación cardiaca se
escuchan S1 y S2 sin agregados. Los campos pulmonares se auscultan con
murmullo vesicular. ¿Cuál sería el siguiente paso en el manejo de esta paciente?
d} Citar en dos semanas para volver a tomar la presión arterial

14. Acude un masculino de 70 años a consulta de control, con antecedentes de


tabaquismo, etilismo, diabetes, hipertensión, hiperlipidemia y angina de pecho,
toma metformina, lisinopril,metoprolol,atorvastati na, nitroglicerina y aspirina.
Puede llevar a cabo todas sus actividades básicas sin problema. Se puso la
vacuna del neumococo hace cinco años, la de herpes zoster hace dos años y se
aplica la vacuna de influenza anualmente. Niega cualquier síntoma, excepto por
ocasionales dolores articulares. Su angina de pecho ha estado estable, sin
disminución de su capacidad física o aumento de frecuencia de sus síntomas. Sus
signos vitales son FC 76/min, FR 16/min, T 37.1 C y PA 128/82. A la auscultación
torácica es normal,en el abdominal se palpa una masa pulsante en mesogastrio, y
se ausculta un soplo que coincide con la sístole. Se confirma por ultrasonografía
un diámetro de 3.5 cm ¿Qué atención se le dará a este paciente?
c} Observación para reevaluación en seis meses

15. Una mujer de 65 años acude a consulta por lumbalgia. En sus antecedentes
patológicos refiere haber padecido osteoartritis, depresión y cáncer de mama,
diagnosticado como carcinoma invasivo ductal, tratado con lumpectomía y
disección axilar,quimioterapia y radioterapia hace seis años. Refiere que el último
mes ha padecido un dolor en la espalda baja, que se irradia hacia el pie izquierdo.
Niega debilidad, cambios de la sensibilidad, parestesias o incontinencia urinaria.
Niega pérdida de peso. A la exploración física,sus signos vitales incluyen FC
82/min, FR 18/min, T 36.2 C y PA 132/78 mm Hg. Se detecta un signo de Lasegue
positivo al elevar la pierna izquierda. El resto de la exploración neurológica se
reporta sin anormalidades. La radiografía revela leve osteofitosis. ¿Cuál sería el
siguiente paso en el manejo de la paciente?
b} Obtener resonancia magnética

16. Una mujer de 65 años acude a consulta para revisión general. Es una persona
sana, refiriendo sólo dolores articulares intermitentes e hipotiroidismo. Sus
medicamentos incluyen levotiroxina y ocasionalmente paracetamol. Refiere
tabaquismo de 20 paquetes/año. Niega cualquier síntoma al IPAS. Sus signos
vitales incluyen FC 82, FR 16, T 37.3 C y PA 120/67. Su preocupación principal es
la osteoporosis. ¿Cuál de las siguientes situaciones es indicación de
densitometría ósea?
e} Mujeres que han tenido terapia de reemplazo por periodos prolongados

17. Paciente femenino de 31 años de edad sin antecedentes cardiovasculares,


presenta cuadro de 2 meses de evolución que consiste en fiebre, pérdida de peso,
artralgias y anorexia. Es atendida en Urgencias, donde se le diagnostica EVC
isquémico. Al tomar la TA se encuentra una discrepancia entre la TA del brazo
derecho y la del izquierdo, con disminución marcada de pulsos en miembro
superior derecho. En la biometría hemática se registra ligera anemia normocítica
normocrómica y elevación de VSG.
17a. El diagnóstico más probable es:
b} Arteritis de Takayasu

17b. Aparte de la aorta, ¿cuál es la arteria que se afecta con mayor frecuencia en
este padecimiento?
d} Subclavia

17c. ¿Cuál de los siguientes no es un criterio de clasificación de la enfermedad de


la que trata el caso clínico?
c} Livedo reticular
17d. Si se tomara una biopsia de la zona afectada, ¿qué reportaría el patólogo?
d}Infiltrado celular con granulomas y células gigantes

17e. Esta enfermedad tiene tres fases, ¿cuáles son las características de la
segunda fase?
b} Disminución y discrepancia delos pulsos enlas extremidades, equimosis,
claudicación,hipertensión renovascular,síncope neurogénico

18. Femenino de 59 años acude a consulta externa refiriendo: plenitud,


estreñimiento, acompañada con resultado de ultrasonido pélvico, imagen quística
en el ovario derecho de 15 por 15 cm. El diagnóstico más probable en esta
paciente es:
a} Cistadenoma seroso

19. ¿Cuál de las siguientes arterias se encarga de la irrigación de la mitad derecha


de la curvatura mayor del estómago?
b} Arteria gastroepiploica derecha

20. Paciente masculino de 64 años de edad se presenta en Urgencias por


presentar desde hace 9 semanas fiebre, confusión mental, anorexia y diaforesis
nocturna. Como antecedente de importancia refiere que se le realizó un recambio
valvular hace varios años; sin embargo, desconoce los detalles. A la exploración
física se notan petequias en conjuntiva y paladar, hemorragias retinianas, y
máculas hemorrágicas en palmas de las manos. A la auscultación cardiaca se
detecta un soplo de insuficiencia mitral.
20a. ¿Qué microorganismo es el que tiene mayor probabilidad de estar
involucrado en este caso?
b} S. viridans

20b. ¿Cuál se los siguientes NO es un criterio mayor de Duke?


e} Fiebre

20c. ¿Cuál es la válvula más afectada en pacientes adictos a drogas


intravenosas?
c} Tricúspide

20d. Si el resultado del hemocultivo es positivo para Streptococcus bovis, ¿qué


patología se descartaría de manera obligada?
c} Carcinoma de colon

20e. ¿Cuál es el antibiótico que se utiliza como profilaxis en procedimientos


dentales o de otra índole para evitar la endocarditis infecciosa?
a} Amoxicilina
21. La amantadina es un fármaco activo contra la influenza tipo A (pero no contra
la influenza tipo B), no obstante, recientemente ha surgido un alto grado de
resistencia a este fármaco. ¿Cuál de los siguientes enunciados se asocia mejor
con el mecanismo de acción de la amantadina?
b}Inhibición dela descapsidación

22. ¿Cuál de los siguientes nervios envía fibras hacia la piel de la región
anteroinferior del abdomen?
e} Nervio iliohipogástrico

23. Paciente de 45 años de edad sin antecedentes de importancia, ingresa al


servicio de urgencias por deterioro neurológico, estado de choque y acidosis
láctica de brecha aniónica aumentada. A la exploración clínica neurológica se
encuentra estado de coma con flacidez y pérdida de los reflejos, pupilas isocóricas
y de lenta respuesta a la luz. La acidosis aniónica continuó empeorando y alcanzó
niveles de ácido láctico de 4.6 mmol/L y brecha osmolar de 11.35, con glucosa,
urea, creatinina y electrólitos normales. Al lavado gástrico se obtiene un material
rojizo. Se solicita un estudio de orina para buscar intencionadamente cristales de
oxalato de calcio. En caso de encontrar cristales de oxalato de calcio, ¿cuál es la
sustancia que más probablemente ingirió este paciente?
c} Etilenglicol

24. ¿Cuál de las siguientes condiciones es más probable que presente un título de
inhibición de hemaglutinina > 1:20?
a}Inmunidad contra elsarampión

25. Paciente masculino de 20 meses de edad se presenta al servicio de urgencias


con múltiples fracturas y hematomas, así como inflamación articular. La
exploración física revela múltiples petequias. En las radiografías se puede
observar engrosamiento de metáfisis y fragmentación alrededor de las epífisis. Los
estudios de laboratorio son característicos por un conteo de glóbulos rojos de 3
500 células/mm3, leucocitos de 2 500 células/mm3, volumen corpuscular medio de
70 fL, y concentración media de hemoglobina de 30%. ¿Cuál es el diagnóstico
más probable en este paciente?
b} Escorbuto

26. Paciente femenino de 23 años de edad la cual se presenta al consultorio por


presentar desde hace varias semanas secreción vaginal maloliente. La paciente
inició hace dos meses su vida sexual activa y está preocupada de que haya
adquirido una enfermedad de transmisión sexual. Al interrogatorio, menciona que
su pareja sexual no tiene ninguna sintomatología, y el método anticonceptivo que
utilizan son los anticonceptivos orales que una amiga le recomendó. A la
exploración física se observa una secreción blanquecina con olor a "pescado". La
medición del pH de la secreción es de 5.5 y la observación al microscopio revela la
presencia de cocobacilos cubiertos de células del epitelio vaginal.
¿Cuál es el tratamiento más adecuado en esta paciente?
d} Metronidazol 500 mg vía oral dos veces al día durante siete días
27. Paciente masculino de 34 años de edad que se presenta con tuberculosis
pulmonar aguda con presencia de bacilos ácido alcohol resistentes en el esputo.
¿Cuál es el tratamiento más adecuado para este paciente?
d}lsoniazida, rifampicina, pirazinamida, etambutol

28. Un paciente de 60 años se presenta con diagnóstico reciente de cáncer de


mama en actual tratamiento con ciclofosfamida, metotrexato y 5-fluorouracilo. El
motivo de consulta es por sintomatología sugerente de cistitis ¿Cuál es el origen
de la mayoría de sus actuales síntomas?
d} Toxicidad por acroleína

29. Se presenta un masculino de 88 años con antecedente de hipertensión arterial


de 12 años de evolución en tratamiento irregular con diltiazem y diuréticos. El
principal motivo de consulta es disnea de inicio subagudo. En los cinco meses
previos ha sido hospitalizado varias veces. A la exploración física presenta PA de
230/11O mm Hg, FC 120 lpm, FR 28 rpm,temperatura de 36.5 ºC y saturación de
89%. La presión venosa yugular es de 14 cm H20,con pulsos hiperdinámicos. La
auscultación pulmonar revela estertores bilaterales. La auscultación cardiaca
indica presencia de tercer y cuarto ruidos cardiacos. Las extremidades muestran
un edema con Godete hasta los tobillos. El electrocardiograma tiene datos en
relación con hipertrofia ventricular izquierda y la radiografía de tórax demuestra un
corazón de tamaño normal con cambios moderados intersticiales consistentes con
edema pulmonar. Se realiza un ecocardiograma que revela hipertrofia ventricular
izquierda con conservación de la función sistólica ventricular izquierda. ¿Cuál es el
diagnóstico más probable en este paciente?
b} Insuficiencia cardiaca diastólica

30. Paciente femenina de 42 años refiere que inicia con fiebre y somnolencia hace
2 semanas, pero que no presenta molestias de ningún tipo. Al interrogatorio niega
molestias faríngeas, urinarias, lesiones en piel o algo que haga sospechar un
origen infeccioso. A la exploración física, llama la atención que presenta equimosis
en piernas. Los resultados de la biometría hemática son:
• Hemoglobina 9.2 g/dL
• Hematocrito 28%
• Leucocitos 7 000 por microlitro
• Plaquetas 56 por microlitro
• Reticulocitos presentes

Usted decide realizar un Coombs, el cual se reporta negativo.


30a. ¿Cuál sería su impresión diagnóstica?
c} Púrpura trombocitopénica trombótica

30b. ¿Cuál es la fisiopatología de la enfermedad?


a} AutoanticuerposlgG que inhibenla actividad de ADAMTS 13, por lo que no se
separan los multímeros de vWF
30c. ¿Cuál es la clínica típica de la enfermedad?
d} Hay una pentada clásica: trombocitopenia, anemia hemolítica
microangiopática, alteraciones del estado mental,insuficiencia renal y fiebre
30d. Usted decide realizar un frotis para tener mayor certeza de su diagnóstico.
¿Qué esperaría ver en el frotis?
a} Esquistocitos

30e. ¿Cuál es el tratamiento ideal para esta paciente?


c} Plasmaféresis urgente y glucocorticoides

31. ¿Cuáles de las siguientes estructuras se desarrollan a partir de los conductos


de Wolff?
b} Epidídimo, vesículas seminales y vas deferens

32. Paciente masculino de 39 años de edad que presenta pápulas pruríticas en el


cuerpo del pene y en el borde de la boca desde hace aproximadamente 20 días.
Al interrogatorio el paciente refiere que a pesar de que tiene varias parejas
heterosexuales, siempre utiliza condón y no realiza sexo oral. No existen
antecedentes personales o familiares de importancia. A la exploración física se
encuentran pápulas en el pene, en la mucosa oral, así como en las muñecas y
región anterior de la pierna. En la boca las lesiones son parecidas a estrías tipo
"encaje". Utilizando únicamente la información previamente descrita, responda las
siguientes preguntas:
32a. ¿Cuál de las siguientes opciones diagnósticas es la más adecuada en este
paciente?
c} Biopsia + panel viral de hepatitis

32b. ¿Cuál de las siguientes opciones representa el agente etiológico más


frecuentemente asociado con esta condición dermatológica?
a} Virus de la hepatitis

32c. ¿Cuál de las siguientes opciones es el tratamiento más adecuado para la


condición dermatológica paciente?
a} Crema de dipropionato de betametasona

33. ¿Cuál de los siguientes fármacos interfiere con el funcionamiento de los


microtúbulos?
e} Griseofulvina

34. ¿Cuál de las siguientes aseveraciones respecto del control respiratorio es


correcta?
c} Los quimiorreceptores periféricos estimulanla respiración cuando la presión
parcial de oxígeno cae por debajo de 60 mm Hg
35. Una paciente de 30 años de edad se presenta dos semanas después de la
realización de cesárea de su primer hijo. La paciente menciona que ella ha estado
llorando desde hace dos semanas y con ánimo "decaído". ¿Cuál de los siguientes
síntomas harían sospechar que la paciente presenta depresión posparto y no tan
sólo tristeza posparto?
a} Sentimiento de desesperanza

36. Acude paciente femenino de 27 años de edad a consulta de control prenatal


por presentar embarazo de 10 semanas de gestación, no cuenta con
antecedentes de importancia, no toma ningún medicamento y no tiene factor de
riesgo alguno para el embarazo. Según la norma oficial mexicana, ¿cuántas
consultas como mínimo deben tenerse durante el embarazo?
b} 5

37. Usted se encuentra en una guardia de urgencias ginecoobstétricas y llega una


mujer de 36 semanas de gestación con trabajo de parto. Nunca había tenido
control prenatal previo. Con fondo uterino de 34 cm desde sínfisis del pubis. Palpa
dos dorsos con la segunda maniobra,con un polo cefálico encajado y el otro a
nivel del fondo uterino, con frecuencia cardiaca fetal de 150 y 135 latidos por
minuto respectivamente. Usted decide que nazcan vía cesárea sí:
a} Monoamnióticos

38. Recién nacido de 20 horas de vida, producto de primera gestación, con tipo
sanguíneo B-, la madre tiene sangre tipo A+. ¿Qué es FALSO en relación con la
incompatibilidad ABO?
b} Nunca ocurre en elprimer embarazo

39. Lactante de dos meses de edad que presenta desde el nacimiento estridor de
forma intermitente, sin taquipnea, aleteo nasal, tiraje subcostal,retracción
subxifoidea. En la exploración física se observa mucosa faríngea de aspecto
rosado, sin hiperemia ni exudados, se realiza laringoscopia directa en la que se
encuentran hallazgos compatibles con laringomalacia (flacidez de la epiglotis con
prolapso sobre la laringe durante la inspiración). De acuerdo al diagnóstico, ¿cuál
es la medida terapéutica más indicada en este caso?
d} Vigilancia

40. Paciente femenino de 21 años, estudiante de preparatoria, no


fumadora,sana,con historia de un año de tos que ocurre esporádicamente o
después de exponerse a polvo o hacer ejercicio, asociada a opresión torácica y
que se autolimita en unas horas. Los síntomas son peores por la mañana y los
presenta menos de una vez a la semana durante el día y no más de dos veces al
mes por la noche. Acudió con un médico, quien le recetó omeprazol 20 mg cada
24 h,sin mejoría. Llega a consulta con unas pruebas de función respiratorias
normales y una radiografía de tórax normal.
40a. ¿Qué prueba sería de utilidad para guiar el diagnóstico?
b} Test de metacolina
40b. ¿En qué clasificación de asma está la paciente?
a)Intermitente

40c. ¿Cuál de los siguientes es el responsable de mayor relevancia en la


fisiopatología del asma?
b} Leucotrieno C4

40d. ¿Cuál es el tratamiento indicado para la paciente?


c} Betaagonista de acción corta de rescate

40e. ¿Cuál de los siguientes tratamientos tiene una muy buena respuesta en asma
inducida por ejercicio?
a} Cromolín

41. Paciente femenino de 30 años, que cursa con su segundo embarazo. Tiene 23
semanas de gestación. Aparentemente normoevolutivo. Grupo sanguíneo B
negativo. Su esposo es grupo sanguíneo O positivo. El primer embarazo tuvo
complicaciones y abortó. La pareja del primer embarazo era distinta al del
segundo. Desconoce el grupo sanguíneo de la primera pareja. ¿Qué acción haría
usted en este momento?
b} Coombs indirecto

42. Secundigesta de 32 semanas de embarazo con antecedente de diabetes


gestacional. Se le realizó prueba de OSullivan la cual resultó positiva, por lo que
se llevó a cabo prueba de sobrecarga oral de glucosa, siendo éste también
positivo. Se le diagnosticó diabetes gestacional. Se inició manejo a base de dieta y
ejercicio, pero no se logró el control metabólico adecuado. ¿Cuál de los siguientes
medicamentos se deberá utilizar?
e} Insulina

43. Niña de cinco años de edad con una dermatosis diseminada, bilateral y
asimétrica que afecta región axilar e inguinal, constituida por pápulas umbilicadas,
en forma de domo translúcidas, bien delimitadas. ¿A qué familia pertenece el
agente causal de esta dermatosis?
a} Poxvirus

44. Paciente femenino de 32 años acude a consulta refiriendo que desde hace 3
semanas siente la necesidad de acudir al baño a orinar; sin embargo, cada vez
que lo intenta no logra sacar la orina por más que puja. Nunca le había sucedido
anteriormente. Es una mujer sana, sin antecedentes de importancia, no fumadora
y no toma alcohol. Al interrogatorio dirigido hace mención de que padece
cansancio extremo desde hace 2 meses e incapacidad para concentrarse. A la
exploración física se registra FV: 78, FR: 22, TA: 120/80 mm Hg, Temp: 36 ºC. El
examen general de orina es normal, no tiene bacterias ni leucocitos. Se le realiza
una cistometría en donde la contractilidad del detrusor es excesiva en respuesta a
un parasimpaticomimético.
44a. ¿Cuál es el diagnóstico más probable?
b} Esclerosis múltiple

44b. ¿Cuál es la fisiopatología de la enfermedad que padece la paciente?


a} Desmielinización y daño axonal por anticuerpos contra oligodendrocitos

44c. ¿En qué consiste el fenómeno de Uthoff?


d} Los síntomas aparecen o empeoran tras el ejercicio o baño caliente

44d. ¿Cómo se encuentra el líquido cefalorraquídeo de estos pacientes?


b} Con bandas oligoclonales de lgG

44e. En caso de iniciar con espasticidad, ¿qué tratamiento le indicaría a esta


paciente?
a} Baclofeno

45. ¿A qué se refiere el "efecto del alba"?


b} Hiperglucemia matutina causada por efectividad reducida de lainsulina en ese
momento

46. La principal e inicial medida terapéutica del estado hiperglucémico


hiperosmolar es:
b} La rehidratación

47. Un paciente de 10 años de edad inhala un cacahuate, el cual se encuentra


obstruyendo uno de sus bronquios principales. Se obtiene una radiografía la cual
revela desviación del mediastino hacia el lado de la obstrucción. ¿Cuál de las
siguientes opciones es la más adecuada para describir las alteraciones
pulmonares que sufre este paciente?
a} Atelectasias de absorción

48. Hombre de 38 años acude por una dermatosis bilateral y simétrica diseminada
a ambas plantas de los pies, a nivel de los primeros metatarsianos de cada lado.
Se caracteriza por dos úlceras, la del pie derecho de 3 cm de diámetro de forma
circular, bien delimitada con bordes queratósicos y en el lecho de la herida se
observa necrosis en 50% y el porcentaje restante corresponde a fibrina, con
eritema perilesional de 2 cm aproximadamente. La úlcera del pie izquierdo está
cubierta por una callosidad, la cual se desbrida y se encuentra úlcera de 2 cm de
diámetro de forma circular, bien delimitada con 70% de granulación y 30% de
fibrina. El paciente tiene diagnóstico de diabetes tipo 2 de larga evolución
controlada, es deportista, refiere que no se dio cuenta cuándo inició con las
lesiones. El diagnóstico es pie diabético. La conducta a seguir es:
e} Todas las respuestas anteriores son correctas
49. Paciente masculino de 49 años de edad, tras sufrir un accidente
automovilísitico se presenta en el servicio con epistaxis, obstrucción nasal y dolor
en misma región. A la exploración física se observa dorso nasal desviado a la
derecha,se palpa crepitación de huesos propios y a la rinoscopia se localiza
luxación del cartílago cuadrangular.
49b. Si el paciente, una vez hecha la reducción cerrada después del accidente,
persiste con deflexión septal importante, es probable que presente epistaxis
posteriores. ¿Qué mecanismo se encuentra alterado en este caso?
a} Flujo nasal

50. Hombre de 38 años de edad. Acude a revisión oftalmológica general. Se


refiere asintomático. Sin antecedentes personales patológicos ni oftalmológicos. A
la exploración oftalmológica muestra agudeza visual de 20/20 en ambos ojos. El
globo ocular sin alteraciones. A la exploración de los párpados, se ubica una
lesión en el borde palpebral inferior derecho ligeramente elevada y pigmentada.
Se hace el diagnóstico de nevo pigmentado. En qué estructura anatómica del
párpado se encuentra localizada la lesión?
e} Lámina anterior delpárpado

51.A la exploración del fondo de ojo de una paciente de 16 años se observa


disminución focal y generalizada de las arteriolas. Al continuar con la exploración
se encuentran hemorragias en la retina izquierda, así como esclerosis. Al
interrogatorio la paciente niega cualquier alteración visual. Debido a estos
hallazgos, se decide tomar la presión arterial, la cual resulta en 190/103 mm Hg.
¿Cuál de los siguientes hallazgos es más probable que también se presente en
este caso?
a} Cefalea

52. Acude a consulta pediátrica un paciente de 12 años de edad que sufrió un


golpe en la región lateral de la rodilla derecha durante un partido de fútbol. A la
exploración física el paciente muestra abducción anormal de la rodilla al
movimiento pasivo y desplazamiento anterior excesivo de la pierna cuando la
articulación de la rodilla se coloca en ángulo recto. Se solicita resonancia
magnética de la región, la cual confirma daño del ligamento colateral medio y del
menisco medio. Es probable que exista una tercera estructura lesionada en este
paciente; durante la reparación quirúrgica, ¿dónde deberá insertarse esta tercera
estructura?
c} Medial y anterior

53. Una paciente de 50 años de edad con diabetes mellitus tipo 2 y antecedente
de síndrome de ovario poliquístico se somete a histerectomía total abdominal para
tratamiento y estadificación de carcinoma endometrial. ¿Cuál de las siguientes
estructuras está en riesgo de ser dañada cuando las arterias uterinas son ligadas?
e} Ureteros
54. Una paciente de 30 años de edad con un embarazo de 40 semanas de
gestación se presenta en trabajo de parto. Una hora posterior al parto la paciente
presenta dolor intenso en la región perianal. El anestesiólogo decide realizar un
bloqueo nervioso para tratar dicho dolor ¿Cuál de los siguientes nervios se
encarga de inervar la región perianal y, por lo tanto, es responsable del dolor en
esta paciente?
c} Pudendo

55. Femenino de 19 años con antecedente de hospitalizaciones frecuentes por


intoxicación alcohólica. En esta ocasión es ingresada después de haber estado
ingiriendo de manera constante bebidas alcohólicas desde hace cinco días. A la
exploración física, la presión arterial es de 110/70 mm Hg, frecuencia cardiaca de
80 latidos/min, peso de 70 kg. Se presenta letárgica y con habla incoherente. En la
sala de Urgencias, presenta una crisis convulsiva, que se yugula con diazepam.
Los estudios de laboratorio reportan sodio de 11O mEq/L, potasio sérico de 3.8
mEq/L, nivel de alcohol de 250 mg/dl, osmolalidad sérica de 230 mOsm/kg, y
concentración de glucosa de 92 mg/dl. No hay alteración en la función renal.
¿Cuál de las siguientes opciones representa el mejor tratamiento para esta
paciente?
a} Solución salina hipertónica (3%) intravenosa a una velocidad de infusión de
130 ml/h

56. Después de sufrir infarto cerebral, un paciente se presenta con debilidad en


ambas extremidades derechas, desviación de la mandíbula hacia la izquierda al
momento de la protrusión, y anestesia de la cara y cuero cabelludo.
¿Cuál es el sitio de lesión más probable?
a} Puente

57. Paciente femenino de 77 años de edad con antecedente de hipertensión,


insuficiencia renal crónica y bronquitis crónica. Es ingresada por exacerbación de
su condición pulmonar. El tratamiento iniciado consiste de agonistas beta
adrenérgicos, corticoesteroides, y trimetoprim-suflametoxazol. Después de siete
días de tratamiento, la condición de la paciente mejora de manera considerable.
Los estudios de laboratorio al momento del ingreso y siete días después se
muestran a continuación:
¿Cuál de las siguientes opciones se puede considerar como la principal causa de
la hiperpotasemia de esta paciente?
c} Tratamiento con trimetoprim-sulfametoxazol

58. Paciente masculino de 66 años de edad acude a consulta de seguimiento por


adenocarcinoma de colon hace cinco años. El paciente tiene antecedente de
hemicolectomía derecha por el proceso neoplásico, y en esta ocasión acude para
revisar los resultados de un enema de bario que se le realizó recientemente. En el
estudio se revela un pólipo pendunculado de 1.7 cm en la flexura esplénica, así
como múltiples divertículos. ¿Cuál de las siguientes opciones es la más adecuada
en este paciente?
d} Realizar una colonoscopia con polipectomía

59. Paciente masculino de seis años con antecedentes de asma es llevado a


urgencias por su madre por presentar dificultad respiratoria posterior a estar
jugando en el jardín. La madre refiere que el paciente presenta dos crisis a la
semana,cuatro crisis nocturnas al mes con dificultad ocasional para conciliar el
sueño, su flujo espiratorio máximo (PEF) es mayor a 80% del PEF teórico con
variabilidad de 25%. A la exploración física se observa al paciente sentado,
agitado, taquipneico, con uso de músculos accesorios y sibilancias espiratorias.
¿Cómo clasificaría la enfermedad asmática de este paciente?
c} Asma persistente leve

60. Bailarina exótica (table dance) de 22 años de edad que es llevada al servicio
de urgencias por fibrilación auricular. La paciente menciona que desde hace varias
semanas había presentado palpitaciones, con mayor frecuencia los lunes. A la
exploración física la paciente tiene una frecuencia cardiaca de 123 latidos/minuto,
frecuencia respiratoria de 18 respiraciones/minuto, presión arterial de 175/98 mm
Hg. Se puede percibir un soplo sistólico. Sin tratamiento, el ritmo cardiaco se
vuelve sinusal. ¿Cuál de las siguientes opciones es la más probable como agente
etiológico del cuadro de esta paciente?
e} Alcohol

61. Se presenta a consulta un paciente de dos meses de edad. Tomando en


cuenta que recibió de manera adecuada las vacunas que se requieren al momento
del nacimiento. ¿Cuál de las siguientes vacunas se le debe administrar en esta
consulta según el esquema de vacunación de México?
b} Pentavalente acelular,rotavirus, antihepatitis B y neumocócica
conjugada
62. Paciente de 21 años de edad que es llevada al Servicio de Urgencias por
presentar disnea y dolor torácico tipo pleurítico del lado derecho. Ha estado
utilizando anticonceptivos orales desde los 18 años de edad. Se encuentra
normotensa, pero taquicárdica y taquipneica. ¿Cuál de las siguientes opciones
sería la de mayor utilidad para excluir el diagnóstico de embolismo pulmonar?"
b} Niveles plasmáticos normales de dímero D

63. Paciente masculino de 1O días de edad es llevado a consulta para evaluación


de rutina. Fue producto de un embarazo de 38 SDG y parto eutócico. La madre
notó que el paciente inició con ictericia a partir del cuarto día de vida y que
aumentó para encontrar su máxima intensidad en el día seis de vida. La madre
menciona que al parecer la ictericia se encuentra disminuyendo. A la exploración
física el paciente está afebril, y sin ninguna otra anormalidad aparte de la ictericia.
Al momento del alta,el día tres de nacido, la bilirrubina total era de 8.7, mientras
que la bilirrubina conjugada fue de 0.5. La madre y el niño tienen sangre de tipo
O+, y al parecer el paciente tolera bien la vía oral. ¿Cuál de las siguientes
opciones es el diagnóstico más probable en este recién nacido?
e} Ictericia fisiológica

64. ¿Cuál de los siguientes agentes fibrinolíticos tiene una vida media prolongada,
lo cual permite su administración en forma de bolo?
c} Tenecteplasa

65. ¿Cuál de los siguientes medicamentos se relaciona con el desarrollo de


leucopenia y exantema eritematoso?
c} Procainamida

66. ¿Cuál de las siguientes descripciones corresponde con los nódulos de Osler?
b} Lesiones pequeñas, dolorosas, elevadas, de color rojo o púrpura,
que se presentan en las falanges terminales delas manos

67. Paciente masculino de 12 años de edad, quien este invierno presenta cuadros
repetitivos de infección de vías aéreas superiores, múltiples tratamientos
sintomáticos sin recibir antibióticos por considerarse cuadros virales y alérgicos;
hace 2 días y el día de hoy por la madrugada se despierta por accesos de tos
disneizante, cianozante, la madre refiere que se escucha un "silbato" cuando el
niño respira, mejora ligeramente cuando no tose y en reposo. A la exploración
física se encuentra paciente con taquipnea, tos, disnea que limita la actividad
física,sibilancias audibles a distancia,acrocianosis, aleteo nasal y tiraje intercostal
bajo, la orofaringe se encuentra hiperémica, con descarga posterior hialina,
hipertrofia amigdalina grado 2/4; resto de la exploración sin alteraciones evidentes.
FC 95 lpm, FR 22 rpm, temp. 36.7ºC, TA 100/70 mm Hg, peso 32 kg, talla 1.30 m.
67a. El diagnóstico clínico del paciente es:
c} Crisis asmática

67b. Al evaluar al paciente usted solicita una espirometría que reporta FEV1
>80%, junto con los datos clínicos puede clasificar esta exacerbación como:
b} Persistente leve

67c. Una vez establecido el grado de severidad, usted inicia tratamiento con:
d} Corticoesteroidesinhalados a dosis bajas y beta-2-agonistas delarga duración
67d. Al solicitar varios estudios, usted espera encontrar:
b} Biometría hemática con eosinofilia

67e. Se controla la crisis con el tratamiento establecido, al egreso del paciente


usted recomienda:"
a} Beta-2-agonistas de corta acción

68. Paciente femenino de 28 años de edad que acude a consulta por disnea y
malestar general. Al interrogatorio refiere edema de extremidades inferiores,
ortopnea y disnea paroxística nocturna; menciona que ha tenido fiebre intermitente
y escalofríos. La presión arterial es de 150/60 mm Hg y la frecuencia cardiaca de
99 latidos/minuto. La presión venosa yugular se encuentra elevada. Los pulsos de
las arterias femorales y carótida son saltones con un colapso brusco. A la
auscultación se encuentra un soplo 11/IV sistólico de eyección y un soplo 111/IV
diastólico en decrescendo a lo largo del borde esternal izquierdo que aumenta de
intensidad durante el empuñamiento. Se puede percibir un tercer ruido cardiaco.
Existen estertores bilaterales y edema de las extremidades inferiores. No hay
datos de endocarditis infecciosa. No hay datos que sugieran endocarditis
infecciosa y el electrocardiograma no es diagnóstico. La radiografía de tórax
demuestra cardiomegal ia moderada y edema pulmonar intersticial. ¿Cuál sería el
estudio diagnóstico más adecuado en este caso?

b} Ecocardiografía transtorácica

69. Lactante de ocho meses de edad que es llevado a consultar por fiebre de 39
ºC y rechazo al alimento de dos días de evolución. La madre niega rinorrea,tos,
expectoración, vómito, diarrea u otro síntoma. En el examen físico se confirma la
fiebre, y no se encuentran hallazgos de relevancia. Se solicita una biometría
hemática que demuestra leucocitos de 17 000. Se solicita radiografía de tórax que
no muestra anormalidades y el examen general de orina revela 20 leucocitos por
campo, con nitritos positivos.
69a. Respecto al diagnóstico que sospecha, ¿cuál de las siguientes opciones es
FALSA?
e} El diagnóstico se hace si en elurocultivo se detectan más de 10 000 unidades
formadoras de colonias de una muestra obtenida por bolsa colectora

69b. ¿Cuál estudio de los siguientes es útil para evaluar la presencia de reflujo
vesicoureteral y se caracteriza por dosis mínimas de radiación?
c} Cistografíaisotópica

70. Paciente masculino de 20 años que pertenece al ejército acude a urgencias


por presentar tos no productiva desde hace una semana asociada con fiebre,
disnea y dolor torácico al respirar profundamente. Las radiografías PA y lateral de
tórax muestran un infiltrado intersticial bilateral. Usted diagnostica neumonía
atípica adquirida en la comunidad.
¿Cuá es el antibiótico de elección para el tratamiento de la neumonía atípica
adquirida en la comunidad?
c} Azitromicina

71. Se trata de una mujer de 28 años de edad, sin antecedentes relevantes, niega
haberse enfermado en el último año o haber tomado antibióticos, presenta un
cuadro de fiebre con escalofrío, náusea, lumbalgia, disuria y polaquiuria, motivo
por el que acudió a la consulta; a la exploración física se encuentra
hipersensibilidad en el ángulo costovertebral derecho, así como hiperbaralgesia en
los puntos ureterales del mismo lado. FC 105 lpm, FR 18 rpm, temp. 38.3 ºC, PA
110/70 mm Hg, peso 58 kg, talla 1.68 m
71a. Con los datos obtenidos ¿cuál es la sospecha diagnóstica?
c} Pielonefritis aguda

71b. Para confirmar el diagnóstico usted decide tomar uno de los siguientes
estudios de laboratorio:
c} Urocultivo

71c. En relación al urocultivo, ¿cuál de las siguientes frases define una "bacteriuria
significativa"?
b} Recuento de > 100000 UFC/ml

71d. Esta paciente no está exenta de complicaciones, ¿cuál es una indicación de


estudios de imagen?
d) Fiebre más de 72 horas

71e. Establecido el diagnóstico y tomando los factores de riesgo de la paciente


¿cuál es el tratamiento de primera elección?
a} Ampicilina 1g cada 6 horas + gentamicina 1mg/kg cada 8 horas

72. Masculino de 64 años, consulta por debilidad muscular,fatiga y dolor leve en


hipocondrio derecho. Refiere ser fumador de larga evolución. Menciona que la
fatiga ha ido en aumento de manera paulatina y actualmente se fatiga al realizar
actividades de mínimo esfuerzo, pero no en reposo. A la exploración se registra
TA: 100/90, FC: 95 lpm, ingurgitación yugular leve, estertores finos bibasales,
precordio rítmico, con S3, hepatomegalia, frialdad en extremidades inferiores,
pulso poplíteo disminuido y pulso pedio ausente.
72a. De acuerdo con las características de este caso, ¿cuál es el estudio que
ayudaría a confirmar el diagnóstico y la causa?
a} Ecocardiografía

72b. ¿Cuál de los siguientes medicamentos ha demostrado disminuir la mortalidad


en insuficiencia cardiaca?
b} Carvedilol

72c. ¿Cuál de las siguientes medidas terapéuticas generales no está indicada en


la insuficiencia cardiaca?
d} Restricción de sodio a 2 g/día en pacientes asintomáticos
72d. De acuerdo con el caso clínico, ¿en cuál estadio de insuficiencia cardiaca de
la AHA/ACC (American Heart Association/American College of Cardiology) se
encuentra el paciente?
c} C

72e. Según la clasificación de la New York Heart Association,¿en qué clase


funcional se encuentra el paciente?
c} Clase 111

73. Se trata de un paciente masculino de la 3ª década de la vida, quien acude a


valoración por presentar cefalea intensa, que cataloga como la "peor" de su vida
de tipo holocraneana y pulsáil,inició hace 48 horas y se acompaña de inestabilidad
de la marcha y malestar general, con escalofríos de predominio nocturno; a la
exploración encontramos datos de irritación meníngea, debilidad asimétrica de
extremidades inferiores y reflejos plantares normales, sin otros datos relevantes.
FC 95 lpm, FR 20 rpm, temp. 37.8 ºC, PA 135/75 mm Hg, peso 67 kg, talla 1.72m.
73a. Con los datos clínicos usted sospecha en:
c} Meningitis bacteriana

73b. Al sospechar de una neuroinfección, usted realiza una tomografía de cráneo


que se encuentra sin alteraciones estructurales y una punción lumbar, ¿cuál de las
alteraciones citoquímicas no corresponde a una meningitis bacteriana?
c} Hiperglucorraquia

73c. Basado en la epidemiología reportada, usted espera encontrar con mayor


frecuencia uno de los siguientes patógenos en el cultivo:
a} Streptococcus pneumoniae

73d. ¿Cuál de los siguientes esquemas de antimicrobianos es de primera línea


para esta neuroinfección?
b} Cefalosporina de tercera generación + Vancomicina

73e. ¿Cuál de los siguientes factores es de mal pronóstico en las neuroinfecciones


bacterianas?
c} Hipoglucorraquia

74. Paciente femenina de 30 años, acude por astenia y mialgia de varias semanas
de evolución que han progresado en intensidad; recientemente inicia con pérdida
de peso, náusea, dolor abdominal y deseo de consumir sal. TA: 100/60 mm Hg,
FC: 65 lpm; se observa hiperpigmentación en nudillos y pliegues palmares. Labs:
glucosa 64 mg/dl, Na: 132 mmol/L, K: 5.5 mmol/L, pH: 7.32, PaCO: 30 mm Hg,
HC03: 15 mEq/L.
74a. ¿Cuál es el diagnóstico más probable en esta paciente?
a} Insuficiencia suprarrenal primaria

74b. ¿Cuál es la causa más frecuente del padecimiento de esta paciente?


b} Adrenalitis autoinmune

74c. ¿Cuál es el estudio indicado para iniciar el protocolo diagnóstico y el


resultado esperado en la insuficiencia suprarrenal primaria?
a} Cortisol plasmático disminuido, ACTH elevada

74d. Una vez diagnosticada la insuficiencia suprarrenal primaria, ¿cuál es el


siguiente paso para demostrar su causa?
d} Autoanticuerpos suprarrenales y tomografía suprarrenal

74e. ¿Cuál es el tratamiento indicado en la insuficiencia suprarrenal primaria?


a} Hidrocortisona + fludrocortisona

75. Lactante de dos meses de vida,con antecedente de síndrome de Down, que es


traído a consulta porque presenta desde su primera semana de vida episodios de
regurgitación posprandial de contenido alimentario después de cada toma. No se
reconocen otros síntomas digestivos. Se niegan antecedentes de dificultad
respiratoria,tos, estridor y sibilancias. Su peso se encuentra en el P75.
Actualmente es alimentado sólo con leche materna.
75a. De acuerdo con la patología que sospecha en el paciente, indique cuáles de
los siguientes factores se consideran predisponentes:
1 = relajaciones transitorias del esfínter esofágico inferior 2 = hipersecreción de
bicarbonato a nivel gástrico
3 = tomas de alimento espeso
4 = hernia de la pared abdominal 5 = posición supina
a} 1 y 5

75. Lactante de dos meses de vida,con antecedente de síndrome de Down, que es


traído a consulta porque presenta desde su primera semana de vida episodios de
regurgitación posprandial de contenido alimentario después de cada toma. No se
reconocen otros síntomas digestivos. Se niegan antecedentes de dificultad
respiratoria,tos, estridor y sibilancias. Su peso se encuentra en el P75.
Actualmente es alimentado sólo con leche materna.
75b. Todas las siguientes pueden ocurrir como complicaciones de la patología que
presenta el paciente, EXCEPTO:
d} Gastritis

75c. ¿Cuál de los siguientes estudios tiene mayor sensibilidad en el diagnóstico de


la patología del caso clínico?
d} pHmetría de 24 horas

75d. De acuerdo al caso clínico, ¿qué abordaje terapéutico es el más adecuado en


este momento?
b} Espesar las tomas de alimento
76. Recibes en la guardia a una menor de 14 años, por un aborto incompleto quien
al día siguiente se le realizará un legrado. El sangrado se ha controlado y no hay
datos de infección. Al revisar el expediente no hay indicios de que se le haya
informado a la paciente del procedimiento.
76a. Ante esta situación, ¿cuál es la mejor conducta a seguir?
c}Informar sobre anestesia,legrado, resultados y complicaciones a la paciente

76b. Como todo procedimiento quirúrgico, éste tiene sus complicaciones. Por ello,
¿qué es lo más recomendable para la paciente?
d} Darleinstrucciones verbales y por escrito delos cuidados posoperatorios

77. Niño de cinco años, sin antecedentes relevantes, que es llevado a consulta por
haber iniciado de forma súbita 3 horas previas a su ingreso con cuadro de náusea
y evacuaciones diarreicas sin moco y sin sangre. No ha presentado fiebre y niega
contacto con alguien más que tuviera síntomas gastrointestinales. La madre
relaciona con el cuadro que 4 horas antes de iniciar con los síntomas, el paciente
ingirió mucho hierro. ¿Cuál es el tratamiento de elección para esta patología?
c} Desferoxamina

78. Paciente masculino de 43 años de edad quien es originario de la zona rural de


Chiapas, agricultor, con antecedente de hipertensión areterial sistémica y diabetes
mellitus tipo 2, antecedentes heredofamilar madre fallecida por cuadro de
tuberculosis. Acude a consulta por dolor a nivel lumbar de 3 semanas de evolución
que no cede a la ingesta de AINE y dificultad progresiva para la marcha, refiere
además parestesias en cara interna de muslo izquierdo. Al interrogatorio dirigido
se identifica la presencia de fiebre de predominio nocturno y pérdida de peso hace
1 año.Niega antecedente de trauma. FC 87 lpm, FR 16 rpm,temp 37.5 ºC, PA
140/90 mm Hg, peso 43 kg, talla 1.76 m.
78a. Su diagnóstico más probable con este paciente sería:
b} Enfermedad de Pott

78b. ¿Qué esperaría observar en el estudio de imagen y PPD?


c} Densidad ósea disminuida L3 y PPD > 1Omm

78c. Una vez que realiza el diagnóstico de enfermedad de Pott. ¿Cuál es el


tratamiento farmacológico que indicaría a este paciente?
b} Rifampicina, Etambutol,lsoniazida, Pirazinamida

78d. Con los estudios de imagen correspondientes se encuentra un grado de lisis


ósea grave, con riesgo alto de fractura. Dentro del tratamiento de las
complicaciones por la enfermedad de Pott: ¿Qué tratamiento adicional sugeriría?
c} Cirugía de Columna
78e. Una vez que se da tratamiento y se continúa con el seguimiento del paciente.
¿qué otra medida debe realizar?
d}Identificar otros posibles casos en el entorno familiar del paciente

79. ¿Cuál es el mecanismo de acción por el que la selegelina puede mejorar la


sintomatología de la enfermedad de Parkinson?
b}Inhibición dela degradación de dopamina porla monoaminooxidasa tipo B

80. ¿Cuál de los siguientes medicamentos es el más adecuado en el caso de


pacientes con enfermedad de Wilson?
a} Penicilamina

81. Femenino de 35 años acude a Urgencias debido a un dolor en la fosa lumbar


derecha que se irradia hacia la ingle del mismo lado, pasando por el flanco
derecho. El dolor inició hace algunas horas y no ha cedido con la ingesta de
diclofenaco. La paciente refiere que es la primera vez que le sucede, pero ha
tenido desde hace días molestia al orinar y sensación de vaciado incompleto; se
observa afectada, sudorosa y cambia de postura continuamente. EF: 1.50m; 85kg;
TA: 145/85; Temp:36.5°C.
81a. ¿Cuál es la principal sospecha diagnóstica?
c} Urolitiasis

81b. ¿cuál es el hallazgo más común en el examen general de orina en pacientes


con litiasis renal?
b} Hematuria

81d. El estándar de oro para el diagnóstico es:


b} TC

81e. Lito más frecuente:


c} Oxalato cálcico

82. Paciente de 46 años con diagnóstico de varios años de esquizofrenia. El


médico tratante decide iniciar un nuevo antipsicótico. Dos semanas después del
cambio de antipsicóticos el paciente se presenta a consulta de urgencia con un
cuadro neumónico grave. La citometría hemática revela reducción del conteo de
neutrófilos, basófilos y eosinófilos.
¿Cuál de los siguientes fármacos es el que con mayor probabilidad pudo haber
causado este trastorno?
b} Clozapina

83. ¿Cuál de los siguientes enunciados respecto a la recurrencia de la infección


por Clostridíum díffícíle es correcta?
b} Las recurrencias delasinfecciones por Clostridium difficile se asocian con
complicaciones serias
84. ¿Cuál de los siguientes movimientos es el que más frecuentemente ocasiona
dolor en pacientes con síndrome del manguito retador?
a} Abducción delbrazo

85. Se presenta a consulta externa paciente del sexo femenino de 20 años de


edad por presentar resequedad y ulceraciones en la piel que cubre las
prominencias óseas, principalmente las rodillas y los codos. Entre los hallazgos de
importancia a la exploración física se pueden observar placas pequeñas grisáceas
en la conjuntiva. ¿Cuál de los siguientes diagnósticos es el más probable en esta
paciente?

b} Deficiencia de vitamina A

86. Paciente femenino de 29 años de edad con antecedente de esquizofrenia.


Había sido internada debido a que sufrió varios episodios en los que decía que
escuchaba voces que la insultaban y le decían qué acciones llevar a cabo. Al
momento del egreso se le prescribió haloperidol. ¿Cuál sintomatología es la más
probable que la paciente llegue a presentar?
d} Galactorrea

87. ¿Cuál de los siguientes hallazgos podría estar presente con mayor
probabilidad en pacientes con lesión de la división superior del nervio facial?
a}Incapacidad ipsilateral para fruncirla frente

88. Paciente femenino de 15 años de edad estudiante de bachillerato que se


presenta a consulta. La paciente presenta antecedente de asma leve intermitente
bajo tratamiento adecuado; sin embargo, desde hace cuatro días ha presentado
fiebre, tos, malestar generaly mialgias intensas. Al interrogatorio menciona que
varios profesores y estudiantes de su centro de estudios presentan sintomatología
similar.Al revisar su expediente, el médico observa que esta paciente no recibió su
última dosis de inmunoprofilaxis contra la infección del virus de la influenza. ¿Cuál
de los siguientes tratamientos es el más adecuado en ella?
b} Tratamiento sintomático basado en antihistamínicos

89. Una mujer de 45 años de edad que se presenta a consulta por padecer dolor
de hombro desde hace aproximadamente ocho semanas, el cual es más intenso
cuando acude a sus entrenamientos de tenis. Durante el reposo el dolor suele
desaparecer.A la exploración física el hombro se encuentra doloroso, con
limitación del movimiento activo y pasivo. En las radiografías de la región se
pueden observar calcificaciones por encima del trocánter mayor. Se le indica a la
paciente que lo más probable es que padezca tendinitis calcificada, por lo que
requiere tratamiento durante seis semanas con un cabestrillo, reposo, calor y
antiinflamatorios no esteroideos. Después del periodo de tratamiento la paciente
acude de nuevo a consulta, pero los síntomas persisten. ¿Cuál de las siguientes
opciones describe mejor el siguiente paso en el manejo de esta paciente?
e} Aspiración de la calcificación guiada por ultrasonido
90. ¿Cuál de los siguientes organelos se encuentra ausente en los eritrocitos
maduros, y por esta razón éstos son incapaces de sintetizar el grupo heme de la
hemoglobina, a pesar de que se les inyecte el mRNA correspondiente?
e} Mitocondria

91. Paciente femenino de 19 años de edad acude a consulta por tener desde hace
varias semanas fatiga, palpitaciones y palidez. A la exploración física muestra un
síndrome anémico, caracterizado por palidez de mucosas y tegumentos,
taquicardia y soplo sistólico. Los exámenes de laboratorio confirman anemia
megaloblástica. La paciente menciona que está consumiendo antibióticos para
una infección de las vías urinarias. Debido a que en el pasado tuvo una infección
de vías urinarias y recibió una receta con el medicamento adecuado, consume el
mismo medicamento cada vez que tiene una infección de vías urinarias. ¿Cuál de
los siguientes medicamentos es el que con mayor probabilidad está consumiendo?
e} Trimetoprim-sulfametoxazol

92. En un paciente que se presenta con "pie diabético", ¿cuál de los siguientes
diagnósticos diferenciales de osteomielitis es el más difícil de descartar a pesar de
la realización de complejos estudios de imagen, como tomografía computarizada,
resonancia magnética, ultrasonido y medicina nuclear ósea?
b} Pie de Charcot

93. Según la clasificación de fracturas del platillo tibial de Schatzker, ¿cuál de las
siguientes opciones representa la que involucra sólo el platillo tibial lateral sin
presentar depresión?
a} Tipo 1

94. ¿Cuál de las siguientes vitaminas contribuye a la carboxilación de residuos de


glutamina de proteínas en el hígado?
a} Vitamina K

95. ¿Cuál de los siguientes medicamentos es el tratamiento de elección en el


contexto de una intoxicación por propranolol?
c} Glucagon

96. Paciente indígena de 20 años de edad la cual se presenta a hospital rural con
embarazo del primer trimestre, febrícula y un exantema maculopapular. La
paciente refiere que el exantema inició hace dos días en la cara y posteriormente
se extendió a tórax y extremidades. Al parecer la paciente jamás ha sido
vacunada. A la exploración física presenta linfadenopatías posauriculares. ¿Cuál
es el agente que más probablemente está causando el cuadro clínico?
c} Virus de la rubeola
97. Paciente femenino de 94 años, sin antecedentes de interés, salvo asma
extrínseca por epitelio de animales, osteoporosis y cirugía de prolapso uterino con
incontinencia urinaria de esfuerzo residual.¿Cuál de estos elementos no es
indispensable para ser un individuo continente?
d} No ser persona dela tercera edad

98. Paciente masculino de dos meses de edad que fue diagnosticado hace poco
con deficiencia del complejo piruvato deshidrogenasa. Actualmente el paciente
sufre grave sintomatología neurológica y acidosis láctica. ¿Cuál de las siguientes
sustancias puede ocasionar una disminución de la acidosis láctica de este
paciente?
a} Lisina

99. lván es un paciente masulino de 45 años de edad con antecedente de


Diabetes Mellittus tipo 2 desde hace 5 años en regular apego al tratamento. Acude
a consulta por presentar desde hace 48 horas secundaria a la realización de
actividad física ptosis del párpado derecho, así como desviación de la comisura
labial hacia la izquierda lo cual dificulta la alimentación y el habla. Dentro del
exámen físico se indentifica disminución de pliegues frontales derechos y
borramiento de surco naso geniano. Resto de la exploración sin alteraciones. FC
98 lpm, FR 18 rpm, temp 36.8ºC, PA 130/85 mm Hg, peso 76 kg, talla 1.56 m.
99a. Con base en el interrogatorio de la paciente, el diagnóstico presuntivo es:
c} Parálisis facial derecha periférica

99b. El término como se le conoce a la parálisis periférica del nervio facial es:
c} Bell

99c. ¿Cuál es el tratamiento para la parálisis facial periférica idiopática?


c} Corticoides

99d. ¿Qué otros cuidados se debe mantener con el paciente que presenta
parálisis facial periférica?:
d} Cuidados de córnea y rehabilitación

99e. El síndrome de Ramsay Hunt se produce por:


b} Varicela zoster

100. ¿Cuál de las siguientes condiciones incrementa el volumen residual del


pulmón?
c} Enfisema pulmonar

101. ¿Cuál de las siguientes anormalidades de la coagulación tiene la prevalencia


más alta en pacientes con
tromboembol ismo venoso?
a} Mutación en elfactor V de Leiden
102. ¿Cuál de las siguientes bacterias es una causa frecuente de meningitis en
pacientes posoperados de procedimientos neuroquirúrgicos?
e} Staphy/ococcus aureus

103. Paciente masculino de nueve meses de edad que presenta dermatosis


localizada a región genital afectando escroto, periné y región perianal;que está
caracterizada por eritema, erosiones, escama y algunas zonas con costras
serohemáticas y maceración, de 1O días de evolución,caracterizando una
dermatitis del pañal. ¿Qué factores predisponen esta enfermedad?
a} Contacto prolongado con evacuaciones

104. Paciente femenino de siete años de edad que padece un infarto miocárdico.
En los estudios de laboratorio se encuentra la presencia de elevaciones de
metionina en sangre. ¿Cuál de los siguientes aminoácidos es considerado como el
más esencial en pacientes con este tipo de trastorno metabólico?
e} Cisteína

105. Usted se encuentra en el quirófano para recibir a un recién nacido por


cesárea. Antecedentes: 42 semanas de gestación, control prenatal regular,sin
complicaciones (diabetes, hipertensión, sangrado) durante el embarazo;
momentos antes de entrar a quirófano se refiere rotura de membranas y líquido
amniótico con meconio. Se indica cesárea por desaceleraciones. A la exploración,
observa al paciente con hipotonía, dificultad respiratoria y frecuencia cardiaca de
90 lpm.
105a. De acuerdo con las características del paciente, ¿cuál es la causa
fisiopatológica más probable de su padecimiento?
b} Aspiración meconial

105b. ¿Cuál de los siguientes antecedentes es de utilidad para apoyar la


sospecha diagnóstica?
c} Meconio enlíquido amniótico

105c. ¿Cuál es el método diagnóstico de este padecimiento?


a} Radiografía de tórax

105d. De acuerdo con las características y antecedentes del paciente, ¿qué


tratamiento específico debe instituirsede manera inmediata?
c}Intubación con aspiración endotraqueal

105e. De acuerdo con las características y antecedentes del paciente, ¿cuál es la


complicación más grave que podría presentar éste?
b} Hipertensión pulmonar persistente neonatal
106. Paciente femenino de 75 años con diagnóstico de DM2 desde hace 15
años(en tratamiento con metformina e insulina), hipertensa desde los 45 años de
edad (en tratamiento con nifedipino e hidralazina), fumadora desde hace 45 años
(cinco cigarrillos diarios),acude a consulta por sus propios medios (caminando)
debido a que sufrió el día de ayer una caída desde su propia altura, de sentón, en
el baño.Según refiere, la caída la relaciona con una pérdida reciente de fuerza en
las piernas; también se queja de dolor en la región de las nalgas.No presenta
limitaciones del movimiento y comenta que ha perdido altura de manera reciente.
La EF: 1.58m, 65kg; TA: 140/80; Temp: 36.5ºC. A la exploración se registra dolor
en la zona de las nalgas, movilidad y fuerza de los miembros pélvicos sin
alteraciones, reflejo anal y continencia fecaly urinaria conservadas, así como dolor
a la palpación de la columna lumbar.
106a. ¿Cuál de los siguientes estudios sería de mayor utilidad diagnóstica?
d} Radiografías simples de columna vertebral

106b. En el estudio anterior se evidencia disminución de la altura anterior de los


cuerpos vertebrales T2 y T3, además de una xifosis prominente. De acuerdo
conesto, ¿cuálsería su diagnóstico?
b} Fractura vertebral por compresión

106c. Por el tipo de fractura y los antecedentes, usted solicita una densitometría
ósea, la cual se la reportan con un valor de T=-2.8.¿Cuálsería su diagnóstico?
c} Osteoporosis

106d. ¿Cuál es un factor de riesgo modificable para osteoporosis?


b} Tabaquismo

107. Paciente femenino de 23 años de edad que ha iniciado su vida sexual activa
hace 20 días. Como método de anticoncepción su médico le ha recetado
anticonceptivos orales. A pesar de que la paciente no fuma, ésta presenta embolia
pulmonar asociada al uso de anticonceptivos orales. ¿Cuál de las siguientes
opciones representa el factor que con mayor probabilidad predispuso a esta
paciente?
a} Alteraciones en el factor V de la coagulación

108. ¿Cuál de las siguientes estructuras se dañaría en caso de una lesión


penetrante en la fisura orbitaria superior?
a} Nervio craneal VI

109. Paciente recién nacido de sexo femenino de madre soltera de 22 años.


Veinticuatro horas después de nacer el paciente presenta llanto agudo, temblor,
rinorrea profusa y diarrea. El cuadro se acompañó minutos antes de crisis
convulsivas mioclónicas. Al interrogatorio la madre dice que nunca llevó control
prenatal. ¿Cuál de los siguientes fármacos puede disminuir la sintomatología del
recién nacido?
c} Opio
11O. ¿Cuál de los siguientes hallazgos histopatológicos óseos son característicos
de la deficiencia de vitamina D?
e} Acumulación de matriz extracelular osteoide alrededor delas trabéculas

111. ¿Con cuál de las siguientes condiciones se asocia la enfermedad celiaca?


d} Dermatitis herpetiforme

112. ¿Cuál de los siguientes virus que ocasionan hepatitis requiere coinfección
con el virus de la hepatitis B para establecerse en el hígado?
d} Virus de la hepatitis D

113. Paciente masculino de 15 años de edad cae durante una competencia de


velocidad en patineta. Informa que, al momento de caer, su codo derecho golpeó
contra el piso y tiene dolor muy intenso en el brazo. A la exploración física no
puede extender la muñeca y tiene disminución de la sensibilidad en la porción
lateral del dorso de la mano ipsolateral. ¿Cuál de las siguientes estructuras es
más probalbe que se encuentre lesionada?
c} Diáfisis del húmero

114. Paciente masculino de 30 años de edad el cual se diagnostica recientemente


con diabetes mellitus tipo 2. Para iniciar el tratamiento se inicia un plan de ejercicio
y dieta;sin embargo, el paciente no tiene apego a estas medidas por lo que se
decide iniciar medicamentos hipoglucemiantes. La principal preocupación es que
debido a las actividades profesionales del paciente, éste suele tener periodos no
planeados y prolongados de ayuno, lo cual podría ocasionar episodios de
hipoglucemia en caso de iniciar con fármacos antidiabéticos. ¿Cuál de los
siguientes fármacos predispone al mayor riesgo de hipoglucemia?
b} Gliburida

115. ¿Cuál de las siguientes características está ausente en la insuficiencia


suprarrenal secundaria?
b} Hiperpotasemia

116. Paciente masculino de 55 años de edad el cual se presenta a consulta por


ardor gástrico desde hace cuatro semanas. Tiene antecedente de enfermedad
acidopéptica (úlceras gástricas), así como diabetes de cuatro años de diagnóstico.
El paciente menciona que al mismo tiempo que inició con el malestar gástrico
empezó a presentar saciedad temprana, pérdida de peso, fatiga, sudación cuando
duerme y episodios febriles. Al paciente se le realiza una prueba de urea en
aliento, la cual resulta positiva. ¿Cuál de las siguientes condiciones es la que
ocasiona directamente y con mayor probabilidad todos los síntomas de este
paciente?
b} Linfoma MALToma

117. ¿Cuál de los siguientes hallazgos se encontraría en un paciente son


síndrome de Bernard-Soulier?
b} Tiempo de sangrado prolongado
118. Paciente masculino de 65 años de edad en posquirúrgico mediato de
colecistectomía electiva. Posterior a la administración de un medicamento, el
paciente presenta eritema facial, diaforesis y náusea con pupilas mióticas pero con
adecuada respuesta a la estimulación luminosa; su presión arterial es de 100/75
mm Hg y la frecuencia cardiaca de 50 latidos/minuto. ¿Cuál de las siguientes
opciones representa con mayor probabilidad la indicación del medicamento que le
administró la enfermera a este paciente?
e} Atonía vesical

119. Se trata de Mónica, femenino de 31 años de edad quien cursa embarzao de


26 semanas de gestación,sin antecedentes patológicos personales. Acude a
consulta por cefalea punzante, disnea, y astenia que iniciaron hace 3 días. Dentro
de los antecedentes gineco obstétricos refiere 2 abortos espontáneos, 1 óbito. En
el examen físico se encuentra pálida, con presencia de edema en miembros
inferiores y tromboflebitis superficial. A nivel cardiovascular taquicardia leve, a la
exploración pulmonar con estertores finos subescapulares bilaterales, de
predominio derecho; un examen general de orina revela la presencia de
proteínuria +++. FC 105 lpm, FR 20 rpm, temp 37.5 ºC, PA 180/110mm Hg, peso
85 kg, talla 1.60 m.
119a. Con base en lo expuesto en el caso clínico, ¿cuáles datos son los más
relevantes para el padecimiento actual?
b} Abortos espontáneos, óbito, proteinuria

119b. En sospecha de colágenopatía es necesario descartar la presencia de


síndrome de anticuerpos antifosfolípidos. ¿Qué estudios solicita para confirmar su
sospecha diagnóstica?
d} Anticuerpos anticardiolipina- anticoagulantelúpico.

119c. Se confirma el diagnóstico de SAF ¿qué alteración hematológica espera


encontrar en este caso?
c} Trombocitopenia

119d. Durante la estancia hospitalaria del paciente se realiza biopsia de piel con
reporte de oclusión de pequeños vasos y anticoagulante lúpico. ¿Cuál es su
sospecha con estos resultados?
b} SAF catastrófico

119e. Una vez estabilizada a la paciente, ¿qué medidas de tromboprofilaxis le


recomendaría?
b} Dosis bajas de aspirina e hidroxicloroquina

120. Paciente masculino de 58 años acude a consulta debido a que presenta


molestias en la vista; tiene miopía desde los 15 años y comenta que en las últimas
semanas le cuesta trabajo ver los objetos que están a sus lados e incluso no ve
cuando la gente se le acerca. Entre sus antecedentes, menciona que padece DM2
desde hace 5 años y que hace 10 años le hicieron una paquimetría y le dijeron
que tenía la córnea muy delgada, por lo cual no se operó. EF: 1.70m; 85kg; TA:
145/85mm Hg; Temp: 36.5ºC y PIO: 23mm Hg; se observa en el fondo del ojo
hemorragia en astilla del disco óptico, sin datos de microaneurismas ni exudados."
120a. ¿Cuál es la principal sospecha diagnóstica?
c} Glaucoma de ángulo abierto

120b. NO son factores de riesgo para su sospecha diagnóstica:


a} Tabaquismo, raza asiática, hipermetropía

120c. Las fases de la historia natural de estetrastorno incluyen a todas las


siguientes, excepto:
b}Islote nasal

120d. El tratamiento médico correcto es:


b} Bloqueadores beta e inhibidores dela anhidrasa carbónica

121. Con relación a la nefropatía por gota:


d} Con el tratamiento adecuado, se mantiene o mejora la función renal en 80%
delos casos

122. Un hombre de 80 años de edad es admitido en un hospital para someterse a


cirugía electiva de reemplazo de rodilla. En su historia médica se incluye
osteoartritis, hipertensión y enfermedad coronaria, habiendo sufrido un infarto de
miocardio con colocación de stent hace siete años. Sus medicamentos incluyen
hidroclorotiazida, metoprolol, aspirina (la cual suspendió una semana previa a la
cirugía) y ocasionalmente paracetamol. Niega uso de alcohol, aunque refiere
tabaquismo de 30 paquetes/año. La cirugía se realiza con éxito, sin embargo,
llaman en la noche porque el paciente se encuentra diaforético. Signos vitales
incluyen FC 102/min FR 25/min,T 36.0 C, PA 142/86 mm Hg. Saturación de
oxígeno es de 90%. Se encuentra consciente, aunque al interrogatorio no está
orientado en tiempo o espacio, se tarda en contestar y no pone completa atención.
Según lo observado en el expediente médico el paciente actualmente vive con su
esposa, con la que lleva casado 43 años. Es capaz de realizar todas las
actividades básicas de vida diaria, y antes de la cirugía, según la hija que se
encuentra presente, no tenía problemas de orientación o memoria.
122a. ¿Cuál es el diagnóstico más probable?
c} Delirio

122b. ¿Cuál de estos estudios de laboratorio es innnecesario para este paciente?


d} Amonio
122c. Los estudios de laboratorio que se realizaron para el paciente resultaron sin
anormalidades. Al interrogatorio con la esposa presente se refiere que el paciente
ha tenido cierto grado de deterioro en los últimos meses. La esposa ha tenido que
ayudarle a controlar sus medicinas, pues el paciente olvida tomarlas o se
confunde de horario. Además, ha tenido dificultad para reconocer a sus amistades
y en dos ocasiones ha requerido la ayuda de un vecino para regresar a casa de su
caminata diaria. La esposa refiere, sin embargo, que aún recuerda sin problema
eventos que ocurrieron mucho tiempo atrás. El paciente se mantiene interesado
en sus actividades diarias y disfruta pasar el tiempo con su familia. Niega
comportamiento extraño, desinhibido, hipersexualidad, pérdida de peso o
temblores de las extremidades. Se ordena una TAC de cráneo, que revela atrofia
cortical difusa. Se sospecha que el paciente tiene algún tipo de demencia
subyacente. ¿Cuál sería el tipo más probable en este paciente?
e} Enfermedad de Alzheimer

122d. El paciente se recupera del episodio de delirio y se le realiza un Mini-Mental,


con resultado de 23/30, falla principalmente en el área de memoria y de
orientación. ¿Qué tratamiento es recomendable para retrasar la progresión de la
enfermedad?
a} Donepezil

123. Paciente masculino de 70 años con antecedentes de hipertensión arterial


acude a urgencias por presentar inicio súbito de dolor intenso en la región
interescapular, desgarrador que se extiende al epigastrio. A la exploración física el
paciente se encuentra taquicárdico y diaforético con presión arterial de 170/102
mm Hg. La radiografía PA de tórax muestra ensanchamiento mediastinal y la
tomografía axial computarizada confirma una disección de la aorta descendente
¿Cuál es el tratamiento inicial más adecuado para este paciente?
b} Nitroprusiato de sodio

124. Se trata de una mujer de 45 años, diabética (tipo 1) insulinodependiente de


12 años de evolución con control metabólico irregular,se presenta a la consulta
quejándose de halos visuales posterior a un descontrol metabólico de 330 mg/dl.
En el examen oftalmológico reporta agudeza visual de ojo derecho (OD) 20/40 que
no mejora con refracción y en ojo izquierdo (01) 20/20, reflejos pupilares normales,
presión intraocular normal en ambos ojos, prueba de ishihara normal en ambos
ojos. En el fondo de ojo derecho se observa papila hiperémica y elevada,con
dilatación de la microvasculatura, no había datos de retinopatía diabética;en ojo
izquierdo se observa papila definida, excavación 20, sin datos de retinopatía
diabética. Se realiza campo visual de Goldman que reporta sólo un aumento de la
mancha ciega de ojo derecho, el ojo izquierdo está sin alteraciones. Ella ya traía
un estudio de TAC que no reportaba alteraciones neurológicas. Se realiza estudio
de FAG en el que se observa mínima fuga a nivel del disco óptico de ojo derecho,
sin datos de fuga por neovascularización. De los siguientes enunciados todos son
falsos, EXCEPTO:
d} La mayoría de los casos no necesita tratamiento específico
1. Paciente femenino de 67 años que acude a urgencias por náusea y vómito.
Refiere que el padecimiento inició hace tres días, y ocurre de manera intermitente.
Niega fiebre o hematemesis; también diarrea, refiriendo que no ha tenido
evacuaciones en cinco días. En sus antecedentes patológicos tiene historia de
colelitiasis diagnosticada hace dos meses, pero negó tratamiento quirúrgico. A la
exploración física sus signos vitales incluyen FC 104/min, FR 19/min, T 38.3 C y
PA 113/70 mm Hg. No hay anormalidades a la exploración torácica. En la
búsqueda abdominal se observa un abdomen distendido, timpánico, con dolor
difuso a la palpación y peristalsis aumentada. ¿Cuál es el estudio inicial más
apropiado para esta paciente?
d} Radiografía simple de abdomen

2. Paciente masculino de 67 años que acude a consulta por fiebre y disnea.


Refiere que inició hace dos días, con fiebre y tos, pero actualmente tiene dificultad
para respirar. También cuenta que expectora flema amarillenta. Cuenta con
tabaquismo de 24 paquetes/año e ingerir tres cervezas diarias. Niega
antecedentes médicos de importancia. El único medicamento que toma es
aspirina. A la exploración física sus signos vitales son FC 101/min, FR 21/min, T
38.6 C, PA 133/80 mm Hg. A la exploración física se auscultan estertores
crepitantes. En una placa de tórax se muestra una consolidación en el lóbulo
medio del pulmón derecho. ¿Cuál es la etiología más común de este padecimiento
en pacientes de este grupo de edad?
d} Streptococcus pneumoniae

3. Paciente femenino de 73 años que acude a urgencias por cefalea. Inició hace
unas horas, pero ha ido aumentando en intensidad y no mejoró con dos tabletas
de paracetamol de 500 mg. El dolor es unilateral, y cuando señala donde coloca
su mano frente a su ojo del lado derecho. También refiere algo de visión borrosa y
náusea. Niega que el dolor hubiera ocurrido antes de esa manera. Al IPAS niega
vómito, fotofobia y sonofobia. Al examen de agudeza visual no puede identificar
números ni letras, sólo movimientos de la mano. Se observa inyección conjuntiva!
ocular y una pupila dilatada, no reactiva a la luz. Sus signos vitales incluyen FC
103/min, FR 18/min, T 37.1 C, PA 134/79 mm Hg. El resto de los pares craneales,
incluyendo los movimientos oculares, está intacto. No hay anormalidades en el
resto de la exploración neurológica. ¿Cuál sería el siguiente paso en el manejo de
este paciente?
d} Medición de la presión intraocular

4. ¿Cuál es la vasculitis a la que el siguiente enunciado hace referencia?:


conocida también como angeítis y granulomatosis alérgica, caracterizada por la
presencia de asma, eosinofilia tisular y periférica, formación de granulomas
extravasculares y vasculitis de varios órganos y sistemas.
a} Síndrome de Churg-Strauss
5. Paciente femenino de 7 meses de edad es traída a consulta. La madre refiere
que ha presentado cuadros frecuentes de vómito, movimientos no coordinados y
crisis convulsivas; menciona que el cabello se le ha vuelto más claro en
comparación al resto de sus hermanos. En la exploración física se identifica piel
seborreica, microcefalia, maxilar prominente, dientes espaciados y retraso en el
crecimiento.
5a. De acuerdo con las características clínicas, ¿cuál es el trastorno más probable
de esta paciente?
d} Fenilcetonuria

5b. ¿Con cuál de los siguientes exámenes de laboratorio confirmaría el


diagnóstico?
a} Niveles de fenilalanina en plasma

5c. Con base en su sospecha clínica, ¿qué tratamiento indicaría?


c} Dieta libre de fenilalanina y cofactor tetrahidrobiopterina

5d. ¿Qué tipo de transmisión tiene este padecimiento?


b} Rasgo autosómico recesivo

6. Paciente masculino de 35 años de edad que regresa de viaje de luna de miel en


África. A su regreso el paciente presenta cefalea y fiebre por lo que acude a
servicio médico. El paciente es diagnosticado con infección por Plasmodíum
falcíparum. En este momento el paciente presenta un hematocrito de 18%,
hiperbilirrubinemia de 9 mg/dl y creatinina sérica de 2.7 mg/dl. El paciente es
ingresado con monitorización de la función renaly administración de fenobarbital
como profilaxis de crisis convulsivas. ¿Cuál de los siguientes antibióticos es el
más adecuado para este paciente?
e} Artesunato

7. En varias ocasiones la hipotermia grave no es diagnosticada durante la


exploración física inicial. La temperatura se puede reportar como el límite inferior
del termómetro, lo cual puede ser de tan sólo unos cuantos grados centígrados
por debajo de lo normal. ¿Cuál de los siguientes es suficientemente adecuado
para indicar el diagnóstico de hipotermia?
d} Electrocardiograma

8. ¿Cuál es el principal efecto secundario de la administración de abacavir en


pacientes con infección del virus de la inmunodeficiencia humana?
b} Reacción de hipersensibilidad severa

9. ¿Cuál de los siguientes hallazgos se puede observar en un paciente con lesión


del nervio safeno?
b} Alteración de la sensibilidad enla porción medial de la pierna
1O. Se presenta a consulta paciente femenino de 14 años por amenorrea primaria
y ausencia de desarrollo glandular mamario. La madre refiere que el único
antecedente de importancia es disminución de la audición que requirió uso de un
dispositivo especial. A pesar de que la paciente fue producto de un embarazo
normal y parto eutócico, la madre recuerda que los médicos le dijeron que su hija
estaba un poco "hinchada" al nacer. A la exploración física la paciente muestra
signos vitales normales, mide 1.50 m y pesa 60 kg. A la inspección, se encuentra
implantación baja de orejas y cuello alado. La exploración cardiovascular revela un
soplo sistólico grado 11/IV,con pezones extremadamente separados y glándulas
mamarias Tanner l. La exploración pélvica revela ausencia de vello púbico y
genitales externos en fase prepuberal. Se puede observar mediante un espéculo
la presencia de cuello uterino (cervix). ¿Cuáles otros hallazgos son probables de
identificar en esta paciente?
a} Anomalías renales, linfedema de extremidades, anomalías cardiovasculares
(en especial coartación de aorta}

11. ¿Cuál de los siguientes medicamentos se utiliza en el tratamiento de la


esclerosis múltiple?
b} lnterferón beta

12. Paciente masculino de seis años de edad es llevado a consulta por sus padres
debido a un cuadro que inicia como un aparente resfriado con rinorrea, pero en
este momento el paciente presenta disnea intensa. Los padres informan que el
paciente muestra notable mejoría durante la mañana pero los síntomas empeoran
conforme se acerca la noche. A la exploración física se pueden auscultar
estertores y sibilancias pero es imposible localizar los ruidos cardiacos. La
radiografía de tórax revela hiperinsuflación pulmonar, bronquiectasias e inversión
izquierda/derecha de varias estructuras del sistema respiratorio. ¿Cuál de las
siguientes complicaciones puede presentar este paciente?
e} Infertilidad

13. Durante la etapa fetal, ¿en cuál de las siguientes estructuras el porcentaje de
saturación de hemoglobina fetal es menor?
a} Ductus arteriosus

14. Paciente femenino de 28 años con diagnóstico de síndrome de Wolff-


Parkinson-White acude a urgencias por presentar palpitaciones de inicio súbito
hace 45 minutos. A la exploración física se encuentra angustiada y destaca ritmo
irregular. El electrocardiograma muestra fibrilación auricular con respuesta
ventricular rápida con una frecuencia de 170 latidos por minuto. ¿Cuál es el
tratamiento más adecuado para la paciente?
c} Procainamida
15. Paciente del género masculino de 35 años de edad con antecedente de
infección por el virus de la inmunodeficiencia humana (diagnosticado desde hace 3
años). El paciente no desea la administración de antirretrovirales, pues cree que
su enfermedad es un ""invento del gobierno y de las compañías farmacéuticas"".
Actualmente, en una revisión de rutina, el paciente refiere una ""erupción"" que
apareció hace 24 horas en la región lumbar y en la región posterior de la pierna
derecha. El paciente refiere las lesiones como dolorosas (tipo ""ardor""). Entre la
información obtenida en el interrogatorio, el paciente mencionó que fue ascendido
en su trabajo recientemente, pero que esto lo ha estresado demasiado pues ahora
tiene mayores responsabilidades. A la inspección de la lesión se observa un
exantema formado por vesículas sobre una base eritematosa (""gotas de rocío
sobre pétalos de rosa roja""). Utilizando únicamente la información previamente
descrita, responda las siguientes preguntas:
15a. ¿Cuál de las siguientes opciones representa la etiología más probable del
exantema en este paciente?
d} Virus herpes humano 3

15b. ¿Cuál de las siguientes opciones representa la conducta inmediata más


adecuada para este paciente?
a} Administración de aciclovir oral por 7-10 días

15c. ¿Cuál es la categoría clínica de infección por el virus de la inmunodeficiencia


humana en que se encuentra este paciente?
b} Categoría B

15d. Después de solicitar estudios de laboratorio, resulta que el paciente presenta


una carga viral de 15,000 copias/mL y un conteo de células T CD4+ de 349/mcL.
¿Cuál de las siguientes opciones representa el mejor tratamiento a seguir?
a}Iniciar tratamiento antirretroviral,pues el conteo de linfocitos T CD4+lo indica

15e. ¿Cuál de las siguientes opciones representa un desenlace esperado del


tratamiento antirretroviral para considerar éste como exitoso?
a} Reducción de porlo menos 10 veces (1logaritmo} en los niveles de RNA del
virus dentro de 1-2 meses siguientes del inicio del tratamiento, y eventualmente
una reducción a < 50 copias/ml

16. Paciente masculino de 1O años de edad el cual es llevado a la sala de


urgencias por la presencia de adenopatía axilar dolorosa. A la exploración física se
encuentra una serie de rasguños en el antebrazo. Se decide realizar una
aspiración del nódulo linfático, el cual produce un material purulento que es
enviado a evaluación en el laboratorio. El reporte revela la presencia de un bacilo
altamente pleomórfico. ¿Cuál
d} Bartonella henselae
17. Paciente femenino de 55 años de edad que acude a consulta por ictericia. Al
interrogatorio dirigido la paciente no revela información de utilidad que oriente al
diagnóstico. A la exploración física presenta signo de Murphy negativo, pero con
vesícula biliar palpable. ¿Cuál es el diagnóstico más probable en esta paciente?
c} Cáncer de páncreas

18. Se presenta a consulta geriátrica paciente de 80 años de edad, quien es


llevado por su esposa debido a que lo ha notado más "olvidadizo" que lo usual. Al
interrogar al paciente acerca de su memoria, éste refiere que fue obligado por su
esposa para acudir a consulta, ya que él mismo no hubiera acudido. La esposa ha
mencionado que el problema de memoria de su esposo ha empeorado desde los
últimos meses. Los antecedentes sólo son de importancia por hipertensión de
cinco años de diagnóstico. A la exploración neurológica el paciente muestra
deficiencias en la memoria de corto plazo, pero no hay problemas en cuanto al
funcionamiento sensitivo y motor,sin ataxia. Se decide obtener una tomografía
computarizada de la cabeza la cual muestra atrofia cerebral. Es claro que el
paciente presenta un síndrome demencial. ¿Cuál de las siguientes causas
reversibles de demencia es probable que presente el paciente?
e} Hipotiroidismo

19. Paciente del sexo femenino de 20 años de edad que consulta por amenorrea
primaria. Durante el interrogatorio la paciente menciona que ocasionalmente
desde la niñez ha presentado cuadros ocasionales de náusea,vómito y debilidad
generalizada. La exploración física revela hipertensión arterial (180/95 mm Hg). Se
solicitan estudios de laboratorio los cuales muestran hipopotasemia inversa. ¿Cuál
de las siguientes deficiencias enzimáticas con mayor probabilidad presenta esta
paciente?
b} Deficiencia de 17 alfa hidroxilasa

20. Paciente masculino de 50 años de edad que mantiene un matrimonio desde


los últimos 1O años, pero recientemente ha expresado que su esposa ha sido
infiel desde el primer día de matrimonio a pesar de que ella lo ha negado
rotundamente. Su problema de desconfianza se ha extendido al resto de sus
relaciones y menciona que la mayoría de las personas están listas para
traicionarlo. Cuando acude a consulta psiquiátrica el paciente se presenta
defensivo a cada comentario del médico. ¿Cuál es el trastorno de la personalidad
que más probablemente
presenta este paciente?
c} Paranoide

21. ¿Cuál de los siguientes anticonvulsivantes produce sedación e inducción del


citocromo P450?
c} Fenobarbital

22. ¿Cuál de los siguientes fármacos puede producir neuropatía periférica?


c) Paclitaxel

23. Acude a consulta de urgencias un paciente del sexo femenino de 38 años de


edad sin antecedentes de importancia. La paciente refiere que tuvo una riña con
uno de sus hermanos; posteriormente cuando trató de salir de inmediato de la
casa, ella fue incapaz de abrir la puerta por un "ataque" de debilidad en el brazo
derecho. A la exploración física la paciente presenta una fuerza muscular en el
brazo derecho 0/5 sin alteraciones de los reflejos de estiramiento en la misma
extremidad. ¿Cuál es el diagnóstico más probable en esta paciente?
a} Trastorno conversivo

24. ¿Cuál de las siguientes neoplasias del sistema nervioso central en pacientes
pediátricos se asocia con el síndrome de van Hippel-Lindau?
b} Hemangioblastoma

25. ¿Cuál de los siguientes marcadores tumorales se eleva en el carcinoma


colorrectal?
b} Antígeno carcinoembrionario

26. Se presenta una paciente de 22 años de edad con su primer embarazo. La


paciente se muestra renuente a abstenerse del consumo de alcohol durante el
resto de su embarazo. En el caso de una mujer embarazada que bebe entre 60 y
120 mlde alcohol a 80%, ¿cuál es el riesgo de presentar un neonato con síndrome
alcohólico fetal?
b} 10 %

27. ¿Cuál de las siguientes translocaciones cromosómicas se asocia con el


linfoma folicular?
d} t(14:18}

28. Paciente masculino de 30 años de edad acude por poliuria y polidipsia. El


paciente menciona que recientemente ha notado un incremento en el tono de la
pigmentación de su piel a pesar de que no se ha expuesto más tiempo del normal
a la radiación solar. Expresa que por las mañanas presenta edema de las
extremidades inferiores y necesita dormir con dos o tres almohadas para evitar la
sensación de disnea nocturna. ¿Cuál es el diagnóstico más probable en este
paciente?
d} Hemocromatosis

29. Paciente masculino de 27 años de edad acude a consulta por dolor de 72


horas de evolución en el muslo izquierdo. Menciona que el dolor se ha
acompañado de un crecimiento en la región, el cual rápidamente ha aumentado de
tamaño. Al revisar el expediente clínico, se revela que el paciente ha acudido en
tres ocasiones en los últimos cuatro meses por lesiones similares, las cuales
describe como ""picaduras de insectos"" en los glúteos y abdomen. A la
exploración física el paciente se encuentra afebril y normotenso. En la zona
dolorosa el paciente tiene una masa fluctuante de 3 cm rodeada de un anillo
eritematoso de 4 cm. ¿Cuál de las siguientes opciones es la más adecuada para
este paciente?
b} Realizar incisión y drenaje, obtener cultivos y prescribir clindamicina oral

30. Paciente de 22 años de edad sin antecedentes heredofamiliares de


importancia. El paciente se presenta con cuadro clínico caracterizado por
lateropulsión, vértigo, cefalea, hipoacusia bilateral y catarata. El estudio
audiométrico reportó hipoacusia superficial grave bilateral. Los potenciales
evocados auditivos mostraron disrupción de la vía auditiva a nivel del tallo
cerebral. En la resonancia magnética de cráneo contrastada con gadolinio, se
observaron lesiones tumorales extraaxiales hiperintensas bilaterales a nivel del
ángulo pontocerebeloso, que afectan la emergencia de los complejos VINlll, del
lado izquierdo más prominente, y condicionan compresión del tallo cerebral,en
especial delpuente. ¿Cuál de los siguientes hallazgos es más probable que se
encuentre en este paciente?
a} Nódulos de Lisch

31. ¿Cuál de las siguientes alteraciones electrolíticas se asocia con la aparición de


la onda de Osborn en el electrocardiograma?
b} Hipercalcemia

32. Acude a urgencias, paciente femenino de 23 años de edad con diagnóstico de


embarazo de 37 semanas de gestación. Presenta dermatosis generalizada, que
respeta palmas y plantas, caracterizada por vesículas sobre fondo eritematoso,
pústulas y costras; así como prurito. Producto sin alteraciones aparentes. ¿Qué
acción tomaría?
d} Gammaglobulina específica

33. Mencione cuál de los siguientes factores está más asociado con el riesgo de
padecer artritis reumatoide:
e} Multifactorial

34. Acude a consulta paciente femenino de 20 años de edad por amenorrea.


Refiere que sí ha presentado menstruaciones en el pasado. No se observan
alteraciones en la talla,fenotipo ni cardiacas, hay vagina corta. Se realiza un
cariotipo encontrando 46XX, los ovarios se encuentran normales. ¿Cuál de las
siguientes es una causa de amenorrea secundaria?
c} Síndrome de Asherman

35. ¿Cuáles son las entidades con las que se debe realizar diagnóstico diferencial
del paciente con artritis reumatoide?
e} Todas las opciones

36. La investigación epidemiológica reconoce situaciones de alerta que demandan


investigación de campo, identifica principios, métodos y procedimientos básicos en
el estudio de brotes y establece bases prácticas para la investigación de campo en
los niveles de salud local. Además en la promoción de la salud se suma la
expansión de la vigilancia, prevención y control de problemas de salud que
incluyen no sólo enfermedades transmisibles, sino estilos de vida, factores de
riesgo y desórdenes genéticos, eventos de salud ocupacional, riesgos
ambientales, discapacidad y enfermedades crónicas, entre otros.
36a. Si en tu comunidad diagnosticas un cuadro de rubéola en menor de 2 años,
estás ante una alerta epidemiológica y debes actuar de manera rápida y eficiente
para interrumpir la infección. Para esta situación la Salud Pública cuenta con dos
métodos de investigación:
a} Epidemiología descriptiva y analítica

36b. La investigación epidemiológica de campo aplica métodos de las ciencias


básicas, estadísticas y sociales que incluye la investigación de brotes como una
tifoidea y suele aplicar un diseño descriptivo. Algunas de sus características son:
b} Ágil,riguroso, técnicamente sencillo, ofrece respuestas urgentes ante
epidemia local

36c. Al aumento inusual en el número de casos relacionados


epidemiológicamente, de aparición súbita y diseminación localizada en un espacio
específico, como pudiera ser una intoxicación alimentaria se le llama:
d} Brote

36d. ¿Cuál es la utilidad de tener el canal endémico actualizado como el de los


casos de influenza en adultos mayores?
a} Para emitir una alerta epidemiológica

36e. En Salud Pública, al indicador que mide la severidad de una enfermedad


como sería para la diabetes tipo 2 se le conoce como:
d} Tasa de Letalidad

36f. Para confirmar un brote se requiere verificar el diagnóstico a través de la


historia clínica y exámenes de laboratorio de los casos notificados. El siguiente
paso es el trabajo de campo y el tercero es la definición operacional de caso que
reúne diferentes criterios. Menciona los criterios epidemiológicos.
c} Distribución,tiempo, espacio y persona, periodo de incubación y de
exposición al probable caso índice, casos secundarios y fuente común,tiempo de
exposición y área geográfica

36g. El diseño epidemiológico de tipo analítico más empleado durante la


investigación de una alerta epidemiológica, por ejemplo de un caso de hepatitis A
en niños, es el estudio de caso-control que investiga a dos grupos de la
comunidad: con la enfermedad parte del brote (casos) y sin la enfermedad
(controles). El análisis que explora la significancia entre exposición y enfermedad
se determina con:
a} Chi-cuadrado

37. Paciente femenino de 27 años de edad, que presenta a nivel genital cuatro
úlceras blandas y dolorosas, con inflamación perilesional. Ganglios en ingle
derecha dolorosos y ulcerados que según la paciente aparecieron cinco días
después de la primera úlcera. Antes de la aparición de la primera úlcera tuvo
contacto sexual con una nueva pareja.
37a. ¿Cuál es el agente etiológico de esta patología?
b} Haemophi/us ducreyi

37b. ¿Cuál es el tratamiento indicado para este microorganismo?


c} Ceftriaxona

38. Paciente femenino de 11 años de edad acude por presencia de masa


eritematosa dolorosa en región nasal del párpado inferior del ojo derecho desde
hace tres días que ha ido en aumento. La madre refiere que había presentado
cuadros previos semejantes y que había resuelto con tratamiento no especificado.
Desde lactante presentaba descarga de secreción mucopurulenta a través del
punto lagrimal del mismo ojo. A la exploración se observa eritema cutáneo,
edema,se palpa indurada por debajo del tendón cantal medio y con presión leve
sobre el saco lagrimal se obtiene material mucopurulento amarillo-verdoso. Se
observa conjuntiva sin hiperemia, córnea transparente, sin alteraciones de la
motilidad ocular. ¿Cuál es el diagnóstico más probable?
c} Dacriocistitis

39. Mujer de 70 años de edad padece diabetes mellitus tipo 2 complicada con
retinopatía proliferativa, neuropatía sensitiva distal e insuficiencia renal crónica,
acude a consulta de revisión. A la exploración física muestra presión arterial
160/95mm Hg, frecuencia respiratoria 24 resp/min, ictericia generalizada y
debilidad muscular; datos de laboratorio con anemia normocítica normocrómica
(Hb 1O g/dl, VCM 90 fl,HCM 30 pg), creatinina 4 mg/dl, BUN 60 mg/dl,
hiperpotasemia, hiperfosfatemia, hipocalcemia (7 mg/dl) y acidosis metabólica de
brecha aniónica elevada (pH 7.32, HC03 18 mEq/L, PaC02 33 mm Hg). Su
depuración de creatinina por Cockcroft-Gault es de 40 mUmin.
39c. La anemia normocítica normocrómica en esta paciente es secundaria a:
c} Déficit de eritropoyetina

40. Paciente masculino de 62 años de edad con diagnóstico de diabetes mellitus


tipo 2 de 13 años de evolución, controlada con hipoglucemiantes orales; última
glucemia central fue de hace un mes en 143 mg/dL. Acude a consulta de
urgencias por presentar dolor ocular intenso en ojo derecho, disminución de la
agudeza visual de 4 horas de evolución, además de náusea y vómito. A la
exploración oftalmológica presenta una agudeza visual de movimiento de manos
en ojo derecho y 20/80 en ojo izquierdo. A la biomicroscopia se encuentra en ojo
derecho hiperemia conjuntiva! e inyección ciliar,córnea edematosa con presencia
de microbulas subepiteliales, reacción inflamatoria en cámara anterior, la cual se
observa poco profunda, pupila midriática, hiporrefléctica, cristalino con catarata.
Presión intraocular de 58 mm Hg. La gonioscopia muestra un ángulo
completamente cerrado, el fondo de ojo no es valorable por la opacidad de
medios. Ojo izquierdo córnea transparente, cámara anterior poco profunda, pupila
fotorreactiva, iris sin alteraciones, cristalino transparente y a la gonioscopia
ángulos moderadamente abiertos. ¿Cuál es el diagnóstico más probable?
c} Glaucoma agudo por bloqueo pupilar

41. Paciente masculino de 40 años, sin antecedentes de importancia, acude


después de trauma contuso con pelota de squash en ojo derecho de 3 horas de
evolución. A la exploración oftalmológica se encuentra agudeza visual de 20/400
observando hiperemia conjuntiva!, inyección ciliar, córnea transparente, cámara
anterior formada con celularidad leve y un nivel hemático de 15% de la cámara
anterior,pupila normorrefléctica, presión intraocular dentro de límites normales,
cristalino transparente, retina aplicada y vítreo sin opacidades. ¿Cuál sería el
tratamiento inicial para el hifema traumático en este paciente?
a} Esteroides y ciclopléjicos tópicos

42. Paciente femenino de 55 años de edad con una úlcera en el tercer dedo de la
mano izquierda. Según refiere, la úlcera se presentó hace siete días y no es
dolorosa. Decidió automedicarse utilizando un antibacteriano tópico e
hidrocortisona en crema. Al interrogatorio no logra recordar algún traumatismo que
explique esta lesión. Latemperatura de la paciente es de 39° C con adenomegalia
epitroclear y axilar izquierda. La úlcera es de alrededor de 3 cm en la cara dorsal
del tercer dedo de la mano izquierda y se encuentra cubierta por una escara negra
y rodeada por edema. ¿Cuál de los siguientes diagnósticos es el más probable en
esta paciente?
b} Ántrax cutáneo

43. Paciente femenino de siete años de edad acude al servicio de urgencias por
presentar fatiga y palidez. La paciente había sido tratada hace poco por infección
de vías aéreas superiores. En esta ocasión, la exploración física revela
esplenomegalia y equimosis en las extremidades superiores. Los estudios de
laboratorio demuestran un conteo de leucocitos de 11 000 células/mm3,
hemoglobina de 6 g/dL, y conteo plaquetario de 40 000 plaquetas/mm3. La prueba
de Coombs es positiva. ¿Cuál es el diagnóstico más probable en esta paciente?
c} Síndrome de Evans

44. ¿Cuál de los siguientes calendarios de administración de la vacuna DPT es el


más correcto según el esquema de vacunación de México?
c} A los dos, cuatro, seis y 18 meses de edad y a los cuatro años de edad

45. ¿Cuál de las siguientes opciones representa al agente infeccioso más


frecuente en pacientes de cuatro meses de edad con secreciones nasales
copiosas, sibilancias y tos, en especial en la época de invierno?
d} Virus sincitial respiratorio

46. ¿Cuál de las siguientes enfermedades se caracteriza por hipertensión


pulmonar?
a} Esclerodermia
47. Paciente femenino de 75 años de edad que se encuentra en la actualidad en
tratamiento por infección de vías urinarias con trimetoprimsulfametoxazol.
Después de una semana de haber completado el tratamiento, acude al servicio de
urgencias por la aparición de vesículas en el abdomen, las cuales evolucionan a
bulas flácidas y erosiones de la piel del tórax, abdomen,espalda y muslos. En la
mucosa oral también tiene erosiones. La biopsia demuestra separación
intraepidérmica. ¿Cuál es el diagnóstico más probable en esta paciente?
b} Pénfigo vulgar

48. ¿Cuál de los siguientes medicamentos se relaciona con trombocitopenia y


microtrombosis?
a} Heparina

49. ¿Cuál de las siguientes descripciones corresponde con las hemorragias en


astilla?
d} Lesiones lineares o enforma de llama

50. ¿Cuál es la causa más probable del dolor de rodillas en corredores?


a} Síndrome patelofemoral

51. Una mujer de 40 años de edad acude para extirpación de ""verrugas"" en el


cuello porque le duelen cuando se pone una cadena. Tiene una dermatosis
localizada, bilateral,simétrica que afecta cuello y axilas. Se caracteriza por
múltiples neoformaciones exofíticas, pediculadas, bien delimitadas, lisas, del color
de la piel. Le molestan con cuellos altos de la ropa y cadenas y refiere aumento
después de un embarazo. Ella tiene sobrepeso y resistencia a la insulina.
51a. ¿Cuál es el diagnóstico más probable?
d} Fibromas blandos

51b. ¿A cuál enfermedad sistémica se relacionan los fibromas blandos?


e} Diabetes mellitus o síndrome metabólico

51c. ¿Cuál es la causa aceptada de los fibromas blandos?


c} Sobrepeso

51d. ¿Cuál es el tratamiento de elección para los fibromas blandos?


d} Rasurado

51e. ¿Cuál es el tratamiento preventivo de los fibromas blandos?


e} Ninguno
52. Paciente masculino de 25 años de edad, con antecedentes familiares por línea
materna de hemorragias. Acude a consultar por hematomas que aparecieron en
ambos brazos tras golpes mínimos desde la infancia. Ha recibido anteriormente
tratamiento con desmopresina por hemofilia tipo A sin resultados.
52a. Señale el tratamiento indicado en este paciente:
a} Factor VIII recombinante
52b. Tipo de herencia de la enfermedad con la que cursa el paciente.
c} Ligada al cromosoma X

52c. ¿Cuál de las siguientes manifestaciones hemorrágicas no forma parte de la


clínica en los pacientes con hemofilia?:
b} Petequias

52d. ¿Cuál de las siguientes alteraciones en el laboratorio se encuentran en los


pacientes con hemofilia?
b} Tiempo tromboplastina parcial alargado-tiempo de protrombina normal

52e. Señale el enunciado correcto respecto a la terapia anticoagulante:


d} El control dela medicación anticoagulante oral se realiza con elTP

53. La Comisión Nacional de los Derechos Humanos, específicamente en el


Programa de Asuntos de la Mujer y de igualdad entre mujeres y hombres, publica
los delitos de hostigamiento sexual, abuso sexual,estupro y violación, de los
cuales es preciso saber:
53a. El abuso sexual por medio de la violencia física o moral y si la víctima fuera la
esposa o concubina, se impondrá una pena de:
c} 8 a 20 años de prisión

53b. Se equipara a la violación, al que sin consentimiento de una persona o con el


consentimiento de un o una menor de 14 años introduzca uno o más dedos o un
objeto de cualquier naturaleza en la región anal o vaginal,a esto se le llama:
b} Violación impropia

54. Ante un niño de cinco años que presentó síndrome anticolinérgico por
intoxicación medicamentosa, ¿cuál de las siguientes NO forma parte del síndrome
de toxicidad de los antihistamínicos?
e} Diarrea

55. Paciente masculino de 65 años de edad, con antecedente de tabaquismo,


refiere padecer dificultad para respirar al caminar por las mañanas desde hace 3
años, por lo que mantiene una vida sedentaria. Acude a consulta por tos
intermitente desde hace 3 meses. Niega dolor torácico. A la auscultación se
detecta disminución del murmullo vesicular. La radiografía de tórax reporta
aplanamiento diafragmático, disminución del trama vascular y silueta cardiaca
alargada.
55a. ¿Cuál es el punto de corte en la espirometría para el diagnóstico de EPOC?
b} FEV1/FVC < 70%

55b. Esquema farmacológico indicado en este paciente.


a} Tiotropio + salbutamol/ipratropio

55c. ¿Cuáles son las medidas que han demostrado disminuir la mortalidad en la
EPOC?
b} Abandono deltabaco-oxigenoterapia

55d. Señale los criterios de Winnipeg que definen exacerbación de la EPOC:


d} Aumento de disnea + aumento de expectoración + purulencia delesputo

55e. ¿Cuál es el tratamiento indicado ante una exacerbación de la EPOC de


causa infecciosa?
c} Amoxicilina + ácido clavulánico

56. Acude a Urgencias un paciente masculino de 35 años, sin antecedentes de


importancia,con fiebre de 3 días de evolución, cefalea,fotosensibilidad y
somnolencia. A la exploración física se nota rigidez de cuello, así como signo de
Kernig y Brudzinski positivos. Los signos vitales son temperatura de 39 ºC, FC: 98,
FR: 25, TA: 110/70 mm Hg. Se toma la decisión de internarlo para estudiarlo.
56a. ¿Cuál es el agente etiológico más frecuente de meningitis en pacientes de
esta edad?
a} S. pneumoniae

56b. ¿Cuál de las siguientes condiciones NO es indicación para realizar una


tomografía de cráneo antes de una punción lumbar?
c} Fiebre

56c. Usted lleva a cabo una punción lumbar, por medio de la cual encuentra los
siguientes datos en el LCR: presión de apertura elevada, 9 000 PMN, glucosa 20
mg/dL, proteínas de 1 000 mg/dL, lo cual lo lleva a sospechar de una meningitis:
b} Bacteriana

56d. El médico de base no va a llegar hasta dentro de 8 h,así que usted tiene que
decidir el tratamiento. ¿Qué tratamiento administraría?
d} Ceftriaxona más vancomicina más dexametasona

56e. Si se tratara de un paciente con meningitis meningocócica, ¿cuál es el


tratamiento profiláctico para los contactos cercanos?
b} Rifampicina

57. Acude a consulta paciente masculino de 67 años, fumador,por un cuadro de


malestar general y dolor en columna vertebral que inició hace 3 meses y ha ido
progresando. A la exploración física solamente llama la atención el dolor difuso en
la columna vertebral. En los laboratorios se encuentra anemia normocítica
normocrómica, plaquetas 115 000/µL,VSG 120, creatinina de 2.8 mg/dL, proteínas
totales 8.5 g/dL e hipoalbuminemia de 2.1 g/dL, orina con proteinuria de 4 g/L e
hipercalciemia de 11.2 mg/dL.
57a. De acuerdo con el caso clínico, ¿cuál es su sospecha diagnóstica?
a} Neoplasia maligna de células plasmáticas

57c. Para estadificar la enfermedad se emplean los siguientes criterios, según


Durie-Salmon:
b} Hb, Ca, lesiones líticas en hueso, lgG olgA o cadenas ligeras en orina

57d. ¿Cuál es el tratamiento de un paciente con esta enfermedad que se


encuentra asintomático o en estadio I?
d} No se da tratamiento

57e. ¿Cuál de los siguientes enunciados no es verdadero?


b} El pico monoclonal en la mayoría de los casos es delgA

58. Paciente masculino de 65 años con diagnóstico de envío de enfermedad


ampollosa no especificada presenta dermatosis diseminada que afecta tronco y
mucosa oraly conjuntiva!, además del ano, caracterizada por ampollas tensas,
eritema y descamación en encías. En la valoración oftalmológica se encuentra
conjuntivitis inespecífica,triquiasis y entropión que se acompañan de disminución
en la agudeza visual. En mucosa anal presenta atrofia y erosiones. Refiere cinco
años de evolución y ha recibido múltiples tratamientos tópicos a base de
antibióticos y corticoesteroides así como neosporina oftálmica. Mencione cuál de
las siguientes afirmaciones es correcta.
c} Afecta hasta en 85% la mucosa oral además de afección oftálmica

59. Paciente masculino de 70 años, con antecedente de hipertensión arterial de 5


años de evolución controlada con captopril 25 mg, es fumador desde hace 20
años a razón de una cajetilla diaria y fue diagnosticado con enfermedad pulmonar
obstructiva crónica hace 2 años. Acude a Urgencias debido a que ha aumentado
bastante la tos y presenta un leve aumento del esputo, así como disnea de
esfuerzos mínimos. La radiografía muestra datos típicos de enfisema pulmonar.
59a. ¿Qué tratamiento le daría a este paciente en Urgencias para mejorar su
estado actual?
c} Salbutamol y bromuro de ipratropio

59c. Su paciente presenta una FEV1 de 48%. ¿En qué estadio de GOLD está?
c} Severo

59d. De acuerdo con el estadio de GOLD en el que se encuentra su paciente,


¿cuál es el tratamiento crónico indicado para él?
a} Tiotropio diariamente y un esteroideinhalado en exacerbaciones
59e. ¿Cuál de las siguientes es una indicación para la utilización de oxígeno
suplementario en casa?
b} Saturación de 02 menor a 89%

60. Acude a consulta un paciente masculino de 52 años por un dolor intenso


periorbitario unilateral, acompañado de inyección conjuntiva,lagrimeo y rinorrea.
Comenta que esto le sucede dos veces al día, aprox. desde hace 2 semanas, e
incluso en una ocasión lo despertó en la madrugada. Como antecedente de
importancia tuvo un infarto agudo de miocardio hace 2 años con aplicación de
stent. Toma diariamente aspirina 85 mg y clopidogrel. A la exploración física, los
signos vitales son FC: 70, FR: 18, TA: 130/85 mm Hg, Temp: 36 ºC. El resto de la
exploración es normal.
60a. ¿Cuál de los siguientes es el tratamiento más efectivo para este paciente?
d} Prednisolona e inhalación de oxígeno

60b. ¿Cuál es de los siguientes datos no sirven como indicadores para tomar una
neuroimagen en el caso de cefalea?
c} La visión de escotomas

60c. En cuanto a la cefalea tensional, ¿cuál de los siguientes medicamentos se


utiliza como profilaxis?
a} Amitriptilina

60d. En cuanto a la cefalea migrañosa, ¿en qué casos se deben de indicar


medicamentos profilácticos?
d} Todos los anteriores

60e. ¿A qué se le conoce como estatus migrañoso?


a} Cuando las manifestaciones clínicas duran más de 72 hy no responden a
tratamiento habitual

61. ¿Cuál de los siguientes organismos se asocia con zonas desérticas y se


presenta como esférulas con endosporas en la biopsia de tejidos infectados?
b} Coccidioides immitis

62. ¿Cuál de los siguientes esquemas profilácticos de tromboembolia venosa es el


más adecuado en pacientes de alto riesgo para desarrollar ésta?
c} Administración subcutánea de heparina de bajo peso molecular

63. Paciente con una tumoración de 3 mm que le produce dolor intenso. El tumor
se encuentra por debajo de la uña del dedo índice de la mano izquierda. Se
someterá a un procedimiento quirúrgico para extirpar esta tumoración. ¿Cuál de
los siguientes nervios debe bloquearse para generar anestesia para la extirpación
de este tumor?
d} Mediano

64. Paciente masculino de 77 años de edad que es llevado al servicio de


urgencias por sus hijos debido a que se encontraba con alteración del estado de
alerta y diciendo que estaba en la casa del presidente de la nación. A la
exploración física se presenta con una presión arterial de 80/60 mm Hg, y
frecuencia cardiaca de 120 latidos/minuto, así como con una temperatura de 39
ºC. Los hemocultivos son positivos para Escherichía colí. ¿Cuál es el origen más
frecuente de la bacteriemia en este paciente?
c} Vías urinarias

65. Paciente masculino de 18 años de edad que presenta episodios recurrentes en


los que aparece exantema vesicular en zonas expuestas a la luz. Durante los
estudios de laboratorio se descubre elevación de las porfirinas plasmáticas. ¿Cuál
de las siguientes enzimas es la que se encuentra con mayor probabilidad afectada
en este paciente?
a} Uroporfirinógeno descarboxilasa

66. ¿Cuál de los siguientes enunciados es correcto respecto de la enfermedad


celiaca?
a} En caso de alta sospecha de enfermedad celiaca, una biopsia de intestino
delgado, los marcadores serológicos como los anticuerpos antigliadina,
antiendomisio y antitransglutaminasa tisular pueden confirmar eldiagnóstico.

67. ¿Cuál de los siguientes nervios se puede lesionar en caso de una fractura del
cuello quirúrgico del húmero?
a} Nervio axilar

68. Paciente masculino de 13 meses de edad con historia de fiebre de tres días de
evolución (temperatura máxima 39.8 C) que se resolvió y después apareció rash
en tronco, diseminándose después a brazos, cuello y cara. En la exploración se
encuentran lesiones maculopapulares rosadas, no confluentes, en cara, cuello,
brazos y tronco.
¿Cuál es el agente etiológico más probable?
e} Herpes virus tipo 6

69. Paciente masculino de 73 años, diabético de 25 años de evolución (en control


con metformina 850 mg por la noche) e hipertenso de 10 años de evolución (en
control con captopril 25 mg por la mañana), acude a consulta para control de un
problema de insuficiencia renal crónica. Comenta que se siente en general
cansado y con prurito en todo el cuerpo. A la exploración física sus signos vitales
son FC: 67, TA: 150/90 mm Hg, FR: 18, Temp: 36ºC; el resto de la exploración
normal. Al calcular su tasa de filtración glomerular usted obtiene un resultado de
29mUmin/1.73 m2 .
69a. ¿En qué estadio de insuficiencia renal crónica se encuentra este paciente?
c} Estadio 4
69b. ¿Cuál es la definición correcta de insuficiencia renal crónica?
c} Tasa de filtración glomerular menor a 60 ml/min/1.73 m2 o daño renal
anatómico o funcional por 3 meses o más

69c. ¿Cuál es la causa principal de insuficiencia renal crónica?


b} Diabetes mellitus

69d. ¿Cuál es la meta de presión arterial en un paciente con insuficiencia renal


crónica?
b} Menor a 130/80 mm Hg

69e. ¿En qué momento se recomienda suspender el uso de metformina en un


paciente diabético con insuficiencia renal?
b} Cuando la creatinina sérica sea mayor a 1.4 mg/dl en hombres y 1.5 mg/d en
mujeres

70. ¿Cuál de las siguientes funciones es la que corresponde a las ondas lentas en
el tracto gastrointestinal?
d} Despolarización de las células de músculo liso

71. Posterior a una infección de vías aéreas superiores, un paciente de sexo


masculino de seis años de edad desarrolla lesiones tipo púrpura en los glúteos y
en las superficies extensoras de las extremidades superiores e inferiores.
Simultáneamente con la aparición de estas lesiones el paciente inicia con dolor
abdominal, vómito y artralgias. En una prueba cualitativa de orina se puede
detectar hematuria. ¿Cuál de las siguientes características se encontraría en una
biopsia renal en este paciente?
e} Depósito de lgA en las regiones mesangiales

72. ¿Cuál de las siguientes lesiones se asocia con más frecuencia a necrosis
avascular?
a} Fractura de escafoides

73. Paciente masculino de 22 años de edad, el cual a pesar de apego al


tratamiento, no ha logrado controlar su cuadro asmático. El médico decide agregar
zileutón al esquema. ¿Cuál de los siguientes estudios debe realizarse antes de
iniciar tratamiento con este medicamento?
d} Pruebas de función hepática

74. ¿Cuál de los siguientes enunciados respecto a la pericarditis urémica es


correcto?
a} Actualmente existen procedimientos quirúrgicos indicados en caso de
derrame pericárdico que ocasiona compromiso
hemodinámico
75. Paciente masculino de 45 años de edad que se encuentra en este momento
en fase de rehabilitación por alcoholismo, el cual inició desde los 19 años de edad.
En esta ocasión el paciente acude a discutir los resultados de una biopsia hepática
que se solicitó previamente. A la exploración física presenta esplenomegal ia y
distensión abdominal. El reporte histopatológico de la biopsia indica presencia de
fibrosis difusa y regeneración nodular. ¿Cuál de las siguientes complicaciones
puede presentar este paciente a pesar de que detenga el consumo de alcohol?
e} Carcinoma hepatocelular

76. Paciente masculino de 55 años de edad que a la exploración neurológica


presenta cierre palpebral a la estimulación de la córnea derecha, pero ausencia de
cierre palpebral consensual en el ojo izquierdo. ¿Cuál de los siguientes hallazgos
será posible encontrar también en este paciente?
c} Hiperacusia del oído izquierdo

77. Paciente masculino de 11 años de edad es sometido a una craneotomía para


extracción de craneofaringioma. En la cirugía recibió halotano; al concluir ésta fue
extubado sin ninguna eventualidad. Durante el periodo de recuperación, la
enfermera colocó al paciente una vía intravenosa e infusión de solución de
dextrosa a 5% en un tercio de solución salina normal a una velocidad de 125 mUh.
Posteriormente, la enfermera llama al médico tratante debido a que el paciente
presenta disminución del estado de alerta. Los resultados de laboratorio más
reciente muestran la concentración sérica de sodio de 157 mEq/L, osmolaridad de
320 mOsm/L, PaC02 x de 28 mm Hg, bicarbonato de 22 mEq/L y pH de 7.3, así
como incremento en el balance positivo de líquidos. ¿Cuál de los siguientes
diagnósticos es el más probable en este caso?

b} Lesión a la hipófisis posterior

78. ¿Cuál de los siguientes factores de la coagulación es dependiente de vitamina


K?
a} Factor II, VII,IX y X

79. Paciente del sexo femenino de 28 años de edad que acude a consulta por
infertilidad. A pesar que ha intentado relaciones sexuales regulares sin protección
anticonceptiva durante los últimos dos años, no ha logrado embarazarse. Al
interrogatorio la paciente menciona que tiene menstruaciones muy irregulares, las
cuales ocurren cada dos a tres meses, e inclusive, cada cinco meses. A la
exploración física es evidente que presenta hirsutismo y obesidad. Se decide
iniciar tratamiento farmacológico, lo cual incrementa sus niveles de progesterona
en sangre; la realización de una biopsia de endometrio revela cambios secretores
posteriores al inicio del medicamento. ¿Cuál de los siguientes medicamentos fue
el que se le administró con mayor probabilidad a esta paciente?
d} Antagonistas de estrógenos

80. Paciente masculino de 33 años de edad con antecedente de alcoholismo


crónico se presenta con confusión,alteración de la marcha y nistagmo. Posterior a
la administración de tiamina el paciente muestra mejoría significativa. ¿Cuál de los
siguientes cambios neuropatológicos presentó este paciente?
a} Necrosis hemorrágica de los cuerpos mamilares y dela materia gris
periacueductal

81. Paciente masculino de 45 años de edad el cual se presenta con fatiga, palidez
de piel y mucosas, y soplo sistólico. En los estudios de laboratorio se presenta un
hematocrito de 27%, volumen corpuscular medio de 80 fl,concentración
corpuscular media de hemoglobina de 25 pg/célula y reticulocitos de 4.5%, así
como una concentración de ferritina de 340 µg/L. El frotis de sangre periférica
demuestra macrocitos policromatófilos. ¿Cuál es el diagnóstico más probable en
este paciente?
a} Disfunción de médula ósea

82. ¿Cuál de los siguientes antimicóticos ejerce su efecto sobre los microtúbulos
de las células micóticas?
c} Griseofulvina

83. Paciente femenino de 45 años de edad con antecedente de cáncer de


glándula mamaria. La paciente fue diagnosticada hace dos meses y se le realizó
una mastectomía radical con extirpación de ganglios cervicales. Posteriormente
fue programada para administración de quimioterapia. Posterior a la
administración de quimioterapia la paciente presenta náusea y vómito
incontrolable por lo que se administra un fármaco para disminuir esta
sintomatología, Aunque el medicamento disminuyó la náusea y el vómito, también
ocasionó constipación. ¿Cuál de los siguientes fármacos es el que más
probablemente se le administró a esta paciente?
a} Un antagonista de los receptores de serotonina 5HT3

84. Paciente masculino de 70 años con dolor en hipocondrio derecho desde hace
1O horas. A la exploración física se presenta ictérico y con fiebre. Los estudios de
laboratorio revelan elevación leve de las transaminasas, hiperbilirrubinemia (6
mg/dL) y leucocitosis (16 000 células/mm3). ¿Cuál de las siguientes conductas es
la más adecuada en este paciente?

c} Solicitar una colangiopancreatografía endoscópica retrógrada

85. ¿Cuál de los siguientes fármacos antihipertensivos es adecuado para el


manejo de hipertensión arterial sistólica aislada en pacientes no
diabéticos?
a} Amlodipino

86. ¿Cuál de las siguientes opciones representa la lesión glomerular intrínseca


primaria más frecuente?

b} Nefropatía por lgA


87. Paciente masculino de 66 años de edad que acude a consulta por presentar
desde hace varias semanas febrícula, poliartralgias, y exantema eritematoso
localizado principalmente en la cara, aunque hace poco tiempo se ha observado
también en la región superior del tórax. Al interrogatorio el paciente menciona que
recientemente sufrió un infarto agudo al miocardio, así como una arritmia (no
especificada en este momento). En los exámenes de laboratorio se revela
positividad para el anticuerpo antinuclear. ¿Cuál de los siguientes medicamentos
es el que está tomando este paciente con mayor probabilidad?
b} Procainamida

88. Paciente masculino de 70 años de edad conocido por el servicio de cardiología


porque acude desde hace tres años por cuadros de angina estable. En esta
ocasión asiste al servicio de urgencias por presentar dolor torácico intenso. La
enfermera toma un electrocardiograma, el cual revela elevación del segmento ST;
las pruebas enzimáticas rápidas determinan elevación de la creatinincinasa-MB. El
paciente es estabilizado y admitido a la unidad coronaria. Varios días después de
este evento presenta de nuevo dolor torácico. A la exploración física presenta
fiebre, frote pericárdico intenso, estertores pulmonares y edema de extremidades
inferiores. ¿Cuál de los siguientes diagnósticos es el más probable en este
paciente?
d} Pericarditis fibrinoide

89. ¿De los métodos diagnósticos presentados a continuación, cuál es el más


adecuado para realizar el diagnóstico de embolia pulmonar en un paciente con un
proceso neumónico subyacente?

e} Tomografía computarizada espiral contrastada deltórax

90. Paciente masculino de 44 años de edad que se presenta a consulta por


intensos dolores cólicos, constipación, irritabilidad y cefalea. El paciente trabaja en
una fábrica de pintura en aerosol. Presenta unos estudios de laboratorio que el
médico laboral le solicitó; la principal característica de éstos es una anemia
microcítica. Al revisar el expediente médico el paciente ha presentado episodios
depresivos mayores en el pasado. ¿Cuál de las siguientes sustancias es con
mayor probabilidad la causante la sintomatología de este sujeto?
e} Plomo

91. Se requiere de su presencia en la sala de parto debido a que se espera el


nacimiento de un masculino de 37 semanas de gestación. Observa los
procedimientos del obstetra, no se sucitan complicaciones y recibe un recién
nacido masculino. Inicia la exploración física de éste y observa movimiento activo,
frecuencia cardiaca de 139 lpm, buen tono muscular, flexión de las cuatro
extremidades, acrocianosis y llanto vigoroso.
91a. De acuerdo con las características del recién nacido, ¿qué puntaje de Apgar
obtiene?
c} 8 puntos

91b. Al recibir a un recién nacido vigoroso como en el caso descrito, ¿cuál de los
siguientes pasos se debe realizar primero?
a} Verificar que la vía aérea esté permeable, secar,y estimular al recién nacido

91c. En caso de no contar con la información de la edad gestacional del recién


nacido descrito anteriormente, ¿cuál de los siguientes signos clínicos le
indicaríamejorque se trata de un recién nacido a término?
c} Tejido mamario palpable

91d. El neonato pasa a cuidados en cuneros y se comienza el protocolo de recién


nacido por las enfermeras, el cual incluye escrutinio de hipoglucemia. En el
neonato a término asintomático, ¿a qué nivel de glucosa debe iniciarse la
evaluación y tratamiento de hipoglucemia?
c} 45 mg/dl

91e. Después de una semana de nacido, el paciente es llevado a consulta de


revisión. ¿Cuál de las siguientes opciones es la correcta respecto al peso de un
recién nacido en la primera semana de vida?
d} Perder aproximadamente 5 a 10% delpeso al nacimiento

92. Paciente masculino de 50 años de edad con esófago de Barrett que se


presenta con gastroenterólogo para endoscopia de seguimiento. Se toman
múltiples biopsias y se envían a estudio histopatológico. El reporte revela la
presencia de displasia de bajo grado. ¿Cuál de las siguientes conductas clínicas
es la más adecuada?
a} Repetir endoscopia en 6 y 12 meses

93. Paciente masculino de tres meses de edad que acude a consulta pediátrica de
rutina. En la evaluación es evidente que padece leucocoria. Después de
evaluación oftalmológica se determina que la leucocoria es debida a un
retinoblastoma. Se realiza enucleación del ojo izquierdo. ¿Cuál de las siguientes
estructuras del ojo extraído es la que merece especial atención por el patólogo
porque determina el pronóstico?
c} Nervio óptico

94. Paciente masculino de 35 años de edad, bajo tratamiento por depresión


mayor, se presenta al servicio de urgencias por presentar confusión intensa y
alucinaciones. A la exploración física el paciente es poco cooperador, pero se
observa que la boca está seca y la cara eritematosa. La enfermera toma los
signos vitales, resultando en hipotensión (85/50 mm Hg) y taquicardia (11O
latidos/minuto). Se realiza un electrocardiograma, el cual revela prolongación del
intervalo QT (0.13 mseg) y contracciones ventriculares prematuras. ¿Cuál de los
siguientes medicamentos es el más adecuado para este paciente?
e} Bicarbonato de sodio

95. Mujer de 24 años de edad que presenta tumoración en fosa ilíaca derecha
dolorosa. Desde hace seis meses presenta palpitaciones, caída de cabello, diarrea
e hiperreflexia. Se diagnostica tumor de ovario benigno. Se observan tejidos de las
tres capas germinativas. ¿Qué es característico de los tumores germinales de
ovario?
a} El gonadoblastoma se relaciona a síndrome de Swyer

96. Femenino de 62 años llevado a sala de urgencias por experimentar episodios


frecuentes de ptosis, diplopía y fatiga generalizada. EF: se encuentra timo
palpable y parálisis del nervio oculomotor que se corrige transitoriamente al
administrar edrofonio.
La entidad nosológica responsable de la sintomatología de este paciente es:
a} Miastenia grave

97. Un paciente bajo tratamiento antipsicótico por esquizofrenia acude a la


consulta por exceso de rigidez muscular, hipertermia y disminución de la sudación.
A su ingreso el paciente presenta cambios bruscos en las cifras tensionales,
acompañados de episodios de taquicardia y bradicardia. ¿Cuál de los siguientes
medicamentos se debe administrar inmediatamente a este paciente?
d} Dantroleno

98. Mujer de 60 años, consulta por aumento de volumen en cuello. Al


interrogatorio refiere fatiga y estreñimiento hace varias semanas, con aumento
leve de peso. TA: 120/90, FC: 65 x', Temp: 36.5 ºC. A la exploración del cuello se
detecta bocio irregular e indoloro, edema pretibial leve sin fóvea, y frialdad de
extremidades. Sus medicamentos incluyen ramipril y metformina. Perfil tiroideo:
TSH elevada, T4 libre baja, anticuerpos antiperoxidasa tifoidea positivos.
98a. Con la clínica y los laboratorios mencionados, usted hace el diagnóstico de:
c} Hipotiroidismo autoinmune

98b. ¿Cuál es la base fisiopatológica de la enfermedad de este paciente?


a}Infiltración de linfocitos T CD4 y CDS activados y linfocitos B enla tiroides

98c. ¿Cuál de los siguientes enunciados es correcto?


d} La T3 es la hormona tiroidea más potente

98d. ¿Cuál es el tratamiento indicado en este paciente?


a} Levotiroxina
98e. ¿Cuál de los siguientes enunciados es correcto respecto altratamiento
restitutivo con levotiroxina en paciente hipotiroideos?
b} El objetivo del tratamiento es lograr una concentración normal de TSH

99. Neonato de seis horas de vida cuya madre fue positiva para antígeno de
superficie de hepatitits B (HBsAg). ¿Cuál es la conducta más adecuada en
relación a profilaxis contra hepatitis B?
c} Administrar la primera dosis de vacuna anti-hepatitis B e inmunoglobulina de
hepatitis B antes de las primeras 12 horas de vida

1OO. Paciente femenino de siete años de edad acude a consulta por diplopía que
ha empeorado en los últimos días. A la exploración oftalmológica demuestra
parálisis del VI nervio craneal. ¿Cuál de los siguientes padecimientos es el que
con mayor probabilidad tiene la paciente?
a} Glioma del puente

101. Paciente masculino de 48 años de edad, sedentario, con circunferencia


abdominal de 108 cm, hipertensión arterial e hipertrigliceridemia. Acude a consulta
de control. Se reporta asintomático. TA: 130/80 mm Hg, hiperpigmentación en
cuello y axilas. Laboratorios: glucosa en ayuno 154 mg/dl, HbA1c: 7.0%, colesterol
LDL: 115 mg/dl, HDL: 48 mg/dl, triglicéridos: 300 mg/dl.
101a. ¿Cuál es el diagnóstico en este paciente?
c} Síndrome metabólico cumpliendo 4 criterios

101b. De acuerdo con el diagnóstico de diabetes mellitus tipo 2 en el paciente,


señale el enunciado correcto:
b} Se debe repetir la prueba para el diagnóstico definitivo

101c. Señale el enunciado correcto respecto a las complicaciones en la DM2:


a} La nefropatía es una complicación microangiopática

101d. En relación con el tratamiento en este paciente, señale el enunciado


incorrecto:
a} La presión arterial se encuentra dentro dela meta para DM

102. Paciente femenino recién nacida producto de un embarazo prematuro de 32


semanas que exhibe ictericia, exantema, rinitis persistente, anemia, linfadenopatía
generalizada y anormalidades óseas visibles en las radiografías.
¿Cuál de los siguientes factores es el más importante durante el interrogatorio de
la madre?
b} Antecedentes sexuales

103. Lactante de siete meses de edad que se presenta con diarrea y vómito. La
madre menciona que hace cinco días inició consumo de leche de fórmula;
previamente sólo había recibido leche materna. De acuerdo al diagnóstico del
paciente, ¿qué componentes de la leche son los responsables del cuadro clínico?"
a} Beta lactoglobulina y caseína

104. ¿Cuál de los siguientes tipos de catarata se asocia con la enfermedad de


Wilson?
b} Catarata de girasol

105. Paciente masculino de 72 años de edad acude a consulta por mareo y ojo
rojo crónico con disminución de la agudeza visual. Se realiza un angiograma, el
cual demuestra retraso en la visualización de los vasos retinianos con hemorragias
en el polo posterior y periferia. ¿Cuál es la condición con más probabilidad de ser
responsable de este cuadro?"
a} Estenosis carotídea

106. Niño de tres años de edad con probable reflujo vesicoureteral,se solicita
cistografía ureteral miccional y se observa reflujo bilateral hacia uréteres dilatados
y tortuosos con dilatación de pelvis y cálices renales. De acuerdo con la
clasificación de reflujo vesicoureteral del paciente, ¿qué abordaje terapéutico es
más adecuado?
d} Cirugía antirreflujo

107. Paciente masculino de 47 años de edad con antecedente de depresión en


tratamiento farmacológico no especificado, abandonado hace 6 meses. Acude
traído por paramédicos al encontrarse en su domicilio inconciente, se identifica
una caja de paracetamol vacía. A su ingreso con Glasgow de 13 puntos,ictérico,
afebril, diaforético, taquicárdico, murmullo vesicular conservado. Abdomen blando
depresible no doloroso a la palpacion superficial no profunda, presencia de
hepatomegalia. Extremidades sin edema, pulsos distales presentes, llenado
capilar 2 segundos. FC 125 lpm, FR 25 rpm, temp 36.5 ºC, PA 90/60 mm Hg, peso
87 kg, talla 1.62 m.
107a. ¿Cuál es su impresión diagnóstica?
b} Intoxicación por paracetamol

107b. ¿Cuál es el metabolito tóxico del paracetamol?


b} Acetil parabenzoquinoneimina

107c. ¿Cuál es la dosis tóxica de paracetamol en adultos?


c} 7g en 24h

107d. ¿Cuál es la etapa de intoxicación en la que se encuentra nuestro paciente?


b} Estadio 11

107e. ¿Cuál es el antídoto y su dosis?


a} N acetil cisteína 140mg/kg inicial 70mg/kg cada 4h,17 tomas

108. Paciente femenina de 18 años de edad. Sin antecedentes patológicos.


Estudiante. Consulta por cefalea bilateral opresiva de 36 h de evolución,
intermitente, la mayor parte del día,y dificultad para conciliar el sueño, sin embargo
el dolor no la despierta una vez dormida; lo anterior se acompaña de náusea sin
llegar al vómito. TA: 110/70 mm Hg, FC: 75 x', FR: 14 x', Temp: 36.5 ºC.
Exploración neurológica sin alteraciones. No ha consumido medicamentos y se
presenta sin laboratorios.
108a. Por la semiología del dolor, usted hace el diagnóstico presuntivo de:
b} Cefalea tensional

108c. ¿Cuál de los siguientes enunciados es cierto respecto a la cefalea


tensional?:
b} Es más frecuente en mujeres

108d. ¿Cuál es el tratamiento para disminuir la sintomatología en esta paciente?


0 c} AINE

108e. En relación con la fisiopatología de la cefalea tensional,señale la respuesta


correcta:
a} No se conoce en detallelos aspectos fisiopatológicos

109. Paciente femenino de 88 años de edad es hospitalizada por una caída en su


casa debido a tropezón. No puede caminar o soportar peso sobre su extremidad
inferior derecha. Las radiografías revelan que tiene una fractura desplazada del
cuello del fémur, sin demostrar algún otro hallazgo de importancia. Al interrogario
niega cualquier otro traumatismo en la región afectada. Posterior a la
estabilización de esta paciente, ¿cuál es el tratamiento más óptimo para su tipo de
fractura?
d} Hemiartroplastia

11O. Angélica, de 24 años de edad, refiere inicio de vida sexual activa hace 5
años y utiliza anticonceptivo de barrera, su ciclo menstrual es irregular y no
recuerda su fecha de última menstruación, niega otros antecedentes; acude por
dolor abdominal, que inició hace 8 horas, localizado en la fosa ilíaca derecha, tipo
cólico, intensidad 8/1O que baja hasta 3/1O, se automedicó butilhiosina con
mejoría parcial, se agregó anorexia, adinamia,escalofrío, náusea y vómito, con
intolerancia a la vía oral, niega otra sintomatología. A la exploración presenta
disminución de la peristalsis, dolor a la palpación de fosa ilíaca derecha, con signo
de von Blumberg positivo, resto negativos. FC 95 lpm, FR 22 rpm, temp. 37.5 ºC,
PA 130/65 mm Hg, peso 55 kg, talla 1.60 m.
11Oa. Usted considera que se trata de una apendicitis aguda, ¿cuál de los
siguientes es el diagnóstico diferencial a descartar?
b} Embarazo ectópico

11Ob. ¿Qué estudio de laboratorio solicita para descartar el embarazo ectópico?


c} Niveles de HGC-P

11Oc. ¿Qué estudio de imagen es de primera elección para confirmar apendicitis


aguda?
c} Tomografía simple

11Od. Se planea egresar al paciente a piso de Cirugía General después de la


apendicectomía. ¿Cuál es el tratamiento antimicrobiano de primera elección para
los pacientes con apendicitis aguda perforada?
a} Piperacilina-tazobactam

11Oe. Los laboratorios reportaron leucocitosis de 18,300 / mm3 con desviación a


la izquierda,con los datos clínicos y paraclínicos obtenidos ¿cuántos puntos de la
clasificación de Alvarado tiene la paciente?
c} 9 puntos

111. Paciente femenino de 33 años de edad que acude a consulta por presentar
debilidad y parestesias en los primeros tres dedos de la mano derecha. La
paciente informa también dolor localizado a la articulación de la muñeca. Durante
el interrogatorio acepta que los síntomas son peores cuando escribe en la
computadora. Lo más probable es que la paciente presente un síndrome del túnel
del carpo. ¿Cuál de las siguientes pruebas es la más sensible para realizar este
diagnóstico?
b} Prueba de compresión

112. Masculino de 55 años de edad, fumador,dislipidémico, refiere dolor precordial


opresivo irradiado a mandíbula al ejercicio que desaparece con el reposo a los 5
min; lleva 2 meses con la misma sintomatología. TA: 145/95, FC: 88 lpm. Ruidos
cardiacos rítmicos sin soplos. Sin laboratorios.
112a. ¿Cuál es el diagnóstico clínico de este paciente?
a} Angina de pecho estable

112b. ¿En qué clase funcional se encuentra el paciente según la clasificación de la


New York Heart Association?
b} Clase II

112c. ¿Cuál es el estudio para continuar con el diagnóstico del paciente?


c} Prueba de esfuerzo

112d. Estándar de oro para la detección de cardiopatía isquémica:


c} Coronariografía

112e. ¿Cuál de los siguientes esquemas terapéuticos es el más indicado para el


paciente?
a} Aspirina + estatina + IECA + betabloqueador

113. Paciente masculino de tres años de edad, con antecedente de otalgia y


otorrea derecha, desde los dos años de edad, en cuatro ocasiones las cuales
remitieron con manejo antimicrobiano. Refiere la madre cuadro de vías
respiratorias superiores de 48 h de evolución;acude por otalgia derecha.
Exploración física. Temperatura 38 ºC. Otoscopia derecha con membrana
timpánica hiperémica, no móvil. Otoscopia izquierda con membrana timpánica
opaca y no móvil. Cavidad oral con amígdalas grado 3, crípticas sin exudados.
Cuello sin adenomegalias.
113a. El diagnóstico más probable es:
c} Otitis media aguda recurrente

113b. El microorganismo con más frecuencia implicado en esta patología es


c} Streptococcus pneumoniae

113c. El tratamiento consiste en:


c} Amoxicilina

113d. Dos meses después el paciente es llevado a consulta porque presenta


sensación de plenitud aural y la madre refiere que escucha el televisor con un
volumen más alto del habitual. En la otoscopia se observan las membranas
timpánicas íntegras, opacas y no móviles. Usted solicita una impedanciometría
que reporta disminución en la complianza de la membrana timpánica. La conducta
siguiente sería:
a} Manejo expectante

114. Renata es una paciente de 27 años de edad que acude al servicio de


Urgencias por presentar pérdida súbita de la agudeza visual de forma completa
con dolor retroocular,que se acompaña de debilidad de las extremidades
inferiores, misma que limita la deambulación. Al interrogatorio dirigido niega otros
antecedentes, alergias, infecciones y vacunación reciente. La exploración visual
revela alteraciones en la campimetría por confrontación, el fondo de ojo con
borramiento de los bordes y eritema de la papila; la exploración neurológica es
normal. FC 75 lpm, FR 18 rpm, temp. 36.5 ºC, PA 110/65 mm Hg, peso 58 kg, talla
1.67 m.
114a. Con base en el cuadro clínico, ¿cuál es su diagnóstico presuntivo?"

d} Eclerosis múltiple
114b. Según las manifestaciones clínicas pueden seguir uno de los cuatro
patrones característicos, ¿cuál es el más común?
a} Brote-remisión

114c. Para establecer el diagnóstico de la paciente usted solicita:


c} Resonancia magnética de cráneo

114d. ¿Cuál de los siguientes es un diagnóstico diferencial de la esclerosis


múltiple?
a} Vasculitis

114e. ¿Cuál es el tratamiento de elección para brotes comunes?


d} Metilprednisolona

115. Paciente masculino de 70 años que acude a consulta por problemas de


visión. Refiere que en los últimos meses ha notado visión borrosa ""justo en el
frente"" de su campo visual, que no se puede corregir con sus lentes. Este
problema le afecta ambos ojos. En sus antecedentes patológicos menciona
hipertensión,tratada con amlodipino. Refiere tabaquismo de 20 paquetes/año. Al
mostrarle la cartilla de Amsler dice notar un área borrosa y asimétrica cerca del
centro. A la oftalmoscopia se observan depósitos de drusen en la mácula y áreas
de depigmentación. No hay datos de neovascularización. ¿Cuál sería el
tratamiento indicado en este paciente?"
e} Observación

116. Un hombre de 68 años que se encuentra hospitalizado por sufrir un infarto


anterior. Como tratamiento se le realizó un cateterismo y se le colocó un stent.
Llaman durante la noche porque el paciente se queja de disnea. A la exploración
física sus signos vitales incluyen FC 52/min, FR 21/min, T 37.3 C y PA 108/69 mm
Hg, con saturación de oxígeno de 88%. A la auscultación se escucha S1 y S2 sin
agregados. Los campos pulmonares se auscultan con estertores crepitantes. Se le
realiza un trazo de electocardiograma, que se muestra a continuación:

¿Cuál de las siguientes opciones representa la conducta más adecuada?


c} Colocación de marcapaso transcutáneo

117. Un hombre de 71 años acude a consulta por lumbalgia. Refiere que el dolor
inició hace cuatro meses, localizado en el área central de la espalda baja. En su
historia clínica refiere hipertensión tratada con hidroclorotiazida y osteoartritis. Al
interrogatorio refiere sentirse un poco más cansado de lo normal,y dolor
intermitente de sus extremidades. Niega incontinencia urinaria, parestesias, o
cambios de la sensibilidad. A la exploración física se observan nódulos de
Heberdeen y de Bouchard. No hay dolor a la palpación de la columna dorsal o el
resto de la espalda. La sensibilidad perianal está intacta,así como el resto de la
exploración neurológica. En la radiografía se observa osteofitosis de la columna
vertebral, sin otras anormalidades aparentes. Se realizan estudios de laboratorio,
que se muestran a continuación:

¿Cuál de estos estudios diagnósticos estaría indicado en este paciente?


a} Electroforesis de proteínas séricas

118. Un hombre de 81 años acude a consulta por "ronchas". Él es residente de un


asilo y se ha reportado que estas lesiones han ocurrido recientemente en otros
residentes. Los antecedentes patológicos del paciente incluyen hipertensión,
hiperlipidemia, enfermedad coronaria y enfermedad de Alzheimer. El paciente no
es capaz de responder el IPAS, aunque la enfermera que lo acompaña niega
fiebre, o síntomas de infección de las vías respiratorias superiores. A la
exploración física sus signos vitales incluyen FC 81/min, FR 16/min, T 37.2C y PA
130/76 mm Hg. Se observan en los brazos múltiples lesiones papulares en
superficies extensoras e interdigitales, con excoriaciones circundantes. ¿Cuál
sería el manejo de este paciente?
a} lvermectina dosis única a él y a todos los residentes y trabajadores del asilo
119. Una paciente de 20 días de edad es llevada a la sala de urgencias. La madre
menciona que la paciente ha presentado fiebre y llanto continuo las últimas 4
horas; la paciente ha sido alimentada el día de hoy en una sola ocasión pero
vomitó toda la fórmula ingerida. Se refiere que la paciente fue producto de un parto
vaginal a término y eutócico de una madre de 22 años gesta 1, para 1. Ningún otro
antecedente de importancia. A la exploración física la paciente presenta
temperatura de 38.3 ºC, pulso de 140 latidos por minuto y frecuencia respiratoria
de 32 respiraciones por minuto. Presenta disminución del tono muscular,sin
respuesta a los estímulos visuales o auditivos. La fontanela anterior se presenta
abombada. Mucosas adecuadamente hidratadas y ausencia de exantemas. Los
estudios de laboratorio revelan leucocitosis.
119a. Se realiza una punción lumbar,en la que se observa un bacilo grampositivo.
¿Cuál es el agente causal más probable?
b} Listeria monocytogenes

119b. ¿Cuál es el tratamiento más adecuado según el organismo causal?


a} Ampicilina y gentamicina

119c. ¿Cuál es la forma más común de infección del organismo causal del cuadro
de meningitis?
a}Ingestión de productos lácteos no pasteurizados

120. Paciente masculino de 67 años que se presenta al consultorio, su


preocupación principal es que un amigo acaba de morir de un infarto cardiaco y
quiere hacer todo lo que pueda para prevenirlo. Su padre murió de un infarto
agudo de miocardio a los 74 años. Niega alguna enfermedad. Refiere tabaquismo
de 24 paquetes/año. A la exploración física sus signos vitales incluyen FC 72/min,
FR 16/min, T 37.2 C y PA 126/78 mm Hg. Refiere que nunca ha tenido una lectura
de presión arterial arriba de 130/80 mm Hg en sus mediciones previas. Su última
medición de HDL fue de 50 mg/dl. A la exploración física, se observa un paciente
delgado, y no se detectan anormalidades. ¿Cuál sería la meta de LDL para este
paciente?
a} LDL s 130 mg/dl

8. Un hombre de 73 años de edad acude al departamento de urgencias por dolor


torácico que inició hace 15 minutos. Describe el dolor como una presión continua
central que se irradia al brazo izquierdo. Este paciente ya es conocido en el
departamento por su antecedente de enfermedad coronaria y angina. El resto de
su historia clínica incluye diabetes y enfermedad vascular periférica. Sus signos
vitales incluyen FC 101/min, FR 20/min, T 37.2 C y PA 182/65 mm Hg. A la
auscultación cardiaca se escucha un tercer ruido. En los campos pulmonares se
auscultan estertores. ¿Cuál es el siguiente paso en el manejo de este paciente?
d} Administrar aspirina

11. Una mujer de 65 años acude a consulta para revisión general. Es una persona
sana, refiriendo sólo dolores articulares intermitentes e hipotiroidismo. Sus
medicamentos incluyen levotiroxina y ocasionalmente paracetamol. Refiere
tabaquismo de 20 paquetes/año y consume bebidas alcohólicas de forma
regular.Al interrogatorio niega otros síntomas. Sus signos vitales incluyen FC 82,
FR 16, T 37.3 C y PA 120/67. Su preocupación principal es la osteoporosis. ¿Cuál
de los siguientes son factores de riesgo para la osteoporosis en esta paciente?
a} Tabaquismo y alcoholismo

16. Femenino de 37 años, referida al servicio de urgencias por partera empírica.


Con diagnóstico de embarazo de término en trabajo de parto de 16 h de evolución.
Le fue aplicada una ampolleta de 5UI de oxitocina IV directa. Antecedentes: Glll,
PI, Al,EF: TA 140/80 mmHg, FC 90 latidos por min, FR 30 respiraciones por min, 5
contracciones uterinas en 1O min, duración de 45 , útero hipertónico, fondo uterino
de 38 cm del BSP. Producto en situación longitudinal, presentación cefálica
abocada, con frecuencia fetalen Cll de 130 por min, rítmico de buena intensidad.
Al tacto vaginal: cérvix con 5 cm de dilatación y presentación libre. Durante la
preparación para su ingreso desaparecen las contracciones, el abdomen ahora es
blando palpándose fácilmente las partes fetales, los signos maternos son ahora
TA 80/40 mmHg, frecuencia fetal de 140 lat/min.
16a. La sintomatología de esta paciente es debida a:
d} Ruptura uterina completa

16b. El dato clínico que establece el diagnóstico de esta paciente es:


b} Dolor

16c. Factor de riesgo que desencadenó el cuadro clínico de esta paciente:


d} Utilización de oxitocina

18. ¿Cuál de los siguientes nervios se encarga de la producción del reflejo


cremasteriano?

b} Nervio genitofemoral

19. Paciente de 33 años de edad con antecedente de alcoholismo crónico y


síndrome depresivo con intentos suicidas previos. Acude al servicio de urgencias
tras presentar varias crisis convulsivas generalizadas y visión borrosa con diplopía
y fotofobia, sin que se pudiera determinar la existencia de síntomas previos. A su
llegada a urgencias se observa cianosis, hipotermia, hipotensión y coma con una
puntuación de 3 en la escala de coma de Glasgow, con pupilas midriáticas y
arreactivas. En los estudios de laboratorio se destaca acidosis de brecha aniónica
elevada y aumento de los niveles de ácido láctico. En el servicio de urgencias la
paciente presenta un episodio de asistolia.
¿Cuál de las siguientes sustancias es la responsable de la sintomatología de esta
paciente?
b} Ácido fórmico

20. Paciente masculino de 38 años de edad con diagnóstico de síndrome de


inmunodeficiencia adquirida se presenta con proteinuria e hipertensión. La biopsia
renal revela la combinación de glomérulos de apariencia normaly algunos otros
con depósitos de material hialino; no existe incremento de la celularidad o necrosis
en los glomérulos afectados; los túbulos renales presentan dilataciones quísticas,
algunas de las cuales se encuentran llenas de material proteináceo. La
microscopia electrónica revela fusión focal de podocitos y la inmunofluorescencia
muestra depósitos granulares de lgM/C3. ¿Cuál de los siguientes diagnósticos es
el más probable en este paciente?
c} Glomerulosclerosis focal y segmentaria

21. Paciente de ocho años de edad con ataques recurrentes de otitis media bajo
tratamiento antibiótico. Los padres del paciente lo llevan al consultorio debido a
que han notado la presencia de hematomas en piernas y sangrado al cepillado de
dientes. La exploración física es relevante por hemorragias en astillas en las uñas
de las manos. ¿Cuál es la función del nutriente con deficiencia en este paciente?
a} Carboxilación de los residuos de ácido glutámico de proteínas hepáticas

22. Paciente de sexo masculino de cinco años de edad es llevado a consulta por
presentar mala agudeza visual progresiva y mancha blanca en ojo izquierdo de
seis meses de evolución. Como antecedentes perinatales: producto de la gesta 1
de 37 SDG obtenido vía vaginal,Apgar 8/9. No incubadora,convulsiones o
administración de oxígeno. Esquema de inmunizaciones completo. A la
exploración oftalmológica se encuentra agudeza visual de ojo derecho 20/20 y ojo
izquierdo 20/400 con presencia de endotropía. A la biomicroscopia sin
alteraciones. En fondo de ojo de ojo izquierdo se observan telangiectasias
retinianas periféricas, exudados amarillentos en periferia y neovascularización.
22a. El diagnóstico más probable es:
b} Enfermedad de Coats

22b. ¿Cuál es el factor etiológico de esta enfermedad?


e}ldiopático

22c. ¿Qué estudios complementarios son de ayuda para el diagnóstico y


tratamiento en esta enfermedad?
a} Angiografía con fluoresceína y ecografía

22d. ¿Cuáles son las alternativas terapéuticas para esta enfermedad?


e} Todas las anteriores

24. Paciente del sexo femenino de 22 años de edad que ejerce la prostitución. Se
presenta a la consulta externa por prurito vaginal y secreción amarillenta
maloliente. La observación bajo el microscopio revela la presencia de
Trichomonas vagina/is. ¿Cuál de las siguientes aseveraciones es correcta en
relación a la infección vaginal tricomoniásica?
d} La tricomoniasis sólo puede ser adquirida a través de vía sexual

25. ¿Cuál de los siguientes enunciados NO es un criterio diagnóstico para el


síndrome de dificultad respiratoria aguda?
c}Incremento respiratorio de la distensibilidad

26. ¿Cuál de las siguientes condiciones tiene mayor influencia en la progresión de


infección inicial por el virus de la inmunodeficiencia humana hacia el síndrome de
inmunodeficiencia adquirida?
b} La carga viral seis meses posteriores a lainfección inicial

27. Una paciente de 60 años se presenta con diagnóstico reciente de cáncer de


mama en actual tratamiento con ciclofosfamida, metotrexato y 5-fluorouracilo. El
motivo de consulta es por náusea y vómito posterior a la quimioterapia, así como
disnea de reciente inicio, úlceras orales y fatiga. A la exploración física la paciente
presenta derrame pleural del lado derecho y mucositis. Los resultados de
laboratorio son los siguientes: creatinina de 2.4 mg/dl, nitrógeno ureico de 25
mg/dl, leucocitos de 1 OOO/mm3, hemoglobina de 9.2 mg/dl y plaquetas de 91
OOO/mm3. ¿Cuál es el origen de la mayoría de sus actuales síntomas?
a} Toxicidad por metotrexato

28. ¿Cuál de los siguientes procesos fisiopatológicos producen principalmente una


hipertrofia concéntrica ventricular?
c} Estenosis aórtica

29. Paciente femenino de 29 años de edad que se presenta con poliartralgias y


rigidez matutina en las manos y rodillas. Hace una semana la paciente presentó
un episodio febril acompañado de exantema eritematoso facial y reticular en las
extremidades, características que ya no se encuentran presentes. A la exploración
física presenta ligero edema y dolor a la presión de las articulaciones del carpo,
metacarpofalá ngicas e interfalá ngicas proximales bilateralmente. ¿Cuál es el
diagnóstico más probable en esta paciente?
d} Infección por el parvovirus 819

30. ¿Cuál de las siguientes hormonas se encarga de la conversión de testosterona


en 17-beta estradiol en las células de la granulosa?
d} Hormona foliculoestimulante

31. Se presenta una paciente de 15 años de edad, con preocupación porque no ha


comenzado a menstruar.Al interrogatorio no se revela ningún antecedente de
importancia. La exploración física es significativa por un adecuado desarrollo
mamario, estatura dentro de rangos normales, vagina corta y ausencia de cervix y
útero a la exploración pélvica bimanual. ¿Cuál es el diagnóstico más probable en
esta paciente?
e} Síndrome de feminización testicular

32. Acude a consulta externa un hombre de 28 años de edad por presentar desde
hace 15 días fatiga, malestar general y disnea. El paciente es originario de un área
rural, y por el momento se encuentra laborando como constructor; ha estado
viviendo debajo de un puente. Al interrogatorio el paciente refiere que sus
síntomas se han acompañado de tos sin flema, la cual en ocasiones lo lleva
alvómito; también refiere anorexia en los últimos días. El paciente niega cualquier
pérdida de peso o sudoración nocturna, sin embargo, menciona que es posible
que haya sufrido de fiebre (subjetiva, no cuantificada). A la edad de 26 años, un
médico le diagnosticó síndrome de inmunodeficiencia humana con un conteo de
linfocitos CD4+ de 134 (el paciente trae reporte con los resultados). Actualmente
no se encuentra consumiendo medicamentos y confiesa ser adicto al uso de
cocaína y etanol. A la exploración física se encuentra una presión arterial de
120/75 mm Hg, taquicardia de 95 latidos/minuto; la saturación de oxígeno se
encuentra reducida; la Pa02 es de 65 mm Hg. La inspección general muestra un
paciente con dificultad respiratoria y hallazgos auscultatorios de neumonía. La
radiografía torácica demuestra un infiltrado difuso bilateral, con predominio basal.
Utilizando únicamente la información previamente descrita, responda las
siguientes preguntas:
32a. ¿Cuál de los siguientes agentes patógenos es el que con mayor probabilidad
está ocasionando los síntomas de este paciente?
a} Pneumocystis jiroveci

32b. ¿Cuál de las siguientes opciones es la mejor conducta terapéutica en este


paciente?
d} Administración de trimetoprim-sulfametoxazol y prednisona

33. ¿Cúal de los siguientes fármacos inhibe la enzima escualeno epoxidasa?


d)Terbinafina

34. Paciente de 18 años de edad que acude a consulta por malestar general de
dos semanas y malestar faríngeo. A la exploración física se revela faringitis,
hepatoesplenomegalia y linfadenopatías axilares. Los resultados de estudios de
laboratorio son: hemoglobina 13 g/dl, hematocrito 43%, volumen corpuscular
medio 95 fl,conteo plaquetario 300 000 plaquetas/mm3 y leucocitos de 9 000
células/mm3. El frotis de sangre periférica demuestra linfocitos atípicos. ¿Cuál fue
el principal origen por el que esta paciente adquirió esta enfermedad?
d} Por su novio

35. Paciente de sexo femenino de 18 meses de edad es llevada al departamento


de urgencias por alteraciones mentales y retraso mental progresivo. A la
exploración física se encuentra hepatoesplenomegalia, linfadenopatías, sin
alteraciones a la exploración del fondo de ojo. Se realiza biopsia hepática,
esplénica y de ganglios linfáticos, los cuales demuestran macrófagos aumentados
de tamaño con lisosomas alargados y distendidos. ¿Cuál es la deficiencia
enzimática que presenta esta paciente?
b} Glucocerebrosidasa

36. ¿Cuál de las siguientes hormonas gastrointestinales estimula la secreción de


bicarbonato del páncreas?
b} Secretina
37. Paciente femenino de 22 años de edad se diagnosticó con un trastorno
hemorrágico desde la adolescencia. Entre los principales síntomas, la paciente
informa hemorragias al cepillado de los dientes, episodios de menorragia y anemia
grave. El mismo cuadro presentaron la madre y abuela de esta paciente. Los
estudios de laboratorio revelan un conteo plaquetario de 250 OOO/mm3,
incremento en el tiempo de sangrado, pero con normalidad del tiempo de
protrombina y el tiempo de tromboplastina parcial. ¿Cuál es el diagnóstico más
probable en esta paciente?
e} Trombastenia de Glanzmann

38. Ante una mujer con tipo sanguíneo AB-, cursando su segundo embarazo en la
semana 27 de gestación, con prueba de Coombs indirecto negativo, ¿cuál es la
conducta más adecuada?
c} Administrar anti-lgM en este momento y después del nacimiento (

39. Secundigesta de 24 años de edad en trabajo de parto. El producto se


encuentra con una variedad de posición occipitoposterior derecha.
Ordena adecuadamente los movimientos cardinales del trabajo de parto.
a} Encajamiento, descenso, flexión,rotación interna, extensión,rotación externa

40. En el caso de una mujer que durante el embarazo estuvo en contacto cercano
con gatos y perros, hay probabilidad de que ocurra una infección connatal. De ser
así, ¿cuáles de las manifestaciones siguientes esperaría de acuerdo con los
antecedentes del caso? 1) Cataratas, 2) convulsiones, 3) coriorretinitis, 4)
hipoacusia sensorial,5) calcificaciones periventriculares, 6) calcificaciones
cerebrales difusas, 7) hidrocefalia, 8) microcefalia.
b}2,3,6,7"

41. Paciente femenino de 12 años de edad que acude a consulta externa de


pediatría. La madre menciona que la niña ha estado cojeando desde hace varias
semanas y refiere dolor intenso en la rodilla izquierda. Al interrogatorio la madre
dice que la paciente ha estado afebril, no recuerda golpes o traumatismos dirigidos
a las extremidades, ni antecedente de enfermedades recientes. A la exploración
física es evidente que la paciente no puede rotar internamente la pierna afectada.
¿Cuál de los siguientes diagnósticos es el más probable en esta paciente?
a} Epifisiólisis de cabeza femoral

42. Primigesta de 35 años de edad que se encuentra en la semana 25 de


embarazo. Tiene como antecedentes de importancia un IMC de 30 previo al
embarazo y es de raza negra. No hay antecedentes de diabetes en la familia y ella
niega diabetes mellitus e hipertensión. Usted decide practicarle una prueba de
detección temprana para diabetes gestacional. ¿Cuál es el método más indicado
para detección de diabetes gestacional con una prueba de 2 pasos?
b} Glucemia posterior a carga con 50 g de glucosa

43. Durante un partido de fútbol americano un paciente de 18 años de edad es


lastimado. Posterior aljuego, el estudiante es llevado a la sala de urgencias debido
a que no puede abducir o rotar el brazo izquierdo a nivel del hombro, flexionar el
codo y extender la muñeca. A la exploración física el paciente presenta depresión
del reflejo bicipital del brazo, pero conservación del resto de los reflejos de la
extremidad superior. ¿Cuál es el sitio más probable de la lesión?
d} Raíces nerviosas C5-C6

44. Paciente masculino de 75 años de edad acude a consulta tras haber sufrido un
síncope. Al interrogatorio menciona que en las últimas semanas ha tenido
dificultad para respirar al llevar a cabo actividades que antes efectuaba sin
problemas, y que ello va acompañado de dolor precordial que cede con el reposo.
Es portador de un stent coronario. Entre sus antecedentes está el hecho de que
fumó hasta hace 1O años, padece de hipertensión y es dislipidémico. Sus
medicamentos incluyen aspirina, ramipril,simvastatina, metoprolol. En la
exploración física se registra TA: 150/90 mm Hg; FC: 60 lpm; FR: 20; Temp: 36.5
ºC; pulso rítmico y débil; un soplo sistólico romboidal con intensidad 2/6, que
predomina en el 2do espacio intercostal paraesternal derecho; S4 presente;
enzimas cardiacas normales. ECG: ritmo sinusal,sin datos de isquemia.
44a. La causa más probable del síncope en este paciente es:
c} Estenosis valvular aórtica

44b. El estudio de gabinete ideal para continuar con el análisis del paciente es:
d} Ecocardiografía

44c. Acerca del tratamiento en este paciente:


a) Referir a cirugía para reemplazo valvular

44d. ¿Cuál de los siguientes hallazgos en la exploración orienta más hacia el


diagnóstico de estenosis valvular aórtica?

b} Aumento de laintensidad delsoplo al sentar al paciente e inclinar el tórax hacia


adelante

44e. ¿Cuál de las siguientes afirmaciones es correcta?:


b} El fenómeno dela disociación acústica, que consiste en una mayor propagación
del soplo sistólico aórtico al ápex que al cuello con carácter musical,es
característico dela estenosis aórtica

45. Son ototóxicos no farmacológicos:


c} Mercurio y plomo

46. Paciente masculino de 63 años de edad acude a consulta por diplopía de inicio
súbito. Entre los antecedentes de importancia se encuentra el de diabetes mellitus
tipo 2 desde hace 14 años. A la exploración física muestra deficiencia del tercer
nervio craneal. ¿Cuál de los siguientes hallazgos corresponde a una parálisis del
tercer nervio craneal secundaria a vasculopatía por diabetes mellitus?
c} Conservación de los reflejos pupilares ala luz
47. Paciente masculino de 32 años acude a consulta por visión borrosa, ojo rojo y
sensación de cuerpo extraño en ojo derecho desde hace 5 días. Al llevar a cabo el
interrogatorio por aparatos y sistemas, comenta que le duele la rodilla izquierda y
siente rigidez en ambas rodillas por la mañana desde hace 3 semanas. Niega
antecedentes de importancia. A la exploración física registra FC: 70, FR: 17, TA:
120/75, Temp: 36.5 ºC, conjuntiva derecha bastante eritematosa y edematosa,
rodilla izquierda edematosa, caliente, y ambas rodillas con dolor al movimiento
pasivo y activo. Al revisar genitales usted nota una úlcera de 2 x 1 cm no dolorosa
a la palpación junto al meato urinario con bordes bien definidos.
47a. ¿Cuál es el diagnóstico más probable?
b} Artritis reactiva

47b. ¿Cuál de los siguientes estudios sería de utilidad para diagnosticar al


paciente?
c} PCR urinario para Chlamydia trachomatis

47c. ¿Cuál de los siguientes patógenos no se asocian a la enfermedad del


paciente?
b} Escherichia coli

47e. ¿A qué grupo de trastornos pertenece la enfermedad del caso clínico?


a} Espondiloartropatías seronegativas

48. Acude a consulta una paciente de 15 años con antecedente de síndrome de


ovarios poliquísticos, la cual inicia con un cuadro de faringoamigdalitis desde hace
una semana, diarrea, adenopatías generalizadas y fiebre no cuantificada.
Comenta que acudió a su centro de salud, en donde le recetaron ampicilina.
Posteriormente presentó erupciones cutáneas en todo el cuerpo. A la exploración
física se registran adenopatías cervicales, petequias en paladar,así como
hipertrofia amigdalina grado 2.
48a. ¿Cuál es el diagnóstico más probable?
c} Virus de Epstein-Barr

48b. ¿Qué se observa en el frotis de sangre periférica?


a} Linfocitos atípicos

48c. ¿Cuál de las siguientes opciones sirve para confirmar el diagnóstico?


a} Anticuerpos heterófilos

48e. ¿Qué complicación mortal puede presentar la paciente?


a} Rotura esplénica

49. Se presenta a la sala de urgencias un paciente del sexo femenino de dos


semanas de vida. Su madre menciona que la bebé tiene una coloración púrpura
principalmente en los dedos de pies y manos, y su color corporal es azul al llanto.
Comenta que el neonato presenta dificultad para la alimentación y no ha
aumentado de peso desde el nacimiento. A la exploración física el neonato se
presenta dormido con cianosis leve en la cara y tronco, y cianosis moderada en
las extremidades. A la auscultación es posible determinar un soplo cardiaco.
¿Cuál de las siguientes es la causa más común de cianosis durante las primeras
semanas de vida?
c} Tetralogía de Fallot

50. ¿Cuál de los siguientes ligamentos es el que más frecuentemente se lesiona


en esguinces de tobillo?
a} Ligamento talofibular anterior

51. Femenino de 85 años con antecedente de depresión, acude aUrgencias por


debilidad y mareo. Toma furosemida 20 mg/día para reducir el edema de
miembros inferiores y hace poco se le agregó hidroclorotiazida para el manejo de
la presión arterial. Al interrogatorio niega fiebre, escalofríos, náusea,o vómito;
menciona que a últimas fechas ha tenido mucha sed, por lo que ha ingerido
grandes cantidades de jugo de manzana. A la exploración física la presión arterial
en decúbito es de 100/50 mm Hg, y sentada de 80/44 mm Hg, con un peso de 60
kg. Los estudios de laboratorio revelan osmolalidad sérica de 260 mOsm/kg, sodio
sérico de 125 mEq/L, potasio sérico de 3.4 mEq/L, creatinina sérica de 0.8 mg/dl.
El examen general de orina revela una concentración de sodio urinario de 50
mEq/L, y la osmolalidad urinaria de 200 mOsm/kg.
¿Cuál de las siguientes opciones representa el manejo más adecuado de esta
paciente?
a} Administración intravenosa de solución salina (0.9%) a una velocidad de
125 m /h

52. Se presenta un paciente con incapacidad para mover el ojo derecho en


cualquier dirección, ptosis del párpado derecho, dilatación pupilar derecha y
alteración de la sensibilidad de la piel sobre el maxilar y el frontal. ¿Cuál de los
siguientes diagnósticos es el más probable?
a} Trombosis del seno cavernoso

53. Paciente masculino de 71 años con diagnóstico previo de osteoartritis,


hipertensión leve, y dolor lumbar de reciente aparición. La exploración física revela
presión arterial dentro de rangos normales y conjuntivas pálidas; a la palpación de
L4 y L5 se produce dolor,y en los tobillos se encuentra edema 2+. Los estudios de
laboratorio revelan anemia grave (6.2 g/dL), y creatinina sérica de 5 mg/dL. El
médico tratante decide realizar una biopsia de médula renal, la cual es consistente
con mieloma múltiple; la recolección de orina de 24 horas demuestra 7 g de
albúmina y 1.5 g de cadena ligera kapa. Se realiza una biopsia renal que se
observa bajo el microscopio electrónico, demostrando depósitos granulares a lo
largo de la membrana basal y en los nódulos glomerulares. No es posible
identificar materialfibrilar en la biopsia. ¿Cuál de las siguientes opciones es la
causa más probable de insuficiencia renal en este paciente?
d} Depósito de cadenas ligeras en elriñón
54. ¿Cuál es la complicación más frecuente de la vasectomía?
c} Hematoma

55. Paciente femenino de 60 años acude a urgencias tras sufrir fractura oblicua del
tercio distal de la diáfisis humeral al caer sobre su mano extendida. A la
exploración física destaca inhabilidad para extender la muñeca y las articulaciones
metacarpofalángicas.
¿Cuál es la lesión asociada más probable?
b} Lesión delnervio radial

56. Un paciente masculino de 23 años de edad acude al servicio de urgencias


después de haber mantenido una relación homosexual con penetración anal con
un hombre con infección por el virus de la inmunodeficiencia humana. Durante su
visita al servicio de urgencias se obtiene una muestra de sangre, la cual resulta
negativa para anticuerpos contra el virus de la inmunodeficiencia humana.
¿Cuándo es el menor tiempo posible en que debe citarse de nuevo al paciente
para detectar seroconversión en caso de que éste se encuentre infectado?
c} Seis a 12 semanas

57. Paciente femenino de 74 años ha estado hospitalizada 5 días por una cirugía
de prótesis de cadera que se complicó. Recibe heparina no fraccionada para
prevenir trombosis por la cirugía y el reposo prolongado, además de utilizar
medias compresivas. Refiere que tiene dolor en la pantorrilla izquierda, y a la
exploración física presenta signos de trombosis venosa profunda, lo cual se
confirma al realizarle un ultrasonido. También se le hace una biometría hemática,
de cuyos resultados llaman la atención las plaquetas de 56 OOO/mm3.
57a. ¿Cuál es el paso a seguir con esta paciente?
 b} Suspender la heparina

57b. ¿A partir de cuántas plaquetas se tiene el riesgo de sangrado espontáneo?


c} menor a 20 000

57c. ¿Cuál es el mecanismo de acción de la heparina no fraccionada y cuál es su


principal efecto secundario?
 b} Acelerala acción dela antitrombina III y provoca trombocitopenia

57d. ¿Cuál es el mecanismo de acción de la heparina de bajo peso molecular?


c}Inhibe al factor de coagulación Xa

57e. ¿Cómo se diferencia un sangrado por defecto plaquetario de uno por defecto
de la coagulación?
c) En los defectos plaquetarios hay equimosis y petequias
58. Paciente masculino de cinco años de edad con un trastorno congénito del
metabolismo. A pesar de que ha estado en un ayuno de 12 horas, la muestra de
sangre que se obtiene es de apariencia lechosa y los estudios de laboratorio
revelan hipertrigliceridemia. ¿Cuál de las siguientes opciones representa a la
enzima deficiente en este paciente?
d} La enzima deficiente se sintetiza porlo regular porlas células adiposas y
musculares

59. Paciente masculino de 49 años de edad que ingresa a hospitalización por


disnea progresiva. Entre los antecedentes de importancia se encuentra el de
hipertensión y está bajo tratamiento con inhibidores de la enzima convertidora de
angiotensina. Los antecedentes sociales revelan que el paciente consume cuatro
copas de whisky diariamente, durante los últimos 10 años. A la exploración física
tiene elevación de la presión venosa yugular,estertores bibasales, desplazamiento
lateral del impulso apical cardiaco, un soplo holosistólico grado 2/6 y presenta de
tercer ruido cardiaco. El ecocardiograma demuestra dilatación del ventrículo
izquierdo y una fracción de eyección de 25%, así como insuficiencia mitral
moderada. ¿Cuál de los siguientes enunciados respecto a los efectos del alcohol
en el corazón es incorrecto?
c} Las mujeres son menos susceptibles que los hombres a padecer
miocardiopatía secundaria a alcoholismo

60. Paciente masculino de 10 años de edad que es llevado al Servicio de


Urgencias posteriormente a que ingirió material cáustico. Se inician maniobras de
reanimación y estabilización. ¿Cuál de las siguientes opciones representa una
complicación a largo plazo que pueda sufrir este paciente?
d} Carcinoma esofágico

61. ¿Cuál de los siguientes agentes fibrinolíticos debe formar un complejo con el
plasminógeno para presentar actividad enzimática?
a} Estreptocinasa

62. ¿Cuál de los siguientes medicamentos se caracteriza por presentar


miocardiopatía dilatada como complicación?
b} Doxorrubicina

63. Paciente femenino de 27 años, primigesta, con embarazo de 32 semanas de


gestación. El embarazo no ha tenido complicaciones. Acude a consulta por
presentar un dolor precordial agudo que se exacerba con la inspiración
profunda,así como por falta de aire de una hora de evolución. Niega dolor en
pantorrillas. Sus signos vitales son Temp: 37.9 ºC, TA: 130/80, FC: 110, FR: 24 y
satura a 90%. A la EF se registran estertores basales derechos y signo de
Homans negativo bilateral. Se toma una radiografía de tórax, la cual es normal;en
el análisis de sangre hay dímero D elevado, y en el electrocardiograma se muestra
taquicardia sinusal con desviación del eje a la derecha.
63a. ¿Cuál es el siguiente paso diagnóstico?
d} Gammagrama ventilación/perfusión
63b. ¿Cuál es la tríada de Virchow?
b} Trauma vascular,estasis venosa e hipercoagulabilidad

63c. ¿Cuál de los siguientes no es un criterio modificado de Wells?


d} Electrocardiograma con S2Q3T3

63d. ¿Cuál es la alteración más frecuente que se espera encontrar en el


electrocardiograma?
b} Taquicardia sinusal

63e. ¿Cuál es el tratamiento indicado para la paciente?


d} Heparina

64. Paciente femenino de 28 años de edad que acude a consulta por disnea y
malestar general. Al interrogatorio refiere edema de extremidades inferiores,
ortopnea y disnea paroxística nocturna; menciona que ha tenido fiebre intermitente
y escalofríos. La presión arterial es de 150/60 mm Hg y la frecuencia cardiaca de
99 latidos/minuto. La presión venosa yugular se encuentra elevada. Los pulsos de
las arterias femorales y carótida son saltones con un colapso brusco. A la
auscultación se encuentra un soplo 11/IV sistólico de eyección y un soplo 111/IV
diastólico en decrescendo a lo largo del borde esternal izquierdo que aumenta de
intensidad durante el empuñamiento. Se puede percibir un tercer ruido cardiaco.
Existen estertores bilaterales y edema de las extremidades inferiores. No hay
datos de endocarditis infecciosa. No hay datos que sugieran endocarditis
infecciosa y el electrocardiograma no es diagnóstico. La radiografía de tórax
demuestra cardiomegalia moderada y edema pulmonar intersticial. ¿Cuál sería el
tratamiento inicial más adecuado en esta paciente?

b} Administración de inhibidores de la enzima convertidora de angiotensina y


diuréticos

65. Paciente femenino de 77 años con historia de hipertensión arterial sistémica


en control con Losartán y cirugía de cadera hace más de 2 meses sin
complicaciones, acude a consulta ya que desde hace una semana inició con
cuadro gripal que ha ido empeorando y ahora presenta tos productiva amarilla con
dolor pleurítico izquierdo, malestar general y fiebre no cuantificada. A la EF se
registra TA: 130/80 mm Hg, FC: 98, FR: 26 y Temp: 39 ºC. A la auscultación se
detecta en base derecha aumento del frémito y estertores.
65a. ¿Cuál es el diagnóstico más probable?
c} Neumonía adquirida en la comunidad
65b. ¿Cuál es la causa más probable?
b} S. pneumoniae

65c. ¿Qué estudio serviría para corroborar el origen de la enfermedad?


c} Antígenos urinarios

65d. ¿Cuántos puntos del CURB 65 tiene esta paciente?


a} 1

65e. ¿Cuál es el tratamiento de primera línea?


b) Macrólidos de manera ambulatoria

66. Un niño de cuatro años se presenta con miosis, sialorrea, broncoespasmo,


náusea, vómito, diarrea, incontinencia urinaria y fecal. Habita en el campo y su
padre es agricultor. Se determina que su cuadro clínico es secundario a una
intoxicación. En este contexto, ¿cuál es el tratamiento de elección?"
d} Atropina

67. Paciente masculino de 26 años de edad, empresario, con historia de migraña


sin aura de seis años de evolución, obeso (IMC = 31kg/m2) desde la
adolescencia, fuma 1O cigarros diarios desde hace ocho años y es sedentario.
Acude a consulta por incremento en frecuencia e intensidad de sus cuadros
cefalálgicos sin causa aparente. En la exploración física destaca TA 135/85 mm
Hg, exploración de área cardiovascular y neurológica normal, piel livedoide en
miembros torácicos y pélvicos. Muestra varios estudios realizados de forma
incidental que reportan: glucosa 115 mg/dl,plaquetas 130 mil, colesterol total 230
mg/dl,colesterol LDL 195 mg/dl, ácido úrico de 7.8 mg/dl, anticardiolipinas lgG
positivas de 50 U/mi.
67a. Una asociación sugestiva como detonante del incremento en sus cuadros de
migraña es:
e} Seropositividad para anticardiolipinas lgG

67c. El tratamiento adecuado para esta condición es:


d} Ácido acetilsalicílico

68. Recién nacido de 4 horas de vida, que desde el nacimiento ha presentado


distrés respiratorio intenso con cianosis e hipertensión pulmonar. Se observó
presencia de abdomen excavado y en la auscultación del hemitórax izquierdo se
perciben ruidos intestinales y ausencia de murmullo vesicular en esta región.
Todas las siguientes son características de la patología del paciente, EXCEPTO:

e} El tratamiento es a base de vigilancia puesto que eldefecto se cierra alos


meses de vida

69. La NOM-0004-SSA3-2012 del expediente clínico tiene como propósito


establecer los criterios científicos, éticos, tecnológicos y administrativos
OBLIGATORIOS en la elaboración, integración, uso, manejo, archivo,
conservación, propiedad, titularidad y confidencialidad de este documento; el cual
es de uso obligatorio para todo el personal de salud de los sectores público, social
y privado del Sistema Nacional de Salud. Al respecto debes saber el uso de los
diferentes documentos que lo integran:
69a. Todas las notas médicas y reportes a que se refiere esta norma deben
contener:
a} Nombre completo del paciente, edad, sexo, número de cama o expediente

69b. El documento escrito firmado por el paciente, su representante legal o el


familiar más cercano en vínculo, mediante el cual acepta un procedimiento
quirúrgico con fines diagnósticos o terapéuticos una vez que ha recibido la
información de los riesgos y beneficios para el paciente, es:
d} La carta de consentimientoinformado que puede ser preelaborada

70. ¿Cuál de los siguientes mecanismos de acción corresponde al de la niacina?


e} Disminución dela lipólisis perifércia

71. ¿Cuál de los siguientes hallazgos podría estar presente con mayor
probabilidad en pacientes con lesión de la división inferior del nervio facial?
e} Debilidad para moverla mitadizquierda dellabioinferior

72. Paciente masculino de 32 años bisexual el cual se presenta a consulta por la


presencia de una lesión no dolorosa en la región anal. A la exploración física se
encuentra una lesión de aproximadamente 2 cm de diámetro y de consistencia
dura. El paciente reporta que esta tumoración inició como una pápula y
posteriormente fue creciendo hasta adquirir la apariencia actual. El paciente niega
sensación dolorosa a la digitopresión de esta tumoración. Se toman muestras para
cultivo de la lesión,sin embargo éste es son negativo; por otro lado, las reaginas
plasmáticas rápidas son negativas. ¿Cuál es el tratamiento más adecuado para el
padecimiento de este paciente?
e} Administración de 2.4 millones de unidades de penicilina G intramuscular

73. Acude a consulta un hombre de 55 años por dolor profundo en la región


cervical que se extiende hacia la escápula derecha desde hace aproximadamente
cinco meses. Al interrogatorio el paciente menciona que el dolor inició después de
que sufrió un accidente en su automóvil. A la exploración física presenta
normalidad en fuerza muscular y sensibilidad. A pesar de que el paciente ha
utilizado varios analgésicos, ninguno ha sido de utilidad. ¿Cuál de las siguientes
opciones es el diagnóstico más probable?
b} Enfermedad cervical derecha interapofisiaria

74. Paciente masculino de 39 años de edad acude a consulta por antecedente de


disnea en reposo desde hace cinco días, así como tos no productiva. Se decide
tomar una radiografía de tórax, la cual revela infiltrados difusos bilaterales. La
citometría hemática reporta un valor de conteo leucocitario de 2 000 células/mm3.
Al ver los resultados de laboratorio se interroga al paciente sobre la posibilidad de
infección por el virus de la inmunodeficiencia humana; el paciente acepta, y
menciona que no lo informó desde el principio porque creía que no era necesario
para que atendieran su padecimiento actual. Se realiza una tinción de metenamina
de plata de esputo, el cual revela la presencia de Pneumocystis j iroveci. Debido a
que el paciente presenta alergia a las sulfas, no se recomienda la administración
de trimetoprim sulfametoxazol. ¿Cuál de los siguientes antibióticos es el que se
debería administrar a este paciente?
d} Pentamidina

75. ¿Cuál de los siguientes medicamentos es capaz de precipitar una crisis de


insuficiencia suprarrenal aguda?
c} Ketoconazol

76. ¿Cuál de los siguientes antibióticos es el que con menor frecuencia se ha


asociado al desarrollo de colitis por Clostridium difficile?
a} Piperacilina/tazobactam

77. Según la clasificación de fracturas del platillo tibial de Schatzker, ¿cuál de las
siguientes opciones representa la que involucra sólo el platillo tibial medial?
d} Tipo IV

78. ¿Cuál de las siguientes deficiencias enzimáticas se presenta en la intolerancia


hereditaria a la fructosa?
c} Aldolasa B

79. ¿Cuál de los siguientes síndromes se asocia con la presencia de distrofia


ungueal,alopecia, hipoalbuminemia e hiperpigmentación cutánea?
b} Síndrome de Cronkhite-Canadá

80. Paciente masculino de 55 años de edad con diagnóstico de depresión


mayor,hipertensión y diabetes mellitus tipo 2. Es llevado a la Sala de Urgencias
por sus familiares debido a que muestra somnolencia excesiva desde hace dos
horas, así como respiración ""rara"". En el cuarto del paciente había un contenedor
de tabletas de propranolol vacío. En la sala de urgencias se determina que tiene
una frecuencia respiratoria de 40 respiraciones/min; la enfermera toma una
muestra capilar de sangre que revela niveles de glucosa de 75 mg/dl. ¿Cuál de los
siguientes fármacos es el tratamiento más adecuado en este caso?
c} Glucagon

81. Paciente del sexo femenino de 45 años de edad, la cual recibió un trasplante
renal hace aproximadamente 10 meses. En esta ocasión la paciente se presenta
al servicio de urgencias por dolor torácico tipo pleurítico y tos. A la exploración
física presenta febrícula. Una radiografía de tórax revela un infiltrado en el lóbulo
inferior del pulmón derecho. Se inicia tratamiento con antibióticos de amplio
espectro (vancomicina, ceftriaxona y azitromicina). A pesar del tratamiento, la
paciente no parece presentar mejoría después de tres días. Se decide realizar un
lavado broncoalveolar el cual revela la presencia de organismos teñidos con
mucicarmina. ¿Cuál es el diagnóstico más probable?
a} Infección por Cryptococcus neoformans

82. ¿Cuál de los siguientes mecanismos de enfermedad es el que mejor se


relaciona con la presencia de hemoglobina S en pacientes con anemia
drepanocítica?
c} La mutación ocasionala creación de interacciones hidrofóbicas entre moléculas
de hemoglobina

83. ¿Cuál de las siguientes opciones es la más adecuada en un paciente con


antecedente de sinusitis estacional y con sintomatología típica de sinusitis aguda
de tres días de inicio?
d} Fluticasona nasal

84. Paciente masculino de 55 años de edad con fiebre intensa, pérdida de peso,
infecciones oportunistas, ausencia de adenomegalias, pancitopenia y
esplenomegal ia. ¿Cuál es el diagnóstico más probable en este paciente?
b} Leucemia de células vellosas

85. ¿Cuál de las siguientes bacterias es una causa de meningitis y epiglotitis en


pacientes pediátricos?
c) Haemophilus influenzae

86. Niño de 12 años con antecedente de dermatitis atópica que presenta


exacerbación del cuadro desde hace unos meses. ¿Cuál de los siguientes signos
espera usted encontrar?
c} Pliegue de Dennie-Morgan y signo de Hertoghes

87. ¿Cuál de las siguientes opciones representa la causa más frecuente de


derrame pleural?
e}Insuficiencia cardiaca

88. ¿Cuál de las siguientes fracturas se presenta con reducción aparente del
tamaño de la extremidad inferior afectada y rotación externa?
b} Fractura de cuello del fémur

89. Paciente masculino de 12 años de edad que acude a consulta con su madre
debido a que presenta sacudidas súbitas periódicas. Al parecer las contracciones
musculares ocurren en las mañanas antes de irse a la escuela, y se ven
aumentadas cuando se desvela. El paciente no pierde el estado de alerta durante
estos episodios. Al interrogatorio, la madre comenta que un hermano de ella
presenta episodios similares.
¿Cuál es el fármaco de elección en este paciente?
c} Ácido valproico
90. ¿Cuál de los siguientes hallazgos histopatológicos óseos son característicos
de la osteoporosis?
d} Adelgazamiento trabecular con pocasinterconexiones

91. Paciente masculino de 13 años de edad, el cual es traído a revisión pediátrica


por su madre noruega (lo cual dificulta comunicación adecuada) por presentar
diarrea abundante y en exageración maloliente (principalmente después de los
alimentos) y flatulencia. El paciente presenta resultados anormales en la prueba
de d-xilosa; en una biopsia de yeyuno se encuentra borramiento de las
vellosidades intestinales e infiltración linfocítica en la lámina propia. ¿Con cuál de
las siguientes condiciones se asocia más probablemente el padecimiento de base
de este paciente?
d} Dermatitis herpetiforme

93. ¿Cuál de los siguientes nervios inerva la piel que se encuentra sobre el
músculo deltoides?
a} Nervio axilar

94. Paciente femenino de 35 años de edad la cual inicia tratamiento con altas
dosis de prednisona por diagnóstico de nefritis lúpica grave.
¿Cuál de los siguientes cambios en la biometría hemática son resultado del
tratamiento con glucocorticoides?
a}Incremento en el conteo de neutrófilos en sangre

95. ¿Cuál de los siguientes fármacos inhibe directamente la liberación de insulina


por el páncreas?
a} Agonistas alfa-2 adrenérgicos

96. ¿Cuál de los siguientes resultados de laboratorio es el que se esperaría


encontrar en un paciente dislipidémico en caso de que éste tenga adecuado
apego y respuesta a la colestiramina?
e} Disminución delcolesterol-LDL y leve disminución del nivel de triglicéridos

97. Paciente masculino de 29 años de edad que se presenta a consulta por


haberse detectado una tumoración en el lado derecho de su cuello. Al
interrogatorio el paciente se muestra preocupado pues no intencionadamente ha
perdido alrededor de 8.5 kg en el último mes; entre otros hallazgos realizados
durante el interrogatorio se descubre que el paciente ha padecido sudación
nocturna. Debido al cuadro clínico tan sugestivo, se decide realizar una biopsia de
la adenomegalia cervical,la cual revela presencia de células de Reed-Sternberg.
Durante el resto de la evaluación se descubre que el involucro ganglionar se limita
a la zona del cuello y la axila. ¿Cuál es el estadio en el que se encuentra este"
paciente?
d} IIB
98. Paciente masculino de 40 años de edad, trabajador de la industria de la
construcción. El principal motivo de consulta es por dolor en la región cervical y
debilidad en el miembro torácico izquierdo. A la exploración física muestra
inclinación de su cabeza hacia el lado izquierdo, así como limitación a la flexión y
extensión del cuello debido al dolor intenso que presenta. Ha perdido la
sensibilidad en la porción radial del antebrazo y en el pulgar. El reflejo
braquiorradial se encuentra debilitado, así como los movimientos extensores de la
muñeca. ¿Cuál de los siguientes diagnósticos es el más probable en este
paciente?
a} Compresión de las raíces nerviosas provenientes delsegmento medular C6
dellado izquierdo

99. ¿Cuál de las siguientes drogas presenta un síndrome de abstienencia


caracterizado por ansiedad, insomnio, anorexia, sudación profusa, dilatación
pupilar,piloerección, fiebre, rinorrea, náusea,cólicos estomacales, bostezos
frecuentes?
c} Opioides
1OO. Paciente del sexo femenino de 71 años de edad la cual cae durante la
realización de sus labores domésticas. En la sala de urgencias se diagnostica una
fractura distal de radio. Debido a la fractura y a la edad de la paciente se decide
programar una densitometría ósea, la cual indica un valor T para esta paciente de
-2.8. ¿Cuál es el sitio más frecuente de fracturas asociadas con osteoporosis?
d} Vértebra

101. ¿Cuál de los siguientes estudios se debe solicitar en pacientes con sospecha
de oclusión de la arteria central de la retina y de arteritis de células gigantes,
inclusive antes que se solicite interconsulta al servicio de oftalmología?

c} Velocidad de sedimentación globular

102. Paciente masculino de 31 años que recibe tratamiento quimioterapéutico por


primera ocasión por tumor testicular. Entre los fármacos administrados se
encuentra el cisplatino. ¿Cuál de los siguientes medicamentos podría disminuir la
nefrotoxicidad ocasionada por el cisplatino en este paciente?
a} Amifostine

103. Se trata de paciente femenino de 25 años de edad, sin carga genética


relevante ni antecedentes previos, con diagnóstico de tumoración ovárica
maligna,se encuentra recibiendo el 2° ciclo de quimioterapia, quien es traída por
sus familiares al Servicio de Urgencias por presentar crisis convulsivas. A su
ingreso se administra diazepam 10 mg intravenoso con lo que remite la
sintomatología, la exploración física es normal a excepción de extrasístoles
ventriculares aisladas que no requieren intervención urgente. FC 100 lpm, FR 24
rpm, T 37ºC, PA 110/60 mm Hg, peso 49 kg, talla 1.68m.
103a. El diagnóstico clínico de sospecha que usted integra es:
c} Probable síndrome de lisis tumoral
103b. Para confirmar el diagnóstico y estadificar la severidad, además de las crisis
convulsivas y las arritmias usted necesita saber:
a} Creatinina sérica

103c. Una vez establecido el diagnóstico clínico y bioquímico de síndrome de lisis


tumoral, usted administra el manejo inicial que es:
b} Soluciones cristaloides (salina 0.9%)

103d. Una vez estabilizada a la paciente, remitidos los síntomas y con control de
los niveles de electrolitos, usted decide el egreso, ¿cuál es el tratamiento de
elección para manejo ambulatorio?
d} Rasburicasa

103e. La mejor estrategia de prevención del síndrome de lisis tumoral, previo al


inicio de quimioterapia es:
a} Hiperhidratación

104. Con relación a la gota poliarticular simétrica:


d} Pueden cursar con factor reumatoide positivo a títulos bajos y en mujeres
posmenopáusicas

105. Se trata de una mujer de 38 años de edad, quien cursa con disfagia a sólidos
y pérdida de peso en forma involuntaria desde hace 6 meses, el último mes se
agregó disfagia a líquidos, regurgitación y pirosis. A la exploración física se
encuentran halitosis, datos de pérdida de peso y mala hidratación. FC 95 lpm, FR
18 rpm, temp. 36.5 ºC, PA 120/70 mm Hg, peso 51 kg, talla 1.60 m.
105a. Con los datos obtenidos, usted sospecha de un trastorno motor esofágico
¿cuál es el más probable?"
c} Acalasia

105b. Usted solicita una radiografía con trago de bario en la cual espera encontrar:

b}Imagen de pico de pájaro

105c. Al realizar una manometría esofágica usted espera encontrar:


a} Relajaciónincompleta en deglución

105d. Usted decide iniciar tratamiento médico, ¿cuál es la terapéutica que reporta
mayor eficacia?
d} Dilataciones neumáticas

105e. Esta paciente es candidata a tratamiento quirúrgico de primera línea, ¿cuál


es la técnica quirúrgica de mayor eficacia?
a} Miotomía esofágica laparoscópica
106. ¿En cuál de los siguientes pacientes con diagnóstico de evento vascular
cerebral (EVC) con déficit neurológico agudo está indicado el tratamiento
fibrinolítico?
e} Paciente masculino de 75 años que es traído por su esposa dos horas después
del comienzo de los síntomas

107. Se trata de Eduardo, un masculino de 76 años de edad, con antecedentes de


diabetes mellitus tipo 2 de 15 años de evolución y buen apego a tratamiento;
refiere que desde hace dos meses presenta otalgia derecha intensa, acompañada
de otorrea purulenta e hipoacusia, ya acudió al Centro de Salud en donde le
dieron manejo analgésico y antimicrobiano para la otitis media aguda hace mes y
medio, sin presentar mejoría, además esta última semana se agregó parálisis
facial. A la exploración se encuentra eritema e induración del canal auditivo, con
importante tejido de granulación y zonas maceradas. FC 751pm, FR 18 rpm, temp.
37.5 ºC, PA 130/65 mm Hg, peso 75 kg, talla 1.68 m, IMC"
26.57 kg/m2 .
107a. ¿Cuál es la sospecha diagnóstica más probable?
b} Otitis externa necrosante

107b. ¿Cuál es el nervio craneal que se afecta con mayor frecuencia?


d} VII (séptimo}

107c. ¿Cuál es el patógeno más común en la otitis externa necrosante?


c} Pseudomonas aeruginosa

107d. ¿Cuál de los estudios de imagen es el de primera elección en este caso?


a} Tomografía de cráneo

107e. Usted decide iniciar tratamiento antimicrobiano con uno de los siguientes
esquemas:
a} Ciprofloxacino 400 mg cada 8 horas

108. Paciente femenina de 23 años, acude a consultar tumefacción en cuello. A la


exploración física se evidencia un nódulo tiroideo fijo, sin adenomegalias. El
ultrasonido reporta nódulo tiroideo de 2 cm con microcalcificaciones en su interior.
La gammagrafía con yodo marcado muestra únicamente un nódulo ""frío"" en la
tiroides. Se realiza una BAAF que reporta papilas recubiertas de células atípicas,
calcificaciones en grano de arena (cuerpos de psamoma) y núcleos escindidos
con aspecto de ""Anita la huerfanita""."
108a. Por epidemiología, ¿cuál es el tumor más probable en esta paciente?
d} Carcinoma papilar de tiroides

108b. ¿Cuál es el tratamiento indicado en esta paciente?


b} Cirugía
108c. Señale la estrategia recomendada para el seguimiento de esta paciente
luego de haberle brindado tratamiento:
a} Medición de tiroglobulina

108d. Según los datos mencionados en el caso, el pronóstico para la vida de esta
paciente es:
c} Bueno

109. Paciente femenino de 23 años de edad acude a reumatología proveniente del


servicio de infectología con el diagnóstico de coriorretinitis por citomegalovirus en
base a lgG elevada. La paciente presenta desde hace un mes pérdida de agudeza
visual en gran medida en ojo izquierdo y del mismo tiempo de evolución crisis
convulsivas tonicoclónicas generalizadas; signos vitales tomados por pasante de
enfermería TA 100/60, FC 90, FR 22, temperatura 37. Se identifica disminución de
intensidad de pulsos carotídeos, carotidinea,soplo subclavio izquierdo y ausencia
de pulsos radiales y cubitales.
109a. ¿Cuál es el diagnóstico más probable?
d} Probable vasculitis sistémica

109b. Por el grupo etáreo y las características clínicas. ¿Cuál es el diagnóstico


específico que puede tener la paciente?
a} Arteritis de Takayasu

109c. Durante la evaluación de la paciente en la consulta externa, qué


determinación de parámetro o signo vital está en duda?
d} Presión arterial

109d. ¿Cuál es el estándar de oro para el diagnóstico de la patología que presenta


la paciente?
c} Arteriografía

11O. Se trata de Sofía, de 20 años de edad sin antecedentes, primigesta,cursando


embarazo de 32 semanas, dos controles previos, el útimo hace 2 meses en donde
se identificó infección de vias urinarias, y completó tratamiento antibiótico. Acude
por presentar hace 2 horas cefalea intensa, pulsátil retroocular que provoca
náusea sin llegar al vómito, y escotomas. Al examen físico corazón rítmico, sin
presencia de ruidos sobreañadidos, pulmones con adecuado ruido respiratorio,
abdomen gestante, fondo uterino a 28 cm, no doloroso a la palpación,sin datos de
irritación peritoneal. Extremidades con presencia de edema ++ pretibial. Durante la
consulta presenta crisis convulsiva de tipo tónico clónica de 1 minuto de duración
con relajación de esfínter urinario. FC 100 lpm, FR 18 rpm, temp 37 ºC, PA
180/120 mm Hg, peso 87 kg, talla 1.68 m.
11Oa. ¿Cuál es el diagnóstico de la paciente?
d} Eclampsia

110b. ¿Cuál es el tratamiento definitivo que se debe realizar con la paciente?


a}Interrupcióninmediata del embarazo

11Oc. ¿En qué consiste la dosis de carga del régimen de Pritchard para la
administración de sulfato de magnesio?
a} 4g por vía intravenosa

11Od. Si bien todos los regímenes comentados son válidos para las pacientes que
se encuentran en riesgo de presentar crisis convulsivas, el régimen de Pritchard
es el más utilizado. Es de gran importancia el mantener el control de magnesio
sérico. Además de la prevención de crisis convulsivas, ¿qué otros parámetros
deben evaluar en la paciente?
c} Función hepática y cuenta de plaquetas

11Oe. ¿Qué recomendaciones requiere la paciente posterior al alta hospitalaria?


d} Mantener vigilancia de cifras tensionales por 12 semanas mínimo

111. Paciente de 54 años de edad, recientemente deportado de la zona Este de


Estados Unidos. Acude a consultar por fiebre de varios días de evolución, mialgia,
astenia, adinamia y artralgia migratoria. TA: 120/80, FC: 85 lpm. Se observa en el
muslo eritema circular de 1O cm de diámetro con aclaramiento parcial, borde
externo rojo y centro en diana; el paciente refiere que inició como una pápula roja
que se extendió poco a poco hasta formar la lesión anular actual.
111a. Según las características clínicas, ¿cuál es el diagnóstico más probable?
b} Enfermedad de Lyme

111b. ¿Cuál es el agente causal de la enfermedad?


a) Borrelia burgdorferi

111c. ¿Cuál es el estudio diagnóstico indicado en este paciente?


d} Serología

111d. ¿Cuál es el tratamiento recomendado en este paciente?


b} Doxiciclina

111e. En caso que el paciente presente afectación neurológica por la enfermedad,


¿cuál es el tratamiento recomendado?
a} Ceftriaxona

112. Masculino de 45 años acude a consulta por presentar desde la semana


pasada dolor en la zona testicular derecha, el cual disminuye al levantarse el
testículo; también comenta que desde hace una semana siente molestias al
orinar.A la exploración se encuentra un epidídimo aumentado de tamaño y
doloroso, así como ligera secreción clara transuretral. Temp: 38ºC, TA:135/85mm
Hg.
112a. ¿Cuál es su sospecha diagnóstica?
d} Epididimitis

112b. ¿Cómo se conoce a la maniobra que alivia el dolor al elevar el testículo?


a} Prehn

112c. El principal diagnóstico diferencial es:


b} Torsión testicular

112d. ¿Cuál es el agente etiológico más probable de epididimitis en este paciente?

b} E.coli

112e. Fármaco con probabilidad de ocasionar epididimitis.


d} Amiodarona

113. Hombre de 55 años que acude por referir pérdida súbita de la visión en ojo
derecho al despertarse en la mañana el día de ayer, refiere pérdida de la visión
inferior que se ha mantenido sin cambios, niega sintomatología agregada. Es
hipertenso de ocho años de evolución, controlado con enalapril 1 x 2, refiriendo
buen control. Cursa con dislipidemia controlada con Lipitor,refiriendo buen
control,tabaquismo positivo a razón de 15 cigarrillos/día, niega antecedentes
heredofamiliares de otras enfermedades sistémicas. Es valorado por el servicio de
oftalmología reportando: agudeza visual ojo derecho (OD) 20/400 que no mejora
con refracción y en 01 20/40 refracción de -0.50 -0.75 a 180 mejora a 20/20.
Defecto pupilar aferente (DPA) + en OD y reflejos pupilares normales en 01. Se
realiza prueba de sensibilidad a color ishihara reportando alterado en OD 0/8 y
normal 01 8/8. El segmento anterior está sin alteraciones. En fondo de OD se
observa papila hiperémica con pérdida de los bordes, hemorragias en astilla, con
presencia de brillo foveolar, relación AV 1/3, con cruces AV,sin lesiones en
periferia. 01 con papila naranja, bordes bien definidos, excavación puntiforme,
mácula sin alteraciones. Se envía campos visuales de Goldman donde se observa
un defecto actitudinal OD y en el ojo izquierdo CV completo. Estudios de
laboratorio reportan Hg 15, Hct 46, triglicéridos 300 mg, colesterol 350 mg, HDL 25
mg/dl, VSG normal,su evolución de la AV es tórpida; cuatro semanas después la
AV continúaa 20/400 y en el fondo de ojo se observa palidez de la papila ++ OD.
¿Cuál es el diagnóstico más probable?
c} Neuropatía óptica isquémica anterior

114. Luisa es una paciente femenina de 65 años de edad con antecedente de


hipotiroidismo hace 10 años en buen apego a levotiroxina. Acude a consulta por
presentar desde hace 7 días visión borrosa, fotofobia,cefalea de moderada
intensidad de tipo punzante sin predominio de horario, así como náusea y vómito.
Al interrogatorio dirigido refiere escotomas. Dentro del exámen físico se indentifica
congestión conjuntiva! y dolor a la presión de globos oculares. Resto de la
exploración sin alteraciones. FC 98 lpm, FR 18 rpm, temp 36.8 ºC, PA 130/85 mm
Hg, peso 76 kg, talla 1.56 m.
114a. Con base en el interrogatorio de la paciente, el diagnóstico presuntivo es:

b} Glaucoma

114b. Al momento de abordar al paciente se realiza tonometría ocular. ¿Cómo se


diagnostica glaucoma de ángulo cerrado?
c} Presiónintraocular 50- 80mmHg más celularidad y fiare en cámara anterior

114c. ¿Cuál es el tratamiento para el glaucoma de ángulo cerrado?


b} Acetazolamida

114d. En el cierre angular intermitente se puede observar al examen


oftalmológico:
d} Sinequias anteriores periféricas

114e. La córnea en el cierre angular crónico se encuentra:


b} Transparente

115. Una mujer de 65 años acude a consulta para revisión general anual. Entre
sus antecedentes familiares importantes se encuentran cáncer de próstata en su
padre (a causa del cual falleció) e hipertensión y diabetes mellitus tipo 11 en su
madre. Ella actualmente padece de hipertensión, manejada con amlodipino. En su
historial medico padeció miomatosis uterina, por la que tuvo que someterse a una
histerectomía total a los 48 años. Refiere además tabaquismo de 20
paquetes/año. Su última mamografía fue hace siete meses, que no mostró
anormalidades. Niega cualquier síntoma al interrogatorio por aparatos y sistemas.
Niega pérdida de peso. A la exploración física sus signos vitales incluyen
frecuencia cardiaca de 72/min,frecuencia respiratoria de 18/min, presión arterial de
132/74 mm Hg y temperatura de 36.9 C. Su principal preocupación actual es que
se haya realizado todos los estudios indicados para el diagnóstico temprano y
prevención de cáncer."
115a. En cuanto a estudios de tamizaje de cáncer,¿cuál de los siguientes está
indicado para esta paciente?
d} Colonoscopia

115b. Los siguientes son factores de riesgo para este padecimiento, EXCEPTO:
d} Nuliparidad

115c. ¿Cuál de las siguientes es la presentación más común de cáncer de colon


ascendente?
b} Anemia
116. Le solicitan valoración de recién nacido con las siguientes características:
pequeño para la edad gestacional, exantema diseminado y ausencia de reflejo
rojo.Se menciona dentro de los antecedentes madre con exantema no
especificado al inicio del embarazo. En la exploración física se observa paciente
femenino muy pequeño, lesiones en piel de tipo purpúrico y cataratas en ambos
ojos. En la auscultación se detecta un soplo cardiaco. Usted sospecha una
infección congénita.
116a. ¿Cuál es el origen más probable de la sintomatología dela paciente?
c} Rubéola

116b. ¿Qué método diagnóstico es el más adecuado para confirmar la sospecha


diagnóstica?
a} Cultivo de secreciones faríngeas

116c. ¿Cuál de las siguientes opciones representa el método ideal de prevención


de la infección de rubéola en el embarazo?
c} Pruebas para anticuerpo contra rubéola antes del embarazo

116d. ¿Qué frecuencia de infección y daño al feto se espera en mujeres infectadas


con rubéola durante el primer trimestre?
d} 80%
116e. En un paciente con la misma infección, pero de 4 años, ¿cuál es el
tratamiento de elección?
d} Sintomático

117. Se trata de Juan Carlos, un paciente masculino de 23 años detenido en el


aeropuerto de la Ciudad de Mexico por la Policía Federal por sospecha de
transporte ilícito de sustancias. En el examen con rayos X se evidencia la
presencia de aproximadamente 22 cápsulas en cámara gastrica. Al interrogatorio
el paciente desconoce el contenido de dichas cápsulas. Duarante su estancia con
los peritos presenta de manera subita, cefalea, deterioro neurológico, taquicardia,
midriasis, las pupilas se encuentran midriáticas ,covulsiones y datos de
focalización. FC 130 lpm, FR 27 rpm, temp 35.8 ºC, PA 180/100 mm Hg, peso 85
kg, talla 1.56 m.
117a. Con base en el interrogatorio y los datos clínicos del paciente, el diagnóstico
presuntivo es:
a} intoxicación por cocaína

117b. Con base en lo expuesto en el caso clínico. ¿usted qué tipo de toxíndrome
integra en este paciente
c} Simpaticomimético

117c. ¿Cuál es el abordaje inicial para intoxicación por cocaína?


d} Permeabilidad de vía aérea, monitoreo hemodinámico, hidratación
117d. Una vez estabilizado el paciente se identifica que durante las últimas tres
horas los flujos urinarios han disminuido y en los paraclínicos se encuentra
elevación importante de CK y CK-MB. ¿Cuál es su sospecha diagnóstica?
c} Rabdomiolisis

117e. Una vez controlado el evento agudo en urgencias, ¿cuál es la


recomendación estándar que indicaría a su paciente?
c} Vigilar datos de hipertensión crónica

118. Niño de dos años de edad que es llevado a urgencias por evacuaciones
intermitentes con abundante sangre roja brillante, niega presencia de dolor
abdominal y se encuentra en buen estado general, con signos vitales estables.
¿Cuál de los siguientes estudios diagnósticos solicitaría para confirmar el
diagnóstico del paciente?
a} Gammagrafía con pertecnetato de tecnecio

119. Paciente masculino de 18 años llega a urgencias traído en helicóptero


después de sufrir accidente automovilístico a gran velocidad con desaceleración
rápida. Se encuentra agitado, quejándose de dolor intenso en el tórax. A la
exploración física se observa desviación de la tráquea a la derecha, fractura
esternal, campos pulmonares bien ventilados, ruidos cardiacos rítmicos y presión
arterial de 140/60 mm Hg. La radiografía de tórax muestra ensanchamiento
mediastinal.

¿Cuál es el diagnóstico más probable?


d} Rotura traumática de la aorta

120. Paciente masculino de 60 años con antecedentes de insuficiencia renal


crónica acude a urgencias por presentar debilidad generalizada desde el día de
ayer.También refiere que en los últimos dos días ha tenido cefalea que mejoró con
la ingestión de ibuprofeno. Su electrocardiograma se muestra a continuación.

¿Cuál es el tratamiento adecuado dadas las anormalidades electrolíticas


responsables de los hallazgos electrocardiográficos?
c} Gluconato de calcio

1. ¿Cuál de los siguientes mecanismos de acción corresponde al de la ezetimiba?


c}Inhibición dela absorción de colesterol en elintestino delgado

2. ¿Cuál de los siguientes defectos es ocasionado por ausencia de la formación


del septo aorticopulmonar en forma de espiral en 180º?
e} Transposición de las grandes arterias

4. ¿Cuál de los siguientes fármacos antibióticos ejerce su efecto mediante la


inhibición de la síntesis de peptidoglucano?

e} Vancomicina

3. ¿Cuál de las siguientes condiciones es la forma de presentación más común de


la infección primaria por el virus del herpes simple tipo 1?
c} Gingivoestomatitis y faringitis

5. ¿Cuál de los siguientes fármacos requiere la suspensión de la ingesta de


alimentos ricos en tiramina?
d} Fenelzina

6. ¿Cuál de los siguientes enunciados es el que más adecuadamente representa


las características de la fractura de Smith?
c} Fractura de la porción distal delradio con angulación palmar de la porción distal

7. Paciente masculino de cinco años de edad con retraso mental grave se


presenta al servicio de urgencias. A pesar que se le atendio inmediatamente, el
paciente muere por un infarto cerebral masivo (corroborado por autopsia, la cual
también revela infartos renales antiguos).
¿Cuál de las siguientes sustancias con mayor probabilidad hubiese evitado la
muerte de este paciente?
e} Piridoxina

8. ¿Cuál de los siguientes síndromes se presenta con lesiones melanóticas en


labios, mucosa oraly piel?
c} Síndrome de Peutz-Jeghers

9. Paciente masculino de 77 años de edad que fue desfibrilado debido a la


presencia de taquicardia ventricular inestable. Se le receta amiodarona oral. A
pesar de que el medicamento ha conseguido normalizar la frecuencia y ritmo
cardiacos durante varias semanas, el paciente muestra efectos adversos de la
administración de este medicamento. ¿Cuál de los siguientes hallazgos es más
probable que pueda estar
presente en este caso?
a} Disminución de peso, temblor y dolor torácico
1O. ¿Cuál de las siguientes bacterias ocasiona infecciones recurrentes en el caso
de deficiencia del complejo de ataque de membrana del complemento, es decir,
los factores C5-8?
a} Neisseria meningitidis

11. Una mujer de 72 años de edad acude al departamento de urgencias por


cefalea. Ésta comenzó hace una semana y la describe como intensa (7/10)
pulsante, generalizada y continua,sin acompañarse de cambios en la visión,
debilidad, mareo o parestesias. Lo que sí refiere es que al comer el dolor
empeora. En su historial médico comenta que padece de diabetes mellitus tipo 2,
hipertensión e hiperlipidemia. Niega historia de migraña. Sus medicamentos
incluyen aspirina, metformina, captopril,atorvastati na y ocasionalmente
paracetamol. Refiere que padece dolores musculares ""cuando cambia el tiempo""
y que intermitentemente sufre artralgias en sus manos y rodillas. A la exploración
física se observa una paciente obesa, normocefálica. Sus signos vitales consisten
en frecuencia cardiaca de 89/min, frecuencia respiratoria 17/min, presión arterial
147/96 mm Hg y temperatura 37.6 C. A la auscultación cardiaca se escucha un
soplo mesosistólico con irradiación al segundo espacio intercostal derecho, que
disminuye cuando la paciente aprieta los puños. Los pulsos carotídeos están
ligeramente disminuidos y se ausculta un soplo carotídeo derecho. Los campos
pulmonares se auscultan con murmullo vesicular sin agregados. La exploración
neurológica se encuentra en límites normales excepto por un ligero dolor a la
palpación en la región temporal derecha. En el fondo de ojo se observan
hemorragias y neovascularización. Se ordena la primera serie de laboratorios:

11a. ¿Cuál es el diagnóstico más probable?


b} Arteritis de células gigantes

11b. ¿Cuál es el siguiente paso en el manejo de esta paciente?


e}Iniciar tratamiento con prednisona
11c. ¿Cuáles de estas medidas de prevención se deben aplicar para esta paciente
al iniciar el tratamiento?
d} A y B son correctas

12. ¿Cuál de los siguientes diagnósticos se asocia a hipoglucemia acompañada


de elevación de los niveles intracelulares de NADH en los hepatocitos?
a}Intoxicación alcohólica

13. ¿Cuál de los siguientes tipos de cáncer NO metastatiza al sistema nervioso


central?
e} Ovario

14. ¿Cuál de las siguientes bacterias es la causa más frecuente de meningitis


bacteriana en adultos?
a} Streptococcus pneumoniae

15. Paciente masculino de 27 años de edad acude a consulta por dolor de 72


horas de evolución en el muslo izquierdo. El paciente menciona que el dolor se ha
acompañado de un crecimiento en la región, el cual rápidamente ha aumentado de
tamaño. Al revisar el expediente clínico, se revela que el paciente ha acudido en
tres ocasiones en los últimos cuatro meses por lesiones similares, las cuales
describe como ""picaduras de insectos"" en los glúteos y abdomen. A la
exploración física el paciente se encuentra afebrily normotenso. En la zona
dolorosa tiene una masa fluctuante de 3 cm rodeada de un anillo eritematoso de 4
cm. ¿Cuál de las opciones listadas es la conducta más adecuada a seguir en este
paciente?
b} Realizar una incisión y drenaje, obtener cultivos y prescribir clindamicina oral

16. Paciente masculino de 40 años de edad acude a evaluación de infertilidad. A


pesar de que se encuentra casado por segunda ocasión, jamás ha tenido hijos. Ha
intentado durante dos años relaciones sexuales sin protección con su esposa,
pero no han logrado el embarazo. Los resultados del análisis de semen
demuestran un conteo normal de espermatozoides pero ninguna movilidad en
éstos. Entre los antecedentes de importancia se encuentran infecciones de senos
paranasales y neumonías frecuentes, y en alguna ocasión un médico le dijo que
padecía de bronquiectasias. ¿Cuál de los siguientes hallazgos se verían en una
radiografía de tórax de este paciente en este momento?
d} Situs inversus

17. Paciente masculino de 14 años de edad que cae desde un árbol a 8 m de


altura. A la exploración física se observa la salida de líquido a través del meato
auditivo externo del lado izquierdo. Los estudios de laboratorio indican que se trata
de líquido cefalorraquídeo. ¿Cuál de las siguientes lesiones es más probable que
haya tenido el paciente?
a} Fractura en el piso de la fosa craneal media
18. ¿Cuál es el anticonvulsivante de elección en caso de crisis convulsivas
generalizadas mioclónicas?
c} Ácido valproico

19. Paciente masculino de 22 años de edad acude a la sala de urgencias por


herida con objeto punzocortante en el primer espacio interdigital de la mano
izquierda. Refiere que estaba jugando con un cuchillo con otro individuo y el
evento fue accidental. ¿Cuál de las siguientes estructuras es más probable que
está afectada?
a} Arteria radial

20. ¿Cuál de las siguientes condiciones es una indicación para el uso de


rifampicina como monoterapia?
a} Exposición a Neisseria meningitidis

21. Paciente masculino de tres años de edad que presenta retraso psicomotor. La
madre del paciente menciona que recientemente este paciente ha presentado
actitudes ""extrañas"", tales como lanzarse contra las paredes hasta dañarse, y
morderse los dedos de las manos ocasionandose graves lesiones. ¿Cuál de las
siguientes enzimas es la que se encuentra deficiente con mayor probabilidad en
este paciente?"
a} Hipoxantina-guanina fosforribosil transferas

22. ¿Cuál de los siguientes resultados de laboratorio es el que se esperaría


encontrar en un paciente dislipidémico en
caso de que éste tenga adecuado apego y respuesta a la simvastatina?
c} Disminuciónintensa en los niveles de colesterol-LDL,incremento en el
colesterol-HDL, y disminución moderada de los triglicéridos

23. ¿Cuáles son los hallazgos histopatológicos encontrados en las neumonitis por
hipersensibilidad?
e} Neumonitisintersticial coninfiltración por linfocitos, células plasmáticas y
macrófagos

24. ¿Cuál de los siguientes hallazgos se presenta en caso de lesión del nervio
hipogloso izquierdo?
a} Desviación dela lengua hacialaizquierda durante la protrusión

25. ¿Cuál de las siguientes drogas se caracteriza más frecuentemente por


epistaxis, depresión grave e inducción de acciones suicidas, así como
hipersomnolencia y fatiga posterior a los efectos agudos de su consumo?
a} Cocaína

26. ¿Cuál de los siguientes tumores NO metastatiza al hígado?


e} Cáncer de próstata
27. Paciente pediátrico del sexo femenino que se presenta con dilatación de los
ventrículos laterales por presencia de un astrocitoma que involucra al cerebelo y
comprime al cuarto ventrículo. ¿Cuál de los siguientes diagnósticos es el más
probable?
c} Hidrocefalia no comunicante

28. Se trata de un masculino de 43 años de edad quien realiza una excursión de


alpinismo desde hace 2 semanas, no tiene antecedentes relevantes y tampoco
manifiesta sintomatología respiratoria, durante el viaje se realiza estudios de
control para monitorizar su estado de salud ¿Cuál de los siguientes cambios se
observa después de varios días de estancia a grandes altitudes?
e} Disminución en la reabsorción renal de bicarbonato

29. ¿Cuál de las siguientes condiciones se relaciona con células rojas


fragmentadas de diferentes tamaños y formas, es decir, esquizocitos?
a} Soplo mecánico del segundo ruido cardiaco

30. ¿Cuál de las siguientes características se presenta típicamente en pacientes


con esotropía congénita?
e} Desviación de gran magnitud y alteraciones dela abducción ocular

31. ¿Cuál de los siguientes enunciados describe de mejor forma el mecanismo de


acción del dantroleno?
a}Inhibición dela liberación de calcio por elreceptor de rianodina

32. ¿Cuál de los siguientes cambios neuropatológicos se observa en casos de


deficiencia de vitamina B12?
c} Degeneración axonal delas columnas posteriores y laterales de la médula
espinal

33. Paciente masculino de siete años de edad que se presenta con evidente
inflamación palpebral derecha. Al interrogatorio la madre es incapaz de relacionar
el inicio del cuadro con algún evento específico, sin embargo menciona que
recientemente el paciente sufrió un cuadro de infección de vías aéreas
respiratorias superiores con desarrollo de sinusitis. ¿Cuál de los siguientes
hallazgos se presentaría con mayor probabilidad en este paciente en caso de que
éste presente una celulitis orbital?
d} Proptosis

34. Paciente masculino de 55 años de edad que acude a consulta externa por
presentar fatiga y disnea progresiva, lo cual ocasiona marcada limitación de sus
actividades diarias. El paciente confiesa que requiere varias almohadas para
dormir durante las noches y en ocasiones tiene que levantarse por episodios de
""asfixia"". Al revisar el expediente, se encuentra que sufrió un infarto anterolateral
hace aproximadamente 18 meses, el cual no fue trombolizado. A la exploración
física el paciente presenta estertores bilaterales y edema de miembros inferiores.
La radiografía de tórax muestra cardiomegalia. En este momento su presión
arterial es de 170/105 mm Hg, y se ha mantenido en estos niveles durante las
últimas 12 horas. ¿Cuál de los siguientes agentes es el más adecuado para el
tratamiento de la hipertensión de este paciente?
a} Lisinopril

35. Paciente masculino de dos años seis meses con antecedentes de íleo
meconial al nacimiento, prolapso rectal, repetidos cuadros de bronquiolitis en los
primeros años de vida, diarrea crónica, reflujo gastroesofágico y poliposis nasal,
que acude por presentar desde hace dos días fiebre y tos productiva con esputo
mucoso. Se detectan estertores crepitantes en la exploración pulmonar.
35a. De acuerdo al diagnóstico que sospecha, ¿cuál es el agente etiológico que
desencadenó el cuadro respiratorio actual?
b} P. aeruginosa

35b. De las siguientes pruebas, ¿cuáles le ayudarían a establecer el diagnóstico


de la enfermedad que padece el paciente? 1 = Cantidad de cloro en el sudor,2 =
tripsina y quimotripsina en heces, 3 = pruebas de función pulmonar, 4= diferencia
de potencialtransepitelial nasal,5 = tripsinógeno inmunorreactivo sérico, 6 =
pruebas serológicas.
a} 1, 2 y 4

35c. ¿Qué prueba se realiza para tamizaje neonatal de esta patología?


c} Tripsinógeno inmunorreactivo sérico

36. Toda persona mayor de 15 años que tenga una glucosa plasmática en ayuno
> 126 mg/dl o una glucemia plasmática > 200 mg/dl después de 2 horas de haber
ingerido 75 g de glucosa anhidra, disuelta en 300 ml de agua o tenga > 6.5% de
hemoglobina glucosilada.
36a. Para fines de Vigilancia Epidemiológica la anterior definición se conoce como:
c} Caso confirmado de diabetes mellitus tipo 2

36b. Con base en esta misma definición, en los hospitales: ¿cuál es el porcentaje
de pacientes que debe estar con una hemoglobina glucosilada < 8%?Este es un
indicador de calidad de la atención médica y ¿cuál es otro indicador de eficacia?
b} 65 % con esta HbA1c y TA < de 130/80

37. Paciente femenino de 20 años de edad que acude a consulta por detectarse
durante el baño una masa mamaria derecha, no dolorosa, la cual no se modifica
de tamaño con el ciclo menstrual. Se palpa masa de 2.5 cm de diámetro en mama
derecha no dolorosa, firme, con bordes bien definidos, no adherida a planos
profundos. No se palpan ganglios. ¿Cuál es el diagnóstico?
c} Fibroadenoma
38. Una mujer de 28 años con diagnóstico de VIH acude a Urgencias por cefalea,
fiebre y sudoración nocturna de 2 semanas de evolución. Se realiza una punción
lumbar y se manda a teñir una muestra de LCR en tinta china,en donde se ve una
levadura encapsulada. La paciente refiere que toma trimetoprim/sulfametoxazol.
Su cuenta de CD4 es de 51/mm3, y sus signos vitales son: Temp: 38 ºC, FC:
74/min, FR: 130/80 mm Hg. A la exploración física presenta rigidez nucal.
38a. ¿Cuál es el diagóstico más probable?
b} Meningitis por criptococo

38b. ¿A partir de cuántos linfocitos T CD4 se presenta esta enfermedad en


pacientes con VIH?
c} Menor a 100 células por microlitro

38c. ¿Cuál de las siguientes NO es una indicación para inicio de tratamiento


antirretroviral?
a} Cuenta de linfocitos T CD4 menor a 300 células por microlitro

38d. ¿Qué antirretroviral produce anemia grave por supresión de médula ósea?
d} Zidovudina (AZT}

38e. ¿Cuál es el tratamiento profiláctico para el complejo Mycobacteríum avíum?


a} Azitromicina

39. Paciente masculino de 18 años acude a consulta luego de realizarse un


estudio de VIH en un laboratorio, pues su compañera sexual le comentó que
talvez ella estaba contagiada. Los resultados fueron positivos y lo que él desea es
informarse acerca de este padecimiento, ya que aunque ha investigado bastante
en Internet, todavía tiene algunas dudas.
39a. ¿Qué porcentaje de pacientes presentan el síndrome retroviral agudo?
c} 40 a 90%

39b. ¿Cuál de las siguientes es una indicación para iniciar tratamiento


antirretroviral?
d} Nefropatía asociada a VIH

39c. ¿Cuáles son las cuatro principales enfermedades oportunistas para las que
se debe de indicar profilaxis primaria en caso necesario?
a} Tuberculosis, toxoplasmosis, P. jiroveci y M. avium

39d. El paciente comenta que su novia, la cual padece VIH, está embarazada.
¿Cuándo sufre mayor riesgo el producto de contagiarse sin ningún tratamiento?

b} Durante elparto

39e. ¿Cuál de las siguientes no es una enfermedad definitoria de SIDA?


c} NIC 1
40. Paciente masculino de 11 años es traído por su madre al médico. Ella refiere
que su caso lo han estudiado durante varios meses debido a que presentó varios
episodios de ictericia y anemia, sin embargo no se le ha dado un diagnóstico
certero. Al observar todos sus estudios y realizarle una meticulosa exploración
física, usted encuentra una esplenomegalia marcada.
40a. Usted decide realizar un estudio de fragilidad osmótica, pues sospecha el
siguiente trastorno:
d} Esferocitosis hereditaria

40b. En relación con su sospecha clínica, ¿cuál es la fisiopatología de la


enfermedad?
c} Defecto enlos genes delas proteínas ankirina, b espectrina o banda 3

40c. ¿Cuál es el tratamiento de la enfermedad del paciente en cuestión?


a} Folatos y esplenectomía

40d. La siguiente alteración hematológica presenta cuerpos de Heinz en el frotis:

b} Deficiencia de glucosa 6 fosfato deshidrogenasa

40e. La siguiente alteración hematológica presenta esquistocitos:


a} Anemia hemolítica microangiopática
41. ¿Cuál de las siguientes aseveraciones es correcta respecto del síndrome de
Lowe?
c} Los pacientes con esta enfermedad desarrollan cataratas congénitas bilaterales

42. Lactante de ocho meses con cuadro de episodios diarreicos y estreñimiento de


forma crónica. Todos los siguientes estudios diagnósticos aportarían información
para confirmar la enfermedad que sospecha, EXCEPTO:
a} Ecografía abdominal

43. Paciente masculino de 15 meses de edad que presenta evacuaciones


diarreicas acuosas y copiosas. Una semana antes tuvo gastroenteritis viral que se
resolvió. Todos los siguientes estudios diagnósticos sirven para confirmar el
diagnóstico de la enfermedad que padece, EXCEPTO:
d} Lactato sérico

44. Paciente masculino de 41 años, con obesidad grado 111, diabético de 10 años
de evolución en control con insulina NPH, hipertenso de 5 años de evolución
controlado con captopril 25 mg cada 12 h, es fumador y toma varias cervezas
diariamente. Acude a consulta con su médico general, pues refiere que en los
últimos meses no logra dormir adecuadamente por las noches, presenta
somnolencia durante el día y se ha quedado dormido manejando varias veces. Su
esposa, la cual lo acompaña, comenta que por las noches ronca. A la exploración
física sus signos vitales son FC: 77, FR: 17, TA: 150/70 mm Hg, Temp: 36 ºC.
44a. ¿Qué estudio realizaría para corroborar su diagnóstico?
b} Polisomnografía

44b. ¿Cuál es la definición de apnea?


b} Cese completo dela señal respiratoria de al menos 1O s

44c. ¿Cuál de los siguientes es el principal factor de riesgo para presentar este
padecimiento?
b} Obesidad

44e. ¿Cuál es la principal manifestación cardiaca del trastorno que presenta el


paciente?
d} Hipertensión arterial sistémica

45. Se presenta a consulta un paciente femenino de 70 años de edad. El principal


motivo es que desde hace cerca de 4 horas ha sufrido dolor agudo en el ojo
izquierdo, disminución de la agudeza visualy ojo rojo. A la exploración física se
puede observar una córnea opaca con edema epitelial, disminución del tamaño de
la cámara anterior,una pupila no reactiva de alrededor de 4 mm, presión
intraocular de 59 mm Hg en el ojo derecho y de 16 mm Hg. La gonioscopia del ojo
izquierdo muestra estrechamiento potencial del ángulo en el ojo derecho sin
visibilidad de las estructuras angulares. Al interrogatorio la paciente menciona que
ésta es la primera ocasión que esto le sucede. ¿Cuál de los siguientes hallazgos
es el más probable que también se pueda observar en esta paciente?
a} Hiperemia y edema del nervio óptico

46. ¿Cuál de los siguientes tipos de catarata se asocia con el síndrome de Alport?
e} Catarata polar anterior

47. Paciente masculino de 65 años acude a urgencias por presentar dolor torácico
intenso asociado a fiebre de 38 ºC. El dolor es retrosternal y se irradia al hombro
izquierdo, aumenta al respirar profundamente y mejora al estar sentado. El
paciente sufrió un infarto agudo del miocardio hace un mes por lo que está muy
preocupado y teme que se trate de otro infarto. A la exploración se ausculta un
leve frote. A continuación aparece el electrocardiograma a su ingreso.
¿Cuál es el diagnóstico más probable?
e} Síndrome de Dressler

48. Paciente femenino de 35 años, arquitecta de profesión, acude a consulta


debido a que últimamente en el trabajo le cuesta mucho trabajo concentrarse
porque se siente muy débily cansada. Además, se le dificulta ver bien en la
computadora; tiene la sensación de ver doble, incluso el párpado derecho está un
poco caído. El día de ayer le empezó a costar trabajo tragar, por lo que se asustó
y decidió acudir a consulta. A la exploración física sus signos vitales son normales,
tiene ptosis derecha, pupilas y reflejos normales, y el resto sin alteraciones.
48a. ¿Cuál es la fisiopatología de la enfermedad?
c} Anticuerpos contra el receptor de Ach

48b. Si la paciente llegara a necesitar asistencia ventilatoria, ¿de qué estaríamos


hablando?"
a} Crisis miasténica

48d. ¿Cuál es la probabilidad de que los síntomas de la paciente sean por un


síndrome paraneoplásico?
a} 10%

48e. ¿Cuál es el tratamiento de la crisis miasténica?


e} Plasmaféresis einmunoglobulina IV

49. Paciente femenino de 57 años con historia de trastorno bipolar desde hace 15
años bien controlada con litio, y osteopenia en tratamiento con vitamina D y calcio,
se encuentra en estudio por hipernatriemia. Refiere poliuria y bastante sed. A la
exploración física sus signos vitales son normales y el resto normal. Los estudios
de laboratorio reportan: glucosa 102 mg/dl, Na 148 mEq/L, K 4.2 mEq/L, CI 109
mEq/L, HC03 23 mEq/L,Ca 9.8 mg/dl, osmolaridad plasmática de 302 mOsm/kg y
osmolaridad urinaria 195 mOsm/kg.
49a. ¿Cuál es el diagnóstico más probable?
a} Diabetes insípida

49b. ¿Qué es probable que haya desencadenado la enfermedad?


a} El litio

49c. ¿Qué tratamiento sería apropiado para esta paciente?


b} Tiazida

49e. Las siguientes son causas de hipernatriemia, excepto:


c} Hipocalciemia

50. Paciente femenino de 41 años de edad que acude al servicio de urgencias por
dolor torácico. Menciona que siempre ha tenido buen estado de salud; sin
embargo, desde hace una semana presenta secreción nasal blanquecina, ardor
faríngeo, tos y malestar general. Describe que el dolor inició hace 5 horas y es de
tipo ""desgarrador"", empeora durante la inspiración profunda, y mejora cuando se
inclina hacia delante. A la exploración física presenta una presión arterial de
110/60 mm Hg y una frecuencia cardiaca de 90 latidos/minuto. En la exploración
física se puede encontrar un frote trifásico. La radiografía de tórax no muestra
ninguna alteración. El electrocardiograma muestra elevación del segmento ST en
la mayoría de las derivaciones. ¿Cuál debe ser el siguiente paso en el manejo de
esta paciente?
a} Se debe tratar de forma ambulatoria con tratamiento sintomático

51. Recién nacido de sexo masculino de 27 SDG con un peso de 1 100 g obtenido
vía cesárea,Apgar 6/8, el cual recibió oxígeno suplementario durante cuatro
semanas, presentando síndrome de insuficiencia respiratoria y sepsis neonatal. A
la exploración oftalmológica se encontró ausencia de respuesta a la luz y reflejo
blanco en ambos ojos.
51a. ¿Cuál es el diagnóstico más probable?
d} Retinopatía delprematuro

51b. ¿Cuál es la fisiopatología de este padecimiento?


a}Interrupción dela vasculogénesis de la retina

51c. Los signos clínicos que se encuentran en esta enfermedad son:


c} Neovascularización retiniana, hemorragia vítrea, proliferación fibrovascular y
desprendimiento de retina

51d. ¿Cuáles son los factores de riesgo más importantes relacionados con esta
entidad?
d} Peso al nacer menor de 1250 g, menos de 32 SDG,exposición a grandes
cantidades de oxígeno

51e. El tratamiento de esta enfermedad consiste en:


b} Crioterapia o fotocoagulación y corrección quirúrgica de vítreo y retina
52. Paciente de 55 años de edad, sin antecedentes de importancia, ingresa al
servicio de urgencias por dolor en fosa" renal, náusea y vómito. Se solicitan
estudios de laboratorio, donde se reportan Leu 13, Hb 12, Plt 333, Glu 130, BUN
14, Cr 1.02, Na 138, K 4, examen general de orina con nitritos positivos, leucocitos
positivos y bacterias 3+. Se ingresa a observación para iniciar manejo antibiótico,
pero se reporta con glucemias capilares preprandiales en promedio de 108. ¿Qué
estudio solicitaría de manera inmediata para orientar el diagnóstico hacia diabetes
de reciente diagnóstico?

b} Hemoglobina glucosilada

53. ¿Cuál es la principal alteración hidroelectrolítica en pacientes con


hiperaldosteronismo?
c} Hipopotasemia

54. Paciente de 55 años de edad que en los últimos años manifiesta varias crisis
de vértigo rotatorio, de entre 1 y 3 horas de duración, con náusea y vómito, sudor
frío y plenitud aural;se acompaña de nistagmo y acúfeno en el oído derecho.
Audiometría con hipoacusia de tipo neurosensorial. Resonancia magnética
cerebral con contraste normal.
54a. ¿Cuál es tu diagnóstico presuncional?
c} Enfermedad de Ménire

54b. ¿Qué tratamiento indicarías para esta paciente en una crisis vertiginosa
aguda?

b} Difenidol

55. Un paciente anciano se presenta en el departamento de urgencias con dolor


torácico. El paciente tiene historia de angina estable y reciente inicio de diabetes
mellitus tipo 2. Como medicamentos toma gliburida, aspirina y metoprolol. En esta
ocasión reporta que el dolor ocurre con menos esfuerzo y toma más tiempo en
desaparecer. A la exploración física sus signos vitales son FC 113/min, FR 21/min,
T 37.1 C y PA 133/80 mm Hg. Si el paciente tuviera angina inestable, ¿qué
hallazgos se esperaría encontrar?

b} Depresión del segmento ST y enzimas cardiacas normales

56. Un hombre de 66 años se encuentra hospitalizado por una urgencia


hipertensiva, que manifestó con edema pulmonar. El paciente fue estabilizado y
presentó mejoría del edema pulmonar con el tratamiento. Posteriormente como
segundo agente antihipertensor se inició captopril. El paciente tiene antecedentes
patológicos de aneurisma abdominal,el cual se monitorea cada año con
ultrasonido, enfermedad vascular periférica, diabetes e hiperlipidemia, Sus
medicamentos actuales incluyen metoprolol, metformina, atorvastatina y aspirina.
El día de hoy,sin embargo, el paciente se encuentra con edema periférico (+). Sus
signos vitales son FC 101/min, FR 17/min, T 37.3C y PA 133/87 mm Hg, con
saturación de oxígeno de 92%. A los campos pulmonares se escucha murmullo
vesicular sin agregados. La auscultación cardiaca revela S1 y S2 sin agregados.
En la exploración abdominal se escucha un soplo que coincide con la sístole. No
hay dolor a la palpación y no se sienten organomegal ias. Entre los estudios de
laboratorio que se piden se realiza un perfil bioquímico, que revela lo siguiente:

¿Cuál es el siguiente paso en el manejo de este paciente?


a} Descontinuar captopril

48c. ¿En qué grado de la clasificación de Osserman se encuentra una paciente


con afección solamente ocular?
a} 1

57. Una mujer de 75 años de edad acude a consulta para revisión general. En sus
antecedentes patológicos se refiere únicamente hipertensión, que controla con
hidroclorotiazida y amlodipino. Niega cualquier síntoma al IPAS. Refiere leve
pérdida de peso de 3 kg con referencia al mes pasado. A la exploración física se
palpan adenopatías cervicales, axilares e inguinales. Se realiza una biometría con
frotis, que se muestra a continuación:
¿Cuál sería el tratamiento indicado para esta paciente?
e} No se requiere tratamiento

58. Un hombre de 79 años acude a consulta para revisión general. Los


antecedentes patológicos refieren diabetes mellitus tipo 2, hipertensión,
hiperlipidemia e hipotiroidismo. Al IPAS refiere disminución de la sensibilidad de
las extremidades inferiores. A la exploración física sus signos vitales son FC
76/min, FR 15/min, T 37.2 C y PA 133/65 mm Hg. En la exploración neurológica se
observa disminución en la sensación a la vibración de ambas piernas. Su
hemoglobina glucosilada se encuentra en 8%. El paciente acepta que no ha sido
tan constante con el medicamento, pues le causa flatulencia, dolor abdominal y
diarrea. ¿Qué medicamento está tomando para controlar su diabetes?

b} lnhibidor de alfa-glucosidasa

59. Paciente masculino de 65 años que acude a consulta por reflujo


gastroesofágico. Refiere que ha tenido la enfermedad por alrededor de 15 años,
pero últimamente ha sido difícilde controlar,al sentir pirosis y sabor ácido en la
boca. Sus medicamentos incluyen ranitidina 150 mg dos veces al día y omeprazol
20 mg en la noche, y refiere que los toma diariamente. No tiene otros
antecedentes, refiriendo estar en buen estado de salud. A la exploración física sus
signos vitales incluyen FC 75/min, FR 17/min, T 37.2 C y PA 125/82 mm Hg. No
hay anormalidades a la exploración física. ¿Cuál seria el siguiente paso para el
diagnóstico de este paciente?

d} Realizar endoscopia
60. Paciente masculino de 68 años que acude a urgencias por dolor abdominal. El
dolor se encuentra en el cuadrante inferior izquierdo. En sus antecedentes refiere
hipertensión y diabetes mellitus tipo 2. Al IPAS refiere haber tenido diarrea y
sensación de distensión abdominal los días anteriores, además de hematoquezia
que inició en las últimas horas. A la exploración física sus signos vitales incluyen
FC 11O/min, FR 18/min, T 38.2 C, PA 88/57 mm Hg. En la palpación abdominal se
describe dolor en el cuadrante inferior izquierdo, con signo de rebote positivo. Se
pide biometría hemática que muestra leucocitosis de 19 OOO/mm3. Se inician
fluidos intravenosos, y al medir la presión arterial en una hora el paciente presenta
92/62 mm Hg. ¿Qué tratamiento le daría a este paciente?
c} Cirugía con procedimiento de Hartmann

61. En las vasculitis cuya afección de órganos pone en riesgo la vida. ¿Cuál es el
tratamiento de elección en fases
agudas?

b} Metilprednisolona

62. Paciente de 17 años de edad que se encontraba en un bar clandestino y con


exceso de personas. Al llegar la policía la gente del establecimiento sale
violentamente y durante este evento el paciente cae al piso sufriendo múltiples
traumatismos (craneoencefálico, abdominal y vertebral). Por fortuna el paciente
recibe atención médica y traslado hospitalario oportuno. Tres semanas después
del suceso el neurólogo reporta que el paciente presenta edema y decoloración de
la mitad izquierda de la cara, párpado caído ipsolateral y constricción pupilar del
lado afectado. El resto de la exploración neurológica no revela ninguna otra
anomalía. ¿Cuál es el sitio más probable de lesión en este paciente?

d} Lesión de la médula espinal torácica

63. Un paciente de cinco años es llevado a la sala de urgencias con dificultad


respiratoria y sibilancias. Su madre menciona que recientemente el niño tuvo un
resfriado. El paciente presenta frecuencia cardiaca de 95 latidos/min, presión
arterial de 130/85 mm Hg, frecuencia respiratoria de 18 respiraciones/min y
saturación de oxígeno de 93%.A la exploración física se detecta dificultad
respiratoria, con sibilancias bilaterales. ¿Cuál de las siguientes sustancias
contribuye a la sintomatología respiratoria de este paciente?

b} Leucotrieno C4

64. ¿Cuál de los siguientes periodos de la vida intrauterina es aquél en que el


alcohol presentará su efecto más
teratogénico?
b} 3 a 9 semana

65. Recién nacido producto de un embarazo a término y parto eutócico. Tanto uno
como 5 minutos después del nacimiento, el paciente presenta un puntaje (score)
de Apgar de 9 puntos. Al momento de realizar la exploración física de rutina se
observa fusión de pliegues escrotales y una prominencia parecida a un pene, pero
de menor tamaño. El paciente está hipotenso e hipovolémico. Se solicita un
estudio de cariotipo que revela un genotipo 46,XX. ¿Cuál de las siguientes
deficiencias enzimáticas es la más común?
c} Deficiencia de 21-alfa hidoxilasa

66. ¿Cuál de los siguientes escenarios clínicos corresponde de mejor manera al


cuadro clínico más frecuente de la infección por Babesia microti?

b} Síndrome gripal autolimitado

67. Paciente hospitalizado, con ventilación mecánica con oxígeno a 100% y los
siguientes resultados en una gasometría arterial: pH de 7.4, PaC02 de 39 mm Hg
y Pa02 de 63 mm Hg. ¿Cuál de las siguientes opciones representa con mayor
probabilidad la etiología de la hipoxemia en este paciente?
d} Cortocircuitointrapulmonar

68. Se trata de un paciente masculino de 65 años de edad, con antecedentes de


diabetes tratado con glibenclamida y metformina, hipertensión arterial sistémica en
tratamiento con captopril,ambas en adecuado control trimestral en su centro de
salud, hipertrigliceridemia en control con dieta y ejercicio, tabaquismo suspendido
hace 7 años, fumaba 1O a 12 cigarrillos al día desde los 16 años de edad. Acude
a Urgencias por presentar dolor torácico de características opresivo, 8/1O, con
disnea y diaforesis de 30 minutos de evolución;a la exploración física con
taquicardia y taquipnea, ligera palidez, Killip 1, no soplos ni frémito, resto de la
exploración aparentemente normal. FC 100 lpm, FR 24 rpm, temp. 36ºC, PA
110/70 mm Hg, peso 82 kg, talla 1.65 m.
68a. Con los datos obtenidos hasta el momento, ¿qué factor predisponente otorga
mayor riesgo de cardiopatía isquémica?

b} Tabaquismo

68b. Usted decide ingresar al paciente, solicita un electrocardiograma de


superficie de 12 derivaciones, en el cual usted espera observar:
c} Desnivelpositivo del segmento ST

68c. Usted solicitó biomarcadores para apoyar el diagnóstico, por el tiempo de


evolución ¿cuál se encuentra más elevado?
d} Mioglobina
68d. Según la tercera definición de infarto, el paciente cursa con un evento tipo:
a} Tipo 1
68e. Según la escala de riesgo TIMI para mortalidad a 30 días en pacientes con
infarto agudo de miocardio con elevación del ST, ¿qué tipo de riesgo tiene este
paciente?
a} 4 puntos

69. ¿Cuál es el cuadro clínico más frecuente en pacientes no inmunodeprimidos


con infección por Coccídioides?
e} Sin sintomatología

70. Paciente masculino de 22 años de edad con diagnóstico previo de diabetes


mellitus tipo 1 se presenta con antecedente de cinco días de náusea y vómito, así
como reciente hematemesis. La frecuencia cardiaca es de 130 latidos/min, presión
arterial de 100/60 mm Hg, temperatura de 36 ºC y frecuencia respiratoria de 18
respiraciones/min. Entre los estudios de laboratorio relevantes se encuentra un pH
de 7.40, PaC02 de 29 mm Hg; Pa02 de 164 mm Hg (respirando oxígeno a 3
Umin); sodio 141 mEq/L, potasio 4.1 mEq/L, cloro 83 mEq/L, bicarbonato 22
mEq/L, nitrógeno ureico 89 mg/dL;creatina de 3.6 mg/dL y glucosa de 899 mg/dL.
¿Cuál es el estado ácido-base de este paciente?
d} Acidosis metabólica con alcalosis metabólica

71. Paciente de dos años es llevado al servicio de urgencias, en esta ocasión por
su quinto episodio de dolor abdominal cuya duración ha sido de 8 horas. En las
dos ocasiones previas el paciente vomitó contenido gástrico. Entre los episodios
de dolor abdominal el paciente se observa asintomático. A la exploración física se
detecta temperatura de 38 ºC y un abdomen doloroso a la palpación, sin signo de
rebote. Se había solicitado un coprocultivo y coproparasitoscopia, pero los
resulados son negativos. En esta ocasión la biometría hemática reporta un conteo"
leucocitario de 7 500 células/mm3. ¿Cuál es el diagnóstico más probable en este
paciente?
d} lntususcepción

72. Acude a la sala de urgencias un paciente del sexo femenino de 40 años de


edad debido a visión borrosa, dificultad para la marcha y confusión mental. A la
exploración física la paciente se muestra desnutrida, con estertores bilaterales y
marcha de base amplia. Un familiar de la paciente menciona que ésta ha tenido
problemas de memoria durante los últimos días. ¿Cuál de los siguientes hallazgos
son los que más probablemente se observen en esta paciente?
e} Lesión simétrica dela región paraventricular del tálamo e hipotálamo, cuerpos
mamilares y región periacueductal del mesencéfalo.

73. Paciente del género masculino de 28 años de edad la cual acude a consulta
externa por presentar "crecimientos" en la zona de los genitales. Al interrogatorio
de antecedentes personales o heredofamiliares, no existe información relevante
para este padecimiento. Al interrogatorio sobre los hábitos sexuales, el paciente
indica que inició actividad sexual hace 6 meses y que hasta el momento sólo ha
tenido una sola pareja heterosexual, utilizando condones como método
anticonceptivo y de protección. A la exploración física del área afectada se
observa un pene no circuncidado; en el surco coronal se encuentran varias
pápulas firmes, indoloras, blanquecinas, de aproximadamente 0.1 cm cada una.
Utilizando únicamente la información previamente descrita, responda las
siguientes preguntas:
73a. ¿Cuál de las siguientes opciones es el diagnóstico más probable en este
paciente?
d} Pápulas penianas perladas

73b. ¿Cuál de las siguientes opciones representa el tratamiento más adecuado en


este paciente?
d} Ningún tratamiento

74. ¿Cuál de los siguientes fármacos actúa mediante la inhibición de la síntesis


ergosterol?
a} Azoles

75. ¿Cuál de los siguientes enunciados explica de mejor manera la mayor afinidad
de la hemoglobina fetal por el oxígeno que la hemoglobina de adultos?
c} La hemoglobina fetal tiene una subunidad gamma que se une al 2,3-
difosfoglicerato con menor afinidad que en adultos

76. Paciente en su sexto decenio de la vida que es llevado a la sala de urgencias


por su esposa. El paciente refiere dolor torácico intenso. Desde hace varios años
ha presentado dolor torácico con irradiación hacia la porción distal del brazo
izquierdo posterior a la realización de actividades físicas. El paciente se presenta
en choque con disminución de la presión arterialy flujos arteriales. A la
auscultación el paciente presenta tercer ruido cardiaco y estertores que no
desaparecen con la tos en más de un tercio de los campos pulmonares. ¿Cuál es
la clasificación de Killip de este
paciente?
d} IV

77. Paciente masculino de 61 años de edad con antecedentes de epilepsia es


traído a urgencias por presentar convulsión tónico-clónica generalizada de más de
5 minutos de duración según su esposa quien refiere que desde hace un mes
suspendió la medicación antiepiléptica. Mientras se encuentra en urgencias el
paciente ha presentado otras dos crisis convulsivas sin recuperar el estado de
conciencia. Usted evalúa la vía aérea, administra oxígeno por puntas nasales y
canaliza una vía periférica para obtención de pruebas de laboratorio y
administración de fármacos.
¿Cuál es el medicamento inicial de primera elección para el tratamiento del estado
epiléptico?
b} Lorazepam
78. Paciente del género masculino de 55 años de edad que es llevado al servicio
de urgencias por alteración del estado de alerta. El paciente ya es conocido por el
servicio de Hematología y Oncología pues hace 1O semanas recibió el
diagnóstico de mieloma múltiple. El paciente fue encontrado en un restaurante
local actuando como si éste fuera un banco y hablando incoherentemente. Los
signos vitales demuestran taquicardia, el resto de los signos vitales son normales.
A la inspección general se encuentra un paciente delgado, de edad aparente
mayor a la real,y con"
mala higiene. Existe hiperreflexia evidente. Utilizando únicamente la información
previamente descrita, responda las siguientes preguntas:
78a. ¿Cuál de las siguientes opciones es el tratamiento inmediato más adecuado
en este paciente?
b} Hidratación intravenosa abundante y administración de furosemida

78b. ¿Cuál de los siguientes hallazgos clínicos se asocian con la condición que
ocasionó la alteración al estado de alerta de este paciente?
a} Constipación,poliuria, estupor,coma, insuficiencia renal,extrasístoles
ventriculares y ritmo idioventricular (exacerbado por la administración de
digitálicos}

78c. ¿Cuál de las siguientes hallazgos radiográficos se encontrarían con mayor


probabilidad en este paciente?
c} Lesiones en sacabocados

78d. ¿Cuál de las siguientes opciones es la mejor forma de monitorear la


respuesta altratamiento del mieloma múltiple?
a} Medición delpico de paraproteínas enla electroforesis sérica proteica

78e. Si este paciente fuera considerado candidato para transplante autólogo de


células troncoales, ¿cuál de los siguientes medicamentos se debería evitar?
a} Melfalan

79. ¿Cuáles de las siguientes regiones gastrointestinales se encarga de la


absorción del hierro y calcio ingerido en la dieta?
d} Duodeno

80. Paciente femenino de 22 años de edad se diagnosticó con un trastorno


hemorrágico desde la adolescencia. Entre los principales síntomas, la paciente
informa hemorragias al cepillado de los dientes, episodios de menorragia y anemia
grave. El mismo cuadro presentaron la madre y abuela de esta paciente. Los
estudios de laboratorio revelan un conteo plaquetario de 250 OOO/mm3,
incremento en el tiempo de sangrado y el tiempo parcial de tromboplastina, pero
con normalidad del tiempo de protrombina. ¿Cuál es el diagnóstico más probable
en esta paciente?
c} Enfermedad de von Willebrand
81. Después de participar en una riña durante una fiesta, un paciente de 20 años
de edad es llevado a la sala de urgencias por sus amigos. A la exploración física
se presenta con diaforesis, taquicardia, epistaxis, hipertensión, midriasis,
hiperactividad, confusión y desorientación. El paciente se mantiene en
observación y al día siguiente de su ingreso presenta sueño y apetito excesivo.
¿Cuál es el tipo de droga que consumió este paciente?
c} Cocaína
82. Paciente del sexo femenino de 39 años de edad la cual es llevada por su
esposo al servicio de urgencias por presentar mareo, vértigo, confusión, fatiga,
palpitaciones y diaforesis intensa. La paciente reporta que ya había presentado
episodios similares en el pasado, pero que esta ocasión fueron demasiado
intensos por lo que le solicitó a su esposo la llevara a un hospital. La
sintomatología inició dos horas posteriores a la ingesta del desayuno. El esposo,
el cual es diabético insulinodependiente, menciona que él ha tenido cuadros
similares cuando se ha inyectado insulina sin consumir alimentos; sin embargo, la
esposa no tiene antecedente de diabetes hasta ahora. A la exploración física la
paciente presenta una frecuencia cardiaca de 140 latidos/minuto, así como
temblor de las regiones distales de las extremidades. La enfermera toma una
muestra de sangre capilar para analizar en el glucómetro, cuyo resultado es de 40
mg/dl. ¿Cuál de las siguientes sustancias en sangre puede ser de utilidad para
discriminar el origen de esta sintomatología como una enfermedad endógena, o
secundaria a un trastorno facticio?
a} Medición delpéptido C en sangre

83. Los resultados de laboratorio de un paciente revelan incremento en el tiempo


de trombina, con normalidad en el tiempo de tromboplasti na parcial y el tiempo de
sangrado y conteo plaquetario dentro de límites normales ¿Cuál de los siguientes
diagnósticos es el más probable?
b} Deficiencia de vitamina K

84. ¿A qué edad desaparece normalmente la respuesta dorsiflexora del reflejo de


Babinski?
a} Después de 12 meses de edad

85. Paciente masculino de 21 años de edad, acude a consulta en su centro de


salud por presentar tos, disnea, fiebre durante las últimas 2 semanas, malestar
general y diaforesis nocturna. Al interrogatorio niega cualquier antecedente, a la
exploración se encuentran adenomegalias en la cadena cervical anterior derecha,
estrías y se observa que la ropa le queda grande. FC 65 lpm, FR 16 rpm, temp.
37.6 ºC, PA 110/70 mm Hg, peso 60 kg, talla 1.70 m.
85a. Con los datos previos, usted sospecha de alguno de los siguientes
diagnósticos:
b} Linfoma de Hodgkin

85b. Usted decide realizar un estudio para obtener el diagnóstico confirmatorio,


por lo que realiza uno de los siguientes:
c} Biopsia de excisión de ganglio
85c. ¿Cuáles son los marcadores histopatológicos positivos en el linfoma de
Hodgkin clásico?
d} CD15 y CD30

85d. ¿Cuál es el estudio de extensión que decide realizar para clasificar la


invasión?
d} Tomografía por emisión de positrones

85e. ¿Cómo agrupa a los pacientes según el pronóstico?


c} Favorable y desfavorable

86. Femenina de 62 años, consulta por aumento de volumen en cuello. Al


interrogatorio refiere insomnio, pérdida leve de peso y palpitaciones. TA: 130/80,
FC: 88 lpm. ECG sin alteraciones. Se palpa nódulo único doloroso en lóbulo
tiroideo izquierdo, desplazable, sin adenomegal ias. Perfil tiroideo: TSH
disminuida, T4 ligeramente aumentada.
86a. ¿Cuál de las siguientes es la prueba más útil para el diagnóstico de este
trastorno?
c} Gammagrafía

86b. Al realizarle una gammagrafía en esta paciente, ¿qué hallazgos esperaría


encontrar?
b} Captación focal en elnódulo hiperfuncional y disminución de la captación en el
resto dela glándula

86c. ¿Cuál es el tratamiento indicado en esta paciente?


a} Yodo radiactivo

86d. Acerca de los nódulos tiroideos solitarios, indique el hallazgo que sugiere
benignidad.
d} Nódulo ""caliente"" en la gammagrafía

86e. Señale el enunciado incorrecto:


c} La elevación de hormonas tiroideas libres y una TSH suprimida es compatible
conla resisitencia a la hormona tiroidea

87. Paciente femenino de 30 años de edad secundigesta de 38 semanas de


gestación con rotura prematura de membranas de 8 horas de evolución, sin datos
de trabajo de parto ni de corioamnionitis, con un Bishop de 7. Producto en
presentación cefálica,situación longitudinal,sin datos de sufrimiento fetal. ¿Cuál es
la mejor conducta obstétrica para esta paciente?
a}Inducción delparto con el uso de oxitocina
88. Neonato de 48 horas de vida, que se encuentra con taquicardia, taquipnea,
fiebre, leucocitosis; la radiografía de tórax muestra infiltrado reticulonodular con
presencia de broncograma aéreo. Se sospecha de sepsis neonatal. ¿Qué
antibióticos iniciaría para su manejo?
a} Ampicilina y gentamicina

89. Usted se encuentra en un parto y está iniciando el tercer periodo del trabajo de
parto, nota que la porción placentaria que se observa primero es la fetal. ¿Qué
mecanismo de separación placentaria sucedió?
a} Schultze

90. Acude a consulta paciente femenino de 27 años con embarazo de 32 semanas


de gestación. Como antecedente de importancia tuvo preeclampsia en el
embarazo previo. Se encuentra normotensa, sin datos de vasoespasmo y la orina
de 24 horas sin alteraciones. ¿Qué complicación es la principal causa de muerte
en pacientes con preeclampsia/eclampsia?
e} Hemorragia cerebral

91. Alejandra es una paciente de 57 años de edad con antecedente de diabetes


mellitus tipo 2 diagnosticada hace 3 años, apendicectomía previa hace 25 años sin
complicaciones. Acude al servicio de urgencias por presentar hace 30 horas
posterior a ingesta de alimentos grasos dolor abdominal de tipo cólico, intensidad
7/10, que se inicia en región subcostal derecha con irradiación hacia espacio
interescapular. Se acompaña de inapetencia, náusea sin llegar al vómito,
diaforesis. Niega alza térmica u otra sintomatologia. Al examen físico abdomen
distendido, doloroso a la palpación profunda en epigastrio e hipocondrio derecho,
sin rebote, peristalsis presente. Puntos ureterales no dolorosos, giordano negativo.
Extremidades simétricas, pulsos distales presentes, sin edema. FC 92 lpm, FR 18
rpm, temp. 37 ºC, PA 130/85 mm Hg, peso 97 kg, talla 1.68 m.
91a. Su diagnóstico presuntivo es:
b} Colecistitis aguda

91b. Señale el signo radiográfico que no corresponde con un cuadro de colecistitis


aguda:
c} Separación del hígado
91c. Ya que hemos realizado el diagnóstico de un cuadro de colecistitis aguda se
debe iniciar el manejo del paciente buscando control del evento de dolor. ¿Cuál es
el analgésico inicial en este paciente?
d} AINE

91d. La complicación más importante de la colecistectomía laparoscópica es:


a} Lesión de la vía biliar

91e. ¿Cuál es el porcentaje de efectividad del ácido urodesoxicólico en litiasis


biliar?
b} 10%

92. Paciente masculino de 45 años de edad, recientemente deportado de la zona


Este de Estados Unidos. Inicia con malestar general, astenia, adinamia, mialgia,
artralgia y fiebre de 40 ºC. A la exploración se registra TA: 11O/ 70, FC: 120 lpm,
esplenomegalia, ictericia leve. Laboratorios: Hb: 10.5 g/dl, VCM: 88, Hb
corspuscular media: 32, LDH: 600 Ul/L, BD: 0.3 mg/dl, 81: 2.7 mg/dl, glucosa: 75
mg/dl, creatinina sérica: 0.8 mg/dl. Se realiza frotis de sangre periférica y se
observa en eritrocitos una ""Cruz de Malta".
92a. Según la clínica y los hallazgos presentados, ¿cuál es el diagnóstico?
b} Babesiosis

92b. ¿Cuál es el vector del agente que ocasiona la enfermedad de este paciente?
d} lxodes scapularis

92c. ¿Cuál es el tratamiento indicado en este paciente?


a} Atovacuona + azitromicina

92d. Menciona cuál es la clasificación taxonómica correcta del agente causal de la


enfermedad de este paciente.
c} Protozoario

92e. ¿Cuál de las siguientes enfermedades es causada por un protozoario?


b} Leishmaniasis

93. Masculino de 40 años de edad consulta por dolor al eyacular, el cual se


presenta desde hace 2 semanas, junto con molestia en el área perineal.El
paciente ha notado que tiene secreción clara uretral;niega urgencia, tenesmo y
prurito, ytiene historia de varias infecciones de las vías urinarias. La exploración
física es normal, se corrobora la secreción uretral. Temp: 36.5ºC.
93a. ¿Cuál sería su diagnóstico para este paciente?
b} Prostatitis

93b. Se le realiza un EGO al paciente, el cual se reporta estéril, ¿cuálsería su


conducta?
c} Prueba delos 4 vasos

93c. Agentes etiológicos más frecuentes.


c} E.coli, Klebsiella y Pseudomona aeruginosa

94. Se trata de Ramiro, un paciente masculino de 35 años de edad, quien desde


hace 3 días presenta dificultad para la deambulación por debilidad en ambas
extremidades inferiores; él niega todo antecedente crónico patológico y no
patológico, solo recuerda un cuadro diarréico hace 3 semanas referido como
intoxicación alimentaria. Usted encuentra que la fuerza distal de ambas piernas es
2/5 y los reflejos tendinosos de Aquiles y patelar están abolidos, el resto de la
exploración es normal. FC 75 lpm, FR 18 rpm, temp 36.5 ºC, PA 110/70 mm Hg,
peso 67 kg, talla 1.68 m.
94a. Con los datos obtenidos usted sospecha en:
d} Síndrome de Guilain - Barré

94b. De las manifestaciones clínicas características de este síndrome, ¿cuál es la


aseveración correcta?
b} 45 a 75 % de los casos involucra nervios craneales

94c. Usted realiza una punción lumbar para confirmar el diagnóstico y elimiar los
diferenciales, encontrando:
a} Hiperproteinorraquia

94d. ¿Qué manejo farmacológico inicia inmediatamente?


b} lnmunoglobulina

94e. En relación al pronóstico de los pacientes con síndrome de Guillain Barré,


usted sabe que:
a} La mortalidad va de 5 a 1O %
95. En un paciente sin ninguna alteración neurológica se irriga el meato auditivo
externo izquierdo con agua fría. ¿Cuál es el hallazgo que más probablemente se
encuentre?
a} Nistagmo con componente rápido haciala derecha

96. Paciente masculino de 25 años de edad, estudiante, sin antecedentes


patológicos. Consulta por cefalea recurrente; refiere que dicho padecimiento le
sucede a diario, por un tiempo aproximado de 2 h, y se caracteriza por ser intenso,
unilateral en la región frontal,con predominio por las noches, y se acompaña de
epífora e hiperemia ocular. TA: 110/70 mm Hg, FC: 75 x', FR: 14 x', Temp: 36.5
ºC. Exploración neurológica sin alteraciones. No ha consumido medicamentos y se
presenta sin laboratorios.
96a. Por la semiología del dolor,usted hace el diagnóstico presuntivo de:
d} Cefalea histamínica de Horton

96b. ¿Cuál de los siguientes enunciados es cierto respecto a la cefalea


histamínica?:
a} Es más frecuente en hombres
96c. ¿Cuál es el tratamiento para disminuir la sintomatología en este paciente?
a} Oxígeno al 100%

96d. En relación con la fisiopatología de la cefalea histamínica, señale la


respuesta correcta:
d} Existe una activación autónoma parasimpática

97. Paciente femenino de 18 años de edad acude a consulta por prurito en ambos
pabellones auriculares; refiere haber iniciado con la sintomatología desde hace un
mes. A la exploración física se encuentra dermatosis localizada en pabellones
auriculares, con el lóbulo afectado de forma bilateral y simétrica; dicha afectación
se caracteriza por placas eccematosas, con base eritematosa. La paciente refiere
haber cambiado de aretes hace un mes y desconoce si son de oro.
97a. ¿Qué elemento es el agente causal de la dermatosis en esta paciente?
c} Níquel

97b. La paciente refiere haber presentado un caso similar en la infancia. ¿Qué tipo
de dermatitis de contacto es la ocasionada por el níquel?
c} Por sensibilización

97c. El diagnóstico es principalmente clínico, pero si requiriera una prueba


diagnóstica, ¿cuál indicaría a la paciente?
b} Pruebas epicutáneas

98. Paciente de 79 años de edad con antecedente de hipertensión arterial de 12


años de diagnóstico, angina estable y diabetes, acude al servicio de urgencias por
disnea de inicio súbito, sin dolor torácico. El paciente presenta edema periférico
desde hace varias semanas, así como ortopnea,dolor en el cuadrante superior
derecho y fatiga progresiva. La exploración física revela presión arterial de 160/60
mm Hg, frecuencia cardiaca de 90 latidos/min, frecuencia respiratoria de 26
respiraciones/min y temperatura de 37 ºC. El paciente se presenta taquipneico,
pulsos reducidos en volumen e intensidad, con una presión venosa yugular de 12
cm H20 y estertores en ambos campos pulmonares. A la exploración cardiaca, el
corazón se encuentra desplazado lateralmente, con presencia de tercer y cuarto
ruidos cardiacos. A la palpación abdominal,el paciente muestra dolor leve en
hipocondrio derecho con hepatomegalia. Las extremidades se presentan con
edema desde los pies hasta las rodillas (3+). Los estudios de laboratorio son
relevantes por creatinina de 1.6 mg/dL, nitrógeno ureico en sangre de 24 mg/dL,
troponina 1 > 0.05 mg/dl. El electrocardiograma muestra ritmo sinusal con cambios
inespecíficos del segmento ST.
98a. ¿Cuál de los siguientes enunciados es el correcto en relación con la
fisiopatología de la insuficiencia cardiaca?
e} Es resultado de disminución de la pendiente presión/volumen telediastólica.

98b. En este paciente se decide administrar un medicamento que incremente el


volumen latido, el gasto cardiaco y la contractilidad. ¿Cuál de los siguientes
medicamentos tiene esta función?
b} lnhibidores de la fosfodiesterasa

98c. ¿Cuál es el significado del tercer ruido cardiaco en este paciente?


b} Volumen elevado en elventrículoizquierdo durante la fase temprana dela
diástole

99. ¿Cuál de los siguientes pacientes con diagnóstico de neumonía adquirida en la


comunidad puede tratarse de manera ambulatoria?
b} Paciente femenino de 50 años, diabética con neuropatía y glucemia de 280
mg/dl

1OO. Se trata de Marco, masculino de 13 años de edad sin antecedentes


relevantes, desde hace varias semanas se queja de dolor en la pierna izquierda,
ha presentado fiebre no cuantificada de forma esporádica sin predominio de
horario ni asociación a otros síntomas, es llevado a consulta por la persistencia del
dolor de la pierna. Al interrogatorio niega antecedentes de trauma o cirugías, no
tiene otra sintomatología; la palpación del tercio distal de la tibia revela aumento
de volumen, de consistencia sólida,que respeta el maleolo medial del tobillo, sin
otros datos anormales. FC 75 lpm, FR 18 rpm, temp 37 ºC, PA 110/65 mm Hg,
peso 46 kg, talla 1.53 m.
1OOa. Con los datos clínicos usted sospecha en:
c} Sarcoma de Ewing

1OOb. ¿Cuál es el estudio de imagen de primera elección que solicita a este


paciente?
a} Radiografía simple
1OOc. Después del protocolo diagnóstico, determina que no hay lesiones por
metástasis y sólo se trata de una lesión localizada; ¿cuál es el tratamiento de
primera elección para este paciente?
a} Quimioterapia

1OOd. ¿Cuál es el sitio de mayor frecuencia de metástasis?


d} Pulmones

1OOe. En relación con la recurrencia de la enfermedad, ¿cuál de los siguientes


enunciados es correcto?
a) La mayoría ocurren enlos primeros 2 años

101. Paciente masculino de 32 años con antecedente de síndrome de Wolff-


Parkinson-White, se presenta a consulta por un ""exantema"" en la cara y dolor
articular episódico, migratorio, que comenzó hace 2 meses. Reporta también
edema y dolor en rodillas desde hace 2 semanas, y fiebre no cuantificada en tres
ocasiones. Niega rigidez. Toma medicamentos para prevenir la fibrilación
auricular. No cuenta con antecedentes hereditarios de enfermedades
reumatológicas. A la exploración física tiene Temp: 38.5 ºC, FC: 90, TA: 120/74
mm Hg, y muestra eritema en cara bilateral,rodillas edematosas y calientes con
dolor al movimiento activo y pasivo.
101a. ¿Cuál de los siguientes es el diagnóstico más probable?
c} Síndrome similar a lupus (lupus-/ike}

101c. ¿Qué autoanticuerpos son los más específicos en esta enfermedad?


d} Anticuerpos antihistona

101d. ¿Cuál es el tratamiento de elección?


b} Dejar de tomar el medicamento

101e. ¿En cuánto tiempo revierte la enfermedad?


b} 4 a 6 semanas

102. Varón de 60 años de edad con dermatosis que afecta la cara plantar del 1er
dedo, caracterizada por una úlcera de 5 x 3 cm, irregular, bien delimitada, en cuyo
lecho se observa 80% de fibrina, 10% de granulación y 10% de necrosis, además
de edema de la extremidad afectada, y un halo de 2 cm de eritema alrededor de la
lesión. El paciente es diabético tipo 2 de larga evolución con control
irregular,asimismo tiene HTA controlada. Al momento de retirarse el zapato para la
curación se observa un cuerpo extraño justo en la zona donde asienta el primer
dedo. Refiere que estos son los zapatos que usa por lo regular.
102a. El diagnóstico más probable es úlcera:
d} Del diabético

102b. La causa más probable de esta herida es:


c} Neuropática
103. Se trata de masculino de 65 años quien falleció a las 02:30 a.m. durante tu
guardia después de un estado de coma de 36 horas y anuria, ambos datos por
efectos de insuficiencia renal crónica de 5 años debida a diabetes mellitus tipo 2
de 23 años de evolución. El paciente tenía hipertensión y obesidad, y cursaba con
infecciones urinarias de repetición.
103a. ¿Cuál fue la causa básica de muerte?
c} Diabetes mellitus tipo 2

103b. ¿Cuál es el intervalo aproximado desde el inicio de la causa básica de la


defunción?
d} 23 años

103c. ¿Cuál fue la causa contribuyente?


c} Hipertensión arterial

104. Femenino de 22 años acude al hospital por presentar cefalea hemicraneana


izquierda de 48 h de evolución,que se incrementa con la actividad física, de
intensidad 7/1O en la escala visual análoga, de tipo pulsátil,acompañada de
fotofobia, fonofobia, náusea y vómito. Quince minutos previos a la aparición de la
cefalea, la paciente presentaba parestesias en el miembro torácico izquierdo y
hemicara ipsilateral, además de teicopsias. La cefalea no la despertaba y
disminuía durante el sueño. Antecedente: refiere varios episodios en el transcurso
del año.
Exploración física neurológica normal. Refiere sólo tratamiento con paracetamol
500mg VO cada 8 horas.
104a. El diagnóstico más probable en esta paciente es:
a} Migraña con aura

104b. Tratamiento de elección para esta paciente:


d} Uso de triptanos

104c. Complicación potencial de esta entidad:


b}Infarto del lóbulo occipital

105. Una paciente de 29 años de edad gesta 4, partos 3, abortos O, cesáreas O,


acude a consulta de seguimiento obstétrico. Durante el interrogatorio la paciente
describe que ha padecido sangrado gastrointestinal y dolor,ambos se acentúan al
momento de defecar. ¿Cuál es el diagnóstico más probable en esta paciente?
b} Hemorroides externas

106. Durante varios años, una paciente posmenopáusica ha presentado acúfenos


(zumbido de oídos, tinnitus) y náusea en ocasiones. Recientemente, la paciente
ha notado disminución de la audición en su oído izquierdo, dificultad para cerrar el
párpado superior izquierdo y no presenta reflejo corneal del mismo lado. También
refiere pérdida de la sensibilidad en la mitad izquierda de la cara. Cuando ingiere
líquidos, éstos se le escapan por la boca, principalmente del lado izquierdo. ¿Cuál
es el diagnóstico más probable en esta paciente?
a} Neuroma acústico

107. Paciente masculino de 30 años de edad con diagnóstico de infección por el


virus de la inmunodeficiencia humana,el cual ha sido manejado con zidovudina y
lamivudina en el pasado. A su esquema se le añade el fármaco indinavir con el
objetivo de reducir la carga viral. También se encuentra bajo tratamiento
profiláctico con trimetoprim-sulfametoxazol. Después de 12 semanas de iniciar el
tratamiento con indinavir,desarrolla náusea sin vómito, anorexia y hematuria
macroscópica. El examen general de orina revela piuria y hematuria. El sedimento
urinario es representativo por depósito de cristales. El nitrógeno ureico y la
creatinina se encuentran elevados, 54 y 2.4 mg/dl, respectivamente. El médico
tratante decide retirar el indinavir y administrar solución salina. ¿Cuál de los
siguientes diagnósticos es el más correcto en este paciente?
b} Insuficiencia renal aguda asociada a la formación de cristales de indinavir

108. ¿Cuál de las siguientes opciones terapéuticas es la más adecuada en un


paciente al que se le encuentra un riñón en herradura durante una cirugía de
reparación de aneurisma en aorta abdominal?
d} Manejo retroperitoneal e implantación de arterias renales anómalas
109. Paciente femenino de 20 años es llevada a urgencias posterior a sufrir
atropellamiento por un automóvil. La paciente presenta dolor extremo en la pierna
izquierda con deformidad, equimosis y excoriaciones. A la exploración física se
observa pierna izquierda edematizada y tensa con dolor a la extensión pasiva de
los dedos y disminución de la fuerza en comparación con la pierna contraria, se
palpan pulsos tibial posterior y pedio en ambas extremidades. La radiografía
confirma una fractura de la tibia izquierda.
¿Cuál de las siguientes aseveraciones es falsa con respecto a la condición de la
paciente?
e} Debe realizarse fasciotomía una vez que se haya confirmado eldiagnóstico

11O. Paciente femenino de 33 años de edad refiere pérdida de 1O kg en los


últimos 30 días, debilidad generalizada y náusea ocasional. No existen
antecedentes de importancia que ayuden a realizar el diagnóstico. A la exploración
física muestra presión arterial de 98/65 mm Hg en posición supina,y 70/55 mm Hg
cuando se encuentra de pie. Es evidente que presenta hiperpigmentación cutánea.
Los estudios de laboratorio revelan hiponatremia, hiperpotasemia y acidosis
metabólica. ¿Cuál de los siguientes diagnósticos es el más correcto en esta
paciente?
a} Enfermedad de Addison

111. ¿Cuál de los siguientes tumores malignos contiene, por lo menos en 85% de
los casos, una translocación entre los cromosomas 11 y 22?
c} Sarcoma de Ewing

112. Paciente masculino de 73 años de edad es llevado al servicio de urgencias


por presentar náusea, vómito, distensión abdominal, y estreñimiento durante las
últimas 72 horas. A la exploración física se encuentra letárgico con una frecuencia
cardiaca de 102 latidos/min. La radiografía de abdomen demuestra gas en el árbol
biliar y datos en relación a obstrucción del intestino delgado. ¿Cuál de las
siguientes opciones representan el tratamiento más adecuado para este paciente?
e} Exploración quirúrgica con enterotomía para liberar la obstrucción

113. ¿Cuál de los siguientes medicamentos se caracteriza por presentar


miopericarditis aguda como complicación?
d} Ciclofosfamida

114. ¿Cuál de los siguientes esquemas profilácticos es el más adecuado para un


paciente al que se le ha implantado con anterioridad un desfibrilador y se
someterá a una laminectomía?
a} No se requiere ningún tipo de profilaxis antibiótica

115. ¿Cuál de las siguientes oraciones respecto a la etiología de la estenosis


aórtica es el más correcto?
c} La válvula aórtica bivalva esla malformación congénita más común encontrada
en adultos

116. Una niña de tres años con antecedentes de irritabilidad, náusea y vómito de
varias semanas de evolución, inicia con un cuadro de convulsiones y entra en
coma. En su familia hay contacto con plomo. Si se atribuyen las manifestaciones a
una intoxicación por este metal, indique cuál es el agente quelante útil para su
manejo.
d} Dimercaprol

117. El síndrome de Sneddon se refiere a:


a} Coexistencia entre livedo reticularis y evento vascular cerebral isquémico

118. Hombre de 60 años de edad con lesiones ampollosas herpetiformes con


costra central que confluyen para formar placas anulares. Las lesiones tienen un
mes de evolución y ha recibido tratamiento con aciclovir,corticoesteroides tópicos
potentes y homeopatía sin respuesta. Se realiza el diagnóstico de probable
dermatosis lineal por lgA.

118a. Es correcto con respecto a esta entidad:


d} El split con NaCI es útil en el diagnóstico para diferenciar de otras
enfermedades ampollosas

118b. Es correcto con respecto a la variante de dermatosis lineal por IgA asociada
a fármacos:

d)La vancomicina es el fármaco por lo común más asociado

1. Recién nacido de término, con antecedente de distrés respiratorio después del


nacimiento, resultó sin complicaciones. A los 13 días de vida inicia con distensión
abdominal importante, intolerancia a los alimentos con vómito y evacuaciones
hemorrágicas abundantes. Se solicita una radiografía abdominal, en la que se
encuentra neumatosis intestinal.
1a. En relación con el diagnóstico que sospecha, todas las siguientes condiciones
se asocian a esta patología, EXCEPTO una, indique cuál:
e} Fibrosis quística

1b. Se decide iniciar manejo médico. Todas las opciones que se indican son
correctas EXCEPTO una, indique cuál:
b} Alimentación entera!

1c. ¿Cuál de las siguientes situaciones justificaría el manejo quirúrgico de este


paciente?
b} Neumoperitoneo

1d. Posterior a manejo quirúrgico el paciente egresa y continúa sin complicaciones


los primeros meses de vida. A los cuatro meses, inicia con distensión abdominal y
vómitos persistentes. De acuerdo con los antecedentes, ¿qué complicación puede
esperarse en este paciente?
a} Estenosis colónica
2. ¿Cuál de los siguientes enunciados respecto de la galactosemia es correcto?
b} La galactosemia es elresultado dela deficiencia de galactocinasa o galactosa-
1-fosfato uridiltransferasa

3. Un recién nacido de sexo masculino, con antecedente de síndrome de Down,


presenta datos de obstrucción intestinal al nacimiento, con imagen de dientes de
sierra a nivel de sigmoides en estudio baritado. Respecto a este caso clínico,
señale la opción FALSA.
c} La enterocolitis esinfrecuente como complicación de esta enfermedad

4. Niña de un año ocho meses de edad con antecedente de gastroenteritis viral


hace dos semanas, resuelta sin complicaciones, que inició hace dos días con
flatulencia y evacuaciones diarreicas acuosas y abundantes. Se encuentra en el
examen físico con aumento de los ruidos peristálticos y eritema perianal. En
relación con la enfermedad que presenta, ¿cuál es el tratamiento indicado?

e} Retirar alimentos con lactosa de la dieta

5. Paciente masculino de 46 años acude a consulta por haber presentado esputo


hemoptoico que se autolimitó en cuatro días. Dentro de sus antecedentes destaca
el diagnóstico de tuberculosis pulmonar a los 26 años que se trató de manera
correcta,quedando lesión cavitaria residual de pared fina.
En la tuberculosis pulmonar la radiografía de tórax es útil porque:

b} Complementa el diagnóstico de tuberculosis


6. Paciente masculino de 45 años con antecedentes de hipertensión arterial en
tratamiento con captopril, hidroclorotiazida y nifedipina acude a urgencias tras una
semana de no haber tomado ninguno de sus medicamentos por razones
económicas. El paciente refiere que desde hace tres días ha tenido cefalea
progresiva, fosfenos, náusea e irritabilidad. A la exploración física se encuentra
agitado y desorientado, hipertenso con presión arterial de 210/130 mm Hg y el
examen de fondo de ojo revela papiledema y hemorragias, no hay déficit
neurológicos focales.
¿Cuál de las siguientes aseveraciones es falsa con respecto al tratamiento de la
emergencia hipertensiva?
b} El diazóxido cruzala barrera hematoencefálica ejerciendo un efecto directo
sobre la circulación cerebral

7. Una paciente de 32 años con antecedente de lupus eritematoso sistémico


acude a su ginecólogo por abortos espontáneos recurrentes. Ha tenido tres
embarazos, los cuales terminaron en aborto antes de la semana 1O de gestación.
Los padres no tienen ninguna alteración cromosómica y a la madre se le realizó un
histerosalpingograma y USG vaginal,que resultaron normales, además de cultivos
vaginales que fueron negativos. Presenta un VDRL positivo.
7a. ¿Cuál de los siguientes estudios hubiera servido para sospechar el riesgo de
abortos espontáneos?
a} Anticoagulante lúpico

7b. ¿Cuál de las siguientes manifestaciones clínicas es la más común en el


síndrome de anticuerpos antifosfolipídicos (SAAF)?
b} Trombosis venosa profunda

7c. En cuanto a las manifestaciones cardiacas en el SAAF, ¿cuál es la más


común?"
a} Lesiones valvulares

7d. ¿Cuál es el tratamiento en pacientes con SAAF asintomático?


a} Sintratamiento o aspirina a dosis bajas

7e. ¿Cuál de los siguientes no es un criterio de SAAF catastrófico?


b} Confirmación de trombosis venosa recurrente

8. Se agrega ampicilina altratamiento empírico de una meningitis cuando se


sospecha que el agente causal puede ser:
b} Listeria monocytogenes

9. El siguiente es un método adecuado para el diagnóstico de la fiebre tifoidea:


b} Mielocultivo enla segunda

1O. Paciente masculino de 55 años de edad que sufrió un infarto con elevación del
segmento ST tratado mediante fibrinolíticos y heparina de bajo peso molecular.
Después del tratamiento menciona que el dolor torácico ha desaparecido, y el
electrocardiograma muestra resolución de la elevación del segmento ST.Tres días
después del infarto inicia con dolor torácico grave, descrito como ""desgarrante"".
¿Cuál de las siguientes opciones explica el cuadro del paciente a tres días del
infarto?
e} Todas las opciones anteriores pueden ocasionar el cuadro que ahora muestra
el paciente

11. Paciente femenino de 56 años con antecedente de tabaquismo (cuatro


cigarrillos diarios desde hace 15 años) y bronquitis crónica sin tratamiento
específico, inicia hace 24 h con fiebre de 39 ºC, escalofríos, cefalea, mialgias,
artralgias, tos y ardor faríngeo. Actualmente es invierno, por lo que usted
sospecha influenza estacional."
11a. ¿A qué familia pertenece el virus de la influenza?
b} Orthomyxoviridae

11b. ¿Cuál es la complicación más importante de la infección por influenza?


a} Neumonía

11c. ¿Cuál es la prueba más específica y sensible para el diagnóstico?


c} PCR en tiempo real

11d. ¿Cuál de los siguientes medicamentos está contraindicado en pacientes con


influenza menores de 18 años?
c} Aspirina

11e. ¿Qué tratamiento le administraría a la paciente del caso clínico?


c} Oseltamivir

12. Es el mecanismo fisiopatológico por medio del cual el hipercortisolismo induce


hipertensión arterial:
a} Aumento de volumen circulante por la retención de Na

13. Un hombre de 91 años que acude a urgencias por disnea de dos días de
evolución. En su historia clínica se refiere hipertensión, hiperlipidemias, hiperplasia
prostática benigna y enfermedad coronaria, habiéndose sometido a un bypass
coronario. Refiere historia de tabaquismo de 12 paquetes/año, habiendo dejado de
fumar hace 30 años. Sus signos vitales incluyen FC 98/min, FR 23/min, T 36.7 C y
PA de 140/88 mm Hg. A la exploración física se observa distensión venosa
yugular, hepatomegalia y edema 3+ de extremidades inferiores. Los campos
pulmonares se auscultan con murmullo vesicular. El electrocardiograma muestra
datos de hipertrofia ventricular izquierda. Se realiza un ecocardiograma, el cual
muestra una fracción de eyección de 60% con distensión auricular izquierda e
hipertrofia ventricular. Se realiza una radiografía de tórax que muestra datos de
atrapamiento de aire. No se observa enfermedad valvular. ¿Cuál de las siguientes
patologías es la causa de estos síntomas?
a} Falla cardiaca diastólica
14. Una mujer de 72 años de edad acude al departamento de urgencias por
disnea y palpitaciones. En sus antecedentes patológicos se refiere EPOC,
hipertensión, hiperlipidemia y enfermedad coronaria, habiendo sufrido un infarto
hace seis años tratado con stent. Sus medicamentos incluyen bromuro de
ipatropio, metoprolol, captopril,simvastatina y oxígeno en casa, el cual mantiene a
3 Umin. A la exploración física sus signos vitales muestran pulso rápido e irregular,
FR 31/min,T 38.2 C, PA 143/88 mm Hg y saturación de oxígeno de 82%. A la
auscultación pulmonar se escuchan roncantes espiratorios. Se realiza un
electrocardiograma, el cual se presenta a continuación:
¿Cuál es el diagnóstico más probable de esta paciente?
e} Taquicardia multifocal paroxística

15. Un hombre de 73 años acude a consulta por hemoptisis de dos meses de


evolución. Refiere además que ha perdido 8 kg de peso, sin cambiar sus hábitos
alimentarios. Refiere tabaquismo de 40 paquetes/año. En sus antecedentes
patológicos refiere hipertensión y osteoartritis, tratadas con amlodipino y
paracetamol en caso de dolor.A la exploración física sus signos vitales son FC
81/min, FR 18, T 37.2 C y PA 133/82. La auscultación cardiaca revela S1 y S2 sin
agregados. No hay distensión venosa yugular.A la auscultación pulmonar se
escuchan estertores crepitantes en el pulmón inferior derecho. Se realiza placa de
tórax, que revela una masa de 3.5 cm. No se palpa hepatomegal ia o
esplenomegalia. Se solicitan estudios de laboratorio, que se muestran a
continuación:
¿Cuál de las siguientes sería la histología más probable de esta paciente?
c} Carcinoma de células pequeñas

16. Femenino de 24 años, cursa con embarazo de 37 semanas atendida en sala


de urgencias por presentar pérdida del estado de alerta posterior a crisis
convulsivas tónico-clónicas. Antecedentes: G-2,signos vitales con TA 170/120mm
Hg, Fe 95x', reflejos osteotendinosos aumentados, FCF de 132x' y edema
importante de miembros inferiores.
La entidad patológica causal de la sintomatología de esta paciente es:
a} Eclampsia

17. Paciente femenino de 6 años de edad que presenta lesiones dérmicas,


máculas y pápulas, que evolucionan a ampollas en horas. La madre informa que
su hija padeció infección de vías respiratorias superiores hace 2 semanas.
¿Cuál es el diagnóstico en este caso?
c} Varicela

18. Paciente femenino de 28 años de edad con neumonía atípica por Mycop/asma
pneumoniae. Al momento más agudo de su enfermedad la paciente presenta
hemoglobina de 1O mg/dL y elevación de los niveles de lactato deshidrogenasa.
La paciente se trata con macrólidos y regresa a consulta tres meses después. Los
estudios de laboratorio de control demuestran ausencia de la anemia o de la
elevación de lactato deshidrogenasa. ¿Cuál es la mejor explicación a este
fenómeno?
c} Normalización de la respuesta inmune

19. ¿Cuál de los siguientes estudios de laboratorio corresponde a un paciente con


intoxicación por vitamina D?
b} Calcio elevado, fosfato elevado, fosfatasa alcalina normal
20. ¿Cuál de las siguientes condiciones se asocia con elevación de la presión
capilar pulmonar de enclavamiento?

b}Insuficiencia ventricularizquierda con hipertensión pulmonar secundaria

21. Paciente masculino de 80 años de edad que acude a consulta externa llevado
por su hija. El paciente enviudó hace cuatro años, y desde entonces vive con su
hija. Refiere que ha presentado frecuencia y urgencia urinarias, así como nocturia
desde la muerte de su esposa, pero en los últimos meses el problema ha
empeorado. El paciente refiere que presenta dificultad para iniciar la micción.
Comunica que la próxima semana acudirá con el ortopedista porque desde hace
varias semanas ha presentado dolor lumbar que no mejora con reposo. El
paciente también refiere pérdida de peso en los últimos cuatro meses. ¿Cuál es el
medicamento más adecuado para el padecimiento principal de este paciente?
d} Flutamida

22. ¿Cuál de los siguientes eventos es el menos probable que se presente durante
un ataque agudo de asma?
b} La presión pleural durante lainspiración se vuelve más negativa

23. ¿Cuál de los siguientes vasos venosos se encarga de conducir sangre de la


circulación porta en el sistema cava a pesar de la obstrucción de la vena porta (p.
ej.,durante cirrosis hepática)?
e} Venas ácigos y hemiácigos

24. Los pacientes con cáncer de colon no polipósico hereditario presentan genes
con inestabilidad de microsatélites, lo que significa que contienen múltiples
regiones con fragmentos de DNA de cadena única y de estructura anormal. ¿Cuál
de las siguientes es el tipo de proteína anormal en estos pacientes?
a} Enzimas de reparación de apareamiento incorrecto

25. Paciente masculino de 15 años de edad el cual sufre una caída desde su
patineta. A la exploración física presenta una lesión sobre la porción lateral de la
rodilla, la cual permite observar parte de la cabeza del peroné. Se solicita una
radiografía de la región la cual es negativa para fractura. Cuando el paciente trata
de levantarse de la silla de ruedas en la que estuvo todo este tiempo es notable la
presencia de ""pie caído"" al intentar caminar.A la exploración intencionada tiene
incapacidad para la dorsiflexión o eversión del pie. ¿Cuál de las siguientes
estructuras es la que más probablemente se encuentra lesionada?
b} Nervio peroneo común

26. Paciente masculino de 45 años de edad con antecedente de infección por el


virus de la inmunodeficiencia humana se presenta con eritema de 12 mm e
induración de 7 mm a la prueba cutánea de la tubercul ina utilizando 5 unidades
de PPD. La radiografía torácica no muestra anormalidades. El paciente se
encuentra bajo tratamiento antirretroviral altamente activo y no existe
sintomatología. ¿Cuál de las siguientes aseveraciones representa la conducta más
adecuada?
c}Iniciar tratamiento con isoniazida por 12 meses

27. ¿Cuál de las siguientes condiciones es un factor protector para el desarrollo de


neumonía por Legionella?
c} Neutropenia

28. Paciente masculino de 29 años de edad sufre un accidente por colisión de su


motocicleta contra un automóvil. El paciente se presenta con fractura de la diáfisis
derecha de fémur, sin trauma craneoencefálico ni pérdida del estado de alerta
debido a que utilizaba casco protector; 24 horas posteriores a su admisión el
paciente se presenta con infiltrados pulmonares bilaterales y difusos. ¿Cuál es la
etiología más probable de los infiltrados pulmonares?
c} Embolia grasa

29. Paciente femenino de 78 años de edad con diagnóstico de diabetes mellitus


tipo 2 desde hace 22 años. El paciente carece de apego al tratamiento, y debido a
que le han comentado que la insulina es muy ""dañina"", ha preferido continuar
esporádicamente con tratamiento a base de bebidas de medicina alternativa y
metformina. Desde hace varios meses el paciente notó la presencia de una úlcera
sobre la cabeza del tercer metatarso del lado derecho. La úlcera ha crecido, y
recientemente el paciente inició con dolor en la zona afectada por lo que acude a
consulta. A la exploración física se encuentra una úlcera de aproximadamente 4
cm de diámetro, con olor y secreción pútrida. Al parecer la úlcera tiene una
profundidad de 3 cm y afecta estructuras óseas. Se toma una muestra de la base
y se envía a tinción de Gram la cual reporta cocos grampositivos en cadenas,
bacilos gramnegativos, diplococos gramnegativos, gramnegativos pleomórficos y
exceso de neutrófilos. ¿En este momento, cuál es el tratamiento más adecuado en
lo que es posible obtener mientras se esperan los resultados de hemocultivos y
cultivos de la secreción de la herida?"
e} Ampicilina con sulbactam

31. ¿Cuál es el principal efecto secundario de la administración de zidovudina en


pacientes con infección del virus de la inmunodeficiencia humana?
d} Neutropenia

32. Paciente de ocho años de edad acude a la consulta pediátrica oftalmológica


debido a problemas de visión. A la exploración ocular, se revela una subluxación
del cristalino. Entre otros hallazgos el paciente presenta extremidades largas y
disminución generalizada del tono muscular. Los padres también mencionan que
su hijo presentó retraso en el desarrollo con leve retraso mental. Los estudios de
laboratorio indican elevación de los niveles de homocisteína en sangre y orina.
¿Cuál de las siguientes condiciones es la responsable de los signos y síntomas de
este paciente?
a} Deficiencia de cistationina sintasa
33. Paciente masculino de 54 años de edad que acude al departamento de
urgencias por dolor torácico de tipo ""desgarrante"" que inició hace 1 hora. El
paciente refiere que el dolor tiene irradiación hacia la región interescapular,y que
sufrió un episodio sincopal de 2 minutos que coincidió con el inicio del dolor
torácico. En el expediente clínico menciona que es conocido por padecer
síndrome de Marfan y dilatación de la raíz aórtica. En sus dos consultas de
seguimiento previas muestra cifras tensiónales elevadas (155/100 y 160/99 mm
Hg). A la exploración física presenta presión arterial de 180/95 mm Hg en la
extremidad escapular derecha y de 100/60 mm Hg en el miembro escapular
izquierdo, así como taquicardia de 105 latidos/minuto; la temperatura y frecuencia
respiratoria se encuentran dentro de los rangos de la normalidad. A la auscultación
cardíaca se puede percibir un soplo diastólico 11/IV en el borde esternal inferior
izquierdo. El ecocardiograma demuestra disección de la aorta torácica con
involucro de la aorta torácica ascendente y descendente. Utilizando únicamente la
información previamente descrita, responda las siguientes preguntas:
33a. ¿Cuál de los siguientes fármacos es el más adecuado para este paciente en
este momento?
a} Labetalol

33b. ¿Cuál de las siguientes opciones corresponde al tipo de de disección aórtica


de este paciente?
a} DeBakey tipo 1
33c. ¿Cuál de las siguientes opciones es el tratamiento definitivo más adecuado
para este paciente?
a} Cirugía urgente.

34. Ante el caso de un recién nacido postérmino, con presencia de líquido


amniótico con meconio, que manifestó apnea y cianosis al nacimiento, ¿qué
situación de las siguientes justificaría el uso de adrenalina para reanimarlo?
d} Frecuencia cardiaca menor a 60latidos por minuto posterior a ventilación con
presión positiva y compresiones torácicas externas

35. Lactante masculino de diez meses es traído a urgencias por su madre por
presentar diarrea acuosa de dos días de evolución asociada a sed intensa e
irritabilidad. Se encuentra taquipneico, taquicárdico, con los ojos hundidos, llanto
sin lágrimas, lengua seca,saliva espesa, signo del lienzo húmedo y llenado capilar
de 4 segundos.
¿Cuál es el tratamiento adecuado para tratar este caso de deshidratación?
e} Administrar Vida Suero Oral 100 ml por kilogramo de peso, cada 30 minutos
durante cuatro horas enla unidad de salud"

36. Paciente masculino de 55 años de edad que es ingresado al servicio de


Cardiología por presentar endocarditis infecciosa. En los cultivos se ha identificado
cepas de Staphylococcus aureus; los resultados del antibiograma se encuentran
pendientes. El paciente refiere antecedente de choque anafiláctico a la
administración de penicilina. Utilizando únicamente la información previamente
descrita, responda las siguientes preguntas:
36a. ¿Cuál de los siguientes tratamientos es el más adecuado para este paciente
en este momento?
a} Administración de vancomicina intravenosamente

36b. Posterior a la conducta realizada en la respuesta anterior el paciente


presenta eritema en varias partes del cuerpo, pero particularmente en el cuello, sin
otro tipo de sintomatología. ¿Cuál es la conducta terapéutica más adecuada en
este momento?
b} Disminuirla velocidad deinfusión delmedicamento

36c. Después de tratamiento adecuado, los síntomas del paciente se resuelven y


es dado de alta. Seis meses después, el paciente acude a consulta y pregunta si
es necesario que tome profilaxis antibiótica pues se someterá a una intervención
dental. ¿Cuál de las siguientes respuestas es la más adecuada para este
paciente?
a} Sí debe someterse a profilaxis previa al procedimiento dental,pues claramente
existe una indicación por sus antecedentes.

36d. En caso de que sea necesario utilizar profilaxis contra endocarditis infecciosa
previamente al procedimiento dental ¿Cuál de las siguientes
opciones es la más adecuada para este paciente?
b} Clindamicina
37. Paciente femenino de 40 años de edad con diagnóstico de embarazo de 14
semanas de gestación. Como antecedentes de importancia tiene los siguientes: 2
embarazos, en los cuales no hubo complicaciones, los dos fueron partos. Los
productos sin alteraciones. Acude por estar preocupada, ya que sabe que los
embarazos a edades mayores se relacionan con alteraciones en los niños. ¿En
qué momentos de la meiosis se realizan las dos detenciones en la ovogénesis?
b} Diplotena y metafase

38. Te encuentras explorando a una paciente embarazada con 33 semanas de


gestación,con fondo uterino de 26 cm desde sínfisis de pubis, situación transversa
y decides auscultar foco fetal. ¿A partir de qué semana de gestación se puede
percibir el latido fetal a través del Pinard?
d} 20 semanas de gestación

39. Neonato de 12 días de vida, que fue dado de alta en condiciones estables, y a
partir del décimo día de vida presentó episodios de fiebre, con temperaturas entre
38 y 39.7 ºC, hiporreactividad, rechazo al alimento, ¡DATOS DE MENINGITIS! De
los siguientes agentes etiológicos: 1)Staphylococcus epidermidis, 2)
Staphylococcus aureus, 3) L. monocytogenes, 4) H. influenzae. ¿Cuáles dos
agentes se relacionan más con sepsis tardía?"
e} 1 y 2

41. Llega a urgencias paciente femenino de 38 años de edad primigesta, por


haber presentado crisis tonicoclónica generalizada hace 30 minutos. Refiere el
marido que previamente había tenido una cefalea muy intensa, fosfenos y dolor en
el epigastrio. Se encuentra en la semana 24 de gestación. TA de 180/11O mm Hg,
estuporosa, en anasarca, ROT incrementados. ¿Qué medicamento
anticonvulsivante es el de
elección?
d} Sulfato de magnesio

42. Acude a consulta una mujer con amenorrea secundaria, que no menstrúa con
progesterona, pero sí con la combinación de estrógeno y progesterona,con TSH y
prolactina normales, pero con FSH y LH disminuidas y prueba de embarazo
negativa. Con los datos previos. ¿Qué patología podría tener esta paciente?"
c} Tumor hipofisiario

43. Paciente masculino de 4 años de edad ingresa al departamento de Urgencias


por presentar accesos de tos y cianosis con periodos de apnea. Los padres
mencionan que previo dicho cuadro el niño se encontraba perfectamente bien. A la
exploración física se encuentra paciente masculino muy ansioso, con paroxismos
de tos, cianosis peribucal, babeo y estridor inspiratorio. Se aspiran secreciones
abundantes y se oxigena adecuadamente.
43a. Con base en las características del paciente, ¿cuál es el diagnóstico más
probable?
d} Cuerpo extraño

43b. De acuerdo con su sospecha clínica,¿cuál es la causa de este


padecimiento?"
c} Cuerpo extraño en vía aérea

43c. En relación con su sospecha clínica, ¿qué hallazgos encontraría en la


radiografía?
c} Enfisema obstructivo

43d. Con base en su sospecha clínica, ¿qué tratamiento le indicaría al paciente?


a} Broncoscopia

43e. De acuerdo con su sospecha clínica,¿cuál es la complicación que se puede


presentar en el paciente?
b} Estenosis bronquial o neumonías recidivantes

44. De acuerdo con la clasificación de la Universidad de Texas, una úlcera


superficial,infectada y no isquémica en el pie de un paciente diabético corresponde
a:
a} 1B

45. Es el primer fármaco de elección en el tratamiento de la diabetes tipo 2 y


obesidad.
d} Metformina
46. Prepúber de 12 años de edad con verrugas plantares.
46a. ¿Cuál es el agente causal de esta infección?
a} Virus delpapiloma humano

46b. ¿Cuál es el serotipo más prevalente en este tipo de verrugas?


a} 1

46c. ¿Qué le explicarías al paciente con respecto a la evolución y respuesta al


tratamiento?
e} Las verrugas pueden llegar ainvolucionar espontáneamente

46d. El paciente decide tratarse ya que le molesta mucho y no le gusta. De los


siguientes, ¿cuál es una opción de tratamiento indicada?
b} Crioterapia

47. Hombre de 31 años, con historia familiar de diabetes tipo 2 en padre y abuelo
paterno, hipertensión arterial sistémica en madre, infarto agudo de miocardio en
abuelo paterno; con diagnóstico de hipertensión hace tres años tratado con
enalapril. Desde hace aproximadamente 15 años ha presentado aumento
progresivo de peso (15 kg). A la exploración física: peso 140 kg, talla 1.85 m,
índice de masa corporal 41.22 kg/m2, tensión arterial 110/80 mm Hg, pulso 78x,
con obesidad de predominio central,acrocordones y acantosis pigmentaria en
región posterior del cuello y en ambas axilas, tiroides palpable y normal. Abdomen
globoso por panículo adiposo sin estrías. Fuerza muscular conservada. Resto sin
alteraciones. En sus exámenes de laboratorio destacan: glucosa 99 mg/dl,
creatinina 0.61 mg/dl, urea 37.1, ácido úrico 9.73 mg/dl, AST 54 Ul/L, ALT 65
Ul/L,bilirrubinas y fosfatasa alcalina dentro de parámetros normales, colesterol
total 202 mg/dl, triglicéridos 247 mg/dl.
47a. El diagnóstico sindromático del paciente es:

b} Síndrome metabólico

47b. La presencia de acrocordones y acantosis pigmentaria se debe a:


b} Resistencia a la insulina

47c. ¿Qué examen paraclínico es el más indicado en este caso?


b} Curva de tolerancia ala glucosa oral

47d. El perfilviral de hepatitis es negativo y en un ultrasonido de hígado y vías


biliares no se reporta obstrucción de la vía biliar ni neoplasia hepática. El
diagnóstico más probable es:
c} Esteatohepatitis no alcohólica

48. Paciente masculino de 4 años de edad se presenta al departamento de


Urgencias por dolor abdominal. En el interrogatorio se refiere cuadro viral hace 2
semanas, dolor abdominal de pocas horas de evolución,vómito, oliguria y erupción
cutánea extensa. En la exploración física se encuentra un exantema
maculopapular en las nalgas y extremidades inferiores que no blanquea con la
digitopresión. La biometría hemática es normal, pero el examen general de orina
muestra hematuria."
48a. Con base en las características del paciente, ¿cuál es el diagnóstico más
probable?
d} Púrpura de Henoch-Schonlein

48b. ¿Qué hallazgos en los exámenes de laboratorio apoyan su sospecha


diagnóstica?
b} Biometría hemática normal,sangre oculta en heces

48c. De acuerdo con su sospecha clínica,¿cuál es la complicación que se puede


presentar a largo plazo?
c}Invaginaciónintestinal

48d. Con base en su sospecha clínica, ¿cuál es el tratamiento indicado?


a} AINE

48e. ¿Qué aspectodel cuadro clínico del paciente es relevante mencionar a los
padres?
d} Hematuria microscópica

49. Paciente masculino de 88 años con múltiples condiciones médicas (angina


estable, enfermedad pulmonar obstructiva crónica, osteoartritis, diabetes mellitus
tipo 2, enfermedad acidopéptica) ingresa a la sala de urgencias por presentar
dolor abdominal de inicio abrupto de 6 horas de evolución. El paciente informa
dolor abdominal difuso, con leve localización en la región periumbilical. A la
exploración física se registran frecuencia cardiaca de 120 latidos por minuto,
presión arterial de 100/60 mm Hg, frecuencia respiratoria de 30 respiraciones por
minuto e hipotermia. A la palpación abdominal se encuentran signos de irritación
peritoneal. Las radiografías de tórax y abdomen demuestran la presencia de aire
subdiafragmático. ¿Cuál es el vaso sanguíneo más frecuentemente afectado en
casos de perforación de úlcera duodenal en la pared posterior del duodeno
(primera porción)?
b} Arteria gastroduodenal

50. Paciente femenino de cinco años que es traída por sus padres tras ingerir
accidentalmente limpiador de drenaje guardado en una botella de refresco. La
paciente está quejumbrosa,con dolor torácico intenso, náusea, disfagia y sialorrea
profusa. A la exploración física se observan quemaduras de los labios, edema y
eritema de lengua y paladar. ¿Cuál de las siguientes está contraindicada en el
manejo de este caso?
d} Lavado nasogástrico
51. Paciente femenino de 67 años de edad con diagnóstico de hipertensión arterial
sistémica, bajo tratamiento con captopril 25 mg cada 12 horas, actualmente
descontrolada, inicia hace 24 horas con epistaxis profusa que no se autolimita ni
cede a la compresión por lo que acude a urgencias hace 6 horas donde se le
coloca taponamiento nasal anterior y es dada de alta. Tres horas después regresa
por goteo hemático a través del tapón nasaly sensación de descarga hemática
posterior, diagnosticándose epistaxis posterior. ¿Cuál es el origen del sangrado?
b} Arteria esfenopalatina

52. Paciente masculino de 2 años de edad es llevado a consulta de rutina. Se


mencionan como antecedentes que nació a las 37 semanas de gestación, fue
alimentado de manera exclusiva mediante seno materno durante el primer año de
vida y que actualmente no es bueno comiendo. Su peso y estatura se encuentran
en el percentil 5. A la exploración física se observa que sus piernas están algo
arqueadas y el hueso frontal es muy prominente. No tiene dientes y todavía no
camina por sí solo.
"52a. Con base en las características del paciente, ¿cuál es el diagnóstico más
probable?"
b} Deficiencia de vitamina D

52b. En relación con su sospecha clínica,¿cuál de las siguientes opciones


representa una característica particular del padecimiento?"
a} Extremos distales ensanchados con forma de copa y con desgaste en la
radiografía

"52c. Respecto a su sospecha clínica, ¿qué función tiene en el cuerpo la vitamina


que se encuentra con deficiencia en el paciente?"
c} Depósito de calcio

52d. De acuerdo con su sospecha diagnóstica, ¿qué tratamiento le indicaría al


paciente?"
d} Suplemento de vitamina O

52e. Referente a su sospecha clínica, ¿cuál es el mecanismo de prevención que


se debió realizar para evitar esta patología?"

b} Exposición al sol

53. Mujer de 54 años de edad con dermatosis bilateral y con tendencia a la


simetría que se disemina a ambas caras pretibiales. Se caracteriza por tres placas
infiltradas amarillo violáceas y esclerodermiformes, irregulares en la forma y mal
delimitadas, que miden entre 5 y 7 cm. La paciente refiere que sus lesiones
iniciaron hace un mes como ""granitos"" de color rojo violáceo y otras como
nódulos de forma redondeada, de bordes no bien definidos y a veces elevados.
Fueron creciendo de forma lenta y excéntrica, tornándose de color amarillo, con
telangiectasias en la superficie. Una de las lesiones se ulceró, dejando cicatriz y
atrofia posterior.A la exploración física presenta en las lesiones sensibilidad
disminuida al pinchazo y al tacto leve, hipohidrosis y alopecia parcial. La paciente
es diabética de larga evolución y al parecer lleva un buen control. Niega
antecedentes de traumatismos o de infecciones previas. El diagnóstico es:
b} Necrobiosis lipoídica

54. Una paciente de 35 años sin antecedentes previos de importancia ingresa a la


sala de urgencias. El motivo es por dolor abdominal de aproximadamente 24 horas
de evolución, el cual inició en la región periumbilical pero recientemente ha
migrado hacia el hipocondrio derecho. La paciente refiere náusea y vómito en tres
ocasiones. A la exploración física, se detectan mucosas deshidratadas así como
dolor abdominal localizado al hipocondrio derecho con signo de rebote positivo.
Los estudios de laboratorio son relevantes por leucocitosis a base de neutrófilos.
¿Cuáles son los nervios que conducen la sensación dolorosa periumbilical en esta
paciente?
b} Nervio esplácnico menor

"55. Paciente femenino de 32 años de edad, quien tiene como antecedentes


relevantes crisis convulsivas por cisticercosis, previamente tratada con valproato;
hace 4 semanas se hizo cambio por carbamazepina. Hace 3 días inicia con
malestar general, astenia, adinamia, hiporexia y hepatalgia, el día de hoy inicia
con eritema morbiliforme generalizado, motivo por el cual acude a consulta. FC 95
lpm, FR 18 rpm, temp. 38.6ºC, PA 110/70, peso 58 kg, talla 1.68 m.
"55a. Con los datos clínicos anteriores, usted realiza el diagnóstico de:"
b} Síndrome de DRESS
"55b. Estadísticamente, ¿qué órgano se afecta con mayor frecuencia en el
síndrome de DRESS?"
b} Hígado

55c. Usted solicita una biometría hemática que reporta eosinofilia > 1.5 x 109 L,
los laboratorios que solicita reportan elevación de TGO, TGP y DHL, con tiempos
de coagulación prolongados. Usted solicita una biopsia de piel,¿qué hallazgos
espera que se reporten?
a} Paraqueratosis, acantosis local e infiltrado linfocitario con epidermotropismo"

55d. Se debe iniciar el tratamiento, empezando por retirar el fármaco que provocó
el síndrome. Usted inicia el tratamiento con:"
d} Esteroides

55e. Se han documentado diversos datos de severidad del síndrome de DRESS,


¿cuál se asociacon mayor mortalidad?
b} Necrosis hepática

56. Paciente masculino de 31 años de edad con antecedente de dos semanas de


fiebre, a lo cual se agregó hace una semana distensión abdominal,estreñimiento y
dolor epigástrico. Entre otras molestias también refiere cefalea, escalofríos,
mialgias y malestar general. La exploración física revela la presencia de un
""exantema rosado"" (exantema maculopapular color salmón que blanquea a la
presión), principalmente en el tórax, así como dolor abdominal generalizado a la
palpación y esplenomegalia. Presenta temperatura de 38.9 ºC, frecuencia cardiaca
de 45 latidos por minuto, frecuencia respiratoria de 20 respiraciones por minuto,
presión arterial de 100/65 mm Hg. Los estudios de laboratorio básicos son
relevantes por leucopenia y neutropenia. Se realiza un hemocultivo el cual resulta
positivo para Salmonella typhi. ¿Cuál es el tratamiento antibiótico más efectivo en
cepas no provenientes de brotes resistentes?
a} Fluoroquinolonas

57. Un paciente de 21 años se presenta a la sala de urgencias posterior a una


lesión en su hombro durante un partido de fútbol. A la exploración física el hombro
parece deprimido y plano y no es capaz de abducir su brazo. Una radiografía de la
región demuestra que el paciente tiene fractura del cuello quirúrgico del húmero.
¿De dónde proviene la inervación del músculo afectado en este paciente?
b} C5 a C6

58. Un paciente de 45 años con sobrepeso es llevado a la sala de urgencias con


pérdida del estado de alerta después de un accidente automovilístico. Luego de
recuperar la conciencia, la exploración física revela debilidad de ambas
extremidades derechas con signo de Babinski positivo en el pie derecho. El
paciente muestra la lengua desviada hacia la izquierda cuando la protruye. A la
evaluación de la sensibilidad corporal,el paciente ha perdido la sensibilidad
vibratoria en el lado derecho del cuerpo. ¿Cuál es el sitio más probable de lesión
en este paciente?
a} Lesión de la región medial de la médula oblongada
59. Paciente femenino de 60 años de edad con diagnósticos previos de
hipertensión,diabetes tipo 2, miocardiopatía isquémica,e insuficiencia renal
crónica; reporta dolor en la rodilla derecha. La presión arterial es de 140/80 mm
Hg, con una frecuencia cardiaca de 66 latidos/min. Debido al edema y al dolor en
la rodilla, se le receta celecoxib 200 mg/día. Después de dos semanas bajo este
tratamiento, la paciente refiere disnea, edema de extremidades inferiores y fatiga.
La presión arterial en esta ocasión es de 180/105 mm Hg, nitrógeno ureico de 67
mg/dL, creatinina sérica de 3.9 mg/dL. ¿Cuál de los siguientes diagnósticos es el
mecanismo de acción más probable por el cual el analgésico administrado
ocasionó insuficiencia renal?"
a} Pérdida del balance entre prostaglandinas, ocasionada por la inhibición de la
ciclooxigenasa tipo 2"

"60. Paciente masculino de 36 años, arqueólogo, diabético. Estuvo en contacto


con pacientes con tuberculosis en su último viaje a Zimbabwe. Comenzó con
fiebre, tos productiva con estrías sanguinolentas y ataque al estado general.
Durante su traslado presentó sudores nocturnos. La radiografía de tórax revela
infiltrados nodulares en lóbulo superior,con una lesión cavitada que está drenando
a través de un bronquio. PPD negativo. Se inicia tratamiento antituberculoso.
Acude a consulta a ajuste de hipoglucemiantes orales debido a que su glucosa
sérica se elevo casi el doble de lo que manejaba hace tres meses. Refiere que ha
respetado su dieta y ha estado tomando su medicación. Pocas semanas después
del inicio del tratamiento antituberculoso el paciente reporto disminución de tos e
incremento de energía."
¿Cuál es la explicación de la hiperglucemia en este paciente?
b} Rifampicina

61. Una paciente de 64 años es llevada al hospital por sensación de vértigo y


sordera del oído izquierdo. A la exploración física la paciente presenta sordera
neurogénica del lado izquierdo; analgesia y anestesia térmica del lado derecho del
cuerpo y del lado izquierdo de la cara; depresión del paladar con dificultad para la
deglución;síndrome de Horner izquierdo; tendencia a caerse hacia el lado
izquierdo; nistagmo horizontal.
61a. ¿Cuál de las siguientes estructuras es la responsable de la depresión del
paladar y la dificultad para la deglución?
a} Núcleo ambiguo

61b. ¿Cuál de los siguientes enunciados es correcto en relación con el nistagmo


del paciente?
a} El componente rápido se dirige hacia la derecha

61c. ¿Cuál de las siguientes opciones representa la etiología más probable del
cuadro clínico de esta paciente?
d} Oclusión dela arteria cerebelar posteroinferiorizquierda

"62. Paciente masculino de 3 años de edad, quien 24 horas previo a la consulta


presentó rinorrea, fiebre y tos no traqueal,el día de hoy se agrega ronquera, tos
""perruna"" y estridor laríngeo en reposo; a la exploración física se encuentra con
adecuado estado mental, ligera palidez y tiraje intercostal bajo, la Sp02 es de
88%. Se niegan antecedentes relevantes. FC 110 lpm, FR 24 rpm, temp. 38.3ºC,
PA 90/60 mm Hg, peso 14 kg, talla 96 cm."
62a. El diagnóstico de este paciente es:
c} Laringotraqueítis

62b. ¿Cuál de los siguientes es un signo de alarma al ingreso?


d} Estridorlaríngeo

62c. Según los datos obtenidos hasta ahora, usted clasifica este cuadro como:"
b} Moderado

62d. El tratamiento farmacológico que usted selecciona es:


d} Glucocortidoides

62e. ¿Cuáles son los datos de alarma para la familia del paciente?
a} Cianosis
63. Paciente masculino de 18 años con antecedentes de anemia de células
falciformes e hipertensión portal es traído a urgencias por presentar hematemesis
masiva. El paciente se encuentra ictérico, taquicárdico e hipotenso. Usted indica
oxígeno por puntas nasales, canalizar dos vías periféricas con catéter de calibre
16, bolos de solución salina y obtiene muestras de sangre para biometría
hemática, estudios de coagulación y cruzar sangre.
¿Cuál es el tratamiento de elección del sangrado de várices esofágicas?
c} Endoscopia con ligadura de várices

64. Paciente masculino de 70 años de edad acude al servicio de urgencias por


dolor torácico. El paciente niega disnea, palpitaciones, náusea, vómito o mareo. El
expediente clínico es relevante por hipertensión, manejada en la actualidad con
diuréticos. La presión arterial del paciente es de 180/100 mm Hg, con una
frecuencia cardiaca de 11O latidos/minuto. La exploración del cuello evidencia un
desplacamiento inferior del cartílago cricoides durante cada contracción,así como
positividad para el signo de Carderelli. ¿Cuál de los siguientes estudios
diagnósticos es el más apropiado en este paciente?
e} Ecocardiograma transesofágico

65. Paciente femenino de 23 años de edad que ingresa a hospitalización por


presentar desde hace 30 días fatiga y disnea al ejercicio. Entre los antecedentes,
se encuentra una infección de vías respiratorias superiores hace un mes. No hay
antecedentes heredofamiliares de importancia. A la exploración física muestra una
presión arterial de 95/55 mm Hg, y un ritmo cardiaco de 115 latidos/minuto. Las
venas yugulares se encuentran distendidas y la auscultación pulmonar revela
estertores bilaterales; existe tercer ruido cardiaco. Se realiza un ecocardiograma
que demuestra dilatación de ambos ventrículos con hipocinesia difusa y fracción
de eyección del ventrículo izquierdo de 18%. ¿Cuál de los siguientes enunciados
NO es correcto respecto al cuadro de esta paciente?"
c} La administración de corticoesteroides puede disminuirla progresión dela
enfermedad

66. Paciente masculino de 70 años de edad que es llevado al servicio de


urgencias por confusión en las últimas 12 horas. Dos semanas previas el paciente
había sufrido un infarto miocárdico, con colocación de stent. Entre los
medicamentos del paciente se encuentra ticlopidina, aspirina, atorvastatina,
metoprolol y captopril. A la exploración física tiene fiebre de 39 ºC y exantema
petequial difuso. El resto de la exploración física no es relevante. Los estudios de
laboratorio presentan elevación de la creatinina (3.5 mg/dL) y caída en el
hematocrito (22%), con un conteo plaquetario de 45 000 plaquetas/mm3. ¿Cuál de
los siguientes medicamentos es el responsable por el cuadro de este paciente?

b} Ticlopidina

67. Mujer de 25 años de edad, acude a consulta por presentar dolor en el pie
derecho, con aumento de volumen y tumefacción del primer ortejo, limitación para
la movilidad de las articulaciones que componen ese dedo del pie; al interrogatorio
dirigido niega antecedentes crónico degenerativos, ovolactovegetariana, sin
toxicomanías ni alergias referidas, refiere haber cursado con cuadro infeccioso
gastrointestinal hace 3 semanas completamente remitido y niega antecedente
traumático; a la exploración se encuentra aumento de volumen del dorso del pie,
sin eritema ni hipertermia. FC 75 lpm, FR 18 rpm,temp. 36.5 ºC, PA 110/70 mm
Hg, peso 55 kg, talla 1.65 m.
67a. ¿Cuál es la causa más probable de la artropatía?
c} Artritis reactiva

67b. ¿Cuál de los siguientes agentes está asociado con artritis reactiva?
a} Salmonella

"67c. Con el diagnóstico y cuadro clínico, usted decide iniciar el tratamiento de


primera línea:
d} Antiinflamatorios no esteroides

67d. ¿Cuál de los siguientes criterios no es diagnóstico de artritis reactiva?


b} Afección simétrica

67e. Además de la artritis reactiva, los pacientes pueden presentar otras


manifestaciones ¿Cuál de las siguientes no está asociada con artritis reactiva?
d} Eritroderma

68. ¿Cuál de los siguientes medicamentos se caracteriza por presentar infarto


miocárdico como complicación?
c} 5-fluorouracilo

69. ¿Cuál de los siguientes esquemas profilácticos se recomienda para un


paciente con miocardiopatía hipertrófica obstructiva que se va a someter a una
cistoscopia?
a} No se requiere ningún tipo de profilaxis antibiótica

70. Javier es un paciente masculino de 42 años de edad con antecedente de


diabetes mellitus tipo 2 desde hace 5 años en tratamiento con insulina NPH,
tabaquismo positivo durante 12 años a razón de 5 unidades al día. Hace 2
semanas cursó cuadro de neumonía adquirida en la comunidad recibiendo
tratamiento ambulatorio a base de amoxicilina y ácido clavulánico durante 1O días
con aparente buena respuesta,sin embargo, hace 48 horas presenta fiebre
intermitente cuantificada en 38.5 ºC, disnea de moderados esfuerzos, tos sin
expectoración y dolor en punta de costado en hemitórax derecho que se
incrementa con movimientos respiratorios. A su ingreso a Urgencias diaforético,
con evidencia de disnea, taquicárdico, a la auscultación pulmonar con disminución
del murmullo vesicular subescapular derecho, disminución de transmisión de
vibraciones vocales, percusión mate en la misma región,resto sin alteraciones.Fe
11O lpm, FR 25 rpm, temp. 38.5 ºC, PA 100/60 mm Hg, peso 72 kg, talla 1.72 m.
70a. El diagnóstico más probable es:
c} Empiema
70b. Una vez que se evidencia la ocupación del espacio pleural el cual
compromete la mecánica ventilatoria se debe realizar lo siguiente:
d} Realización de toracocentesis

7Oc. Una vez que se realiza el estudio citoquímico de líquido pleural se reporta:
proteínas de líquido pleural / proteínas séricas 1.1. Usted considera lo siguiente:
b} Líquido pleural corresponde a exudado

70d. Dentro del estudio macroscópico del líquido pleural se determina la presencia
de pus por lo que el siguiente paso consiste en:
a} Drenaje costal cerrado

70e. ¿Qué esquema antibiótico empírico recomendaría en este caso?


b} Cefuroxima + Metronidazol

71. Femenino de 45 años de edad es traída por los servicios de emergencia al


hospital donde usted está de guardia. La paciente ha sufrido un accidente con un
coche, se encuentra inconsciente y se observan lesiones graves en todo el macizo
facial y zona del ojo derecho. TA: 130/80, FC: 95x ', FR: 23x ', Temp: 37.SºC."
71a. La fractura más característica de la órbita es:
b} Polo inferior

71b. La fractura más frecuente de la órbita, que además puede lesionar la tróclea
del oblicuo superior,es:
e} Medial

71c. ¿Qué huesos conforman la pared inferior de la órbita?


b} Maxilar superior,malar y palatino"

71d. Ante un trauma ocular, ¿qué estudio de gabinete se debe solicitar?"


d} TC
72. Hombre de 35 años de edad, fumador y bebedor social desde los 17 años,
consulta por haber observado sangre fresca en el retrete y en el papel higiénico
tras obrar; no refiere dolor ni cambios en sus hábitos intestinales, así como
tampoco sensación de masas o tumoración. A la exploración física se observa
región anal sin sangre ni tumoraciones. TA: 120/SOmm Hg, FC: 70x', Temp:
36.SºC.
72a. Su principal sospecha diagnóstica es:
c} Hemorroides

72b. ¿Cuál es la causa de este padecimiento?


b} Dilatación anormal delas anastomosis arteriovenosas

72c. Es la referencia para la clasificación de esta enfermedad.


b} Línea dentada

72d. El primer paso dentro del tratamiento es:


a} Fibra en la dieta y ablandadores de heces

72e. La escleroterapia y la ligadura con banda elástica están contraindicadas en:


d} Hemorroides externas

73. Masculino de 30 años acude a consulta por presentar un cuadro diarreico de 3


días de evolución, entre seis y ocho evacuaciones al día, abundantes, líquidas, sin
presencia de moco o sangre. Presentó vómito en dos ocasiones al inicio, y en las
últimas horas ha mostrado mareo. Niega fiebre. Antecedente: haber asistido a
competencia de triatlón a Nueva Delhi la semana previa al padecimiento actual."
73a. El germen causal de la sintomatología de este paciente es:
d} Vibrio cholerae

73b. Tratamiento de elección para este paciente:


d} Doxiciclina

73c. El cuadro clínico de este paciente es causado por aumento en :


d} cAMP

74. Paciente masculino de seis años de edad con dolor abdominal difuso y
estreñimiento crónicos. Recientemente se ha mostrado muy distraído, con
problemas en su desempeño escolar. El padre del paciente trabaja en una fábrica
de vidrio. Se sospecha de una posible intoxicación por plomo. Señale, cuál es el
mecanismo de acción de este metal.
b} Unión a ligandos sulfhidrilo con inhibición de múltiples enzimas
75. Paciente femenino de 50 años es llevada al servicio de urgencias por
presentar disnea de una semana de evolución que ha progresado hasta ser de
pequeños esfuerzos y se acompaña en ocasiones de palpitaciones y dolor torácico
que dura pocos minutos. La disnea fue máxima esta mañana, asociada con dolor
retroesternal 7/10 que aumenta con la respiración. La paciente no tiene historia de
tabaquismo, niega cirugías recientes. Su esposo refiere que la ha visto cansada y
que ya casi no se levanta de la cama desde que le colocaron una férula en el
tobillo izquierdo debido a que sufrió un esguince hace dos semanas. A la
exploración física es evidente la dificultad respiratoria con uso de músculos
accesorios, signos vitales con frecuencia respiratoria de 26, frecuencia cardiaca
105 lpm, presión arterial 85/50 mm Hg, temperatura 37.5 ºC. Se aprecia
ingurgitación yugular de 8 cm, taquicardia,S1 y S2 sin soplos ni ruidos accesorios,
campos pulmonares bien ventilados y edema de miembros pélvicos,
especialmente del lado izquierdo, con dolor a la palpación de la pantorrilla
izquierda.
La sospecha clínica de que esta paciente presente una embolia pulmonar es:
d} Alta

76. Paciente femenino de cinco años es traída a urgencias por sus padres quienes
la encontraron jugando en un campo que había sido regado con pesticidas. La
paciente presentó inicialmente abundante salivación y lagrimeo, vómito, diarrea e
incontinencia urinaria seguida de confusión y contracciones musculares
generalizadas. A la exploración física se encuentra letárgica, diaforética, con
pupilas mióticas y fasciculaciones.
Usted de inmediato sospecha intoxicación por organofosforados.
¿Cuál es el medicamento adecuado para tratar los efectos nicotínicos en esta
paciente?
e} Pralidoxima

77. Paciente masculino de 35 años, albañil, acude a consulta por presentar prurito
intenso en las manos, así como fisuras y costras entre los dedos de 15 días de
evolución.Se ha aplicado Vitacilina sin mejoría. A la EF se observa dermatosis
diseminada a palmas de manos y a región interdigital, caracterizada por múltiples
placas eritematoescamosas, algunas fisuras y costras hemorrágicas. Niega
ingesta de medicamentos. Comenta que se encuentra bastante preocupado, ya
que no le es posible laborar en la obra ni lavarse las manos porque nota mucho
ardor al contacto con acetona.
77a. ¿Cuál es el diagnóstico de este paciente?
b} Dermatitis por contacto

"77c. Dentro de las enfermedades ocupacionales, ¿qué porcentaje representa la


dermatitis por contacto de manos?"
a} 25 a 50%

77d. ¿Quétipo de reacción es la dermatitis de contacto por irritante primario?


d} No inmunitaria

77e. ¿Qué tipo de reacción inmunitaria es la dermatitis por contacto con


sensibilización o alérgica?
d}IV
78. Masculino de 14 años, sin antecedentes patológicos,es traído al hospital por
sus padres tras habersufrido una lesión mientras jugaba futbol americano. El
paciente comenta que al ser tacleado cayó sobre el hombro derecho e
inmediatamente sintió dolor en la región (EVA 10/1O); dicho dolor ha ido en
aumentoy le impide mover la extremidad superior derecha. A la exploración usted
comprueba que a la movilización de dicha extremidad el dolor es insoportable y
observa que el hombro derecho está descendido con respecto al izquierdo; al
palpar la zona superior derecha del tórax, usted nota edema, crepitación y
movilidad anormal en la región."
78a. ¿Cuálsería su sospecha diagnóstica?
a} Fractura clavicular

78b. ¿Qué estudio solicitaría para confirmar su diagnóstico?


a} Radiografía de tórax anteroposterior

78c. ¿Cómo se clasifica esta lesión?


c} Terciolateral,tercio medio o tercio medial"

78d. El tratamiento clásico de esta lesión es:


a} Vendaje o dispositivo en ""8"""

78e. Fractura más frecuente durante el trabajo de parto:


c} Clavicular

79. Se trata de Vanesa, una paciente femenino de 27 años de edad previamente


sana, que cursa la semana 24 de gestación, ha acudido a todas la consultas de
control prenatal hasta el momento, en esta ocasión corresponde realizar tamizaje
en búsqueda de alteraciones de la glucosa. La exploración física solo resalta un
abdomen globoso por estado de gravidez, sin edema periférico, sin otros datos
relevantes. FC 90 lpm, FR 20 rpm,temp 36.7 ºC, PA 130/70 mm Hg, peso 69 kg,
talla 1.62 m.
79a. El diagnóstico de diebetes gestacional se define como:
c} Intolerancia a carbohidratos reconocida durante el embarazo

79b. Usted realiza una prueba de carga de glucosa de 75 g en ayuno y curva de 2


horas ¿cuáles son los criterios para considerar una prueba
positiva?
d} Niveles de glucosa > 153 mg/dl en 2 horas

79c. Si esta paciente obtuviera un resultado positivo en ambas pruebas de


tolerancia a la glucosa ¿qué modificaciones haría al seguimiento?
b} Realizar 1cita cada 15 días de la semana 28 a 35 y una semanal de la 36 en
adelante
"79d. Se detecta que en ayuno la glucemia es > 95 mg/dl,¿cuál de las siguientes
terapéuticas será la ideal?"
c} Hipoglucemiantes orales
79e. ¿Cuáles son las complicaciones fetales más comunes durante el segudo y
tercer trimestre del embarazo?
d} Macrosomía

80. Paciente femenino de 45 años acude a consulta por ""hinchazón"" de manos


desde hace 5 meses; también ha notado un cambio de textura en su piel,que
ahora está más firme y dura. Tiene antecedentes de enfermedad por reflujo
gastroesofágico y disfagia a sólidos. A la exploración física se nota inflamación en
las articulaciones de sus dedos y presenta nódulos subcutáneos en éstos;
asimismo, se percibe leve edema e induración en ambas manos.
80a. ¿Cuál sería el diagnóstico más probable?
d} Esclerosis sistémica cutánea limitada

80b. De acuerdo con el diagnóstico clínico, ¿cuáles de los siguientes anticuerpos


ayudarían a tener un diagnóstico más certero?:
a} Anticuerpos anticentrómero

80c. ¿Cuál es la fisiopatología de la enfermedad?


d} Proliferación de fibroblastos con aumento enlos depósitos de colágeno

80d. En cuanto a las alteraciones renales de esta enfermedad, ¿cuál es la más


característica?
a} No hay afección renal

81. Masculino de 11 años de edad es traído a consulta por su madre, la cual


comenta que la maestra del niño le ha reportado que lo nota distraído en las
clases y que, por otra parte, en la casa ve la TV muy de cerca;además de que por
la tarde trae los ojos muy rojos. Durante la entrevista usted se da cuenta de que el
niño entrecierra mucho los ojos para observar los objetos en la pared. Al interrogar
al niño, éste comenta que le cuesta trabajo ver de lejos, por lo que se acerca para
ver bien,y que después de ver mucho tiempo la TV le da dolor de cabeza.
81a. ¿Cuál sería su diagnóstico para este paciente?
b} Miopía

81c. ¿Cómo se define un dioptrio?


c} Toda superficie que separa dos medios con distinto índice de refracción

81d. Del sistema óptico del ojo, ¿cuál estructura es la más potente?
b} Córnea

81e. El tratamiento de primera elección para este paciente sería:


c} Lentes divergentes
82. Paciente femenino de 50 años con dermatosis diseminada que afecta tronco y
extremidades superiores e inferiores caracterizada por ampollas flácidas,
erosiones y costras, algunas sanguíneas y otras melicéricas, y se acompaña de
estomatosis caracterizada por erosiones. No ha recibido tratamiento, refiere fiebre
de hasta 38.5 C. Inició hace alrededor de dos semanas con ampollas que han ido
evolucionando a costras. Las lesiones en boca iniciaron un mes antes y recibió
tratamiento con aciclovir por probable estomatitis herpética, sin mejoría. A la
exploración física muestra además mal estado de hidratación de mucosas y
taquicardia. Los hallazgos histológicos de la biopsia por sacabocado de una de las
ampollas son:

b} Ampolla suprabasal con células acantolíticas y células basales unidas a la


membrana basal

83. ¿Cuál de los siguientes diagnósticos se presenta con un reporte de biopsia de


ganglios linfáticos que menciona expansión de los folículos linfoides, preservación
de la arquitectura ganglionar con linfocitos atípicos en la zona paracortical?"
d} Infección aguda por el virus de Epstein-Barr

84. Paciente femenino de 42 años de edad que se presenta con adenomegalia


cervical única sin fiebre, sudación nocturna y pérdida de peso. Se realiza una
biopsia del ganglio linfático la cual revela presencia de linfocitos pequeños y pocas
células de Reed-Sternberg. Se efectúa una resonancia magnética para etapificar a
esta paciente; el estudio de imagen indica enfermedad limitada a sólo un grupo
cervical. ¿Cuál sería la clasificación adecuada de esta paciente?
a} Linfoma de Hodgkin etapa IA

85. Paciente femenino de 36 meses de edad que se presenta con fiebre de 38 ºC


y dolor en ambos oídos. Los padres mencionan que esto ha ocurrido en varias
ocasiones, y que casi siempre le recetan dos medicamentos, los cuales no
recuerdan en este momento. A la exploración física la paciente se encuentra
alerta, con agitación secundaria al dolor,congestión nasal y tos. Las membranas
timpánicas se observan eritematosas, con movimiento positivo a la insuflación. El
timpanograma demuestra una curva ""picuda"". ¿Cuál es el tratamiento más
adecuado en esta paciente?
c} Administración de paracetamol

86. ¿Cuál de los siguientes trastornos del sueño es el más común en la población
general?
d}Insomnio

87. Paciente masculino de 59 años de edad con antecedente de enfermedad


acidopéptica y reflujo gastroesofágico. Debido a que presenta insuficiencia
cardiaca y fibrilación se encuentra bajo tratamiento con digoxina. El paciente
decide iniciar por sí mismo tratamiento con antiácidos de hidróxido de aluminio.
¿Cuál de las siguientes alteraciones electrolíticas puede ser ocasionada por el
antiácido y con alta morbilidad en este paciente?
d} Hipopotasemia
88. Paciente femenino de 85 años de edad que se presenta con alteración del
estado de alerta, desorientación,combativa,con pupilas midriáticas no reactivas a
la luz, taquicardia sinusal, piel seca, hipertermia, constipación y retención urinaria.
Todos estos síntomas aparecieron después que por error de una estudiante de
enfermería se le administró una dosis excesivamente alta de atropina. ¿Cuál de
los siguientes medicamentos es el mejor antídoto en esta paciente?
a} Fisostigmina

89. Paciente masculino de 24 horas de vida producto de una paciente de 42 años


de edad. A la exploración física el neonato se presenta con microcefalia,
implantación baja de orejas, occipital prominente y micrognatia. ¿Cuál de las
siguientes anormalidades en el cariotipo es la que se puede encontrar con más
probabilidad en este paciente?"
b} Trisomía 18

90. Paciente masculino de 65 años se presenta en la madrugada al servicio de


urgencias por presentar disnea en reposo y edema remitente de miembros
inferiores pero que va en ascenso. Cuenta con antecedente de hipertensión y
diabetes mellitus tipo 2, así como de tres infartos miocárdicos, de los cuales dos
de ellos no pudieron ser trombolizados porque el paciente llegó fuera de ventana
terapéutica. Hace dos días presentó un cuadro de infección de vías aéreas
superiores, que ha progresado hasta convertirse en una neumonía incipiente.
¿Cuál de las siguientes combinaciones de fármacos, a pesar de ser efectiva en
este paciente, también puede resultar peligrosa?
a} Digoxina + furosemida

91. ¿Cuál de los siguientes antiarrítmicos tiene su principal mecanismo de acción


en células de despolarización automática y de lenta conducción?
e} Esmolol

92. ¿Cuál de los siguientes medicamentos se asocia con hiperpotasemia como


efecto adverso?
e} Ciclosporina

93. Paciente masculino de dos días de vida el cual presenta acidosis, vómito,
hipotonía y alteraciones neurológicas. Se solicitan estudios de laboratorio los
cuales revelan niveles elevados de lactato y alanina. ¿Cuál de las siguientes
enzimas es la que con mayor probabilidad se encuentra deficiente en este
paciente?
b} Piruvato deshidrogenasa

94. ¿Cuál de los siguientes mecanismos de acción corresponde al de las


estatinas?
b}Inhibición dela enzima hidroxi-metil-glutaril-coenzima A

95. ¿Cuál de los siguientes defectos es ocasionado por ausencia de la formación


del septo aorticopulmonar?
b} Comunicacióninterventricular de tipo membranosa
96. Una mujer de 18 años de edad se presenta a consulta externa de Obstetricia
en su 18 semana de gestación. Hace tres años, la paciente fue diagnosticada con
depresión moderada y se le recetó paroxetina 20 mg/día, la cual continuó tomando
hasta hace aproximadamente seis meses, debido a que consideró que no
necesitaba el medicamento. La paciente informa que recientemente no ha dormido
""en semanas"", y lo atribuye al embarazo. Al interrogatorio menciona que también
se ha mostrado recientemente letárgica, sin ideación suicida y que desea
alimentar a su futuro hijo con leche de su propio seno. ¿Cuál de los siguientes
esquemas de tratamiento es el más adecuado para el manejo de depresión de
esta paciente?
a} Sertralina 50 mg al día, con consulta psiquiátrica de seguimiento en un mes

97. ¿Cuál de las siguientes cadenas de globinas se produce exclusivamente


durante la etapa embrionaria?
e} Zeta globina

98. Paciente femenino de 22 años de edad se presenta a consulta por ictericia y


malestar general desde hace 20 días. No había acudido antes porque su novio
había tenido un tipo de hepatitis algunos meses atrás, y éste le recomendó que no
acudiera por tratamiento médico, pues seguramente era un cuadro autolimitado.
Los estudios de laboratorio revelan una concentración de alanina transaminasa de
250 U/L, aspartato transaminasa de 201 U/L, bilirrubina de 7 mg/dL, positividad
para el antígeno de superficie del virus de la hepatitis B y anticuerpo lgM contra el
antígeno central (core) del virus de la hepatitis B. La paciente muestra un claro
cuadro de hepatitis aguda por el virus de la hepatitis B. ¿Cuál de los siguientes
enunciados respecto de la hepatitis tipo B es incorrecto?
b} La administración de interferón enla etapa aguda dela infección evita el
desarrollo delestado de portador crónico del virus de la hepatitis B

99. ¿Cuál de los siguientes fármacos antibióticos ejerce su efecto mediante la


inhibición de la enzima DNA topoisomerasa en bacterias?
c} Quinolonas

1OO. Paciente masculino de 31 años de edad que acude a consulta de rutina.


Entre los antecedentes de importancia el paciente refiere que a la edad de seis
años sufrió un cuadro de fiebre reumática aguda, lo cual lo confinó a la cama por
varios meses. Desde entonces ha estado bajo tratamiento antibiótico con
penicilina V oral a dosis de 500 mg al día. Cuatro años después del episodio de
fiebre reumática aguda,es decir,a los 1O años, tuvo un cuadro de exacerbación
relacionada a la suspensión del medicamento antibiótico. Actualmente el paciente
trabaja como enfermero en un hospital pediátrico. A la exploración física presenta
un soplo holosistólico grado 3/4, el cualtiene su punto de mayor intensidad en el
ápex. ¿Cuál de los siguientes enunciados es el más adecuado en relación a la
conducta a seguir en este paciente?"
e} El paciente debe continuar tomando penicilina por tiempoindefinido

101. Paciente femenino de 82 años presenta historia de frecuencia urinaria y


urgencia con restos de moderadas cantidades de orina, la cual comienza después
de un evento vascular cerebral tres meses antes. Sin antecedentes familiares de
importancia. A la exploración física nada relevante. Orina residual 35 ml. La
biometría hemática y el examen general de orina son normales. El tratamiento que
más probablemente mejore los síntomas de la paciente es:
a} Oxibutina

102. ¿Cuál de los siguientes enunciados es el que más adecuadamente


representa las características de la fractura de Colles?
b} Fractura de la metáfisis distal del radio en la que existe desplazamiento dorsal
del fragmento distal

103. ¿Cuál de las siguientes hormonas contribuye a mantener niveles normales de


glucosa en la sangre durante periodos prolongados de
ayuno y actúa mediante un receptor intracelular?
e} Cortisol

104. ¿Cuál de los siguientes síndromes se presenta con tumores desmoides y


osteomas mandibulares?
a} Síndrome de Gardner

105. ¿Cuál de los siguientes medicamentos representa un alto riesgo de hepatitis


colestásica?
c} Eritromicina

106. ¿Cuál de los siguientes hallazgos incrementa la probabilidad del diagnóstico


de epididimitis bacteriana?
c} Secreción uretral

107. Paciente femenino de 34 años de edad. Inicia su padecimiento tres días


previos a su ingreso con dolor en región dorsal, progresivo, de tipo punzante,
intenso, de predominio nocturno con irradiación hacia región anterior del tórax, se
administran analgésicos y antiinflamatorios sin mejoría, después se agrega
retención urinaria aguda, debilidad muscular de los miembros inferiores,
parestesias de los miembros pélvicos y en región torácica anterior. Evolucionando
con pérdida progresiva de la fuerza en miembros inferiores y disminución de la
sensibilidad hasta la región torácica. Presenta dolor a la palpación en región dorsal
a nivel de T3-T4. Extremidades superiores con fuerza 5/5, inferior derecha 3/5,
inferior izquierda 1/5, reflejos osteotendinosos de +++ en extremidades inferiores y
++ en extremidades superiores, con respuesta plantar extensora bilateral. Nivel
sensitivo de hipoestesia en T4, incontinencia urinaria y pérdida en el tono del
esfínter anal. Se realizaron exámenes de laboratorio de rutina en rangos normales.
Se programa para tomografía computarizada simple y contrastada de columna
dorsal con foco en T3- T4, sin encontrar lesiones espinales ni lesiones
intrarraquídeas o intramedulares. Se realiza IRM de columna dorsal donde se
encuentra una lesión quística a nivel de T1 con reforzamiento anular y una imagen
hiperdensa dentro de la lesión con edema perilesional compatible con cisticerco
intramedular. En relación a la fisiología de la micción y la incontinencia urinaria
todo es característico EXCEPTO:
d} Laincontinencia urinaria aguda tiene un inicio brusco, generalmente en relación
con una enfermedad aguda o problema iatrogénico y no remite tras resolverse la
enfermedad o el problema de medicación.Se cura aproximadamente 10%"

108. ¿Cuál de los siguientes mecanismos de acción productora de diarrea


corresponde al de Tropheryma whipplei?
b} Colonizaciónintracelular del organismo delas células epiteliales intestinales

109. Paciente masculino de 67 años de edad al cual se le realizó una


colonoscopia como método de detección temprana de cáncer colónico. Se
encontró un adenoma velloso sésil de 1.2 cm en el colon ascendente. La
tumoración fue extirpada. ¿Cuál de las siguientes conductas terapéuticas es la
más adecuada?
b} Repetirla colonoscopia en tres años

110. ¿Cuál de los siguientes organismos se asocia con levaduras de base ancha
en la biopsia de los tejidos infectados?
d} Blastomyces dermatitidis

111. Paciente de cuatro días de edad con deficiencia de la enzima galactosa-1-


fosfato uridiltransferasa. ¿Cuál es la evolución más probable de este paciente en
caso de que su enfermedad no sea tratada oportunamente?
e} El paciente sufriráinsuficiencia hepática

112. ¿Cuál de los siguientes esquemas profilácticos de tromboembol ia venosa es


el más adecuado en pacientes de riesgo moderado para desarrollar ésta?
e} La combinación de A y C es correcta

113. ¿Cuál es el nervio que se daña con mayor frecuencia durante las inyecciones
intramusculares administradas con técnica deficiente en la región glútea?
d} Nervio ciático

114. ¿Cuál es el anticonvulsivante de elección en caso de crisis convulsivas


parciales complejas?
a} Carbamacepina

115. ¿Cuál de los siguientes hallazgos histopatológicos óseos son característicos


del hiperparatiroidismo primario?
c} Resorción de hueso subperióstico con degeneración quística

116. Paciente masculino de cinco años de edad que se presenta con debilidad y
calambres musculares después del ejercicio. A la exploración física el paciente no
presenta ninguna anormalidad aparente, ni retraso mental o alteraciones en el
desarrollo. ¿Cuál de las siguientes enzimas es la que más probablemente se
encuentra deficiente en este paciente?"
d} Glucógeno fosforilasa

117. ¿Cuál de los siguientes resultados de laboratorio es el que se esperaría


encontrar en un paciente dislipidémico en caso de que éste tenga adecuado
apego y respuesta al gemfibrozil?
b} Disminución intensa enla concentración de triglicéridos, ligera disminución en
la concentración de colesterol-LDL, y un ligero incremento del colesterol-HDL"

118. Ante un niño de 2 días de vida con antecedente diagnostico fibrosis quística,
¿qué manifestaciones de patología digestiva se pueden encontrar?
a} Íleo meconial y prolapso rectal

119. Paciente masculino de 29 años de edad que acude al servicio de urgencia


por presentar disnea en reposo. Menciona que necesita utilizar tres o cuatro
almohadas al dormir para evitar la disnea. Recientemente el paciente ha notado
que se le hinchan los pies. Entre los antecedentes el paciente niega el consumo
de alcohol, hipertensión o enfermedad arterial coronaria. Hace una semana tuvo
un cuadro de infección de vías respiratorias. A la exploración física la presión es
de 115/76 mm Hg y la frecuencia cardiaca de 105 latidos/minuto. La presión
venosa yugular es de 14 mm Hg. Existen estertores en ambos campos
pulmonares que se encuentran hasta las escápulas. A la auscultación cardiaca no
se encuentran soplos, pero es posible identificar el tercer y cuarto ruido cardiaco.
En las extremidades inferiores tiene edema 2++. El electrocardiograma no muestra
cambios en el segmento ST o en las ondas T. La radiografía de tórax muestra
datos en relación a edema pulmonar y aumento del tamaño de la silueta cardiaca.
Existe elevación leve la fracción MB de la creatinina cinasa. ¿Cuál es la
probabilidad de que este paciente se recupere de este evento?
e} 50%

120. ¿Cuál de las siguientes articulaciones es la más afectada en la enfermedad


por depósito de cristales de fosfato cálcico básico?
a} Hombro

121. Paciente femenino de 29 años de edad, con lateralidad manual derecha,que


se presenta al servicio de urgencias por pérdida súbita de fuerza en hemicuerpo
izquierdo, desorientada y con lenguaje farfulleante. Tiene el antecedente de dos
cuadros similares: el primero hace un año, que provocó hemiparesia faciocorporal
derecha y afasia motora, parcialmente reversible tras 24 horas, dejó como secuela
paresia de ambos miembros derechos; el segundo hace dos semanas, con
desorientación súbita y parálisis de miembro pélvico derecho. Éste último fue
reversible tras 30 minutos. Al interrogatorio a familiares ha cursado con úlceras
crónicas en miembros pélvicos y piel ""amoratada"". En la exploración física existe
tensión arterial 160/11O mm Hg, frecuencia cardiaca 90 latidos por minuto,
frecuencia respiratoria 28 respiraciones por minuto, temperatura 38 ºC. Tiene
puntaje por escala de Glasgow de 13 puntos. Se corrobora hemiparálisis
faciocorporal derecha, signo de babinsky ipsolateral, sin alteración en pulsos
periféricos y los ruidos cardiacos son normales."
121a. ¿Cuál es el diagnóstico más probable?
e} Evento vascular cerebral isquémico

121b. Además del estudio de tomografía axial de cráneo simple, los estudios de
mayor utilidad para el abordaje diagnóstico de esta entidad en un paciente joven
son:
e} a y c son correctas

121d. El aspecto violáceo o amoratado en los miembros pélvicos de la paciente en


el contexto de su probabilidad diagnóstica causal de la trombofilia es:
d} Livedo reticularis

1. Paciente masculino de 87 años que acude a consulta por fatiga. Refiere que
desde hace seis meses ha sentido gradualmente menor energía, y al inico no le
había dado importancia. En sus antecedentes dice tener enfermedad de Crohn,
habiéndose sometido a resección intestinal de íleon y parte del colon hace cinco
años. Cuenta que inició hace unos meses con una terapia de medicina
alternativa,en donde dejó de ingerir productos animales. Niega actualmente
síntomas de diarrea, distensión abdominal,fiebre o sangrado. Lo que sí refiere es
inestabilidad, a veces perdiendo el equilibrio sin sentirse mareado. Sus signos
vitales incluyen FC 100/min, FR 17/min,T 37.2 C y PA 134/88 mm Hg. Se
auscultan S1 y S2 sin agregados y campos pulmonares con murmullo vesicular.
No hay anormalidades a la exploración abdominal. En la neurológica se observa
disminución a la sensibilidad de la vibración en extremidades inferiores. La prueba
de Romberg es positiva. Se muestran a continuación los estudios de laboratorio.

¿Cuál es la causa más probabledel padecimiento de este paciente?


d} Deficiencia de cobalamina

2. Paciente masculino de 84 años que acude a consulta por dolor abdominal.


Refiere al dolor de tipo punzante, localizado en el hipogastrio. En su historial
médico hay hipertensión,enfermedad coronaria e hiperlipidemias. Sus
medicamentos incluyen metoprolol,lisinopril, aspirina y simvastatina. Al IPAS
refiere dolor ""de los huesos"" en las últimas semanas, principalmente de las
extremidades y la espalda. También al interrogatorio menciona que no ha ido a
orinar en 24 horas. A la exploración física sus signos vitales incluyen FC 104/min,
FR 18/min, T 36.7 C y PA 145/80 mm Hg. No hay anormalidades a la exploración
torácica. En el reconocimiento abdominal se palpa una masa en el hipogastrio,
dolorosa altacto, timpánica a la percusión. Hay edema 1+ de extremidades
inferiores. Al tacto rectal se siente próstata agrandada, con una nodulación. Se
realiza una cateterización,y el examen de orina se observa sin anormalidades. A
continuación se muestran estudios de laboratorio:

¿Cuál es la causa más probable de la falla renal de este paciente?


c} Nefropatía obstructiva

3. Lo siguiente es cierto sobre la erisipela:


e} Los márgenes de la lesión están bien definidos y sonindurados

4. ¿Qué vasculitis se encuentra relacionada con una respuesta inmunitaria


aberrante a la hepatitis C crónica?

d} Crioglobulinemia mixta esencial

5. De los siguientes microorganismos, ¿en cuál NO se ha demostrado una relación


importante con algún tipo de cáncer?
d} Virus herpes simple tipo 1

6. Paciente femenina de 65 años de edad. Nivel socioeconómico bajo.


Antecedente de hipertensión arterial sin tratamiento. Acude a consulta por fatiga y
malestar general de 3 meses de evolución. TA: 140/90, palidez de tegumentos,
edema. Laboratorio reporta BUN elevado, creatinina 2.8 mg/dl, anemia
normocítica-normocrómica. Riñones pequeños por ultrasonido. Filtrado glomerular
por creatinina sérica estimado en 35 mUmin/1.73 m2 .
6a. Según la clasificación KDOQI de la enfermedad renal crónica, ¿en qué estadio
se encuentra la paciente?
b} III

6b. Señale la causa principal de enfermedad renal crónica:


a} Nefropatía diabética

6c. Señale las alteraciones electrolíticas y metabólicas con las que podría cursar la
paciente:
a} Hiperpotasiemia, acidosis metabólica, hiperfosfatemia, hipocalciemia
6d. ¿Cuál es la principal causa de muerte en la insuficiencia renal crónica?
c} Anomal ías cardiovasculares

6e. ¿Cuál de las siguientes opciones no corresponde a una indicación de diálisis?:


a} Edema

7. Masculino de 45 años de edad, con antecedente de fiebre reumática. Acude a


consulta por fatiga de varios meses de evolución y reciente inicio de palpitaciones
y disnea de esfuerzo. A la exploración física se registra TA: 120/80, FC: 88 lpm,
pulso irregular. Campos pulmonares sin ruidos agregados. Choque de la punta
hiperdinámico. Se ausculta un soplo holosistólico intenso descendente irradiado a
axila, S1 ausente.
7a. ¿Cuál es la patología más probable en este paciente?
a} Insuficiencia mitral

7b. ¿Cuál es el mecanismo fisiopatológico más común de la insuficiencia mitral?


c} Prolapso dela válvula mitral

7c. ¿Cuál es el estudio indicado para continuar con el abordaje diagnóstico y


terapéutico en este paciente?
c} Ecocardiograma

7d. Se realiza un ecocardiograma confirmando insuficiencia mitral con FE de 40%,


¿cuál es el tratamiento indicado?
b} Tratamiento quirúrgico conservador

7e. ¿Cuál de los siguientes hallazgos en la exploración nos orienta más hacia el
diagnóstico de insuficiencia mitral?
c} Aumento de laintensidad del soplo al apretar los puños

8. Ante un recién nacido pretérmino que cursó con enfermedad de membrana


hialina y requirió de oxigenoterapia y ventilación mecánica; al completar un mes de
vida persiste con dependencia del ventilador,se le solicita una radiografía de tórax.
De acuerdo al diagnóstico sospechado, ¿qué imagen espera encontrar en la
radiografía?
e} Patrón en esponja
9. A los 1O días de vida un recién nacido presenta caída del cordón umbilical y
posterior a esto, se observa a nivel de la cicatriz umbilical una masa rojiza, blanda
y granular que en ocasiones produce una secreción blanquecina espesa. ¿Cuál es
el manejo más adecuado en este caso?
b} Cauterización con nitrato de plata

1O. Masculino de 32 años, llevado a servicio de urgencias presentando: colapso


pulmonar completo con desplazamiento de mediastino, disminución del retorno
venoso e inestabilidad hemodinámica. Los datos clínicos de este paciente
corroboran el diagnóstico de:
d} Neumotórax a tensión

11. ¿Cuál de los siguientes anticonvulsivantes tiene una alta tasa de asociación
con el síndrome de Stevens-Johnson?
b} Etosuximida

12. Masculino de 22 años presenta síndrome nefrítico, la presencia de anticuerpos


anticitoplásmicos de neutrófilos con patrón citoplasmático (cANCA) positivos, nos
orienta hacia el diagnóstico de:
a} Granulomatosis de Wegener

13. ¿Cuál de los siguientes fármacos puede producir ototoxicidad?


b) Cisplatino

14. Se presenta al servicio de urgencias un paciente masculino de 22 años de


edad, el cual es traído por su novia. Ella refiere que habían estado en una fiesta
tipo rave y que al parecer el paciente ingirió algún tipo de droga. El paciente
presenta midriasis, taquicardia, hipertensión e incremento de la tensión de los
músculos mandibulares. ¿Cuál de las siguientes opciones es la droga que más
probablemente consumió este paciente?
a} Éxtasis

15. ¿Cuál de las siguientes neoplasias del sistema nervioso central es la más
frecuente de localización supratentorial en niños?
a} Craneofaringioma

16. Paciente masculino de 5 años con diagnóstico de escarlatina, se indica


tratamiento. En caso de encontrar una complicación tardía esta es:
c} Fiebre reumática y glomerulonefritis

17. ¿Cuál de los siguientes marcadores tumorales se eleva en los cánceres


ováricos?
a} CA-125

18. Paciente masculino de nueve años de edad que refiere desde hace una
semana malestar faríngeo, así como cefalea y dolor desde hace un día en la
región de la cadera izquierda. En la radiografía de cadera no se observan
anormalidades, sin embargo un aspirado guiado por ultrasonido demuestra la
presencia de un líquido estéril. Inmediatamente después de la aspiración los
síntomas mejoran, a pesar de que marcadores de inflamación se encuentran
elevados (velocidad de sedimentación eritrocitaria de 60 mm durante una hora y
proteína C reactiva de 77 mg/L). Se realiza diagnóstico de artritis viral y es dado
de alta. Tres días después el paciente regresa por dolor en ambos tobillos,
febrícula y eritema en tórax. A la exploración física presenta un soplo sistólico
grado 2/6 en la región de la base. Un ecocardiograma demuesta derrame
pericárdico, únicamente con electrocardiograma normal. En cuanto a estudios de
laboratorio presenta un título de antiestreptolisina O de 1 200 unidades Todd y de
antiDNAsa B de 1 920 unidades Todd, así como cultivo faríngeo positivo para
estreptococo hemolítico del grupo A.

18a. ¿Cuáles son los criterios mayores para el diagnóstico de fiebre reumática?
c} Carditis, poliartritis, corea, eritema marginado, nódulos subcutáneos

18b. ¿Cuál es el tratamiento más adecuado en este paciente?


a} Penicilina benzatínica y salicilatos orales en altas dosis

18c. En una evaluación posterior,se determina que el paciente no presenta daño


valvular cardiaco. ¿Cuál es la duración recomendada del tratamiento profiláctico?
b} Diez años o hasta la edad adulta (la que sea de mayor duración)

19. Paciente del sexo femenino de 25 años de edad que acude a consulta médica
porque desde hace tres meses la paciente presenta fatiga, lo cual ha repercutido
en su desempeño académico. Al interrogatorio la paciente describe que ha perdido
aproximadamente 8 kg en dos meses. A la exploración física la paciente presenta
adenomegalias en la región posterior del triá ngulo cervical y en la axila izquierda.
Se realiza una biometría hemática, la cual sólo es relevante por conteo leucocitaria
cercano al rango superior de normalidad. El médico decide realizar una biopsia de
ganglio linfático; los resultados de la biopsia revelan la sobrexpresión de la
proteína antiapoptótica bcl-2. ¿Cuál es la translocación que más frecuentemente
se presenta en este tipo de neoplasia?
d} t(14:18}

20. Femenino de 43 años es ingresada al servicio de Medicina Interna por


cetoacidosis. Posterior a su recuperación metabólica inicia con fiebre, cefalea,
dolor facial,disminución del nivel de conciencia y enrojecimiento nasal con lesión
negruzca en fosa nasal derecha. Antecedentes: portadora de DM tipo 2y HTAS.
La entidad causal de la sintomatología de esta paciente es:
b} Infección por Mucor

21. ¿Cuál de las siguientes opciones es una manifestación de la deficiencia de


cobalamina?
b} Manifestaciones neuropsiquiátricas

22. Un paciente masculino de 15 años de edad acude a consulta pediátrica de


seguimiento debido a su padecimiento actual, el cual le produce retraso mental
leve. A la exploración física el paciente presenta un nevo pigmentado de forma
oval en la región lumbar, así como angiofibromas en la frente, los cuales la madre
dice que se han presentado desde el nacimiento. En el expediente se menciona
que en el pasado el paciente tuvo un rabdomioma cardiaco, el cual se encuentra
en regresión. ¿Cuál de las siguientes enfermedades incide con mayor riesgo de
presentar este paciente?
c} Epilepsia

23. Un niño de ocho años de edad tiene el diagnóstico de faringoamigdalitis de


repetición, por lo que se programa para amigdalectomía.
23a. ¿Cuál es una indicación absoluta de la cirugía programada para este
paciente?
a} Amigdalitis aguda que cause crisis convulsivas febriles

23b. ¿Cuál de las siguientes es una complicación temprana después de


amigdalectomia?
a} Sangrado

24. ¿Cuál de las siguientes neoplasias malignas se asocia con policitemia como
síndrome paraneoplásico?
d} Carcinoma de células renales

25. Paciente femenino de 31 años de edad acude dos semanas después de su


primer parto por presentar una tumoración dolorosa en la porción inferior de la
glándula mamaria derecha. La paciente refiere que la leche que produce ha
adquirido una coloración verdosa. A la exploración se encuentra una masa
fluctuante y dolorosa. ¿Cuál es el tratamiento más adecuado en esta paciente?
a} Continuar con alimentación delneonato vía seno materno y administrar
antibióticos

26. ¿Cuál de las siguientes alteraciones electrolíticas se asocia con la aparición de


la onda J en el electrocardiograma?
b} Hipercalcemia

27. Se presenta al departamento de urgencias un lactante masculino de cuatro


meses de edad que sufrió crisis convulsivas hace dos horas. Al interrogatorio los
padres mencionan que el paciente fue producto de un embarazo a término y parto
eutócico, con desarrollo psicomotor normal. La exploración física revela que el
paciente presenta una gran hepatomegalia e hipotonía. Lafrecuencia cardiaca se
mantiene entre 180 y 200 latidos/min,con frecuencia respiratoria de 75
respiraciones/min. Los estudios de laboratorio indican hipoglucemia intensa con
acidosis láctica, hipertrigliceridemia. ¿Cuál es la deficiencia enzimática más
probable en este lactante?
b} Deficiencia de glucosa-6-fosfatasa

28. Paciente masculino de 19 años de edad que se presenta a consulta después


de haber notado la aparición de una úlcera no dolorosa en el pene posterior a una
relación sexual sin protección. ¿Cuál de los siguientes métodos es el mejor para
realizar el diagnóstico del agente etiológico del paciente?
b} Toma de exudado de la úlcera y observación bajo microscopio de campo
oscuro

29. Femenino de 59 años acude a consulta externa refiriendo: plenitud,


estreñimiento, acompañada con resultado de ultrasonido pélvico, imagen quística
en el ovario derecho de 15 por 15 cm. El diagnóstico más probable en esta
paciente es:
a} Cistadenoma seroso

30. Paciente masculino de 18 años de edad que conduce su automóvil en la


madrugada de regreso a casa. Durante el trayecto sufre colisión con otro
automóvil, cuyo conductor manejaba en estado de ebriedad. El paciente refiere
que sólo sufrió un impacto en la cabeza, sin pérdida del estado de alerta y que
tiene leve cefalea. Sin embargo el personal paramédico insiste en llevarlo a un
hospital para evaluación adicional. Una vez en el hospital el paciente relata lo
sucedido cuando de repente se muestra somnoliento y después con pérdida del
estado de alerta.
¿Cuál es el diagnóstico más probable?
e} Hemorragia epidural

31. ¿Cuál es la prevalencia en población general de la artritis reumatoide?


e} 0.5 a 1%

32. ¿Cuál es el mecanismo de acción de las quinolonas?


d}Inhibición dela síntesis del DNA

33. Paciente masculino de 49 años, sin antecedentes heredofamiliares de


importancia, fue sometido a colecistectomía hace cinco años. Tiene un índice
tabáquico de 2, niega alcoholismo. Realiza ejercicio aeróbico tres veces por
semana. Es remitido a consulta por prostatismo moderado, en tratamiento con
fitoterapia. Trae consigo una ecografía abdominal que muestra riñones y vejiga
normales, próstata de 45 g con calcificaciones en su interior.
33a. De las siguientes opciones, cual es la aseveración verdadera:

b} Haría un tacto rectal

33b. Tras hacerle un tacto rectal, la próstata impresiona de adenomatosa, volumen


11/IV.
Lo siguiente que procedería sería:
b} Solicitar antígeno prostático específico e iniciar tratamiento con alfa-
bloqueadores

33c. En la siguiente consulta, el paciente comenta que ha tenido que acudir al


servicio de urgencias por presentar prostatitis aguda, posterior a esto se le realizó
un antígeno prostático específico, cuyo resultado es de 25 ng/mL. Pregunta: ¿Cuál
sería la conducta a seguir?
d} Mantener el tratamiento antibiótico durante cuatro semanas, así como los alfa-
bloqueadores. Para finalmente repetir elantígeno prostático específico

33d. Cinco semanas después de haber completado el tratamiento adecuado, el


antígeno prostático específico reportado es de 14 ng/ml. Además, el paciente fue
sometido a sondeo vesical por presentar episodio de retención aguda de orina.
¿Qué le sugeriría al paciente como abordaje?
e} Solicitar ecografía transrectal con planimetría y toma de biopsias

33e. Los estudios realizados arrojan los siguientes resultados: próstata de 85 ce


con calcificaciones en su interior, compatible con hiperplasia prostática benigna.
Las biopsias prostáticas se reportan sin evidencia de malignidad.
¿Cuál de los siguientes tratamientos sería el más adecuado?
d} Adenectomía

34. ¿Cuál de las siguientes hormonas gastrointestinales inhibe la contracción de la


vesícula biliar?
a} Somatostatina

35. ¿A partir de qué momento se debe iniciar el tratamiento con FARME (fármacos
modificadores de la enfermedad) en pacientes con artritis reumatoide?
a} Desde elmomento deldiagnóstico

36. Paciente de 59 años de edad que acude al servicio de urgencias por dolor
torácico asociado con disnea y náusea; el paciente tiene dificultad para describir
exactamente el tipo de dolor. Él estaba bajo control por hipertensión y no se le
conoce ningún otro antecedente de importancia. Los signos vitales al momento del
ingreso son: frecuencia cardiaca de 60 latidos/min, presión arterial 140/115 mm
Hg, frecuencia respiratoria de 15 respiraciones por minuto. A la auscultación se
puede detectar un soplo diastólico. La exploración física no se pudo completar
adecuadamente ya que el paciente desarrolla un edema de pulmón agudo, con
aumento en intensidad del soplo diastólico. Se realiza manejo agudo del paciente
y estabilización. Se solicita radiografía de tórax portátil y un ecocardiograma en la
cama del paciente que demuestran la presencia de una raíz aórtica dilatada,
asociada con insuficiencia aórtica moderada. Se realiza una tomografía
computarizada de urgencia la cual confirma la presencia de una disección aórtica
tipo A que involucra la válvula aórtica y ocluye el ostium de la arteria coronaria
derecha.
¿Cuál de los siguientes hallazgos a la exploración física debe crear la sospecha
de disección aórtica aguda?
a} Dolor torácico anterior

37. ¿Cuál de las siguientes arterias se encarga de la irrigación de la mitad derecha


de la curvatura mayor del estómago?
b} Arteria gastroepiploica derecha
38. La amantadina es un fármaco activo contra la influenza tipo A (pero no contra
la influenza tipo B), no obstante, recientemente ha surgido un alto grado de
resistencia a este fármaco. ¿Cuál de los siguientes enunciados se asocia mejor
con el mecanismo de acción de la amantadina?
b}Inhibición dela descapsidación

39. Mujer de 30 años de edad, procedente de la ciudad de Puebla. Acude a


consulta por presentar dificultad para embarazarse. Refiere dolor pélvico leve
desde hace 12 meses. Presenta dolor a la movilización cervicaly anexial. Tiene un
familiar que hace cinco años tuvo mucha tos con hemoptisis y convivió con él por
dos años. ¿Qué es cierto sobre esta patología?
a} Las trompas son el primerlugar de afección

40. Paciente femenino, producto de segunda gesta, 41 semanas de gestación,


obtenido por parto vaginal eutócico, al momento de su nacimiento se observan
datos de dificultad respiratoria; por lo cual se procede a evaluar con la prueba de
Silverman. Indique cuál de los siguientes parámetros no es valorado por esta
evaluación:
a} Coloración

41. Femenino de 75 años acude a consulta externa por referir iniciar con cuadro
diarreico severo de 2 días de evolución. Antecedentes: hipertensión arterial sin
control y manejo médico. Exploración física: deshidratado, oligúrico, adinámico,
refiere visión borrosa. La tensión arterial sistólica es de 80 mmHg. Exámenes de
laboratorio reportan: urea en sangre 440 mg/dL y creatinina 13.5 mg/dL. La
excreción fracciona! de sodio es de 5%. Vd. Cilindros hemáticos en orina.
41a. Los datos clínicos de esta paciente corroboran el diagnóstico:
c} Necrosis tubular aguda

41b. La aparición de cilindros hemáticos en el sedimento urinario en esta paciente


corroboran:
d} Daño glomerular severo

42. Acude paciente femenino de 27 años de edad a consulta de control prenatal


por presentar embarazo de 1O semanas de gestación, no cuenta con
antecedentes de importancia, no toma ningún medicamento y no tiene factor de
riesgo alguno para el embarazo. Según la norma oficial mexicana, ¿cuántas
consultas como mínimo deben tenerse durante el embarazo?
b} 5

43. Usted se encuentra en una guardia de urgencias ginecoobstétricas y llega una


mujer de 36 semanas de gestación con trabajo de parto. Nunca había tenido
control prenatal previo. Con fondo uterino de 34 cm desde sínfisis del pubis. Palpa
dos dorsos con la segunda maniobra, con un polo cefálico encajado y el otro a
nivel del fondo uterino, con frecuencia cardiaca fetal de 150 y 135 latidos por
minuto respectivamente. Usted decide que nazcan vía cesárea sí
a} Monoamnióticos

44. Recién nacido de 20 horas de vida, producto de primera gestación, con tipo
sanguíneo B-, la madre tiene sangre tipo A+. ¿Qué es FALSO en relación con la
incompatibilidad ABO?
b} Nunca ocurre en elprimer embarazo

45. ¿En cuál de las siguientes situaciones se establece con certeza el diagnóstico
de diabetes mellitus?
e} Concentraciones plasmáticas sucesivas de glucosa en ayuno de 147, 165 y
152 mg/dl en una mujer asintomática

46. Niña de cinco años de edad con una dermatosis diseminada, bilateral y
asimétrica que afecta región axilar e inguinal, constituida por pápulas umbilicadas,
en forma de domo translúcidas, bien delimitadas. ¿A qué familia pertenece el
agente causal de esta dermatosis?
a} Poxvirus

47. ¿A qué se refiere el ""efecto del alba""?"


b} Hiperglucemia matutina causada por efectividad reducida de lainsulina en ese
momento

48. Un joven de 17 años de edad es traído a la sala de urgencias con diaforesis,


náusea,vómito y dolor abdominal en hipocondrio derecho. La madre refiere como
antecedentes principales depresión de 2 meses de evolución y el hecho de
encontrarlo en su cuarto junto a un frasco de pastillas no identificadas. En el
interrogatorio revela que tiene 5 h de evolución. Se toman exámenes de
laboratorio reportándose biometría hemática y química sanguínea normal,pero
pruebas de función hepática ligeramente aumentadas. Usted piensa en una
intoxicación.
48a. Con base en los datos obtenidos en el interrogatorio y los resultados de
laboratorio, ¿cuál es su principal sospecha diagnóstica de intoxicación?

b} Acetaminofén
48b. En un paciente con sospecha de intoxicación de este medicamento, ¿cuál de
los siguientes exámenes de laboratorio se deben realizar?"
d} Todos los anteriores

48c. ¿Cuál es la dosis tóxica del medicamento que sospecha es el responsable de


la intoxicación?
d} 140 mg/kg

48d. Según su sospecha clínica, ¿cuál es el tratamiento específico para esta


intoxicación?
c} N-acetil-cisteína

48e. ¿Cuál es la dosis del antídoto que se debe administrar (en este caso vía
intravenosa)?
b} 150 mg/kg en 60 min,después 50 mg/kg en 4 h

49. La principal e inicial medida terapéutica del estado hiperglucémico


hiperosmolar es:
b} La rehidratación

50. Acude a urgencias paciente femenino de 32 años de edad por dolor abdominal
tipo cólico que inició en fosa ilíaca derecha y ha ido incrementando en intensidad y
frecuencia;se asocia anorexia, astenia y adinamia. Refiere nueve semanas de
amenorrea secundaria. A la exploración física muestra abdomen con ruidos
intestinales disminuidos en frecuencia,abdomen depresible, con hiperestesia e
hiperbaralgesia, doloroso a la palpación en fosa ilíaca derecha,signo de psoas,
obturador y talopercusión negativos. A la exploración vaginal se observa
hemorragia leve, con dolor a la movilización del cuello uterino y se palpa masa en
anexo. Se decide realizar prueba de embarazo y ecografía transvaginal. ¿Cuál es
la localización más frecuente de esta patología?
b} Ampular

51. Masculino de 1O años de edad que es llevado a consulta externa por


presentar exantema maculopapular que comenzó en la cabeza (detrás de las
orejas) y se diseminó de forma cefalocaudal al resto cuerpo. La madre refiere que
el cuadro inició 5 días antes, con fiebre, conjuntivitis, tos, coriza y malestar
general, seguidos de lesiones eritematosas con un centro azul grisáceo en la
mucosa bucal. En la exploración física usted confirma la presencia de un
exantema maculopapular con áreas de confluencia.
51a. Con base en las características del paciente, ¿cuál es el diagnóstico más
probable?
d} Sarampión

51b. De acuerdo con su sospecha clínica, ¿cuál es el agente etiológico del


padecimiento de este paciente?
c} Paramyxovirus
51c. En relacióncon su sospecha clínica, ¿qué complicación es posible que se
presente?
d} Complicaciones respiratorias

51d. En relación con su sospecha clínica, ¿cuáles son los hallazgos que esperaría
encontrar en los exámenes complementarios?
c} Linfopenia

51e. De acuerdo con su sospecha clínica, ¿cuál es el tratamiento indicado para


este paciente?
c} Paracetamol,protección de la luz fuerte y antibióticos para complicaciones
52. Paciente masculino de 29 años de edad con dolor bilateral en las rodillas. Al
realizar la historia clínica menciona que todos los días trota una distancia de 8 a
1O km. ¿Cuál es la causa más probable del dolor de rodillas en este paciente?
a} Síndrome patelofemoral

53. Paciente masculino de 6 años y 9 meses de edad es traído a consulta.Como


antecedente se menciona candidiasis oral a los 3 meses de edad, por lo que fue
hospitalizado en dos ocasiones en las cuales recibió corticoesteroides y
antibióticos no especificados;además, cursó con múltiples infecciones en las vías
respiratorias altas. A la exploración física se observa paciente con apariencia a su
edad cronológica, peso y talla en percentil 25, tórax con estertores crepitantes en
ambas bases,desarrollo psicomotor adecuado. Por el antecedente de infecciones
de repetición, se le solicitaron exámenes que reportaron leucocitosis e
hipogammaglobulinemia.
53a. Con base en las características del paciente, ¿cuál es el diagnóstico más
probable?
a} Agammaglobulinemia ligada al cromosoma X

53b. ¿Qué hallazgos apoyan su diagnóstico en los estudios complementarios?


c} Concentraciones séricas de lgG, lgA, lgM elgE muy por debajo dellímite mínimo
normal y cuenta deleucocitos normal,pero ausencia de linfocitos B

53c. De acuerdo a su sospecha clínica, ¿cuál es la alteración genética asociada?


b} Proteína Btk

53d. En relación con su sospecha clínica, ¿cuál es el tratamiento que estaría


indicado en este paciente?
b} lnmunoglobulina intravenosa

53e. De acuerdo con su sospecha clínica, ¿cuál es la complicación que se podría


presentar en el paciente?
c} Parálisis por vacuna de polio

54. Paciente masculino de 65 años de edad con factores de riesgo cardiovascular


se presenta a la sala de urgencias con hemiplejía derecha, así como parálisis
facial derecha con preservación de los movimientos de la mitad derecha de la
frente. El paciente también muestra dificultad para la comunicación oral. El
paciente se encuentra alerta con frecuencia cardiaca de 88 latidos por minuto y
presión arterial de 175/115 mm Hg, sin dificultad ventilatoria y con una cifra de
glucosa capilar de 190 mg/dL. Los familiares refieren que el cuadro inició hace
aproximadamente 1 hora. Una tomografía computarizada cerebral resulta normal.
¿Cuál de los siguientes medicamentos tiene lugar en el tratamiento de este
paciente?
c} Aspirina
55. Si en un paciente con hipertiroidismo clínico y bioquímico el gammagrama
tiroideo con 1311 muestra captación del radiofármaco disminuida o ausente, el
diagnóstico es:

b} Tiroiditis subaguda

56. Un paciente ha perdido la habilidad para discriminar entre dos puntos de


presión aplicados directamente a la piel de la mano izquierda. ¿Cuál de los
siguientes hallazgos es más probable que también se presente en este paciente?
b} Lesión dellemnisco medio en el lado derecho del puente

57. Acude a consulta pediátrica un paciente de 12 años de edad que sufrió un


golpe en la región lateral de la rodilla derecha durante un partido de fútbol. A la
exploración física el paciente muestra abducción anormal de la rodilla al
movimiento pasivo y desplazamiento anterior excesivo de la pierna cuando la
articulación de la rodilla se coloca en ángulo recto. Se solicita resonancia
magnética de la región,la cual confirma daño del ligamento colateral medio y del
menisco medio. Es probable que exista una tercera estructura lesionada en este
paciente; durante la reparación quirúrgica, ¿dónde deberá insertarse esta tercera
estructura?
c} Medial y anterior

58. Una paciente de 50 años de edad con diabetes mellitus tipo 2 y antecedente
de síndrome de ovario poliquístico se somete a histerectomía total abdominal para
tratamiento y estadificación de carcinoma endometrial. ¿Cuál de las siguientes
estructuras está en riesgo de ser dañada cuando las arterias uterinas son ligadas?
e} Ureteros

59. Se presenta a consulta femenino de 14 años de edad que acude por molestia
en ojos con secreción gruesa. En el interrogatorio resalta historia de 2 días de
evolución con secreción purulenta amarilla viscosa que inició en el ojo derecho y
posteriormente se presentó en el ojo izquierdo. A la exploración física se observa
secreción amarillenta en ambos ojos y conjuntiva congestionada (véase Figura). El
resto de la exploración es normal,excepto por la hiperemia de faringe. El frotis del
exudado reporta neutrófilos y bacterias.

59a. Con base en las características dela paciente, ¿cuál es el diagnóstico más
probable?
c} Conjuntivitis bacteriana
59b. De acuerdo con su sospecha clínica, ¿cuál es el tratamiento que indicaría?
b} Antibióticos tópicos

59c. De acuerdo con su sospecha clínica, ¿cuál es la causa más frecuente?


a} Streptococcus pneumoniae

59d. En relación con su sospecha clínica,¿cuál es la complicación que se puede


presentar en esta paciente?
d} Manifestaciones sistémicas

59e. En conclusión, ¿cuál de los siguientes conjuntos de signos y síntomas orientó


su sospecha diagnóstica?
c} Exudado purulento, neutrófilos en frotis y faringe hiperémica

60. Femenino de 19 años con antecedente de hospitalizaciones frecuentes por


intoxicación alcohólica. En esta ocasión es ingresada después de haber estado
ingiriendo de manera constante bebidas alcohólicas desde hace cinco días. A la
exploración física, la presión arterial es de 110/70 mm Hg, frecuencia cardiaca de
80 latidos/min, peso de 70 kg. Se presenta letárgica y con habla incoherente. En la
sala de Urgencias, presenta una crisis convulsiva, que se yugula con diazepam.
Los estudios de laboratorio reportan sodio de 11O mEq/L, potasio sérico de 3.8
mEq/L, nivel de alcohol de 250 mg/dl, osmolalidad sérica de 230 mOsm/kg, y
concentración de glucosa de 92 mg/dl. No hay alteración en la función renal.
¿Cuál de las siguientes opciones representa el mejor tratamiento para esta
paciente?
a} Solución salina hipertónica (3%) intravenosa a una velocidad de infusión de
130 ml/h

61. Paciente masculino de 1 mes de edad es referido a su consulta por soplo.


Como antecedentes se menciona parto prematuro a las 35 semanas de
gestación,con cuidado prenatal deficiente. En la exploración física se identifican
pulsos periféricos saltones y soplo característico en maquinaria, se encuentra en
percentil 50. En la radiografía de tórax no se aprecia nada significativo.
61a. Con base en las características del paciente, ¿cuál es el diagnóstico más
probable?
d} Conducto arterioso persistente
61b. En relación con su sospecha clínica, ¿con qué problema médico se ha
asociado este diagnóstico?
c} Nacimiento pretérmino

61c. De acuerdo con su sospecha clínica, ¿qué hallazgos esperaría encontrar en


la radiografía de tórax?
a} Aumento de la vascularización pulmonar

61d. Conbase en su sospecha clínica, ¿cuál es el tratamiento indicado en este


paciente?
c} Administración deindometacina

61e. De acuerdo con su sospecha clínica, ¿cuál es la complicación que se puede


presentar a largo plazo?
d}Insuficiencia cardiaca en lactantes

62. Paciente masculino de 12 años de edad, quien este invierno presenta cuadros
repetitivos de infección de vías aéreas superiores, múltiples tratamientos
sintomáticos sin recibir antibióticos por considerarse cuadros virales y alérgicos;
hace 2 días y el día de hoy por la madrugada se despierta por accesos de tos
disneizante, cianozante, la madre refiere que se escucha un ""silbato"" cuando el
niño respira, mejora ligeramente cuando no tose y en reposo. A la exploración
física se encuentra paciente con taquipnea, tos, disnea que limita la actividad
física,sibilancias audibles a distancia, acrocianosis, aleteo nasal y tiraje intercostal
bajo, la orofaringe se encuentra hiperémica,con descarga posterior hialina,
hipertrofia amigdalina grado 2/4; resto de la exploración sin alteraciones evidentes.
FC 95 lpm, FR 22 rpm, temp. 36.?ºC, TA 100/70 mm Hg, peso 32 kg, talla 1.30 m."
62a. El diagnóstico clínico del paciente es:
c} Crisis asmática

62b. Al evaluar al paciente usted solicita una espirometría que reporta FEV1
>80%, junto con los datos clínicos puede clasificar esta exacerbación como:
b} Persistente leve

62c. Una vez establecido el grado de severidad, usted inicia tratamiento con:
d} Corticoesteroidesinhalados a dosis bajas y beta-2-agonistas delarga duración

62d. Al solicitar varios estudios, usted espera encontrar:

b} Biometría hemática con eosinofilia


62e. Se controla la crisis con el tratamiento establecido, al egreso del paciente
usted recomienda:
a} Beta-2-agonistas de corta acción
63. Paciente masculino de 20 años que pertenece al ejército acude a urgencias
por presentar tos no productiva desde hace una semana asociada con fiebre,
disnea y dolor torácico al respirar profundamente. Las radiografías PA y lateral de
tórax muestran un infiltrado intersticial bilateral. Usted diagnostica neumonía
atípica adquirida en la comunidad.
¿Cuá es el antibiótico de elección para el tratamiento de la neumonía atípica
adquirida en la comunidad?
c} Azitromicina

64. Lactante femenino de siete meses de edad que desde el nacimiento fue
diagnosticada con pie equino varo y ha sido tratada con múltiples yesos sucesivos,
que no han corregido en su totalidad el defecto. ¿Cuál es el tratamiento más
adecuado en este momento?
c} Tenotomía percutánea deltendón de Aquiles

65. Niña de tres años de edad que desde hace dos días presenta fiebre y otalgia
izquierda. Tiene antecedente de rinofaringitis hace cinco días. En el examen físico
se observa membrana timpánica izquierda eritematosa y edematosa, abultada, y
se observa líquido en oído medio. Hay pérdida del reflejo luminoso. Todas las
siguientes son potenciales complicaciones de la patología que presenta la
paciente, EXCEPTO:
e} Colesteatoma

66. Se presenta a consulta neurológica un paciente del sexo femenino de 55 años


de edad la cual presenta alteraciones de la coordinación y marcha tambaleante. A
la exploración física la paciente presenta nistagmo, disartria,y alteraciones en la
prueba dedo-nariz. Los estudios de laboratorio muestran elevación de los
anticuerpos anti-Yo. Se ordena una resonancia magnética la cual revela atrofia de
ambos lóbulos del cerebelo. ¿Cuál es la causa que con mayor probabilidad está
ocasionando la sintomatología de la paciente?
c} Cáncer de glándula mamaria

67. Paciente del sexo femenino de 45 años de edad que se presenta al Servicio de
Urgencias por presentar diarrea con moco y sanguinolenta. Al interrogatorio la
paciente niega cualquier antecedente que pudiese explicar su padecimiento
actual. A la exploración física se encuentra que en las piernas la paciente tiene
lesiones ulceradas con bordes violáceos; al tacto rectal se evidencia la presencia
de sangre en recto. La biopsia de colon revela inflamación en la mucosa y
submucosa. ¿Cuál de los siguientes medicamentos es el más adecuado para el
tratamiento de esta
paciente?
b} Sulfasalazina

68. ¿Cuál de las siguientes condiciones ocasiona una disminución de la


osmolaridad urinaria y un incremento del flujo urinario sin cambiar la fracción
excretada de sodio ni la depuración osmolar?
c} Administración de litio
69. Paciente masculino de 60 años de edad con diagnóstico reciente de
hipertensión arterial sistólica aislada. Después de tres semanas de tratamiento la
presión arterial del paciente se ha reducido a los valores esperados, sin embargo
el paciente menciona que presenta edema tibial bilateraly cuadros de
enrojecimiento facial. ¿Cuál de los siguientes medicamentos es el que más
probablemente está consumiendo este paciente?
a} Amlodipino

70. ¿Cuál de los siguientes efectos adversos corresponde a la administración de


mercaptopurina?
e) Dolor abdominal e ictericia

71. Paciente femenino de 87 años consulta por molestias al comer y dolor en la


boca del estómago; reporta que al ingerir alimento siente que le queman el
estómago y que tomar una pastilla para la gastritis le confiere alivio. Tiene
antecedentes de artritis desde hace 5 años, por lo que toma una pastilla de 500mg
de ASA al día, más dos de 275mg de naproxeno sódico/300mg de paracetamol.
Además, refiere haber bajado de peso. TA: 140/85, FC: 85x', FR: 18x', Temp:
37°C."
71a. ¿Cuáles su sospecha diagnóstica?
c} Enfermedad ácido péptica

71b. ¿Cuál es la causa más común de sangrado del tubo digestivo alto?
a} Enfermedad ácido péptica

71c. En esta paciente el factor de riesgo más importante es:


c} Uso de AINE

71d. ¿Cuál complicación de esta enfermedad representa mayor peligro (mayor


mortalidad)?
a} Perforación gástrica

71e. El tratamiento más adecuado para esta paciente es:


d} Omeprazol + inhibidores dela COX-2

72. Paciente femenino de 50 años, fumadora actual, en tratamiento de reemplazo


hormonal, se presenta a la sala de urgencias con dolor pleurítico y disnea. Ésta es
la segunda vez que acude al hospital en la última semana; hace cuatro días
acudió por una neumonía lobar de localización inferior en el pulmón derecho, la
cual fue diagnosticada utilizando radiografía de tórax. En esta ocasión a la
exploración física la pantorrilla izquierda de la paciente se presenta con edema y
eritema. Debido a estos datos, se sospecha de embolia pulmonar como la causa
del padecimiento actual de la paciente. ¿Cuál de los siguientes métodos
diagnósticos es el más adecuado en este caso?
e} Tomografía computarizada espiral contrastada deltórax
73. Paciente femenina de 65 años de edad. Consulta por cefalea y disnea de
esfuerzo. Al interrogatorio refiere dispepsia de larga evolución. A la exploración
física se registra TA: 120/80, FC: 11O lpm, palidez de tegumentos, disminución de
la sensibilidad vibratoria en miembros inferiores. Laboratorios: Hb: 5.5 g/dl, VCM:
120 ft, reticulocitos: 2%, LDH aumentada, leucocitos: 3 000, plaquetas: 85 000.
73a. Señale la causa más probable de la anemia en esta paciente:
b} Deficiencia de vitamina 812

73b. ¿Cómo se espera que sean los valores de ácido metilmalónico sérico y la
homocisteína total en un paciente con deficiencia de vitamina B12?
d} Ácido metilmalónico aumentado y homocisteína aumentada

73c. ¿Cuál es el estudio indicado para confirmar el diagnóstico de anemia


perniciosa?
a} Prueba de Schilling

73d. ¿En qué región del sistema gastrointestinal se absorbe la vitamina 812?
d} Íleon

73e. Se confirma el diagnóstico de anemia perniciosa. ¿Cuál es el tratamiento


indicado en el paciente?
c} Vitamina B12 parenteral

74. ¿Cuál de los siguientes enunciados es correcto respecto a la granulomatosis


de Wegener?

b} Los pacientes con enfermedad sinusal limitada se pueden beneficiar del


tratamiento crónico con trimetoprim/sulfametoxazol

75. Paciente masculino de 44 años de edad cuyo conteo más reciente de linfocitos
T CD4+ es de 55 células/mm3. Se decide iniciar tratamiento indicado en este
paciente; sin embargo después de varias semanas se presenta con anemia
megaloblástica. ¿Cuál de los siguientes fármacos es el que más probablemente
está causando este cuadro?
e} Trimetoprim-sulfametoxazol

76. Se presenta a la consulta externa un paciente de cuatro años de edad el cual


presenta celulitis, descamación de gran parte del cuerpo con signo de Nikolsky
positivo. ¿Cuál de los siguientes diagnósticos es el más probable en este
paciente?
e} Síndrome de la piel escaldada estafilocócica

77. ¿Cuál de los siguientes antiarrítmicos disminuye la fase O del potencial de


acción?
c} Propafenona
78. Paciente masculino de 45 años de edad con antecedente de alcoholismo
crónico es llevado al servicio de urgencias posterior a sufrir un leve traumatismo al
ser golpeado por un automóvil. Una vez en el hospital el paciente se diagnostica
con una fractura de cadera. Se decide realizar reparación quirúrgica. A pesar que
durante el posquirúrgico inmediato no suceden eventualidades, al cuarto día de
hospitalización el paciente desarrolla mialgias y debilidad. Los estudios de
laboratorio revelan elevación de la creatinina sérica,con hematuria sin glóbulos
rojos.
¿Cuál de los siguientes estudios de laboratorio revelará con mayor probabilidad la
causa de los síntomas de este paciente?
b} Medición de los niveles séricos de fosfato

79. Paciente femenino de 45 años de edad con antecedente de cáncer de


glándula mamaria. La paciente fue diagnosticada hace dos meses y se le realizó
una mastectomía radical con extirpación de ganglios cervicales. Posteriormente
fue programada para administración de quimioterapia. Posterior a la
administración de quimioterapia presenta náusea y vómito incontrolables. ¿Cuál
de los siguientes medicamentos es el más adecuado utilizar en este contexto para
disminuir la náusea y el vómito?
a} Ondansetrón

80. Paciente femenino de 22 años de edad que acude al servicio de urgencias


ginecobstétricas del hospital regional por hemorragia vaginal. La paciente se había
realizado una prueba de embarazo hace dos semanas la cual resultó positiva, sin
embargo, por algunas razones aún no había acudido para consulta de evaluación
y control prenatal. En la sala de urgencias se realiza un ultrasonido de útero el
cual revela la presencia de estructuras quísticas en forma de ""racimo de uvas"",
sin evidencia de un embrión. Si esta paciente presenta diagnóstico de mola
hidatidiforme completa, ¿cuál de los siguientes hallazgos sería más probable de
encontrar?
a} El genotipo de las células de la mola es 46,XX y es completamente de origen
paterno

81. ¿Cuál de las siguientes condiciones se asocia con disminución de la


capacidad pulmonar total, incremento del volumen residual, disminución de la
presión inspiratoria máxima y normalidad del índice fracción espirada de volumen
en el primer segundo/capacidad vital forzada?
d} Miastenia grave

82. Paciente femenino de 60 años de edad la cual es ingresada por cuarta ocasión
durante los últimos tres meses por presentar palpitaciones y disnea secundaria a
una fibrilación auricular con respuesta ventricular rápida. El equipo médico tratante
está considerando administrar amiodarona por tiempo indefinido para evitar este
tipo de ataques. ¿Cuál de los siguientes estudios es recomendable realizar antes
de administrar este medicamento?
a} Pruebas de función tiroidea

83. Paciente femenino de 19 años de edad que se presenta con menorragia desde
su menarca y recientemente ha sido hospitalizada por padecer anemia grave
(hemoglobina 6 g/dl). Los estudios de laboratorio demuestran niveles normales del
conteo plaquetario, tiempo de protrombina y del índice de internacional
normalizado. Entre las anormalidades se encuentran prolongación leve del tiempo
de tromboplastina parcial y del tiempo de sangrado. ¿Cuál es el diagnóstico más
probable en esta paciente?
e} Enfermedad de von Willebrand

84. Paciente masculino de una semana de vida, producto de una madre de 42


años de edad. El neonato presenta un fenotipo caracterizado por pliegues
epicánticos, implantación baja de orejas, abertura oral pequeña y pliegue palmar
único. A la exploración cardiovascular se puede percibir un soplo holosistólico en
todo el borde esternal. ¿Cuál de las siguientes anormalidades cromosómicas es la
que con mayor probabilidad causa este cuadro?
d} Mosaicismo

85. ¿Cuál de los siguientes trastornos representa una contraindicación para la


administración de concentrados plaquetarios?
c} Púrpura trombocitopénica trombótica

86. ¿Cuál de los siguientes fármacos se asocia con la vasodilatación de la


circulación esplácnica?
c} Dosis bajas de dopamina (2 a 4 µg/kg/min}

87. ¿Cuál de los siguientes medicamentos es una benzodiacepina de larga


duración de acción?
a} Clorodiazepóxido

88. Paciente masculino de 65 años de edad con diagnóstico de diabetes mellitus


tipo 2 y enfisema con antecedente de tabaquismo intenso. El paciente se presenta
al servicio de urgencias por un cuadro neumónico ocasionado por Streptococcus
pneumoniae. El paciente recibe tratamiento antibiótico adecuado y es egresado
sin eventualidades. Al momento del egreso se le recomienda abandonar el hábito
tabáquico. ¿Cuál de las siguientes medidas también se debe realizar en relación a
este paciente previamente a su egreso?
c} Administrar vacuna contra la influenza y el neumococo

89. Paciente masculino de 31 años de edad que se presenta a consulta por tener
una tumoración en el lado derecho del cuerpo, la cual ha aumentado
considerablemente de tamaño desde que apareció hace tres meses. Entre los
antecedentes de importancia, en este paciente se encuentran tabaquismo intenso
desde los 15 años de edad y consumo moderado de alcohol, llegando a la
embriaguez una vez por mes. Se realiza una biopsia de la lesión, la cual revela
una translocación t(14;18). ¿Cuál de los siguientes productos génicos es el que
corresponde a la translocación cromosómica que presenta este paciente?
a} Sobrexpresión de Bcl2

90. Paciente masculino de 45 años de edad sin ningún antecedente de


enfermedad hepática o digestiva. Durante una exploración física de rutina se
encuentra que tiene una tumoración abdominal en el cuadrante superior derecho y
que existe dolor a la palpación de ésta. Se realiza un ultrasonido el cual revela
hepatomegalia,así como la presencia de múltiples lesiones quísticas hepáticas. Al
interrogatorio el paciente comenta que hace algunas semanas viajó a visitar un
familiar en una granja,y tuvo contacto con perros y ovejas. ¿Cuál de los siguientes
agentes es el que con mayor probabilidad está ocasionando el cuadro de este
paciente?
d} Echinococcus granulosus

91. ¿Cuál es el mecanismo de acción por el que la selegelina puede mejorar la


sintomatología de la enfermedad de Parkinson?
b}Inhibición de la degradación de dopamina por la monoaminooxidasa tipo B

92. ¿Cuál de los siguientes medicamentos es el más adecuado en el caso de


pacientes con enfermedad de Wilson?
a} Penicilamina

93. ¿Cuál de los siguientes medicamentos se asocia con síndrome nefrótico como
efecto adverso?
c} Sales de oro

94. ¿Cuál de los siguientes trastornos se caracteriza por niveles en rangos


normales de hormona luteinizante y testosterona,aumento de la hormona
foliculoestimulante en combinación con un bajo conteo de espermatozoides?
c} Criptorquidia

95. Femenino de 35 años acude a Urgencias debido a un dolor en la fosa lumbar


derecha que se irradia hacia la ingle del mismo lado, pasando por el flanco
derecho. El dolor inició hace algunas horas y no ha cedido con la ingesta de
diclofenaco. La paciente refiere que es la primera vez que le sucede, pero ha
tenido desde hace días molestia al orinar y sensación de vaciado incompleto; se
observa afectada, sudorosa y cambia de postura continuamente. EF: 1.50m;85kg;
TA: 145/85; Temp: 36.5ºC.
95a. ¿Cuál es la principal sospecha diagnóstica?
c} Urolitiasis
95b. ¿cuál es el hallazgo más común en el examen general de orina en pacientes
con litiasis renal?
b} Hematuria

95d. El estándar de oro para el diagnóstico es:


b} TC

95e. Lito más frecuente:


c} Oxalato cálcico

96. Paciente de 46 años con diagnóstico de varios años de esquizofrenia. El


médico tratante decide iniciar un nuevo antipsicótico. Dos semanas después del
cambio de antipsicóticos el paciente se presenta a consulta de urgencia con un
cuadro neumónico grave. La citometría hemática revela reducción del conteo de
neutrófilos, basófilos y eosinófilos. ¿Cuál de los siguientes fármacos es el que con
mayor probabilidad pudo haber causado este trastorno?
b} Clozapina

97. Paciente femenina de 35 años, ama de casa, inicia hace 2 meses con malestar
general,fatiga, artralgia en ambos carpos y articulaciones interfalángicas distales, y
rigidez matutina. Dactilitis en el 2do dedo de la mano derecha. Lesiones:
hiperqueratosis y onicólisis en uñas de las manos de más de un año de evolución.
Dolor a la palpación de la articulación sacroilíaca izquierda.
97a. Señale el diagnóstico más probable con base en la clínica de la paciente:
b} Artritis psoriásica

97b. ¿Cual de los siguientes signos clínicos es característico de la artritis


psoriásica?
a} Acortamiento de los dedos

97c. Hallazgo radiográfico encontrado en la artritis psoriásica.


d}Imagen de ""lápiz en copa"" en articulacionesinterfalángicas"

97d. El diagnóstico de la artritis psoriásica se realiza con base en:


c} La clínica

97e. Tratamiento indicado en esta paciente para evitar la progresión de la


enfermedad.
a} AntiTNF

98. ¿Cuál de los siguientes enunciados respecto a la recurrencia de la infección


por Clostridium difficile es correcta?
b} Las recurrencias delasinfecciones por Clostridium difficile se asocian con
complicaciones serias

1OO. ¿Cuál de los siguientes movimientos es el que más frecuentemente


ocasiona dolor en pacientes con síndrome del manguito retador?
a} Abducción delbrazo

101. Se presenta a consulta externa paciente del sexo femenino de 20 años de


edad por presentar resequedad y ulceraciones en la piel que cubre las
prominencias óseas, principalmente las rodillas y los codos. Entre los hallazgos de
importancia a la exploración física se pueden observar placas pequeñas grisáceas
en la conjuntiva. ¿Cuál de los siguientes diagnósticos es el más probable en esta
paciente?
b} Deficiencia de vitamina A

102. Paciente femenino de 29 años de edad con antecedente de esquizofrenia.


Había sido internada debido a que sufrió varios episodios en los que decía que
escuchaba voces que la insultaban y le decían qué acciones llevar a cabo. Al
momento del egreso se le prescribió haloperidol.
¿Cuál sintomatología es la más probable que la paciente llegue a presentar?
d} Galactorrea

103. ¿Cuál de los siguientes hallazgos podría estar presente con mayor
probabilidad en pacientes con lesión de la división superior del nervio facial?
a}Incapacidad ipsilateral para fruncirla frente

104. Individuo masculino de 40 años, sin antecedentes heredofamiliares de


importancia, presenta dolor abdominal de inicio súbito que se irradia del flanco
derecho a la región inguinal,persiste aun en reposo; se acompaña de náusea,
sudación y urgencia urinaria. Signos vitales: temperatura = 37 C, frecuencia
respiratoria = 12 respiraciones/min, frecuencia cardiaca = 68 latidos/min, presión
arterial = 130/80 mm Hg. A la exploración física nada relevante. ¿Cuál es el
diagnóstico más probable?
d} Cálculo ureteral derecho

105. Una mujer de 45 años de edad que se presenta a consulta por padecer dolor
de hombro desde hace aproximadamente ocho semanas, el cual es más intenso
cuando acude a sus entrenamientos de tenis. Durante el reposo el dolor suele
desaparecer. A la exploración física el hombro se encuentra doloroso, con
limitación del movimiento activo y pasivo. En las radiografías de la región se
pueden observar calcificaciones por encima del trocánter mayor. Se le indica a la
paciente que lo más probable es que padezca tendinitis calcificada, por lo que
requiere tratamiento durante seis semanas con un cabestrillo, reposo, calor y
antiinflamatorios no esteroideos. Después del periodo de tratamiento la paciente
acude de nuevo a consulta, pero los síntomas persisten. ¿Cuál de las siguientes
opciones describe mejor el siguiente paso en el manejo de esta paciente?
e} Aspiración de la calcificación guiada por ultrasonido

106. ¿Cuál de los siguientes organelos se encuentra ausente en los eritrocitos


maduros, y por esta razón éstos son incapaces de sintetizar el grupo heme de la
hemoglobina, a pesar de que se les inyecte el mRNA correspondiente?
e} Mitocondria
107. Paciente femenino de 94 años, sin antecedentes de interés, salvo asma
extrínseca por epitelio de animales, osteoporosis y cirugía de prolapso uterino con
incontinencia urinaria de esfuerzo residual.¿Cuál de estos elementos no es
indispensable para ser un individuo continente?
d} No ser persona dela tercera edad

108. Paciente masculino con antecedente de cirugía de catarata seis días previos.
Acude por disminución visual,dolor y ojo rojo. Clínicamente se observa quemosis e
hipopión. La ecografía reporta vítreo con formación de seudomembranas y grosor
retinocoroideo de 1.8 mm. Estamos ante un caso de endoftalmitis posoperatoria,
considerada urgencia oftalmológica al igual que la quemadura por álcali y la
oclusión de arteria central de la retina. El agente causal más frecuente es:
a} S. epidermidis

109. Paciente masculino de nueve meses de edad que presenta dermatosis


localizada a región genital afectando escroto, periné y región perianal; que está
caracterizada por eritema, erosiones, escama y algunas zonas con costras
serohemáticas y maceración, de 1O días de evolución, caracterizando una
dermatitis del pañal. ¿Qué factores predisponen esta enfermedad?
a} Contacto prolongado con evacuaciones

11O. Paciente femenino de 75 años con diagnóstico de DM2 desde hace 15


años(en tratamiento con metformina e insulina), hipertensa desde los 45 años de
edad (en tratamiento con nifedipino e hidralazina), fumadora desde hace 45 años
(cinco cigarrillos diarios),acude a consulta por sus propios medios (caminando)
debido a que sufrió el día de ayer una caída desde su propia altura, de sentón,en
el baño.Según refiere, la caída la relaciona con una pérdida reciente de fuerza en
las piernas; también se queja de dolor en la región de las nalgas.No presenta
limitaciones del movimiento y comenta que ha perdido altura de manera reciente.
La EF: 1.58m, 65kg; TA: 140/80; Temp: 36.5ºC. A la exploración se registra dolor
en la zona de las nalgas, movilidad y fuerza de los miembros pélvicos sin
alteraciones, reflejo anal y continencia fecal y urinaria conservadas, así como dolor
a la palpación de la columna lumbar.
11Oa. ¿Cuál de los siguientes estudios sería de mayor utilidad diagnóstica?
d} Radiografías simples de columna vertebral

11Ob. En el estudio anterior se evidencia disminución de la altura anterior de los


cuerpos vertebrales T2 y T3, además de una xifosis prominente. De acuerdo
conesto, ¿cuálsería su diagnóstico?
b} Fractura vertebral por compresión
11Oc. Por el tipo de fractura y los antecedentes, usted solicita una densitometría
ósea, la cual se la reportan con un valor de T=-2.8.¿Cuálsería su diagnóstico?
c} Osteoporosis

110d. ¿Cuál es un factor de riesgo modificable para osteoporosis?


b} Tabaquismo
111. Se presenta a la sala de urgencias un paciente masculino de 75 años de
edad que fue encontrado desmayado en el baño por sus familiares. Sus hijas
refieren que el paciente es fumador y que previamente se le habían diagnosticado
niveles elevados de colesterol. A la exploración física el paciente presenta presión
arterial de 180/100 y frecuencia cardiaca de 90 latidos por minuto. Cuando se trata
de interrogar al paciente, éste muestra una expresión de confusión;cuando intenta
comunicarse, su habla es extremadamente fluida pero sin propósitos informativos,
sin patrones gramaticales correctos ni significado. ¿Cuál es el sitio lesionado más
probable en este paciente?
a} Enla mitad posterior de la circunvolución temporal superior y en la parte
adyacente dela circunvolución temporal media
izquierda

112. Paciente masculino de 58 años acude a consulta debido a que presenta


molestias en la vista; tiene miopía desde los 15 años y comenta que en las últimas
semanas le cuesta trabajo ver los objetos que están a sus lados e incluso no ve
cuando la gente se le acerca. Entre sus antecedentes, menciona que padece DM2
desde hace 5 años y que hace 1O años le hicieron una paquimetría y le dijeron
que tenía la córnea muy delgada, por lo cual no se operó. EF: 1.70m;85kg; TA:
145/85mm Hg; Temp: 36.5ºC y PIO: 23mm Hg; se observa en el fondo del ojo
hemorragia en astilla del disco óptico, sin datos de microaneurismas ni exudados.
112a. ¿Cuál es la principal sospecha diagnóstica?
c} Glaucoma de ángulo abierto

112b. NO son factores de riesgo para su sospecha diagnóstica:


a} Tabaquismo, raza asiática, hipermetropía

112c. Las fases de la historia natural de estetrastorno incluyen a todas las


siguientes, excepto:
b}Islote nasal

112d. El tratamiento médico correcto es:

b} Bloqueadores beta e inhibidores dela anhidrasa carbónica

113. Paciente masculino de 59 años de edad que acude a evaluación por padecer
disnea de medianos esfuerzos y dolor torácico. En el expediente clínico no hay
ningún antecedente de importancia para este padecimiento. A la exploración física
los pulsos de las carótidas se encuentran retrasados respecto al ciclo cardiaco, así
como disminuidos en intensidad. El punto de máxima intensidad cardiaco se
muestra hiperdinámico, pero sin desplazamiento. A la auscultación se encuentra
un soplo en crescendo-decrescendo a lo largo del borde esternal izquierdo con
máxima intensidad en telediástole. El componente aórtico del segundo ruido
cardiaco se encuentra ausente. En el electrocardiograma se encuentran datos en
relación a hipertrofia ventricular izquierda. La radiografía de tórax no muestra
incremento del tamaño de la silueta cardiaca y campos pulmonares sin
alteraciones. ¿Cuál de los siguientes estudios es el más adecuado para confirmar
el diagnóstico de este paciente?
c} Ecocardiografía

114. ¿Cuál de las siguientes complicaciones del infarto miocárdico se presenta


como dolor torácico recurrente dos días después del infarto?
d} Pericarditis

115. Con relación a la nefropatía por gota:


d} Con el tratamiento adecuado, se mantiene o mejora la función renal en 80%
delos casos

116. Femenino de 48 años obesa, acude a consulta externa por iniciar con dolor
tipo cólico en hipocondrio derecho, irradiante hacia la espalda, el cual se
acompaña de náusea,anorexia y temperatura de 38.3 ºC, niega ictericia.
Antecedntes: multigesta, con malos hábitos alimentarios. A la EF presenta signo
de Murphy positivo a la maniobra de Pron, palpación profunda dolorosa, rebote
positivo.
La entidad patológica responsable del cuadro clínico de esta paciente es:
b} Colecistitis aguda

117. Acude a Urgencias un paciente de 56 años diabético y con leucemia mieloide


crónica, actualmente en tratamiento con metformina y quimioterapia. Llega con
fiebre y taquicárdico. Se le realiza una biometría hemática,donde se registran
leucocitos de 11 OOO/mm3. Asimismo, se le practica hemocultivo, urocultivo,
cultivo de esputo y radiografía de tórax, todos los cuales resultan normales. Se
decide iniciar un tratamiento empírico con anfotericina B debido al alto riesgo que
presenta el paciente de una infección oportunista por hongos y a que los hongos
son difíciles de aislar.
117a. ¿Cuál es el riesgo que presenta el paciente con el nuevo tratamiento?
a} Acidosis tubular renal tipo 1

117b. ¿En cuál de los siguientes trastornos es común la nefrolitiasis y la


nefrocalcinosis?
a} Acidosis tubular renal distal
117c. En relación con el síndrome de Gitelman, ¿cuál de las siguientes oraciones
no es verdad?

c} Hay una disfunción global del túbulo proximal


117d. ¿Cuál de las siguientes asociaciones lo guiaría a usted al diagnóstico de
síndrome de Fanconi?
d} Glucosuria, aminoaciduria, acidosis metabólica e hipofosfatemia

117e. En cuanto al síndrome de Bartter, ¿cuál de los siguientes datos es


incorrecto?
b} Es autosómica dominante

118. Lactante de cinco meses de edad que recibió al nacimiento, de acuerdo con
el esquema de vacunación 2007, dosis de BCG y hepatitis B. Faltó a la cita de los
dos meses, quedando pendientes las vacunas propias de esta edad. Respecto a
la interrupción en las vacunas de hepatitis B,señale cuál es la conducta más
adecuada en este momento.
c} Administrar la segunda dosis en este momento y ocho semanas después
aplicar la tercera dosis

119. Masculino de 9 meses de edad es traído a consulta por fiebre de 38.7ºC e


irritabilidad de 2 días de evolución. La madre comenta que su bebé se manipula
mucho la oreja derecha. A la exploración física, el paciente se observa irritable, no
cooperador,con signos vitales estables; confirma la presencia de fiebre y realiza
una otoscopia. Observa membrana timpánica eritematosa y abombada, y al fondo
detrás de la membrana percibe niveles sugerentes de derrame purulento. Usted
hace el diagnóstico de otitis media.
119a. ¿Cuáles el organismo que más probablemente esté ocasionando este
cuadro?
b} Streptococcus pneumoniae

119b. De las siguientes opciones, ¿cuál es el mecanismo inicial que produce la


otitis media?
d} Alteración en la función dela trompa de Eustaquio

119c. En caso de contar con timpanometría, ¿qué tipo de curva tendría el paciente
descrito anteriormente?

b} Curva B

119d. ¿Cuál es el tratamiento de elección para este paciente?


c} Amoxicilina 90 mg/kg/día
119e. ¿En qué situación le recomendaría a la madre realizar una miringotomía con
colocación de tubos a su hijo?
b} 3 episodios en 6 meses o 4 en un año (con un episodio enlos últimos 6 meses}

120. Paciente femenino de 88 años de edad es hospitalizada por una caída en su


casa debido a tropezón. No puede caminar o soportar peso sobre su extremidad
inferior derecha. Las radiografías revelan que tiene una fractura desplazada del
cuello del fémur,sin demostrar algún otro hallazgo de importancia. En comparación
con otros pacientes similares con fractura de cadera y desplazamiento
intertrocantérico, ¿cuál de las siguientes condiciones es la que se encuentra en
mayor riesgo de sufrir?
b} Osteonecrosis

121. Paciente femenino de 22 años de edad sufrió una crisis convulsiva en el


servicio de urgencias que le ocasionó luxación del hombro derecho. El residente
de urgencias solicita una interconsulta ortopédica debido a que la paciente exhibe
dolor y limitación grave del rango de movilidad. Antes del episodio convulsivo, no
tenía ninguna alteración en el hombro. A la exploración física,tanto los
movimientos pasivos como activos están limitados, en especial por dolor. La
rotación externa está limitada a menos de 20 grados. ¿Cuál de las siguientes
opciones representa el mejor paso siguiente en el manejo y evaluación de esta
paciente?
a} Radiografía axilar

122. Paciente masculino de seis años de edad con síndrome de Down, acude a
consulta porque la madre refiere conducta hipoacúsica de dos meses de
evolución; niega haber presentado fiebre y otalgia, el diagnóstico más probable es:
b} Otitis media serosa

123. Paciente femenino de 22 años de edad, refiere traumatismo nasal de 3 horas


de evolución,de dirección frontal. Presenta edema moderado de dorso nasal,sin
laterorrinia,con crepitación de huesos propios; en la rinoscopia se observa
abombamiento en región septal anterior bilateral, doloroso a la palpación. El
diagnóstico más probable es:
c} Fractura nasal y hematoma septal

124. Paciente femenino de 71 años que acude a urgencias por náusea y vómito.
Refiere que ha vomitado en cuatro ocasiones, siendo de contenido alimenticio a
simplemente un líquido claro amarillento. En sus antecedentes lo único que refiere
esosteoartritis, tratada con paracetamol de 750 mg. Al IPAS menciona que
últimamente ha requerido tomar más de este medicamento debido a que su
osteoartritis la ha molestado mucho. Cuenta que ha requerido tomarlo en seis
ocasiones en las últimas 24 horas. La hija, que acompaña a la paciente, sin
embargo cree que lo tomó en más ocasiones. Niega fiebre, dolor abdominal,
diarrea o hematoquezia. A la exploración física sus signos vitales incluyen FC
113/min, FR 19/min,T 37.1 C y PA 115/68 mm Hg. No hay anormalidades a la
exploración física. A continuación se muestran estudios de laboratorio:

¿Cuál de las siguientes aseveraciones es correcta respecto a la intoxicación con


paracetamol?
c} El riesgo de muerte por falla hepática es prácticamente nulo si se administra N-
acetilcisteína en las primeras 8 horas posterior a la sobredosis

125. Paciente masculino de 63 años de edad que acude a consulta por dolor
torácico en varias ocasiones, el cual es prolongado y continuo. El paciente
describe el dolor como una sensación ardorosa,que ocurre más frecuente durante
la noche; a veces el paciente se despierta con disnea después de que ya había
conciliado el sueño. Aunque estos episodios no han ocurrido durante el ejercicio,
el paciente evita cualquier actividad física extenuante como medida preventiva. A
pesar de que en el pasado se le recetó nitroglicerina, el paciente no ha adquirido
el medicamento. En una ocasión, un amigo de él le administró nitroglicerina
durante uno de estos episodios de dolor retroesternal,ocasionando desaparición
del dolor. El día de ayer,tuvo una riña con uno de sus hijos, lo cual le ocasiono
dolor torácico de gran intensidad, el cual mejoró cuando se sentó. ¿Cuál de las
siguientes opciones es con mayor probabilidad el diagnóstico de este paciente?
d} Reflujo esofágico y espasmos

126. Paciente masculino de 29 años, carnicero. Acude a urgencias por presentar


tos persistente sin expectoración de seis meses de evolución, acompañada de
dolor retrosternal relacionado con los accesos de tos. Niega disnea, hemoptisis,
fiebre o afectación del estado general. La exploración física reveló presencia de
adenopatías submandibulares e inguinales, siendo el resto normal. La radiografía
de tórax mostró infiltrados alveolares bilaterales en campos superiores y
adenopatías en hilio izquierdo. La baciloscopia del esputo en tres determinaciones
fue negativa para BAAR, al igual que los cultivos para Mycobacteríum
tuberculosis. El Mantoux también fue negativo. Se realizo una
fibrobroncoscopia,con exploración macroscópica normal y la anatomía patológica
de la biopsia bronquial reveló granulomas no necrosantes compatibles con
sarcoidosis. Se inició tratamiento con corticoesteroides orales en dosis
descendentes, con buena respuesta clínica y radiológica.
En este caso se confundió el diagnóstico de tuberculosis con sarcoidosis. ¿Cuáles
son los otros diagnósticos diferenciales que hay que tener
en cuenta ante la sospecha de tuberculosis?
d} Bronquiectasias, linfoma, vasculitis, infección pulmonar por hongos, cáncer
pulmonar, sarcoidosis

127. Paciente masculino de 60 años originario de una zona endémica de


tuberculosis, proveniente de zona de bajos recursos. Ha trabajo en una fundidora
por varios años. Dentro de sus antecedentes personales patológicos padece
enfermedad pulmonar obstructiva crónica, ha recibido corticosteroides los últimos
seis meses. Ingreso a piso hace una semana evolucionando mal, con
empeoramiento de la falla respiratoria, ha requerido ventilación mecánica asistida
por una posible neumonía.
127a. El tubo endotraqueal se debería enviar para:
d} Tinción de Ziehl-Neelsen

127b. Si la tinción anterior fue positiva para muchos organismos que se tiñen de
rojo, ¿cuál sería el régimen terapéutico adecuado?
a} Rifampicina,isoniazida, vitamina 86, pirazinamida y etambutol

128. Paciente masculino de 35 años con antecedente de uso de drogas


intravenosas acude a urgencias por presentar fiebre y escalofríos de una semana
de evolución y disnea progresiva. Hace dos días presentó parestesias en el brazo
izquierdo con debilidad de inicio súbito que duró aproximadamente 3 h y se
resolvió de manera espontánea. Sus signos vitales son temperatura de 38.5 ºC,
pulso de 120 latidos por minuto, frecuencia respiratoria de 24 respiraciones por
minuto, presión arterial de 116/76 mm Hg. A la exploración física se ausculta soplo
sistólico 3/6 en el borde inferior esternal derecho y campos pulmonares bien
ventilados. Usted diagnostica endocarditis infecciosa y procede a tomar
hemocultivos previo a iniciar tratamiento antibiótico empírico.
¿Cuál es el agente etiológico más común en usuarios de drogas intravenosas y
que debe ser cubierto por el esquema empírico de antibióticos?
b} Staphy/ococcus aureus

129. Paciente masculino de 60 años de edad que acude por disminución de la


agudeza visual central de ojo derecho de cuatro días de evolución, la cual ha sido
lentamente progresiva. No tiene ningún otro síntoma asociado. Nunca ha sido
estudiado para enfermedades sistémicas. A la exploración oftalmológica se
encuentran agudeza visual de 20/200 ojo derecho y 20/30 ojo izquierdo,
discromatopsia y defecto pupilar aferente de ojo derecho. La presión intraocular de
ambos ojos es normal. La papila del ojo derecho está hiperémica, tiene edema
difuso y alrededor hay hemorragias en flama y estrechamiento arteriolar, la mácula
es normal. La fundoscopia de ojo izquierdo no muestra alteraciones. La
campimetría muestra un defecto altitudinal inferior del ojo derecho. El campo
visual del ojo izquierdo es normal. El diagnóstico es neuropatía óptica isquémica
anterior no arterítica.
129a. ¿Qué enfermedades sistémicas deben ser descartadas en este paciente, ya
que son factores de riesgo para el desarrollo de la neuropatía óptica isquémica no
arterítica?
d} A y B

129b. El diagnóstico diferencial más importante debido a que se considera una


urgencia oftalmológica es:
b} Neuropatía ópticaisquémica arterítica

1. Paciente masculino de cinco años de edad que se presenta con debilidad y


calambres musculares después del ejercicio. A la exploración física el paciente no
presenta ninguna anormalidad aparente, ni retraso mental o alteraciones en el
desarrollo. ¿Cuál de las siguientes enzimas es la que más probablemente se
encuentra deficiente en este paciente?
d} Glucógeno fosforilasa

2. ¿Cuál de los siguientes resultados de laboratorio es el que se esperaría


encontrar en un paciente dislipidémico en caso de que éste tenga adecuado
apego y respuesta al gemfibrozil?
b} Disminución intensa enla concentración de triglicéridos, ligera disminución en
la concentración de colesterol-LDL, y un ligero incremento del colesterol-HDL

3. Ante un niño de 2 días de vida con antecedente diagnostico fibrosis quística,


¿qué manifestaciones de patología digestiva se pueden encontrar?
a} Íleo meconial y prolapso rectal

4. ¿Cuál de las siguientes articulaciones es la más afectada en la enfermedad por


depósito de cristales de fosfato cálcico básico?
a} Hombro

5. Paciente femenino de 29 años de edad, con lateralidad manual derecha, que se


presenta al servicio de urgencias por pérdida súbita de fuerza en hemicuerpo
izquierdo, desorientada y con lenguaje farfulleante. Tiene el antecedente de dos
cuadros similares: el primero hace un año, que provocó hemiparesia faciocorporal
derecha y afasia motora, parcialmente reversible tras 24 horas, dejó como secuela
paresia de ambos miembros derechos; el segundo hace dos semanas, con
desorientación súbita y parálisis de miembro pélvico derecho. Éste último fue
reversible tras 30 minutos. Al interrogatorio a familiares ha cursado con úlceras
crónicas en miembros pélvicos y piel ""amoratada"". En la exploración física existe
tensión arterial 160/11O mm Hg, frecuencia cardiaca 90 latidos por minuto,
frecuencia respiratoria 28 respiraciones por minuto, temperatura 38 ºC. Tiene
puntaje por escala de Glasgow de 13 puntos. Se corrobora hemiparálisis
faciocorporal derecha, signo de babinsky ipsolateral, sin alteración en pulsos
periféricos y los ruidos cardiacos son normales.
5a. ¿Cuál es el diagnóstico más probable?
e} Evento vascular cerebral isquémico

5b. Además del estudio de tomografía axial de cráneo simple, los estudios de
mayor utilidad para el abordaje diagnóstico de esta entidad en un paciente joven
son:
anticitoplasma de neutrófilos, lipoproteínas de alta y baja densidad"
e} a y c son correctas

5d. El aspecto violáceo o amoratado en los miembros pélvicos de la paciente en el


contexto de su probabilidad diagnóstica causal de la trombofilia es:
d} Livedo reticularis

6. Recién nacido de término, con antecedente de distrés respiratorio después del


nacimiento, resultó sin complicaciones. A los 13 días de vida inicia con distensión
abdominal importante, intolerancia a los alimentos con vómito y evacuaciones
hemorrágicas abundantes. Se solicita una radiografía abdominal, en la que se
encuentra neumatosis intestinal.
6a. En relación con el diagnóstico que sospecha, todas las siguientes condiciones
se asocian a esta patología, EXCEPTO una, indique cuál:
e} Fibrosis quística

6b. Se decide iniciar manejo médico. Todas las opciones que se indican son
correctas EXCEPTO una, indique cuál:
b} Alimentación entera!

6c. ¿Cuál de las siguientes situaciones justificaría el manejo quirúrgico de este


paciente?

b} Neumoperitoneo

6d. Posterior a manejo quirúrgico el paciente egresa y continúa sin complicaciones


los primeros meses de vida. A los cuatro meses, inicia con distensión abdominal y
vómitos persistentes. De acuerdo con los antecedentes, ¿qué complicación puede
esperarse en este paciente?
a} Estenosis colónica

7. ¿Cuál de los siguientes enunciados respecto de la galactosemia es correcto?


b} La galactosemia es elresultado dela deficiencia de galactocinasa o galactosa-
1-fosfato uridiltransferasa

8. Un recién nacido de sexo masculino, con antecedente de síndrome de Down,


presenta datos de obstrucción intestinal al nacimiento, con imagen de dientes de
sierra a nivel de sigmoides en estudio baritado. Respecto a este caso clínico,
señale la opción FALSA.
c} La enterocolitis esinfrecuente como complicación de esta enfermedad

9. Paciente masculino recién nacido producto de un embarazo pretérmino de 29


semanas de gestación y peso de 930 g ha ingresado a la unidad de cuidados
intensivos neonatales. ¿Cuándo se deberá solicitar una consulta oftalmológica
para detectar de manera oportuna retinopatía del prematuro en este paciente?
c} Entre cuatro y seis semanas posteriores al nacimiento

1O. Niña de un año ocho meses de edad con antecedente de gastroenteritis viral
hace dos semanas, resuelta sin complicaciones, que inició hace dos días con
flatulencia y evacuaciones diarreicas acuosas y abundantes. Se encuentra en el
examen físico con aumento de los ruidos peristálticos y eritema perianal. En
relación con la enfermedad que presenta, ¿cuál es el tratamiento indicado?
e} Retirar alimentos con lactosa de la dieta

11. Niño de nueve años que inicia con dolor escrotal moderado hace 6 horas que
ha progresado en intensidad hasta ser intenso. Niega traumatismo en la región
escrotal. En el examen físico se encuentra paciente afebril, con signos vitales
estables, escroto de apariencia normal, dolor a la palpación más intenso en el polo
superior del testículo izquierdo, con reflejo cremastérico ausente.
11a. ¿Cuál de los siguientes hallazgos es patognomónico de la patología que
presenta el paciente?
c} Nódulo paratesticular en polo superior, azuly puntiforme

11b. Se solicitó una ecografía Doppler testicular, ¿qué hallazgos espera encontrar
de acuerdo con el diagnóstico?
a} Lesión hipoecogénica en polo superior testicularizquierdo con flujo ausente

11c. ¿Qué tratamiento está indicado para este paciente?


c} Reposo en cama y antiinflamatorios

12. Paciente masculino de 46 años acude a consulta por haber presentado esputo
hemoptoico que se autolimitó en cuatro días. Dentro de sus antecedentes destaca
el diagnóstico de tuberculosis pulmonar a los 26 años que se trató de manera
correcta,quedando lesión cavitaria residual de pared fina.
En la tuberculosis pulmonar la radiografía de tórax es útil porque:

b} Complementa el diagnóstico de tuberculosis

13. Paciente masculino de 45 años con antecedentes de hipertensión arterial en


tratamiento con captopril, hidroclorotiazida y nifedipina acude a urgencias tras una
semana de no haber tomado ninguno de sus medicamentos por razones
económicas. El paciente refiere que desde hace tres días ha tenido cefalea
progresiva, fosfenos, náusea e irritabilidad. A la exploración física se encuentra
agitado y desorientado, hipertenso con presión arterial de 210/130 mm Hg y el
examen de fondo de ojo revela papiledema y hemorragias, no hay déficit
neurológicos focales.
¿Cuál de las siguientes aseveraciones es falsa con respecto al tratamiento de la
emergencia hipertensiva?

b} El diazóxido cruzala barrera hematoencefálica ejerciendo un efecto directo


sobre la circulación cerebral

14. Una paciente de 32 años con antecedente de lupus eritematoso sistémico


acude a su ginecólogo por abortos espontáneos recurrentes. Ha tenido tres
embarazos, los cuales terminaron en aborto antes de la semana 1O de gestación.
Los padres no tienen ninguna alteración cromosómica y a la madre se le realizó un
histerosalpingograma y USG vaginal,que resultaron normales, además de cultivos
vaginales que fueron negativos. Presenta un VDRL positivo.
14a. ¿Cuál de los siguientes estudios hubiera servido para sospechar el riesgo de
abortos espontáneos?
a} Anticoagulante lúpico

14b. ¿Cuál de las siguientes manifestaciones clínicas es la más común en el


síndrome de anticuerpos antifosfolipídicos (SAAF)?
b} Trombosis venosa profunda

14c. En cuanto a las manifestaciones cardiacas en el SAAF, ¿cuál es la más


común?
a} Lesiones valvulares

14d. ¿Cuál es el tratamiento en pacientes con SAAF asintomático?


a} Sintratamiento o aspirina a dosis bajas

14e. ¿Cuál de los siguientes no es un criterio de SAAF catastrófico?


b} Confirmación de trombosis venosa recurrente

15. Se agrega ampicilina al tratamiento empírico de una meningitis cuando se


sospecha que el agente causal puede ser:
b} Listeria monocytogenes

16. Paciente femenino que se encuentra en puerperio mediato y solicita la


colocación de un dispositivo intrauterino. No tiene
contraindicaciones para su empleo. Usted decide realizar la técnica con insertor.
¿En qué otra situación se debe utilizar la técnica con insertor?
a} lntergenésico

17. El siguiente es un método adecuado para el diagnóstico de la fiebre tifoidea:


b} Mielocultivo enla segunda semana
18. Paciente masculino de 55 años de edad que sufrió un infarto con elevación del
segmento ST tratado mediante fibrinolíticos y heparina de bajo peso molecular.
Después del tratamiento menciona que el dolor torácico ha desaparecido, y el
electrocardiograma muestra resolución de la elevación del segmento ST.Tres días
después del infarto inicia con dolor torácico grave, descrito como ""desgarrante"".
¿Cuál de las siguientes opciones explica el cuadro del paciente a tres días del
infarto?
e} Todas las opciones anteriores pueden ocasionar el cuadro que ahora muestra
el paciente

19. Paciente femenino de 56 años con antecedente de tabaquismo (cuatro


cigarrillos diarios desde hace 15 años) y bronquitis crónica sin tratamiento
específico, inicia hace 24 h con fiebre de 39 ºC, escalofríos, cefalea, mialgias,
artralgias, tos y ardor faríngeo. Actualmente es invierno, por lo que usted
sospecha influenza estacional."
19a. ¿A qué familia pertenece el virus de la influenza?
b} Orthomyxoviridae

19b. ¿Cuál es la complicación más importante de la infección por influenza?


a} Neumonía

19c. ¿Cuál es la prueba más específica y sensible para el diagnóstico?


c} PCR en tiempo real

19d. ¿Cuál de los siguientes medicamentos está contraindicado en pacientes con


influenza menores de 18 años?
c} Aspirina

19e. ¿Qué tratamiento le administraría a la paciente del caso clínico?


c} Oseltamivir

20. Es el mecanismo fisiopatológico por medio del cual el hipercortisolismo induce


hipertensión arterial:
a} Aumento de volumen circulante por la retención de Na
21. Un hombre de 91 años que acude a urgencias por disnea de dos días de
evolución. En su historia clínica se refiere hipertensión, hiperlipidemias, hiperplasia
prostática benigna y enfermedad coronaria, habiéndose sometido a un bypass
coronario. Refiere historia de tabaquismo de 12 paquetes/año, habiendo dejado de
fumar hace 30 años. Sus signos vitales incluyen FC 98/min, FR 23/min, T 36.7 C y
PA de 140/88 mm Hg. A la exploración física se observa distensión venosa
yugular, hepatomegalia y edema 3+ de extremidades inferiores. Los campos
pulmonares se auscultan con murmullo vesicular. El electrocardiograma muestra
datos de hipertrofia ventricular izquierda. Se realiza un ecocardiograma, el cual
muestra una fracción de eyección de 60% con distensión auricular izquierda e
hipertrofia ventricular. Se realiza una radiografía de tórax que muestra datos de
atrapamiento de aire. No se observa enfermedad valvular. ¿Cuál de las siguientes
patologías es la causa de estos síntomas?
a} Falla cardiaca diastólica

22. Una mujer de 72 años de edad acude al departamento de urgencias por


disnea y palpitaciones. En sus antecedentes patológicos se refiere EPOC,
hipertensión, hiperlipidemia y enfermedad coronaria, habiendo sufrido un infarto
hace seis años tratado con stent. Sus medicamentos incluyen bromuro de
ipatropio, metoprolol, captopril,simvastatina y oxígeno en casa, el cual mantiene a
3 Umin. A la exploración física sus signos vitales muestran pulso rápido e irregular,
FR 31/min,T 38.2 C, PA 143/88 mm Hg y saturación de oxígeno de 82%. A la
auscultación pulmonar se escuchan roncantes espiratorios. Se realiza un
electrocardiograma, el cual se presenta a continuación:
¿Cuál es el diagnóstico más probable de esta paciente?
e} Taquicardia multifocal paroxística

23. Paciente femenino de 56 años, sin antecedentes de importancia, acude a


consulta por fatiga e intolerancia al ejercicio de una semana de evolución. Refiere
orina oscura en la micción el día anterior. A la exploración física se encuentra
pálida, levemente ictérica, bazo palpable. FC: 11O, FR: 26, Temp: 36 ºC. Se le
realizan estudios de laboratorio, en donde se reporta lo siguiente:
• Hemoglobina 6.2 g/dl
• Leucocitos 7 OOO/mm3
• Plaquetas 255 000/ml
• Reticulocitos 7%
• Cr .6 mg/dl
• Bilirrubina total 6.4 mg/dl a expensas de la indirecta
23a. ¿Cuál es el diagnóstico más probable?
c} Anemia hemolítica autoinmune

23b. ¿Qué anticuerpos están implicados en la fisiopatología de la enfermedad?


b} lgG

23c. ¿Qué prueba guiaría con certeza al diagnóstico del trastorno?


d} Coombs positivo y esferocitos

23d. ¿Cuál es el tratamiento ideal para esta paciente?


a} Esteroides o rituximab

23e. ¿Cuál de las siguientes anemias hemolíticas no se considera congénita sino


adquirida?
c} Hemoglobinuria paroxística nocturna

24. Un hombre de 73 años acude a consulta por hemoptisis de dos meses de


evolución. Refiere además que ha perdido 8 kg de peso, sin cambiar sus hábitos
alimentarios. Refiere tabaquismo de 40 paquetes/año. En sus antecedentes
patológicos refiere hipertensión y osteoartritis, tratadas con amlodipino y
paracetamol en caso de dolor.A la exploración física sus signos vitales son FC
81/min, FR 18, T 37.2 C y PA 133/82. La auscultación cardiaca revela S1 y S2 sin
agregados. No hay distensión venosa yugular.A la auscultación pulmonar se
escuchan estertores crepitantes en el pulmón inferior derecho. Se realiza placa de
tórax, que revela una masa de 3.5 cm. No se palpa hepatomegal ia o
esplenomegalia. Se solicitan estudios de laboratorio, que se muestran a
continuación:
Examen Resultado
Leucocitos 8 500/mm;j
Eritrocitos 4 200 000/mm;j
Hb 1 16.0 g/dL
HCT 1 48.0%
VCM 1 79 mm3
MHC 1 27 pg
CMHC 1 34 g/dL
RDW 14%
Plaquetas 220 OOO/mm3
Glucosa 123 mg/dL
Creatinina 1.1 mg/dL
BUN 10 mg/dL
Albúmina 4.0 mg/dL
Proteínas totales 6.2 mg/dL
Na 128 mEq/L
K 4.2 mEq/L
CI 104 mEq/L
HC03 1 24 mEq/L
Na urinario 32 mEq/L

¿Cuál de las siguientes sería la histología más probable de esta paciente?


c} Carcinoma de células pequeñas

25. Una mujer de 80 años acude a consulta por problemas de memoria. Refiere
que los últimos meses ha tenido mayor dificultad de recordar personas que
conoce, y de eventos que ocurrieron recientemente. Su hija, que la acompaña,
confirma estos acontecimientos. Niega cambios en la personalidad,
desinhibición,temblores, falta de fuerza o parestesias. Aún puede hacer la mayoría
de las actividades básicas, aunque su hija le ayuda a controlar sus medicamentos.
Los antecedentes patológicos de la paciente incluyen hipertensión, enfermedad
coronaria (habiendo sufrido un infarto de miocardio hace tres años, tratado con
stent), e hiperlipidemia. Al IPAS refiere fatiga, sensación de escalofríos,
estreñimiento, resequedad de cabello y de piel. A la exploración física se observa
una paciente obesa. Sus signos vitales incluyen FC 58/min, FR 14/min,T 36.0 C,
PA 112/68 mm Hg y saturación de oxígeno de 96%. No se detectan
anormalidades en la exploración neurológica. Al examen de Mini-Mental califica
27/30, fallando principalmente en su capacidad de recordar objetos. ¿Cuál de los
siguientes estudios daría el diagnóstico de la paciente?
a} TSH

26. Femenino de 24 años, cursa con embarazo de 37 semanas atendida en sala


de urgencias por presentar pérdida del estado de alerta posterior a crisis
convulsivas tónico-clónicas. Antecedentes: G-2,signos vitales con TA 170/120mm
Hg, Fe 95x', reflejos osteotendinosos aumentados, FCF de 132x' y edema
importante de miembros inferiores.
La entidad patológica causal de la sintomatología de esta paciente es:
a} Eclampsia

27. Paciente femenino de 6 años de edad que presenta lesiones dérmicas,


máculas y pápulas, que evolucionan a ampollas en horas. La madre informa que
su hija padeció infección de vías respiratorias superiores hace 2 semanas.
¿Cuál es el diagnóstico en este caso?
c} Varicela

28. Paciente masculino de 80 años de edad que acude a consulta externa llevado
por su hija. El paciente enviudó hace cuatro años, y desde entonces vive con su
hija. Refiere que ha presentado frecuencia y urgencia urinarias, así como nocturia
desde la muerte de su esposa, pero en los últimos meses el problema ha
empeorado. El paciente refiere que presenta dificultad para iniciar la micción.
Comunica que la próxima semana acudirá con el ortopedista porque desde hace
varias semanas ha presentado dolor lumbar que no mejora con reposo. El
paciente también refiere pérdida de peso en los últimos cuatro meses. ¿Cuál es el
medicamento más adecuado para el padecimiento principal de este paciente?
d} Flutamida

29. ¿Cuál de los siguientes eventos es el menos probable que se presente durante
un ataque agudo de asma?
b} La presión pleural durante lainspiración se vuelve más negativa

30. ¿Cuál de los siguientes vasos venosos se encarga de conducir sangre de la


circulación porta en el sistema cava a pesar de la obstrucción de la vena porta (p.
ej.,durante cirrosis hepática)?
e} Venas ácigos y hemiácigos

31. Los pacientes con cáncer de colon no polipósico hereditario presentan genes
con inestabilidad de microsatélites, lo que significa que contienen múltiples
regiones con fragmentos de DNA de cadena única y de estructura anormal. ¿Cuál
de las siguientes es el tipo de proteína anormal en estos pacientes?
a} Enzimas de reparación de apareamiento incorrecto

32. Paciente masculino de 15 años de edad el cual sufre una caída desde su
patineta. A la exploración física presenta una lesión sobre la porción lateral de la
rodilla, la cual permite observar parte de la cabeza del peroné. Se solicita una
radiografía de la región la cual es negativa para fractura. Cuando el paciente trata
de levantarse de la silla de ruedas en la que estuvo todo este tiempo es notable la
presencia de ""pie caído"" al intentar caminar.A la exploración intencionada tiene
incapacidad para la dorsiflexión o eversión del pie. ¿Cuál de las siguientes
estructuras es la que más probablemente se encuentra lesionada?
b} Nervio peroneo común

33. Paciente masculino de 45 años de edad con antecedente de infección por el


virus de la inmunodeficiencia humana se presenta con eritema de 12 mm e
induración de 7 mm a la prueba cutánea de la tubercul ina utilizando 5 unidades
de PPD. La radiografía torácica no muestra anormalidades. El paciente se
encuentra bajo tratamiento antirretroviral altamente activo y no existe
sintomatología. ¿Cuál de las siguientes aseveraciones representa la conducta más
adecuada?
c}Iniciar tratamiento con isoniazida por 12 meses

34. ¿Cuál de las siguientes condiciones es un factor protector para el desarrollo de


neumonía por Legionella?
c} Neutropenia

35. Paciente masculino de 29 años de edad sufre un accidente por colisión de su


motocicleta contra un automóvil. El paciente se presenta con fractura de la diáfisis
derecha de fémur, sin trauma craneoencefálico ni pérdida del estado de alerta
debido a que utilizaba casco protector; 24 horas posteriores a su admisión el
paciente se presenta con infiltrados pulmonares bilaterales y difusos. ¿Cuál es la
etiología más probable de los infiltrados pulmonares?"
c} Embolia grasa

36. Paciente femenino de 78 años de edad con diagnóstico de diabetes mellitus


tipo 2 desde hace 22 años. El paciente carece de apego al tratamiento, y debido a
que le han comentado que la insulina es muy ""dañina"", ha preferido continuar
esporádicamente con tratamiento a base de bebidas de medicina alternativa y
metformina. Desde hace varios meses el paciente notó la presencia de una úlcera
sobre la cabeza del tercer metatarso del lado derecho. La úlcera ha crecido, y
recientemente el paciente inició con dolor en la zona afectada por lo que acude a
consulta. A la exploración física se encuentra una úlcera de aproximadamente 4
cm de diámetro, con olor y secreción pútrida. Al parecer la úlcera tiene una
profundidad de 3 cm y afecta estructuras óseas. Se toma una muestra de la base
y se envía a tinción de Gram la cual reporta cocos grampositivos en cadenas,
bacilos gramnegativos, diplococos gramnegativos, gramnegativos pleomórficos y
exceso de neutrófilos. ¿En este momento, cuál es el tratamiento más adecuado en
lo que es posible obtener mientras se esperan los resultados de hemocultivos y
cultivos de la secreción de la herida?"
e} Ampicilina con sulbactam

37. Un paciente se presenta con botulismo posterior a la ingesta de miel


contaminada. ¿Cuál de los siguientes hallazgos es factible en este paciente?
b} Diplopía

38. ¿Cuál es el principal efecto secundario de la administración de zidovudina en


pacientes con infección del virus de la inmunodeficiencia humana?
d} Neutropenia

39. Paciente de ocho años de edad acude a la consulta pediátrica oftalmológica


debido a problemas de visión. A la exploración ocular, se revela una subluxación
del cristalino. Entre otros hallazgos el paciente presenta extremidades largas y
disminución generalizada del tono muscular. Los padres también mencionan que
su hijo presentó retraso en el desarrollo con leve retraso mental. Los estudios de
laboratorio indican elevación de los niveles de homocisteína en sangre y orina.
¿Cuál de las siguientes condiciones es la responsable de los signos y síntomas de
este paciente?
a} Deficiencia de cistationina sintasa

40. Paciente masculino de 54 años de edad que acude al departamento de


urgencias por dolor torácico de tipo ""desgarrante"" que inició hace 1 hora. El
paciente refiere que el dolor tiene irradiación hacia la región interescapular,y que
sufrió un episodio sincopal de 2 minutos que coincidió con el inicio del dolor
torácico. En el expediente clínico menciona que es conocido por padecer
síndrome de Marfan y dilatación de la raíz aórtica. En sus dos consultas de
seguimiento previas muestra cifras tensiónales elevadas (155/100 y 160/99 mm
Hg). A la exploración física presenta presión arterial de 180/95 mm Hg en la
extremidad escapular derecha y de 100/60 mm Hg en el miembro escapular
izquierdo, así como taquicardia de 105 latidos/minuto; la temperatura y frecuencia
respiratoria se encuentran dentro de los rangos de la normalidad. A la auscultación
cardíaca se puede percibir un soplo diastólico 11/IV en el borde esternal inferior
izquierdo. El ecocardiograma demuestra disección de la aorta torácica con
involucro de la aorta torácica ascendente y descendente. Utilizando únicamente la
información previamente descrita, responda las siguientes preguntas:"
40a. ¿Cuál de los siguientes fármacos es el más adecuado para este paciente en
este momento?
a} Labetalol
40b. ¿Cuál de las siguientes opciones corresponde al tipo de de disección aórtica
de este paciente?
a} DeBakey tipo 1

40c. ¿Cuál de las siguientes opciones es el tratamiento definitivo más adecuado


para este paciente?
a} Cirugía urgente.

41. ¿Cuál de los siguientes fármacos actúa mediante la inhibición de la síntesis del
DNA en infecciones fúngicas?
b} Flucitosina

42. Lactante masculino de diez meses es traído a urgencias por su madre por
presentar diarrea acuosa de dos días de evolución asociada a sed intensa e
irritabilidad. Se encuentra taquipneico, taquicárdico, con los ojos hundidos, llanto
sin lágrimas, lengua seca,saliva espesa, signo del lienzo húmedo y llenado capilar
de 4 segundos.
¿Cuál es el tratamiento adecuado para tratar este caso de deshidratación?
e} Administrar Vida Suero Oral 100 ml por kilogramo de peso, cada 30 minutos
durante cuatro horas enla unidad de salud"

43. Paciente masculino de 55 años de edad que es ingresado al servicio de


Cardiología por presentar endocarditis infecciosa. En los cultivos se ha identificado
cepas de Staphylococcus aureus; los resultados del antibiograma se encuentran
pendientes. El paciente refiere antecedente de choque anafiláctico a la
administración de penicilina.
Utilizando únicamente la información previamente descrita, responda las
siguientes preguntas:"
43a. ¿Cuál de los siguientes tratamientos es el más adecuado para este paciente
en este momento?
a} Administración de vancomicina intravenosamente

43b. Posterior a la conducta realizada en la respuesta anterior el paciente


presenta eritema en varias partes del cuerpo, pero particularmente en el cuello, sin
otro tipo de sintomatología. ¿Cuál es la conducta terapéutica más adecuada en
este momento?
b} Disminuirla velocidad deinfusión delmedicamento

43c. Después de tratamiento adecuado, los síntomas del paciente se resuelven y


es dado de alta. Seis meses después, el paciente acude a consulta y pregunta si
es necesario que tome profilaxis antibiótica pues se someterá a una intervención
dental. ¿Cuál de las siguientes respuestas es la más adecuada para este
paciente?
a} Sí debe someterse a profilaxis previa al procedimiento dental,pues claramente
existe una indicación por sus antecedentes.
43d. En caso de que sea necesario utilizar profilaxis contra endocarditis infecciosa
previamente al procedimiento dental ¿Cuál de las siguientes opciones es la más
adecuada para este paciente?
b} Clindamicina

44. Paciente femenino de 40 años de edad con diagnóstico de embarazo de 14


semanas de gestación. Como antecedentes de importancia tiene los siguientes: 2
embarazos, en los cuales no hubo complicaciones, los dos fueron partos. Los
productos sin alteraciones. Acude por estar preocupada, ya que sabe que los
embarazos a edades mayores se relacionan con alteraciones en los niños. ¿En
qué momentos de la meiosis se realizan las dos detenciones en la ovogénesis?"
b} Diplotena y metafase II

45. Te encuentras explorando a una paciente embarazada con 33 semanas de


gestación,con fondo uterino de 26 cm desde sínfisis de pubis, situación transversa
y decides auscultar foco fetal. ¿A partir de qué semana de gestación se puede
percibir el latido fetal a través del Pinard?
d} 20 semanas de gestación

46. Neonato de 12 días de vida, que fue dado de alta en condiciones estables, y a
partir del décimo día de vida presentó episodios de fiebre, con temperaturas entre
38 y 39.7 ºC, hiporreactividad, rechazo al alimento, ¡DATOS DE MENINGITIS! De
los siguientes agentes etiológicos: 1)Staphylococcus epidermidis, 2)
Staphylococcus aureus, 3) L. monocytogenes, 4) H. influenzae. ¿Cuáles dos
agentes se relacionan más con sepsis tardía?"

e} 1 y 2

47. Acude a urgencias primigesta de 19 años de edad en segundo periodo de


trabajo de parto. El producto se encuentra en el tercer plano de Hodge con una
variedad de posición occipitoposterior. Lleva una hora en el segundo periodo e
inició con datos de sufrimiento fetal. Sabiendo que no hay anestesiólogo. Qué
acción se realizaría en este momento?
c} Uso de fórceps

48. Llega a urgencias paciente femenino de 38 años de edad primigesta, por


haber presentado crisis tonicoclónica generalizada hace 30 minutos. Refiere el
marido que previamente había tenido una cefalea muy intensa, fosfenos y dolor en
el epigastrio. Se encuentra en la semana 24 de gestación. TA de 180/11O mm Hg,
estuporosa, en anasarca, ROT incrementados. ¿Qué medicamento
anticonvulsivante es el de"
elección?
d} Sulfato de magnesio
49. Acude a consulta una mujer con amenorrea secundaria, que no menstrúa con
progesterona, pero sí con la combinación de estrógeno y progesterona,con TSH y
prolactina normales, pero con FSH y LH disminuidas y prueba de embarazo
negativa. Con los datos previos. ¿Qué patología podría tener esta paciente?"
c} Tumor hipofisiario

50. De acuerdo con la clasificación de la Universidad de Texas, una úlcera


superficial,infectada y no isquémica en el pie de un paciente diabético corresponde
a:
a} 18

51. Es el primer fármaco de elección en el tratamiento de la diabetes tipo 2 y


obesidad.
d} Metformina

53. Hombre de 31 años, con historia familiar de diabetes tipo 2 en padre y abuelo
paterno, hipertensión arterial sistémica en madre, infarto agudo de miocardio en
abuelo paterno; con diagnóstico de hipertensión hace tres años tratado con
enalapril. Desde hace aproximadamente 15 años ha presentado aumento
progresivo de peso (15 kg). A la exploración física: peso 140 kg, talla 1.85 m,
índice de masa corporal 41.22 kg/m2, tensión arterial 110/80 mm Hg, pulso 78x,
con obesidad de predominio central,acrocordones y acantosis pigmentaria en
región posterior del cuello y en ambas axilas, tiroides palpable y normal. Abdomen
globoso por panículo adiposo sin estrías. Fuerza muscular conservada. Resto sin
alteraciones. En sus exámenes de laboratorio destacan: glucosa 99 mg/dl,
creatinina 0.61 mg/dl, urea 37.1, ácido úrico 9.73 mg/dl, AST 54 Ul/L, ALT 65
Ul/L,bilirrubinas y fosfatasa alcalina dentro de parámetros normales, colesterol
total 202 mg/dl, triglicéridos 247 mg/dl.

53a. El diagnóstico sindromático del paciente es:


b} Síndrome metabólico

53b. La presencia de acrocordones y acantosis pigmentaria se debe a:


b} Resistencia a la insulina

53c. ¿Qué examen paraclínico es el más indicado en este caso?


b} Curva de tolerancia ala glucosa oral

53d. El perfilviral de hepatitis es negativo y en un ultrasonido de hígado y vías


biliares no se reporta obstrucción de la vía biliar ni neoplasia hepática. El
diagnóstico más probable es:
c} Esteatohepatitis no alcohólica

54. Paciente masculino de 4 años de edad se presenta al departamento de


Urgencias por dolor abdominal. En el interrogatorio se refiere cuadro viral hace 2
semanas, dolor abdominal de pocas horas de evolución,vómito, oliguria y erupción
cutánea extensa. En la exploración física se encuentra un exantema
maculopapular en las nalgas y extremidades inferiores que no blanquea con la
digitopresión. La biometría hemática es normal, pero el examen general de orina
muestra hematuria."
54a. Con base en las características del paciente, ¿cuál es el diagnóstico más
probable?"
d} Púrpura de Henoch-Schonlein

54b. ¿Qué hallazgos en los exámenes de laboratorio apoyan su sospecha


diagnóstica?
b} Biometría hemática normal,sangre oculta en heces"

54c. De acuerdo con su sospecha clínica,¿cuál es la complicación que se puede


presentar a largo plazo?"
c}Invaginaciónintestinal

54d. Con base en su sospecha clínica, ¿cuál es el tratamiento indicado?"


a} AINE

54e. ¿Qué aspectodel cuadro clínico del paciente es relevante mencionar a los
padres?
d} Hematuria microscópica

55. Paciente masculino de 88 años con múltiples condiciones médicas (angina


estable, enfermedad pulmonar obstructiva crónica, osteoartritis, diabetes mellitus
tipo 2, enfermedad acidopéptica) ingresa a la sala de urgencias por presentar
dolor abdominal de inicio abrupto de 6 horas de evolución. El paciente informa
dolor abdominal difuso, con leve localización en la región periumbilical. A la
exploración física se registran frecuencia cardiaca de 120 latidos por minuto,
presión arterial de 100/60 mm Hg, frecuencia respiratoria de 30 respiraciones por
minuto e hipotermia. A la palpación abdominal se encuentran signos de irritación
peritoneal. Las radiografías de tórax y abdomen demuestran la presencia de aire
subdiafragmático. ¿Cuál es el vaso sanguíneo más frecuentemente afectado en
casos de perforación de úlcera duodenal en la pared posterior del duodeno
(primera porción)?"
b} Arteria gastroduodenal

56. Paciente femenino de cinco años que es traída por sus padres tras ingerir
accidentalmente limpiador de drenaje guardado en una botella de refresco. La
paciente está quejumbrosa,con dolor torácico intenso, náusea, disfagia y sialorrea
profusa. A la exploración física se observan quemaduras de los labios, edema y
eritema de lengua y paladar. ¿Cuál de las siguientes está contraindicada en el
manejo de este caso?"
d} Lavado nasogástrico

57. Paciente femenino de 67 años de edad con diagnóstico de hipertensión arterial


sistémica, bajo tratamiento con captopril 25 mg cada 12 horas, actualmente
descontrolada, inicia hace 24 horas con epistaxis profusa que no se autolimita ni
cede a la compresión por lo que acude a urgencias hace 6 horas donde se le
coloca taponamiento nasal anterior y es dada de alta. Tres horas después regresa
por goteo hemático a través del tapón nasaly sensación de descarga hemática
posterior, diagnosticándose epistaxis posterior. ¿Cuál es el origen del sangrado?"
b} Arteria esfenopalatina

58. Paciente masculino de 2 años de edad es llevado a consulta de rutina. Se


mencionan como antecedentes que nació a las 37 semanas de gestación, fue
alimentado de manera exclusiva mediante seno materno durante el primer año de
vida y que actualmente no es bueno comiendo. Su peso y estatura se encuentran
en el percentil 5. A la exploración física se observa que sus piernas están algo
arqueadas y el hueso frontal es muy prominente. No tiene dientes y todavía no
camina por sí solo."
58a. Con base en las características del paciente, ¿cuál es el diagnóstico más
probable?"
b} Deficiencia de vitamina D

58b. En relación con su sospecha clínica,¿cuál de las siguientes opciones


representa una característica particular del padecimiento?"
a} Extremos distales ensanchados con forma de copa y con desgaste en la
radiografía

58c. Respecto a su sospecha clínica, ¿qué función tiene en el cuerpo la vitamina


que se encuentra con deficiencia en el paciente?"
c} Depósito de calcio

58d. De acuerdo con su sospecha diagnóstica, ¿qué tratamiento le indicaría al


paciente?"
d} Suplemento de vitamina O

58e. Referente a su sospecha clínica, ¿cuál es el mecanismo de prevención que


se debió realizar para evitar esta patología?"
b} Exposición al sol

59. Mujer de 54 años de edad con dermatosis bilateral y con tendencia a la


simetría que se disemina a ambas caras pretibiales. Se caracteriza por tres placas
infiltradas amarillo violáceas y esclerodermiformes, irregulares en la forma y mal
delimitadas, que miden entre 5 y 7 cm. La paciente refiere que sus lesiones
iniciaron hace un mes como ""granitos"" de color rojo violáceo y otras como
nódulos de forma redondeada, de bordes no bien definidos y a veces elevados.
Fueron creciendo de forma lenta y excéntrica, tornándose de color amarillo, con
telangiectasias en la superficie. Una de las lesiones se ulceró, dejando cicatriz y
atrofia posterior.A la exploración física presenta en las lesiones sensibilidad
disminuida al pinchazo y al tacto leve, hipohidrosis y alopecia parcial. La paciente
es diabética de larga evolución y al parecer lleva un buen control. Niega
antecedentes de traumatismos o de infecciones previas. El diagnóstico es:
b} Necrobiosis lipoídica
60. Paciente femenino de 32 años de edad, quien tiene como antecedentes
relevantes crisis convulsivas por cisticercosis, previamente tratada con valproato;
hace 4 semanas se hizo cambio por carbamazepina. Hace 3 días inicia con
malestar general, astenia, adinamia, hiporexia y hepatalgia, el día de hoy inicia
con eritema morbiliforme generalizado, motivo por el cual acude a consulta. FC 95
lpm, FR 18 rpm, temp. 38.6ºC, PA 110/70, peso 58 kg, talla 1.68 m."
60a. Con los datos clínicos anteriores, usted realiza el diagnóstico de:"
b} Síndrome de DRESS

60b. Estadísticamente, ¿qué órgano se afecta con mayor frecuencia en el


síndrome de DRESS?"
b} Hígado

60c. Usted solicita una biometría hemática que reporta eosinofilia > 1.5 x 109 L,
los laboratorios que solicita reportan elevación de TGO, TGP y DHL, con tiempos
de coagulación prolongados. Usted solicita una biopsia de piel,¿qué hallazgos
espera que se reporten?
a} Paraqueratosis, acantosis local e infiltrado linfocitario con epidermotropismo"

60d. Se debe iniciar el tratamiento, empezando por retirar el fármaco que provocó
el síndrome. Usted inicia el tratamiento con:"
d} Esteroides

60e. Se han documentado diversos datos de severidad del síndrome de DRESS,


¿cuál se asociacon mayor mortalidad?"
b} Necrosis hepática

61. Paciente masculino de 31 años de edad con antecedente de dos semanas de


fiebre, a lo cual se agregó hace una semana distensión abdominal,estreñimiento y
dolor epigástrico. Entre otras molestias también refiere cefalea, escalofríos,
mialgias y malestar general. La exploración física revela la presencia de un
""exantema rosado"" (exantema maculopapular color salmón que blanquea a la
presión), principalmente en el tórax, así como dolor abdominal generalizado a la
palpación y esplenomegalia. Presenta temperatura de 38.9 ºC, frecuencia cardiaca
de 45 latidos por minuto, frecuencia respiratoria de 20 respiraciones por minuto,
presión arterial de 100/65 mm Hg. Los estudios de laboratorio básicos son
relevantes por leucopenia y neutropenia. Se realiza un hemocultivo el cual resulta
positivo para Salmonella typhi. ¿Cuál es el tratamiento antibiótico más efectivo en
cepas no provenientes de brotes resistentes?
a} Fluoroquinolonas

62. Un paciente de 21 años se presenta a la sala de urgencias posterior a una


lesión en su hombro durante un partido de fútbol. A la exploración física el hombro
parece deprimido y plano y no es capaz de abducir su brazo. Una radiografía de la
región demuestra que el paciente tiene fractura del cuello quirúrgico del húmero.
¿De dónde proviene la inervación del músculo afectado en este paciente?
b} C5 a C6

63. Un paciente de 45 años con sobrepeso es llevado a la sala de urgencias con


pérdida del estado de alerta después de un accidente automovilístico. Luego de
recuperar la conciencia, la exploración física revela debilidad de ambas
extremidades derechas con signo de Babinski positivo en el pie derecho. El
paciente muestra la lengua desviada hacia la izquierda cuando la protruye. A la
evaluación de la sensibilidad corporal,el paciente ha perdido la sensibilidad
vibratoria en el lado derecho del cuerpo. ¿Cuál es el sitio más probable de lesión
en"
este paciente?
a} Lesión de la región medial de la médula oblongada

64. Paciente femenino de 60 años de edad con diagnósticos previos de


hipertensión,diabetes tipo 2, miocardiopatía isquémica,e insuficiencia renal
crónica; reporta dolor en la rodilla derecha. La presión arterial es de 140/80 mm
Hg, con una frecuencia cardiaca de 66 latidos/min. Debido al edema y al dolor en
la rodilla, se le receta celecoxib 200 mg/día. Después de dos semanas bajo este
tratamiento, la paciente refiere disnea, edema de extremidades inferiores y fatiga.
La presión arterial en esta ocasión es de 180/105 mm Hg, nitrógeno ureico de 67
mg/dL, creatinina sérica de 3.9 mg/dL. ¿Cuál de los siguientes diagnósticos es el
mecanismo de acción más probable por el cual el analgésico administrado
ocasionó insuficiencia renal?"
a} Pérdida del balance entre prostaglandinas, ocasionada por la inhibición de la
ciclooxigenasa tipo 2"

65. Paciente masculino de 36 años, arqueólogo, diabético. Estuvo en contacto con


pacientes con tuberculosis en su último viaje a Zimbabwe. Comenzó con fiebre,
tos productiva con estrías sanguinolentas y ataque al estado general. Durante su
traslado presentó sudores nocturnos. La radiografía de tórax revela infiltrados
nodulares en lóbulo superior,con una lesión cavitada que está drenando a través
de un bronquio. PPD negativo. Se inicia tratamiento antituberculoso. Acude a
consulta a ajuste de hipoglucemiantes orales debido a que su glucosa sérica se
elevo casi el doble de lo que manejaba hace tres meses. Refiere que ha respetado
su dieta y ha estado tomando su medicación. Pocas semanas después del inicio
del tratamiento antituberculoso el paciente reporto disminución de tos e
incremento de energía."
¿Cuál es la explicación de la hiperglucemia en este paciente?
b} Rifampicina

66. Una paciente de 64 años es llevada al hospital por sensación de vértigo y


sordera del oído izquierdo. A la exploración física la paciente presenta sordera
neurogénica del lado izquierdo; analgesia y anestesia térmica del lado derecho del
cuerpo y del lado izquierdo de la cara; depresión del paladar con dificultad para la
deglución;síndrome de Horner izquierdo; tendencia a caerse hacia el lado
izquierdo; nistagmo horizontal.
66a. ¿Cuál de las siguientes estructuras es la responsable de la depresión del
paladar y la dificultad para la deglución?
a} Núcleo ambiguo

66b. ¿Cuál de los siguientes enunciados es correcto en relación con el nistagmo


del paciente?
a} El componente rápido se dirige hacia la derecha

66c. ¿Cuál de las siguientes opciones representa la etiología más probable del
cuadro clínico de esta paciente?
d} Oclusión dela arteria cerebelar posteroinferiorizquierda

67. Paciente masculino de 3 años de edad, quien 24 horas previo a la consulta


presentó rinorrea, fiebre y tos no traqueal,el día de hoy se agrega ronquera, tos
""perruna"" y estridor laríngeo en reposo; a la exploración física se encuentra con
adecuado estado mental, ligera palidez y tiraje intercostal bajo, la Sp02 es de
88%. Se niegan antecedentes relevantes. FC 110 lpm, FR 24 rpm, temp. 38.3ºC,
PA 90/60 mm Hg, peso 14 kg, talla 96 cm."
67a. El diagnóstico de este paciente es:
c} Laringotraqueítis

67b. ¿Cuál de los siguientes es un signo de alarma al ingreso?


d} Estridorlaríngeo

67c. Según los datos obtenidos hasta ahora, usted clasifica este cuadro como:"
b} Moderado

67d. El tratamiento farmacológico que usted selecciona es:


d} Glucocortidoides

67e. ¿Cuáles son los datos de alarma para la familia del paciente?
a} Cianosis

68. Paciente masculino de 18 años con antecedentes de anemia de células


falciformes e hipertensión portal es traído a urgencias por presentar hematemesis
masiva. El paciente se encuentra ictérico, taquicárdico e hipotenso. Usted indica
oxígeno por puntas nasales, canalizar dos vías periféricas con catéter de calibre
16, bolos de solución salina y obtiene muestras de sangre para biometría
hemática, estudios de coagulación y cruzar sangre.
¿Cuál es el tratamiento de elección del sangrado de várices esofágicas?
c} Endoscopia con ligadura de várices
69. Paciente masculino de 70 años de edad acude al servicio de urgencias por
dolor torácico. El paciente niega disnea, palpitaciones, náusea, vómito o mareo. El
expediente clínico es relevante por hipertensión, manejada en la actualidad con
diuréticos. La presión arterial del paciente es de 180/100 mm Hg, con una
frecuencia cardiaca de 11O latidos/minuto. La exploración del cuello evidencia un
desplacamiento inferior del cartílago cricoides durante cada contracción,así como
positividad para el signo de Carderelli. ¿Cuál de los siguientes estudios
diagnósticos es el más apropiado en este paciente?"
e} Ecocardiograma transesofágico

70. ¿Cuál de las siguientes neoplasias óseas es más probable de presentar


histológicamente osteoclastos?
b} Tumor de células gigantes

71. Paciente femenino de 23 años de edad que ingresa a hospitalización por


presentar desde hace 30 días fatiga y disnea al ejercicio. Entre los antecedentes,
se encuentra una infección de vías respiratorias superiores hace un mes. No hay
antecedentes heredofamiliares de importancia. A la exploración física muestra una
presión arterial de 95/55 mm Hg, y un ritmo cardiaco de 115 latidos/minuto. Las
venas yugulares se encuentran distendidas y la auscultación pulmonar revela
estertores bilaterales; existe tercer ruido cardiaco. Se realiza un ecocardiograma
que demuestra dilatación de ambos ventrículos con hipocinesia difusa y fracción
de eyección del ventrículo izquierdo de 18%. ¿Cuál de los siguientes enunciados
NO es correcto respecto al cuadro de esta paciente?
c} La administración de corticoesteroides puede disminuirla progresión dela
enfermedad

72. Paciente masculino de 70 años de edad que es llevado al servicio de


urgencias por confusión en las últimas 12 horas. Dos semanas previas el paciente
había sufrido un infarto miocárdico, con colocación de stent. Entre los
medicamentos del paciente se encuentra ticlopidina, aspirina, atorvastatina,
metoprolol y captopril. A la exploración física tiene fiebre de 39 ºC y exantema
petequial difuso. El resto de la exploración física no es relevante. Los estudios de
laboratorio presentan elevación de la creatinina (3.5 mg/dL) y caída en el
hematocrito (22%), con un conteo plaquetario de 45 000 plaquetas/mm3. ¿Cuál de
los siguientes medicamentos es el responsable por el cuadro de este paciente?"
b} Ticlopidina

73. Mujer de 25 años de edad, acude a consulta por presentar dolor en el pie
derecho, con aumento de volumen y tumefacción del primer ortejo, limitación para
la movilidad de las articulaciones que componen ese dedo del pie; al interrogatorio
dirigido niega antecedentes crónico degenerativos, ovolactovegetariana, sin
toxicomanías ni alergias referidas, refiere haber cursado con cuadro infeccioso
gastrointestinal hace 3 semanas completamente remitido y niega antecedente
traumático; a la exploración se encuentra aumento de volumen del dorso del pie,
sin eritema ni hipertermia. FC 75 lpm, FR 18 rpm,temp. 36.5 ºC, PA 110/70 mm
Hg, peso 55 kg, talla 1.65 m.
73a. ¿Cuál es la causa más probable de la artropatía?
c} Artritis reactiva

73b. ¿Cuál de los siguientes agentes está asociado con artritis reactiva?
a} Salmonella

73c. Con el diagnóstico y cuadro clínico, usted decide iniciar el tratamiento de


primera línea"
d} Antiinflamatorios no esteroides

73d. ¿Cuál de los siguientes criterios no es diagnóstico de artritis reactiva?


b} Afección simétrica

73e. Además de la artritis reactiva, los pacientes pueden presentar otras


manifestaciones ¿Cuál de las siguientes no está asociada con artritis reactiva?
d} Eritroderma

74. Mujer de 35 años de edad, consulta por malestar general,palpitaciones, diarrea


escasa,fiebre y dolor unilateral en mandíbula y oído derecho. Refiere haber
presentado una infección de vía aérea superior hace un par de semanas. TA:
130/70, FC: 90 lpm, Temp: 37.0 ºC. A la exploración se encuentra bocio leve y
dolor a la palpación de tiroides de característica nodular. Perfil tiroideo: TSH
suprimida, T4 y T3 elevadas, VSG elevada. Usted sospecha de tiroiditis
subaguda."
74a. Al realizarle una gammagrafía a esta paciente, ¿qué hallazgos esperaría
encontrar?"
d} Muy baja captación a causa de lalesión de las células foliculares y la supresión
de TSH

74b. ¿Cuál es la base fisiopatológica de la enfermedad de esta paciente?


d}Infección viral

74c. ¿Cuál es el tratamiento indicado en esta paciente?


c} AINE + bloqueador beta

74d. ¿Cuál es el pronóstico de la función glandular en esta paciente?


a} Recuperación completa

75. ¿Cuál de los siguientes medicamentos se caracteriza por presentar infarto


miocárdico como complicación?
e} 5-fluorouracilo

76. ¿Cuál de los siguientes esquemas profilácticos se recomienda para un


paciente con miocardiopatía hipertrófica obstructiva que se va a someter a una
cistoscopia?
a} No se requiere ningún tipo de profilaxis antibiótica

78. Femenino de 45 años de edad es traída por los servicios de emergencia al


hospital donde usted está de guardia. La paciente ha sufrido un accidente con un
coche, se encuentra inconsciente y se observan lesiones graves en todo el macizo
facial y zona del ojo derecho. TA: 130/80, FC: 95x ', FR: 23x ', Temp: 37.SºC."
78a. La fractura más característica de la órbita es:
b} Polo inferior

78b. La fractura más frecuente de la órbita, que además puede lesionar la tróclea
del oblicuo superior,es:"
e} Medial

78c. ¿Qué huesos conforman la pared inferior de la órbita?


b} Maxilar superior,malar y palatino"

78d. Ante un trauma ocular, ¿qué estudio de gabinete se debe solicitar?"


d} TC

79. Masculino de 30 años acude a consulta por presentar un cuadro diarreico de 3


días de evolución, entre seis y ocho evacuaciones al día, abundantes, líquidas, sin
presencia de moco o sangre. Presentó vómito en dos ocasiones al inicio, y en las
últimas horas ha mostrado mareo. Niega fiebre. Antecedente: haber asistido a
competencia de triatlón a Nueva Delhi la semana previa al padecimiento actual."
79a. El germen causal de la sintomatología de este paciente es:
d} Vibrio cholerae

79b. Tratamiento de elección para este paciente:


d} Doxiciclina

79c. El cuadro clínico de este paciente es causado por aumento en :


d} cAMP

80. Paciente masculino de seis años de edad con dolor abdominal difuso y
estreñimiento crónicos. Recientemente se ha mostrado muy distraído, con
problemas en su desempeño escolar. El padre del paciente trabaja en una fábrica
de vidrio. Se sospecha de una posible intoxicación por plomo. Señale, cuál es el
mecanismo de acción de este metal."
b} Unión a ligandos sulfhidrilo con inhibición de múltiples enzimas

81. Paciente femenino de 50 años es llevada al servicio de urgencias por


presentar disnea de una semana de evolución que ha progresado hasta ser de
pequeños esfuerzos y se acompaña en ocasiones de palpitaciones y dolor torácico
que dura pocos minutos. La disnea fue máxima esta mañana, asociada con dolor
retroesternal 7/10 que aumenta con la respiración. La paciente no tiene historia de
tabaquismo, niega cirugías recientes. Su esposo refiere que la ha visto cansada y
que ya casi no se levanta de la cama desde que le colocaron una férula en el
tobillo izquierdo debido a que sufrió un esguince hace dos semanas. A la
exploración física es evidente la dificultad respiratoria con uso de músculos
accesorios, signos vitales con frecuencia respiratoria de 26, frecuencia cardiaca
105 lpm, presión arterial 85/50 mm Hg, temperatura 37.5 ºC. Se aprecia
ingurgitación yugular de 8 cm, taquicardia,S1 y S2 sin soplos ni ruidos accesorios,
campos pulmonares bien ventilados y edema de miembros pélvicos,
especialmente del lado izquierdo, con dolor a la palpación de la pantorrilla
izquierda."
La sospecha clínica de que esta paciente presente una embolia pulmonar es:
d} Alta

82. Paciente femenino de cinco años es traída a urgencias por sus padres quienes
la encontraron jugando en un campo que había sido regado con pesticidas. La
paciente presentó inicialmente abundante salivación y lagrimeo, vómito, diarrea e
incontinencia urinaria seguida de confusión y contracciones musculares
generalizadas. A la exploración física se encuentra letárgica, diaforética, con
pupilas mióticas y fasciculaciones."
Usted de inmediato sospecha intoxicación por organofosforados.
¿Cuál es el medicamento adecuado para tratar los efectos nicotínicos en esta
paciente?
e} Pralidoxima

83. Paciente masculino de 35 años, albañil, acude a consulta por presentar prurito
intenso en las manos, así como fisuras y costras entre los dedos de 15 días de
evolución.Se ha aplicado Vitacilina sin mejoría. A la EF se observa dermatosis
diseminada a palmas de manos y a región interdigital, caracterizada por múltiples
placas eritematoescamosas, algunas fisuras y costras hemorrágicas. Niega
ingesta de medicamentos. Comenta que se encuentra bastante preocupado, ya
que no le es posible laborar en la obra ni lavarse las manos porque nota mucho
ardor al"
contacto con acetona.
83a. ¿Cuál es el diagnóstico de este paciente?
b} Dermatitis por contacto

83c. Dentro de las enfermedades ocupacionales, ¿qué porcentaje representa la


dermatitis por contacto de manos?"
a} 25 a 50%

83d. ¿Quétipo de reacción es la dermatitis de contacto por irritante primario?


d} No inmunitaria

83e. ¿Qué tipo de reacción inmunitaria es la dermatitis por contacto con


sensibilización o alérgica?
d}IV

84. Masculino de 14 años, sin antecedentes patológicos,es traído al hospital por


sus padres tras habersufrido una lesión mientras jugaba futbol americano. El
paciente comenta que al ser tacleado cayó sobre el hombro derecho e
inmediatamente sintió dolor en la región (EVA 10/1O); dicho dolor ha ido en
aumentoy le impide mover la extremidad superior derecha. A la exploración usted
comprueba que a la movilización de dicha extremidad el dolor es insoportable y
observa que el hombro derecho está descendido con respecto al izquierdo; al
palpar la zona superior derecha del tórax, usted nota edema, crepitación y
movilidad anormal en la región."

84a. ¿Cuálsería su sospecha diagnóstica?


a} Fractura clavicular

84b. ¿Qué estudio solicitaría para confirmar su diagnóstico?


a} Radiografía de tórax anteroposterior

84c. ¿Cómo se clasifica esta lesión?


c} Terciolateral,tercio medio o tercio medial"

84d. El tratamiento clásico de esta lesión es:


a} Vendaje o dispositivo en ""8"""

84e. Fractura más frecuente durante el trabajo de parto:


r a} Fémur
e} Clavicular

85. Se trata de Vanesa, una paciente femenino de 27 años de edad previamente


sana, que cursa la semana 24 de gestación, ha acudido a todas la consultas de
control prenatal hasta el momento, en esta ocasión corresponde realizar tamizaje
en búsqueda de alteraciones de la glucosa. La exploración física solo resalta un
abdomen globoso por estado de gravidez, sin edema periférico, sin otros datos
relevantes. FC 90 lpm, FR 20 rpm,temp 36.7 ºC, PA 130/70 mm Hg, peso 69 kg,
talla 1.62 m."

85a. El diagnóstico de diebetes gestacional se define como:


c} Intolerancia a carbohidratos reconocida durante el embarazo

85b. Usted realiza una prueba de carga de glucosa de 75 g en ayuno y curva de 2


horas ¿cuáles son los criterios para considerar una prueba positiva?
d} Niveles de glucosa > 153 mg/dl en 2 horas

85c. Si esta paciente obtuviera un resultado positivo en ambas pruebas de


tolerancia a la glucosa ¿qué modificaciones haría al seguimiento?
b} Realizar 1cita cada 15 días de la semana 28 a 35 y una semanal de la 36 en
adelante

85d. Se detecta que en ayuno la glucemia es > 95 mg/dl,¿cuál de las siguientes


terapéuticas será la ideal?"
c} Hipoglucemiantes orales

85e. ¿Cuáles son las complicaciones fetales más comunes durante el segudo y
tercer trimestre del embarazo?
d} Macrosomía
86. Paciente femenino de 45 años acude a consulta por ""hinchazón"" de manos
desde hace 5 meses; también ha notado un cambio de textura en su piel,que
ahora está más firme y dura. Tiene antecedentes de enfermedad por reflujo
gastroesofágico y disfagia a sólidos. A la exploración física se nota inflamación en
las articulaciones de sus dedos y presenta nódulos subcutáneos en éstos;
asimismo, se percibe leve edema e induración en ambas manos.
86a. ¿Cuál sería el diagnóstico más probable?
d} Esclerosis sistémica cutánea limitada

86b. De acuerdo con el diagnóstico clínico, ¿cuáles de los siguientes anticuerpos


ayudarían a tener un diagnóstico más certero?:"
a} Anticuerpos anticentrómero

86c. ¿Cuál es la fisiopatología de la enfermedad?


d} Proliferación de fibroblastos con aumento enlos depósitos de colágeno

86d. En cuanto a las alteraciones renales de esta enfermedad, ¿cuál es la más


característica?"
a} No hay afección renal

87. Masculino de 11 años de edad es traído a consulta por su madre, la cual


comenta que la maestra del niño le ha reportado que lo nota distraído en las
clases y que, por otra parte, en la casa ve la TV muy de cerca;además de que por
la tarde trae los ojos muy rojos. Durante la entrevista usted se da cuenta de que el
niño entrecierra mucho los ojos para observar los objetos en la pared. Al interrogar
al niño, éste comenta que le cuesta trabajo ver de lejos, por lo que se acerca para
ver bien,y que después de ver mucho tiempo la TV le da dolor de cabeza."
87a. ¿Cuál sería su diagnóstico para este paciente?
b} Miopía

87c. ¿Cómo se define un dioptrio?


c} Toda superficie que separa dos medios con distinto índice de refracción

87d. Del sistema óptico del ojo, ¿cuál estructura es la más potente?"
b} Córnea

87e. El tratamiento de primera elección para este paciente sería:


c} Lentes divergentes

88. Paciente femenino de 50 años con dermatosis diseminada que afecta tronco y
extremidades superiores e inferiores caracterizada por ampollas flácidas,
erosiones y costras, algunas sanguíneas y otras melicéricas, y se acompaña de
estomatosis caracterizada por erosiones. No ha recibido tratamiento, refiere fiebre
de hasta 38.5 C. Inició hace alrededor de dos semanas con ampollas que han ido
evolucionando a costras. Las lesiones en boca iniciaron un mes antes y recibió
tratamiento con aciclovir por probable estomatitis herpética, sin mejoría. A la
exploración física muestra además mal estado de hidratación de mucosas y
taquicardia. Los hallazgos histológicos de la biopsia por sacabocado de una de las
ampollas son:"
b} Ampolla suprabasal con células acantolíticas y células basales unidas a la
membrana basal

89. ¿Cuál de los siguientes trastornos del sueño es el más común en la población
general?
d}Insomnio

90. Paciente masculino de 59 años de edad con antecedente de enfermedad


acidopéptica y reflujo gastroesofágico. Debido a que presenta insuficiencia
cardiaca y fibrilación se encuentra bajo tratamiento con digoxina. El paciente
decide iniciar por sí mismo tratamiento con antiácidos de hidróxido de aluminio.
¿Cuál de las siguientes alteraciones electrolíticas puede ser ocasionada por el
antiácido y con alta morbilidad en este paciente?
d} Hipopotasemia r

91. Paciente femenino de 85 años de edad que se presenta con alteración del
estado de alerta, desorientación,combativa,con pupilas midriáticas no reactivas a
la luz, taquicardia sinusal, piel seca, hipertermia, constipación y retención urinaria.
Todos estos síntomas aparecieron después que por error de una estudiante de
enfermería se le administró una dosis excesivamente alta de atropina. ¿Cuál de
los siguientes medicamentos es el mejor antídoto en esta paciente?"
a} Fisostigmina

92. Paciente masculino de 24 horas de vida producto de una paciente de 42 años


de edad. A la exploración física el neonato se presenta con microcefalia,
implantación baja de orejas, occipital prominente y micrognatia. ¿Cuál de las
siguientes anormalidades en el cariotipo es la que se puede encontrar con más
probabilidad en este paciente?"
b} Trisomía 18

93. Paciente masculino de 65 años se presenta en la madrugada al servicio de


urgencias por presentar disnea en reposo y edema remitente de miembros
inferiores pero que va en ascenso. Cuenta con antecedente de hipertensión y
diabetes mellitus tipo 2, así como de tres infartos miocárdicos, de los cuales dos
de ellos no pudieron ser trombolizados porque el paciente llegó fuera de ventana
terapéutica. Hace dos días presentó un cuadro de infección de vías aéreas
superiores, que ha progresado hasta convertirse en una neumonía incipiente.
¿Cuál de las siguientes combinaciones de fármacos, a pesar de ser efectiva en
este paciente, también puede resultar peligrosa?"
a} Digoxina + furosemida

94. ¿Cuál de los siguientes antiarrítmicos tiene su principal mecanismo de acción


en células de despolarización automática y de lenta
conducción?
e} Esmolol
95. ¿Cuál de los siguientes medicamentos se asocia con hiperpotasemia como
efecto adverso?
e} Ciclosporina

96. Paciente masculino de dos días de vida el cual presenta acidosis, vómito,
hipotonía y alteraciones neurológicas. Se solicitan estudios de laboratorio los
cuales revelan niveles elevados de lactato y alanina. ¿Cuál de las siguientes
enzimas es la que con mayor probabilidad se encuentra deficiente en este
paciente?
b} Piruvato deshidrogenasa

97. ¿Cuál de los siguientes mecanismos de acción corresponde al de las


estatinas?
b}Inhibición dela enzima hidroxi-metil-glutaril-coenzima A

98. ¿Cuál de los siguientes defectos es ocasionado por ausencia de la formación


del septo aorticopulmonar?
b} Comunicacióninterventricular de tipo membranosa

99. Una mujer de 18 años de edad se presenta a consulta externa de Obstetricia


en su 18 semana de gestación. Hace tres años, la paciente fue diagnosticada con
depresión moderada y se le recetó paroxetina 20 mg/día, la cual continuó tomando
hasta hace aproximadamente seis meses, debido a que consideró que no
necesitaba el medicamento. La paciente informa que recientemente no ha dormido
""en semanas"", y lo atribuye al embarazo. Al interrogatorio menciona que también
se ha mostrado recientemente letárgica, sin ideación suicida y que desea
alimentar a su futuro hijo con leche de su propio seno. ¿Cuál de los siguientes
esquemas de tratamiento es el más adecuado para el manejo de depresión de
esta paciente?"
a} Sertralina 50 mg al día, con consulta psiquiátrica de seguimiento en un mes"

1OO. ¿Cuál de las siguientes cadenas de globinas se produce exclusivamente


durante la etapa embrionaria?
e} Zeta globina

101. Paciente femenino de 22 años de edad se presenta a consulta por ictericia y


malestar general desde hace 20 días. No había acudido antes porque su novio
había tenido un tipo de hepatitis algunos meses atrás, y éste le recomendó que no
acudiera por tratamiento médico, pues seguramente era un cuadro autolimitado.
Los estudios de laboratorio revelan una concentración de alanina transaminasa de
250 U/L, aspartato transaminasa de 201 U/L, bilirrubina de 7 mg/dL, positividad
para el antígeno de superficie del virus de la hepatitis B y anticuerpo lgM contra el
antígeno central (core) del virus de la hepatitis B. La paciente muestra un claro
cuadro de hepatitis aguda por el virus de la hepatitis B. ¿Cuál de los siguientes
enunciados respecto de la hepatitis tipo B es incorrecto?
b} La administración de interferón enla etapa aguda dela infección evita el
desarrollo delestado de portador crónico del virus de la hepatitis B

102. ¿Cuál de los siguientes fármacos antibióticos ejerce su efecto mediante la


inhibición de la enzima DNA topoisomerasa en bacterias?
c} Quinolonas

103. Paciente masculino de 31 años de edad que acude a consulta de rutina.


Entre los antecedentes de importancia el paciente refiere que a la edad de seis
años sufrió un cuadro de fiebre reumática aguda, lo cual lo confinó a la cama por
varios meses. Desde entonces ha estado bajo tratamiento antibiótico con
penicilina V oral a dosis de 500 mg al día. Cuatro años después del episodio de
fiebre reumática aguda,es decir,a los 1O años, tuvo un cuadro de exacerbación
relacionada a la suspensión del medicamento antibiótico. Actualmente el paciente
trabaja como enfermero en un hospital pediátrico. A la exploración física presenta
un soplo holosistólico grado 3/4, el cualtiene su punto de mayor intensidad en el
ápex. ¿Cuál de los siguientes enunciados es el más adecuado en relación a la
conducta a seguir en este paciente?
e} El paciente debe continuar tomando penicilina por tiempoindefinido

104. ¿Cuál de los siguientes enunciados es el que más adecuadamente


representa las características de la fractura de Colles?
b} Fractura de la metáfisis distal del radio en la que existe desplazamiento dorsal
del fragmento distal

105. ¿Cuál de las siguientes hormonas contribuye a mantener niveles normales de


glucosa en la sangre durante periodos prolongados de ayuno y actúa mediante un
receptor intracelular?
e} Cortisol

106. ¿Cuál de los siguientes síndromes se presenta con tumores desmoides y


osteomas mandibulares?
a} Síndrome de Gardner

107. ¿Cuál de los siguientes medicamentos representa un alto riesgo de hepatitis


colestásica?
c} Eritromicina

108. ¿Cuál de los siguientes hallazgos incrementa la probabilidad del diagnóstico


de epididimitis bacteriana?
c} Secreción uretral

109. Paciente femenino de 34 años de edad. Inicia su padecimiento tres días


previos a su ingreso con dolor en región dorsal, progresivo, de tipo punzante,
intenso, de predominio nocturno con irradiación hacia región anterior del tórax, se
administran analgésicos y antiinflamatorios sin mejoría, después se agrega
retención urinaria aguda, debilidad muscular de los miembros inferiores,
parestesias de los miembros pélvicos y en región torácica anterior. Evolucionando
con pérdida progresiva de la fuerza en miembros inferiores y disminución de la
sensibilidad hasta la región torácica. Presenta dolor a la palpación en región dorsal
a nivel de T3-T4. Extremidades superiores con fuerza 5/5, inferior derecha 3/5,
inferior izquierda 1/5, reflejos osteotendinosos de +++ en extremidades inferiores y
++ en extremidades superiores, con respuesta plantar extensora bilateral. Nivel
sensitivo de hipoestesia en T4, incontinencia urinaria y pérdida en el tono del
esfínter anal. Se realizaron exámenes de laboratorio de rutina en rangos normales.
Se programa para tomografía computarizada simple y contrastada de columna
dorsal con foco en T3- T4, sin encontrar lesiones espinales ni lesiones
intrarraquídeas o intramedulares. Se realiza IRM de columna dorsal donde se
encuentra una lesión quística a nivel de T1 con reforzamiento anular y una imagen
hiperdensa dentro de la lesión con edema perilesional compatible con cisticerco
intramedular. En relación a la fisiología de la micción y la incontinencia urinaria
todo es característico EXCEPTO:
d} Laincontinencia urinaria aguda tiene un inicio brusco, generalmente en
relación con una enfermedad aguda o problema iatrogénico y no remite tras
resolverse la enfermedad o el problema de medicación.Se cura aproximadamente
10%"

110. ¿Cuál de los siguientes mecanismos de acción productora de diarrea


corresponde al de Tropheryma whipplei?
b} Colonizaciónintracelular del organismo delas células epiteliales intestinales

111. Paciente masculino de 67 años de edad al cual se le realizó una


colonoscopia como método de detección temprana de cáncer colónico.
Se encontró un adenoma velloso sésil de 1.2 cm en el colon ascendente. La
tumoración fue extirpada. ¿Cuál de las siguientes conductas terapéuticas es la
más adecuada?

b} Repetirla colonoscopia en tres años

112. ¿Cuál de los siguientes organismos se asocia con levaduras de base ancha
en la biopsia de los tejidos infectados?
d} Blastomyces dermatitidis

113. Paciente de cuatro días de edad con deficiencia de la enzima galactosa-1-


fosfato uridiltransferasa. ¿Cuál es la evolución más probable de este paciente en
caso de que su enfermedad no sea tratada oportunamente?
e} El paciente sufriráinsuficiencia hepática

114. ¿Cuál de los siguientes esquemas profilácticos de tromboembol ia venosa es


el más adecuado en pacientes de riesgo moderado para desarrollar ésta?
e} La combinación de A y C es correcta
115. ¿Cuál es el nervio que se daña con mayor frecuencia durante las inyecciones
intramusculares administradas con técnica deficiente en la región glútea?
d} Nervio ciático

116. ¿Cuál es el anticonvulsivante de elección en caso de crisis convulsivas


parciales complejas?
a} Carbamacepina

117. ¿Cuál de los siguientes hallazgos histopatológicos óseos son característicos


del hiperparatiroidismo primario?
c} Resorción de hueso subperióstico con degeneración quística

118. ¿Cuál de las siguientes opciones representa de mejor forma el significado de


una diferencia entre la concentración de albúmina en suero y líquido de ascitis
mayor a 1?
d} Cirrosis

119. Paciente femenino de 44 años de edad que se sometió a una resección de


cáncer medular tiroideo. Posterior a la cirugía presenta voz de tono nasal. ¿Cuál
de las siguientes estructuras fue la que con mayor probabilidad se lesionó?
 e} Nervio laríngeo recurrente

1. Paciente femenino de 22 años de edad que acude al servicio de urgencias por


malestar general, mialgias, fiebre, diaforesis, y claudicación de la extremidad
inferior derecha y superior izquierda. La presión arterial es de 180/100 en el brazo
derecho y de 120/80 en el brazo izquierdo. Tanto el pulso radial derecho como el
femoral izquierdo se encuentran disminuidos. Se puede auscultar un soplo en la
arteria subclavia derecha. La velocidad de sedimentación globular se encuentra
muy elevada. ¿Cuál de los siguientes enunciados es el más correcto respecto al
padecimiento de esta paciente?
c} Esta condición es 10 veces más frecuente en el sexo femenino que en el sexo
masculino

2. ¿Cuál de los siguientes hallazgos se presenta en caso de lesión del nervio


hipogloso izquierdo?
a} Desviación dela lengua hacialaizquierda durante la protrusión

3. Una paciente de 50 años de edad con diabetes mellitus tipo 2 y antecedente de


síndrome de ovario poliquístico se somete a histerectomía total abdominal para
tratamiento y estadificación de carcinoma endometrial. ¿Cuál de las siguientes
estructuras está en riesgo de ser dañada cuando las arterias uterinas son ligadas?
e} Ureteros
4. ¿Cuál de los siguientes antiarrítmicos prolonga el potencial de acción y
disminución del flujo de salida celular de potasio, sin afectar los canales de sodio
ni de potasio?"
a} lbutilide

5. ¿Cuál de los siguientes mecanismos de acción corresponde al de la


colestiramina y el colestipol?
a} Unión y excreción de las secreciones biliares
6. ¿Cuál de las siguientes enzimas es responsable de la activación de la
ciclofosfamida en el cuerpo humano?
a} Citocromo P450

7. ¿En cuál de los siguientes organelos celulares el etanol es metabolizado a


acetaldehído?
c} Peroxisomas

8. ¿Cuál de las siguientes hormonas gastrointestinales se libera posterior a la


distensión gástrica?
e} Gastrina

9. ¿Cuál de los siguientes receptores es al que se une con más afinidad la


ketamina?
d} NMDA

1O. ¿Cuál de las siguientes estructuras forma el límite lateral y medial del triá
ngulo femoral?
d} Músculo aductor largo y sartorio

11. ¿Cuál de las siguientes enzimas se encuentra involucrada en la producción de


ribosa a partir de la fructosa-6-fosfato, pero no en la producción de NADPH?"
b} Transcetolasa

12. En varias ocasiones la hipotermia grave no es diagnosticada durante la


exploración física inicial. La temperatura se puede reportar como el límite inferior
del termómetro, lo cual puede ser de tan sólo unos cuantos grados centígrados
por debajo de lo normal. ¿Cuál de los siguientes es suficientemente adecuado
para indicar el diagnóstico de hipotermia?
d} Electrocardiograma

13. ¿Cuál de las siguientes arterias se encarga de la irrigación del colon


ascendente?
a} Arteria mesentérica superior

14. ¿Cuál de las siguientes hormonas gastrointestinales funciona principalmente


como reguladora de los complejos mioeléctricos migratorios en el tubo digestivo?
b} Motilina
15. Desde del punto de vista embriológico, ¿cómo se forma la aurícula?"
b} Mesodermo

16. ¿Cuál es el efecto adverso más grave de los bolos de glucocorticoides?


a} Arritmias cardiacas

17. Varón de 40 años que presenta una dermatosis localizada en la región


perianal,constituida por una neoformacion exofítica en forma de coliflor, de
aspecto vegetante que usted diagnostica como condiloma acuminado. ¿Cuáles
son los serotipos más frecuentes en este padecimiento?
b} 6 y 11

18. ¿Cuál de los siguientes cambios endometriales se observaría en caso de que


se realice una biopsia de endometrio a una paciente al final de la fase proliferativa
del ciclo menstrual?
c} Crecimiento de las arterias espirales

19. ¿Cómo está formado el conducto auditivo externo?


a} El canal cartilaginoso constituye un tercio del canal,dos tercios es óseo

20. ¿Cuál de los siguientes antiarrítmicos actúa con mayor selectividad en células
de despolarización de alta frecuencia?
c} Lidocaína

21. ¿Cuál de los siguientes mecanismos de acción corresponde al de los fibratos?


d}Incremento de la actividad de la lipoproteína lipasa

22. ¿Cuál de las siguientes sustancias favorece la vía de la gluconeogénesis por


medio de estimulación alostérica en el hígado?
c} Acetil-coenzima A

23. ¿Cuál de los siguientes antiarrítmicos tiene su principal mecanismo de acción


en células de despolarización automática y de lenta conducción?
e} Esmolol

24. ¿Cuál de los siguientes mecanismos de acción corresponde al de las


estatinas?
b}Inhibición dela enzima hidroxi-metil-glutaril-coenzima A

25. El síndrome de Stein-Leventhal se presenta con hirsutismo por:


a} Trastorno funcional ovárico, con relación LH:FSH mayor a 2"

26. ¿Cuál de las siguientes enzimas se inhibe irreversiblemente por el fármaco


pentostatina?
e} Adenosina deaminasa
27. ¿Cuál de las siguientes hormonas gastrointestinales funciona como un
modulador de la transmisión de los impulsos dolorosos al sistema nervioso?
e} Sustancia P

28. ¿Cuál es el gasto cardiaco de un paciente que tiene un consumo de oxígeno


en reposo de 200 mUmin, contenido arterial de oxígeno de 0.20 ml 02
xlmin,contenido venoso de oxígeno de 0.15 ml 02 x ml de sangre?
b} 4 L/min
29. Paciente femenino de 25 años de edad la cual acude a consulta por padecer
dispareunia, fiebre,1 dolor abdominal en hipogastrio principalmente y escasa
secreción vaginal maloliente y sa nguinolenta. Entre los antecedentes de
importancia la paciente menciona que es sexualmente activa desde los 14 años de
edad, con múltiples parejas sexuales, y no siempre utiliza protección contra
enfermedades de transmisión sexual. A la exploración física se presenta dolor a la
movilización cervical. Se sospecha enfermedad pélvica inflamatoria. ¿Cuál de los
siguientes organismos se relaciona con esta enfermedad?
a} Neisseria gonorrhoeae

30. ¿Cuál de las siguientes NO es una causa de masa mediastinal posterior?


b} Timoma

31. ¿Cuál de las siguientes hormonas gastrointestinales se libera posterior a la


distensión intestinal?
d} Péptido intestinal vasoactivo

32. Paciente masculino de 55 años de edad, que se presenta con una tumoración
en la región lumbar derecha, la cual se extiende hacia la región inguinal
ipsolateral. Después el paciente fue diagnosticado con tuberculosis. De acuerdo
con la presentación clínica, ¿cuál de los siguientes músculos es el que más
probablemente se encuentra afectado por el absceso tuberculoso que presenta
este sujeto?"
a} Psoas

34. Paciente femenino de 17 años es traída a urgencias por sufrir quemaduras en


ambos miembros inferiores al caer de rodillas en una tina con agua hirviendo.
Usted observa que únicamente la piel de la parte anterior y posterior de ambos
muslos, piernas y pies presenta quemaduras de segundo grado de espesor
parcial, muy dolorosas, con eritema,edema y ampollas. La enfermera le indica que
la paciente pesa 50 kg, tiene ya insertadas dos líneas periféricas de calibre 16 y
espera su indicación para iniciar la reposición de líquidos.
¿Cuál es el requerimiento de líquidos en las primeras 24 horas de esta paciente
con quemaduras?
c} 7.2 L de Ringer lactato

35. Paciente femenino de 87 años es encontrada tirada en el suelo de su


recámara por su nieta; la señora no responde y no parece respirar normal.
35a. La primera medida que debe tomar la persona que la encuentra (la nieta) es:
a} Activar elsistema de emergencias

35b. En caso de ser un rescatador no entrenado, deberá de:


c} Posterior a activar elsistema de emergencias, dar solamente compresiones"

35c. ¿Cómo se definen las compresiones torácicas de calidad en adultos?


b} Ritmo de 100/min y profundidad de al menos 5cm

35d. ¿Cuál es el 3er eslabón en la cadena de supervivencia del ACLS?


a} Rápida desfibrilación

35e. En caso de ser dos rescatadores, ¿cada cuánto tiempo se deben de rotar?
d} 2 min

37. ¿Desde dónde se extiende el conducto auditivo externo?


b} De la aurícula a la porción externa de la membrana timpánica

38. ¿Cuál de los siguientes antiarrítmicos ejerce su efecto farmacológico


únicamente durante la fase O del potencial de acción?
d} Flecainamida

39. ¿Cuál de las siguientes enzimas requiere de la vitamina K como coenzima?


d} Gammaglutamil carboxilasa

40. ¿Cuál de los siguientes mecanismos de acción corresponde al de la niacina?


e} Disminución dela lipólisis perifércia

41. ¿Cuál de los siguientes nervios inerva la piel que se encuentra sobre el
músculo deltoides?
a} Nervio axilar

42. Paciente femenino de 28 años de edad acude a la sala de urgencias por sentir
un dolor abdominal localizado a la fosa ilíaca derecha, de leve intensidad. Al
interrogatorio niega la presencia de náusea o vómito. Entre los antecedentes de
importancia se encuentra el de apendicectomía a la edad de 21 años. La paciente
menciona que no ha tenido relaciones sexuales desde hace cuatro meses y su
último periodo menstrual fue hace dos semanas. ¿Cuál de los siguientes cambios
endometriales se observarían en caso de que se realice una biopsia de
endometrio?"

c} Crecimiento de las arterias espirales

43. Paciente femenino de 31 años de edad con gesta 2, para 2, sexualmente


activa y con uso de dispositivo intrauterino desde hace cuatro años. El motivo de
la consulta es por dolor abdominal y pélvico bilateral que aparece desde hace tres
días, con sensación de pesantez pélvica. La sintomatología inició al día siguiente
después de su última menstruación. El día de hoy la paciente ha presentado
náusea y cefalea. A la exploración física la paciente se encuentra levemente
taquicárdica, presión arterial dentro de rangos normales, temperatura de 39 ºC.
Reporta dolor a la palpación de la región abdominal pélvica. La exploración
ginecológica revela secreción vaginal purulenta y dolor extremo a la movilización
cervical; no se palpan masas en anexos. La biometría indica ligera anemia y
leucocitosis de 14 000 células/µL. El ultrasonido transvaginal demuestra
septaciones de la pared tubaria (signo de la rueda dentada). Se realiza prueba de
embarazo en orina la cual resulta negativa. ¿Cuál es el diagnóstico más probable?
a} Peritonitis-salpingitis aguda

44. ¿Cuál de los siguientes antiarrítmicos actúa tanto en la fase O como en la fase
3 del potencial de acción?
b} Quinidina

45. Una paciente de 14 años se presenta a consulta pediátrica debido a que no ha


tenido su primera menstruación ni ha desarrollado crecimiento mamario. A la
exploración física la paciente tiene una altura de 1.60 m, obesidad y cuello corto.
Tanto genitales externos como glándulas mamarias se encuentran en un estadio 1
de Tanner. Los resultados de los estudios de laboratorio revelan elevación de la
hormona foliculoestimulante y disminución del estradiol en sangre.
¿Cuál de los siguientes hallazgos es probable que también se observe en esta
paciente?
e} Presión arterial en las extremidades superiores mucho mayor que en las
inferiores

46. ¿Cuál de los siguientes mecanismos de acción corresponde al de la


ezetimiba?
c}Inhibición dela absorción de colesterol en elintestino delgado

2. Llega a urgencias paciente femenino de 38 años de edad primigesta, por haber


presentado crisis tonicoclónica generalizada hace 30 minutos. Refiere el marido
que previamente había tenido una cefalea muy intensa, fosfenos y dolor en el
epigastrio. Se encuentra en la semana 24 de gestación. TA de 180/11O mm Hg,
estuporosa, en anasarca, ROT incrementados. ¿Qué medicamento
anticonvulsivante es el de elección?
d} Sulfato de magnesio

4. ¿Cuál de las siguientes arterias se encarga de la irrigación del colon


ascendente?
a} Arteria mesentérica superior

5. Paciente femenino de 19 años de edad, con embarazo de 12 semanas de


gestación por fecha de última menstruación, acude a urgencias por presentar
hemorragia genital persistente, con dolor en hipogastrio tipo cólico, sin
irradiaciones que ha ido aumentando en intensidad y frecuencia. A la exploración
física: frecuencia cardiaca 105 latidos por minuto, frecuencia respiratoria 20
respiraciones por minuto, temperatura 37 C. Abdomen blando, depresible,
doloroso a la palpación del hipogastrio, sin datos de irritación peritoneal. Se
observa hemorragia genital y se realiza tacto vaginal encontrando cuello uterino
central, con 4 cm de dilatación y borramiento de 30%. ¿Cuál es su impresión
diagnóstica?
c} Aborto en evolución

6. Se encuentra realizando la exploración física inicial de un neonato de 12 horas


de vida, producto de primera gestación, peso al nacer 3 250 g. Explora
intencionadamente la cadera para descartar la presencia de una luxación
congénita.
6a. ¿Cuál de las siguientes NO es una característica de esta enfermedad?
e} Es más frecuente en varones
6b. ¿Cuál de los siguientes hallazgos en el examen físico del neonato le hacen
sospechar luxación congénita de cadera?
d} Chasquido al abducirla cadera y presionar eltrocánter mayor hacia una posición
anterior,conla cadera y rodilla en flexión

6c. ¿Qué información aporta la prueba de Barlow?


b} Detecta una cadera que esluxable al momento del examen físico

6d. Ante pruebas positivas de Barlow y Ortolani, usted decide repetir la


exploración a las seis semanas de vida y a los seis meses. Para la evaluación a
los síes meses de vida, ¿qué método diagnóstico es de elección para diagnosticar
la displasia congénita de cadera?
e} Radiografía simple

6e. El paciente acude a consulta a los cinco meses de vida y se confirma el


diagnóstico. ¿Cuál es la opción más adecuada de manejo de acuerdo con la edad
del paciente?
a} Arnés de Pavlik

7. Paciente femenino de 18 años acude a consulta por embarazo sin conocer las
semanas de gestación. Al interrogatorio no refiere datos de importancia. Usted
decide calcular las semanas de gestación y la fecha probable de parto, sabiendo
que el primer día de la última menstruación fue el 18 de marzo de 2008 y hoy es
septiembre 14 de 2008. ¿En qué semana de gestación está y cuál es la probable
fecha de parto?
c} 25 de diciembre de 2008 y 25 semanas de gestación

9. Se presenta una paciente de 22 años de edad con su primer embarazo. La


paciente se muestra renuente a abstenerse del consumo de alcohol durante el
resto de su embarazo. En el caso de una mujer embarazada que bebe entre 60 y
120 mlde alcohol a 80%, ¿cuál es el riesgo de presentar un neonato con síndrome
alcohólico fetal?
b} 10 %
1O. Acude a consulta paciente femenino de 20 años de edad por amenorrea.
Refiere que sí ha presentado menstruaciones en el pasado. No se observan
alteraciones en la talla, fenotipo ni cardiacas, hay vagina corta. Se realiza un
cariotipo encontrando 46XX, los ovarios se encuentran normales. ¿Cuál de las
siguientes es una causa de amenorrea secundaria?
c} Síndrome de Asherman

11. Paciente masculino, producto de primera gestación de 39 semanas de


duración, obtenido por histerotomía indicada por desproporción cefalopélvica. Se
evalúa al nacer y se encuentra con una frecuencia cardiaca de 98 latidos por
minuto, respirando de forma irregular y lenta, no llora, hay presencia de algunos
movimientos de extremidades con predominio de flexión, cuando se estimula con
sonda nasogástrica se desencadena llanto vigoroso y su piel se observa rosada
en el tronco, pero los dedos de manos y pies se encuentran cianóticos. De
acuerdo con estas observaciones, ¿cuál es el puntaje de Apgar que se le
otorgaría?
d} 6

12. Un neonato de 20 días de vida es llevado a consulta por presentar masa


abdominal. En el examen físico se observa una masa a nivel de la cicatriz
umbilical, sin cambios en las características de la piel suprayacente, se palpa
masa de consistencia blanda,depresible, de aproximadamente 1 cm de diámetro,
no es dolorosa en apariencia. ¿Cuál es el diagnóstico más probable?
d} Hernia umbilical

13. ¿Cómo está formado el conducto auditivo externo?


a} El canal cartilaginoso constituye un tercio del canal,dos tercios es óseo

14. Paciente femenino de 69 años que acude a consulta porque ha manchado su


ropa interior con sangre. Al interrogarla de manera apenada refiere que ha
ocurrido en dos a tres ocasiones todas relacionadas con el coito. Refiere tener
relaciones sexuales sólo con su esposo, con el que lleva casada 42 años. Su
fecha de última menstruación fue hace 20 años. Menciona además dispareunia,
refiriéndolo como ardor poscoital. Niega disuria,fiebre o exudados purulentos. A la
exploración pélvica se observa una mucosa delgada y friable, con un poco de
petequias y eritema,con muy pocos pliegues. Se observa leucorrea serosa con un
pH de 6. Un frotis muestra la presencia de leucocitos y disminución de la población
de lactobacilos. ¿Qué tratamiento le daría a esta paciente?
d} Estrógenos tópicos vaginales

15. Paciente femenino gesta 1, partos O, abortos O, cesáreas O, acude para


evaluación obstétrica. En un ultrasonido de control se reporta un peso fetal
estimado por debajo del décimo percentil para la edad gestacional. Debido a que
la paciente no acudió a control obstétrico desde el inicio del embarazo, no es
posible obtener una edad gestacional con precisión. La paciente menciona que
desde hace más de cinco meses notó el crecimiento del perímetro abdominal."
Se realiza una amniocentesis que revela un índice lecitina/esfingomielina > 2.
¿Cuál es la conducta terapéutica a seguir?
e} Interrumpir elembarazo

16. ¿Cuál de los siguientes antiarrítmicos tiene su principal mecanismo de acción


en células de despolarización automática y de lenta conducción?
e} Esmolol

17. ¿Cuál de los siguientes mecanismos de acción corresponde al de las


estatinas?
b}Inhibición dela enzima hidroxi-metil-glutaril-coenzima A

18. Paciente femenino de 30 años de edad secundigesta de 38 semanas de


gestación con rotura prematura de membranas de 8 horas de evolución, sin datos
de trabajo de parto ni de corioamnionitis, con un Bishop de 7. Producto en
presentación cefálica,situación longitudinal,sin datos de sufrimiento fetal. ¿Cuál es
la mejor conducta obstétrica para esta paciente?
O a}Inducción delparto con el uso de oxitocina

19. Recién nacido de 39.5 semanas de gestación, obtenido por cesárea,presenta


dificultad respiratoria durante las primeras horas de vida, con frecuencias
respiratorias de 90 respiraciones por minuto y cianosis que mejora con oxígeno.
Los campos pulmonares están bien ventilados. La radiografía de tórax muestra
líneas de líquido en las cisuras y marcas vasculares prominentes.
19a. Con base en las características del paciente, ¿cuál es su sospecha
diagnóstica?
a} Taquipnea transitoria del recién nacido

19b. ¿Cuál es la fisiopatología que ocasiona la taquipnea transitoria del recién


nacido?
b} Lenta absorción dellíquido de los pulmones fetales

20. ¿Cuál de las siguientes NO es una causa de masa mediastinal posterior?


b} Timoma

21. Paciente femenino de 25 años de edad, que acude para control prenatal de su
segundo embarazo, con 14 semanas de gestación por fecha de última
menstruación. Sin antecedentes heredofamiliares ni personales patológicos.
Únicamente refiere que el padre de este segundo embarazo es distinto al primero.
Refiere que ha tenido más náusea y vómitos que en comparación de su primer
embarazo y que últimamente se ha sentido un poco nerviosa, con caída de cabello
y diarrea. Refiere que hace dos días tuvo dolor en hipogastrio de moderada
intensidad, tipo cólico y ha presentado hemorragias intermitentes de color marrón
desde hace varias semanas. A la exploración física se observa abdomen no
doloroso a la palpación, con fondo uterino mayor para su edad gestacional. Resto
sin alteraciones. ¿Cuál de las siguientes características es la menos frecuente de
esta condición?
e} No metastatiza

22. Se requiere de su presencia en la sala de parto debido a que se espera el


nacimiento de un masculino de 37 semanas de gestación. Observa los
procedimientos del obstetra, no se sucitan complicaciones y recibe un recién
nacido masculino. Inicia la exploración física de éste y observa movimiento activo,
frecuencia cardiaca de 139 lpm, buen tono muscular,flexión de las cuatro
extremidades, acrocianosis y llanto vigoroso.
22a. De acuerdo con las características del recién nacido, ¿qué puntaje de Apgar
obtiene?
c} 8 puntos
22b. Al recibir a un recién nacido vigoroso como en el caso descrito, ¿cuál de los
siguientes pasos se debe realizar primero?
a} Verificar que la vía aérea esté permeable, secar,y estimular al recién nacido

22c. En caso de no contar con la información de la edad gestacional del recién


nacido descrito anteriormente, ¿cuál de los siguientes signos clínicos le
indicaríamejorque se trata de un recién nacido a término?
c} Tejido mamario palpable

22d. El neonato pasa a cuidados en cuneros y se comienza el protocolo de recién


nacido por las enfermeras, el cual incluye escrutinio de hipoglucemia. En el
neonato a término asintomático, ¿a qué nivel de glucosa debe iniciarse la
evaluación y tratamiento de hipoglucemia?
c} 45 mg/dl

22e. Después de una semana de nacido, el paciente es llevado a consulta de


revisión. ¿Cuál de las siguientes opciones es la correcta respecto al peso de un
recién nacido en la primera semana de vida?
d} Perder aproximadamente 5 a 10% delpeso al nacimiento

23. Paciente masculino de 55 años de edad, que se presenta con una tumoración
en la región lumbar derecha, la cual se extiende hacia la región inguinal
ipsolateral. Después el paciente fue diagnosticado con tuberculosis. De acuerdo
con la presentación clínica, ¿cuál de los siguientes músculos es el que más
probablemente se encuentra afectado por el absceso tuberculoso que presenta
este sujeto?
a} Psoas

24. Paciente femenino de 87 años es encontrada tirada en el suelo de su


recámara por su nieta; la señora no responde y no parece respirar normal.
24a. La primera medida que debe tomar la persona que la encuentra (la nieta) es:
a} Activar elsistema de emergencias
24b. En caso de ser un rescatador no entrenado, deberá de:
c} Posterior a activar elsistema de emergencias, dar solamente compresiones

24c. ¿Cómo se definen las compresiones torácicas de calidad en adultos?


b} Ritmo de 100/min y profundidad de al menos 5cm

24d. ¿Cuál es el 3er eslabón en la cadena de supervivencia del ACLS?


a} Rápida desfibrilación

24e. En caso de ser dos rescatadores, ¿cada cuánto tiempo se deben de rotar?
d} 2 min

25. Recién nacido de término, con antecedente de distrés respiratorio después del
nacimiento, resultó sin complicaciones. A los 13 días de vida inicia con distensión
abdominal importante, intolerancia a los alimentos con vómito y evacuaciones
hemorrágicas abundantes. Se solicita una radiografía abdominal,en la que se
encuentra neumatosis intestinal.
25a. En relación con el diagnóstico que sospecha, todas las siguientes
condiciones se asocian a esta patología, EXCEPTO una, indique cuál:
e} Fibrosis quística

25b. Se decide iniciar manejo médico. Todas las opciones que se indican son
correctas EXCEPTO una, indique cuál:
b} Alimentación entera!

25c. ¿Cuál de las siguientes situaciones justificaría el manejo quirúrgico de este


paciente?
b} Neumoperitoneo

25d. Posterior a manejo quirúrgico el paciente egresa y continúa sin


complicaciones los primeros meses de vida. A los cuatro meses, inicia con
distensión abdominal y vómitos persistentes. De acuerdo con los antecedentes,
¿qué complicación puede esperarse en este paciente?
a} Estenosis colónica

26. Se trata de paciente femenino de 18 semanas de embarazo que acude a


consulta para recibir inmunizaciones. De las siguientes vacunas, ¿cuál sí se
podría administrar?
b} Tétanos

27. ¿Desde dónde se extiende el conducto auditivo externo?


b} De la aurícula a la porción externa de la membrana timpánica

28. ¿Cuál de las siguientes enzimas requiere de la vitamina K como coenzima?


d} Gammaglutamil carboxilasa

29. Paciente femenino de 42 años de edad, fumadora, que acude por embarazo
de 14 SDG porque ""escuchó en la televisión que embarazos de mujeres grandes
tienen riesgos"". Sin antecedentes heredofamiliares de importancia. G2 PO AO
C1. El primer embarazo fue normoevolutivo sin alteraciones y la cesárea se realizó
por sufrimiento fetal. Este es un embarazo no planeado. No toma fumarato ferroso
ni ácido fálico, aunque refiere que hace seis meses tomaba vitamina A ""para
mejorar su vista"". Ya recibió su primer dosis de toxoide tetánico y se le realizó un
examen general de orina que no reporta alteraciones. Usted decide practicarle un
USG.
29a. ¿Qué datos ecográficos no harían sospechar de una cromosomopatía?
a} Gastrosquisis

29b. ¿Qué datos de laboratorio no nos harían pensar en un síndrome de Down?


e} iAFP

29c. Sobre métodos de diagnóstico, señale la relación incorrecta.


c} Biopsia corial:a las 6 SDG

30. Acude paciente femenino de 27 años de edad a consulta de control prenatal


por presentar embarazo de 10 semanas de gestación, no cuenta con
antecedentes de importancia, no toma ningún medicamento y no tiene factor de
riesgo alguno para el embarazo. Según la norma oficial mexicana, ¿cuántas
consultas como mínimo deben tenerse durante el embarazo?
b} 5

31. Usted se encuentra en una guardia de urgencias ginecoobstétricas y llega una


mujer de 36 semanas de gestación con trabajo de parto. Nunca había tenido
control prenatal previo. Con fondo uterino de 34 cm desde sínfisis del pubis. Palpa
dos dorsos con la segunda maniobra,con un polo cefálico encajado y el otro a
nivel del fondo uterino, con frecuencia cardiaca fetal de 150 y 135 latidos por
minuto respectivamente. Usted decide que nazcan vía cesárea sí
a} Monoamnióticos

32. Recién nacido de 20 horas de vida, producto de primera gestación, con tipo
sanguíneo B-, la madre tiene sangre tipo A+. ¿Qué es FALSO en relación con la
incompatibilidad ABO?
b} Nunca ocurre en elprimer embarazo

1. Paciente masculino de 29 años de edad con dolor bilateral en las rodillas. Al


realizar la historia clínica menciona que todos los días trota una distancia de 8 a
1O km. ¿Cuál es la causa más probable del dolor de rodillas en este paciente?
a} Síndrome patelofemoral
2. Paciente masculino de 6 años y 9 meses de edad es traído a consulta.Como
antecedente se menciona candidiasis oral a los 3 meses de edad, por lo que fue
hospitalizado en dos ocasiones en las cuales recibió corticoesteroides y
antibióticos no especificados;además, cursó con múltiples infecciones en las vías
respiratorias altas. A la exploración física se observa paciente con apariencia a su
edad cronológica, peso y talla en percentil 25, tórax con estertores crepitantes en
ambas bases,desarrollo psicomotor adecuado. Por el antecedente de infecciones
de repetición, se le solicitaron exámenes que reportaron leucocitosis e
hipogammaglobulinemia.
2a. Con base en las características del paciente, ¿cuál es el diagnóstico más
probable?
a} Agammaglobulinemia ligada al cromosoma X

2b. ¿Qué hallazgos apoyan su diagnóstico en los estudios complementarios?


c} Concentraciones séricas de lgG, lgA, lgM elgE muy por debajo dellímite mínimo
normal y cuenta deleucocitos normal,pero ausencia de linfocitos B
2c. De acuerdo a su sospecha clínica, ¿cuál es la alteración genética asociada?
b} Proteína Btk

2d. En relación con su sospecha clínica, ¿cuál es el tratamiento que estaría


indicado en este paciente?
b)lnmunoglobulina intravenosa

2e. De acuerdo con su sospecha clínica, ¿cuál es la complicación que se podría


presentar en el paciente?
c} Parálisis por vacuna de polio

3. Paciente masculino de 65 años de edad con factores de riesgo cardiovascular


se presenta a la sala de urgencias con hemiplejía derecha, así como parálisis
facial derecha con preservación de los movimientos de la mitad derecha de la
frente. El paciente también muestra dificultad para la comunicación oral. El
paciente se encuentra alerta con frecuencia cardiaca de 88 latidos por minuto y
presión arterial de 175/115 mm Hg, sin dificultad ventilatoria y con una cifra de
glucosa capilar de 190 mg/dL. Los familiares refieren que el cuadro inició hace
aproximadamente 1 hora. Una tomografía computarizada cerebral resulta normal.
¿Cuál de los siguientes medicamentos tiene lugar en el tratamiento de este
paciente?
C} Aspirina

4. Si en un paciente con hipertiroidismo clínico y bioquímico el gammagrama


tiroideo con 1311 muestra captación del radiofármaco disminuida o ausente, el
diagnóstico es:
b} Tiroiditis subaguda
5. Un paciente ha perdido la habilidad para discriminar entre dos puntos de
presión aplicados directamente a la piel de la mano izquierda. ¿Cuál de los
siguientes hallazgos es más probable que también se presente en este paciente?
b} Lesión dellemnisco medio en el lado derecho del puente

6. Acude a consulta pediátrica un paciente de 12 años de edad que sufrió un golpe


en la región lateral de la rodilla derecha durante un partido de fútbol. A la
exploración física el paciente muestra abducción anormal de la rodilla al
movimiento pasivo y desplazamiento anterior excesivo de la pierna cuando la
articulación de la rodilla se coloca en ángulo recto. Se solicita resonancia
magnética de la región,la cual confirma daño del ligamento colateral medio y del
menisco medio. Es probable que exista una tercera estructura lesionada en este
paciente; durante la reparación quirúrgica, ¿dónde deberá insertarse esta tercera
estructura?
C} Medial y anterior

7. Una paciente de 50 años de edad con diabetes mellitus tipo 2 y antecedente de


síndrome de ovario poliquístico se somete a histerectomía total abdominal para
tratamiento y estadificación de carcinoma endometrial. ¿Cuál de las siguientes
estructuras está en riesgo de ser dañada cuando las arterias uterinas son ligadas?
e} Ureteros

9. Una paciente de 30 años de edad con un embarazo de 40 semanas de


gestación se presenta en trabajo de parto. Una hora posterior al parto la paciente
presenta dolor intenso en la región perianal. El anestesiólogo decide realizar un
bloqueo nervioso para tratar dicho dolor ¿Cuál de los siguientes nervios se
encarga de inervar la región perianal y, por lo tanto, es responsable del dolor en
esta paciente?
C} Pudendo

1O. Femenino de 19 años con antecedente de hospitalizaciones frecuentes por


intoxicación alcohólica. En esta ocasión es ingresada después de haber estado
ingiriendo de manera constante bebidas alcohólicas desde hace cinco días. A la
exploración física, la presión arterial es de 110/70 mm Hg, frecuencia cardiaca de
80 latidos/min, peso de 70 kg. Se presenta letárgica y con habla incoherente. En la
sala de Urgencias, presenta una crisis convulsiva, que se yugula con diazepam.
Los estudios de laboratorio reportan sodio de 11O mEq/L, potasio sérico de 3.8
mEq/L, nivel de alcohol de 250 mg/dl, osmolalidad sérica de 230 mOsm/kg, y
concentración de glucosa de 92 mg/dl. No hay alteración en la función renal.
¿Cuál de las siguientes opciones representa el mejor tratamiento para esta
paciente?
a} Solución salina hipertónica (3%) intravenosa a una velocidad de infusión
de

11. Paciente masculino de 1 mes de edad es referido a su consulta por soplo.


Como antecedentes se menciona parto prematuro a las 35 semanas de
gestación,con cuidado prenatal deficiente. En la exploración física se identifican
pulsos periféricos saltones y soplo característico en maquinaria, se encuentra en
percentil 50. En la radiografía de tórax no se aprecia nada significativo.
11a. Con base en las características del paciente, ¿cuál es el diagnóstico más
probable?
d} Conducto arterioso persistente

11b. En relación con su sospecha clínica, ¿con qué problema médico se ha


asociado este diagnóstico?
c} Nacimiento pretérmino

11c. De acuerdo con su sospecha clínica, ¿qué hallazgos esperaría encontrar en


la radiografía de tórax?
a} Aumento de la vascularización pulmonar

11d. Conbase en su sospecha clínica, ¿cuál es el tratamiento indicado en este


paciente?
c} Administración deindometacina

11e. De acuerdo con su sospecha clínica, ¿cuál es la complicación que se puede


presentar a largo plazo?
d}Insuficiencia cardiaca en lactantes
12. Bailarina exótica (table dance) de 22 años de edad que es llevada al servicio
de urgencias por fibrilación auricular. La paciente menciona que desde hace varias
semanas había presentado palpitaciones, con mayor frecuencia los lunes. A la
exploración física la paciente tiene una frecuencia cardiaca de 123 latidos/minuto,
frecuencia respiratoria de 18 respiraciones/minuto, presión arterial de 175/98 mm
Hg. Se puede percibir un soplo sistólico. Sin tratamiento, el ritmo cardiaco se
vuelve sinusal. ¿Cuál de las siguientes opciones es la más probable como agente
etiológico del cuadro de esta paciente?
e} Alcohol

13. Paciente masculino de 12 años de edad, quien este invierno presenta cuadros
repetitivos de infección de vías aéreas superiores, múltiples tratamientos
sintomáticos sin recibir antibióticos por considerarse cuadros virales y alérgicos;
hace 2 días y el día de hoy por la madrugada se despierta por accesos de tos
disneizante, cianozante, la madre refiere que se escucha un ""silbato"" cuando el
niño respira, mejora ligeramente cuando no tose y en reposo. A la exploración
física se encuentra paciente con taquipnea, tos, disnea que limita la actividad
física,sibilancias audibles a distancia, acrocianosis, aleteo nasal y tiraje intercostal
bajo, la orofaringe se encuentra hiperémica,con descarga posterior hialina,
hipertrofia amigdalina grado 2/4; resto de la exploración sin alteraciones evidentes.
FC 95 lpm, FR 22 rpm, temp. 36.?ºC, TA 100/70 mm Hg, peso 32 kg, talla 1.30 m.
13a. El diagnóstico clínico del paciente es:
c} Crisis asmática

13b. Al evaluar al paciente usted solicita una espirometría que reporta FEV1
>80%, junto con los datos clínicos puede clasificar esta exacerbación como:
b} Persistente leve

13c. Una vez establecido el grado de severidad, usted inicia tratamiento con:
d} Corticoesteroidesinhalados a dosis bajas y beta-2-agonistas delarga duración

13d. Al solicitar varios estudios, usted espera encontrar:


b} Biometría hemática con eosinofilia

13e. Se controla la crisis con el tratamiento establecido, al egreso del paciente


usted recomienda:
a} Beta-2-agonistas de corta acción

14. Paciente masculino de 20 años que pertenece al ejército acude a urgencias


por presentar tos no productiva desde hace una semana asociada con fiebre,
disnea y dolor torácico al respirar profundamente. Las radiografías PA y lateral de
tórax muestran un infiltrado intersticial bilateral. Usted diagnostica neumonía
atípica adquirida en la comunidad.
¿Cuá es el antibiótico de elección para el tratamiento de la neumonía atípica
adquirida en la comunidad?
c} Azitromicina

15. Se trata de una mujer de 28 años de edad, sin antecedentes relevantes, niega
haberse enfermado en el último año o haber tomado antibióticos, presenta un
cuadro de fiebre con escalofrío, náusea, lumbalgia, disuria y polaquiuria, motivo
por el que acudió a la consulta; a la exploración física se encuentra
hipersensibilidad en el ángulo costovertebral derecho, así como hiperbaralgesia en
los puntos ureterales del mismo lado. FC 105 lpm, FR 18 rpm, temp. 38.3 ºC, PA
110/70 mm Hg, peso 58 kg, talla 1.68 m
15a. Con los datos obtenidos ¿cuál es la sospecha diagnóstica?
c} Pielonefritis aguda

15b. Para confirmar el diagnóstico usted decide tomar uno de los siguientes
estudios de laboratorio:
C)Urocultivo

15c. En relación al urocultivo, ¿cuál de las siguientes frases define una


""bacteriuria significativa""?"
b} Recuento de > 100000 UFC/ml

15d. Esta paciente no está exenta de complicaciones, ¿cuál es una indicación de


estudios de imagen?
d) Fiebre más de 72 horas

15e. Establecido el diagnóstico y tomando los factores de riesgo de la paciente


¿cuál es el tratamiento de primera elección?
a} Ampicilina 1g cada 6 horas + gentamicina 1mg/kg cada 8 horas
16. Lactante femenino de siete meses de edad que desde el nacimiento fue
diagnosticada con pie equino varo y ha sido tratada con múltiples yesos sucesivos,
que no han corregido en su totalidad el defecto. ¿Cuál es el tratamiento más
adecuado en este momento?
c} Tenotomía percutánea deltendón de Aquiles

17. Niña de tres años de edad que desde hace dos días presenta fiebre y otalgia
izquierda. Tiene antecedente de rinofaringitis hace cinco días. En el examen físico
se observa membrana timpánica izquierda eritematosa y edematosa, abultada, y
se observa líquido en oído medio. Hay pérdida del reflejo luminoso. Todas las
siguientes son potenciales complicaciones de la patología que presenta la
paciente, EXCEPTO:
e} Colesteatoma

18. Paciente del sexo femenino de 45 años de edad que se presenta al Servicio de
Urgencias por presentar diarrea con moco y sanguinolenta. Al interrogatorio la
paciente niega cualquier antecedente que pudiese explicar su padecimiento
actual. A la exploración física se encuentra que en las piernas la paciente tiene
lesiones ulceradas con bordes violáceos; al tacto rectal se evidencia la presencia
de sangre en recto. La biopsia de colon revela inflamación en la mucosa y
submucosa. ¿Cuál de los siguientes medicamentos es el más adecuado para el
tratamiento de esta paciente?
b} Sulfasalazina

19. ¿Cuál de las siguientes condiciones ocasiona una disminución de la


osmolaridad urinaria y un incremento del flujo urinario sin cambiar la fracción
excretada de sodio ni la depuración osmolar?
c} Administración de litio

20. Paciente masculino de 60 años de edad con diagnóstico reciente de


hipertensión arterial sistólica aislada. Después de tres semanas de tratamiento la
presión arterial del paciente se ha reducido a los valores esperados, sin embargo
el paciente menciona que presenta edema tibial bilateraly cuadros de
enrojecimiento facial. ¿Cuál de los siguientes medicamentos es el que más
probablemente está consumiendo este paciente?
a} Amlodipino

21. ¿Cuál de los siguientes efectos adversos corresponde a la administración de


mercaptopurina?
e} Dolor abdominal eictericia

22. Paciente femenino de 87 años consulta por molestias al comer y dolor en la


boca del estómago; reporta que al ingerir alimento siente que le queman el
estómago y que tomar una pastilla para la gastritis le confiere alivio. Tiene
antecedentes de artritis desde hace 5 años, por lo que toma una pastilla de 500mg
de ASA al día, más dos de 275mg de naproxeno sódico/300mg de paracetamol.
Además, refiere haber bajado de peso. TA: 140/85, FC: 85x', FR: 18x', Temp:
37°C."
22a. ¿Cuáles su sospecha diagnóstica?
c} Enfermedad ácido péptica

22b. ¿Cuál es la causa más común de sangrado del tubo digestivo alto?
a} Enfermedad ácido péptica

22c. En esta paciente el factor de riesgo más importante es:


C} Uso de AINE

22d. ¿Cuál complicación de esta enfermedad representa mayor peligro (mayor


mortalidad)?
a} Perforación gástrica

22e. El tratamiento más adecuado para esta paciente es:


d} Omeprazol + inhibidores dela COX-2

23. Paciente femenino de 50 años, fumadora actual, en tratamiento de reemplazo


hormonal, se presenta a la sala de urgencias con dolor pleurítico y disnea. Ésta es
la segunda vez que acude al hospital en la última semana; hace cuatro días
acudió por una neumonía lobar de localización inferior en el pulmón derecho, la
cual fue diagnosticada utilizando radiografía de tórax. En esta ocasión a la
exploración física la pantorrilla izquierda de la paciente se presenta con edema y
eritema. Debido a estos datos, se sospecha de embolia pulmonar como la causa
del padecimiento actual de la paciente. ¿Cuál de los siguientes métodos
diagnósticos es el más adecuado en este caso?
e} Tomografía computarizada espiral contrastada deltórax

24. Paciente femenina de 65 años de edad. Consulta por cefalea y disnea de


esfuerzo. Al interrogatorio refiere dispepsia de larga evolución. A la exploración
física se registra TA: 120/80, FC: 11O lpm, palidez de tegumentos, disminución de
la sensibilidad vibratoria en miembros inferiores. Laboratorios: Hb: 5.5 g/dl, VCM:
120 ft, reticulocitos: 2%, LDH aumentada, leucocitos: 3 000, plaquetas: 85 000."
24a. Señale la causa más probable de la anemia en esta paciente:
b} Deficiencia de vitamina 812

24b. ¿Cómo se espera que sean los valores de ácido metilmalónico sérico y la
homocisteína total en un paciente con deficiencia de vitamina B12?
d} Ácido metilmalónico aumentado y homocisteína aumentada

24c. ¿Cuál es el estudio indicado para confirmar el diagnóstico de anemia


perniciosa?
a} Prueba de Schilling

24d. ¿En qué región del sistema gastrointestinal se absorbe la vitamina 812?
d} Íleon
24e. Se confirma el diagnóstico de anemia perniciosa. ¿Cuál es el tratamiento
indicado en el paciente?
c} Vitamina 812 parenteral

25. ¿Cuál de los siguientes enunciados es correcto respecto a la granulomatosis


de Wegener?
O b} Los pacientes con enfermedad sinusal limitada se pueden beneficiar del
tratamiento crónico con trimetoprim/sulfametoxazol

26. Paciente masculino de 44 años de edad cuyo conteo más reciente de linfocitos
T CD4+ es de 55 células/mm3. Se decide iniciar tratamiento indicado en este
paciente; sin embargo después de varias semanas se presenta con anemia
megaloblástica. ¿Cuál de los siguientes fármacos es el que más probablemente
está causando este cuadro?
e} Trimetoprim-sulfametoxazol
27. Se presenta a la consulta externa un paciente de cuatro años de edad el cual
presenta celulitis, descamación de gran parte del cuerpo con signo de Nikolsky
positivo. ¿Cuál de los siguientes diagnósticos es el más probable en este
paciente?
e} Síndrome de la piel escaldada estafilocócica

28. ¿Cuál de los siguientes antiarrítmicos disminuye la fase 0 del potencial de


acción?
C} Propafenona

29. Paciente masculino de 45 años de edad con antecedente de alcoholismo


crónico es llevado al servicio de urgencias posterior a sufrir un leve traumatismo al
ser golpeado por un automóvil. Una vez en el hospital el paciente se diagnostica
con una fractura de cadera. Se decide realizar reparación quirúrgica. A pesar que
durante el posquirúrgico inmediato no suceden eventualidades, al cuarto día de
hospitalización el paciente desarrolla mialgias y debilidad. Los estudios de
laboratorio revelan elevación de la creatinina sérica,con hematuria sin glóbulos
rojos.
¿Cuál de los siguientes estudios de laboratorio revelará con mayor probabilidad la
causa de los síntomas de este paciente?
b} Medición de los niveles séricos de fosfato

31. Paciente femenino de 22 años de edad que acude al servicio de urgencias


ginecobstétricas del hospital regional por hemorragia vaginal. La paciente se había
realizado una prueba de embarazo hace dos semanas la cual resultó positiva, sin
embargo, por algunas razones aún no había acudido para consulta de evaluación
y control prenatal. En la sala de urgencias se realiza un ultrasonido de útero el
cual revela la presencia de estructuras quísticas en forma de ""racimo de uvas"",
sin evidencia de un embrión. Si esta paciente presenta diagnóstico de mola
hidatidiforme completa, ¿cuál de los siguientes hallazgos sería más probable de
encontrar?
a} El genotipo de las células de la mola es 46,XX y es completamente de origen
paterno"

32. ¿Cuál de las siguientes condiciones se asocia con disminución de la


capacidad pulmonar total, incremento del volumen residual, disminución de la
presión inspiratoria máxima y normalidad del índice fracción espirada de volumen
en el primer segundo/capacidad vital forzada?
d} Miastenia grave

33. Paciente femenino de 60 años de edad la cual es ingresada por cuarta ocasión
durante los últimos tres meses por presentar palpitaciones y disnea secundaria a
una fibrilación auricular con respuesta ventricular rápida. El equipo médico tratante
está considerando administrar amiodarona por tiempo indefinido para evitar este
tipo de ataques. ¿Cuál de los siguientes estudios es recomendable realizar antes
de administrar este medicamento?
a} Pruebas de función tiroidea

34. Paciente femenino de 19 años de edad que se presenta con menorragia desde
su menarca y recientemente ha sido hospitalizada por padecer anemia grave
(hemoglobina 6 g/dl). Los estudios de laboratorio demuestran niveles normales del
conteo plaquetario, tiempo de protrombina y del índice de internacional
normalizado. Entre las anormalidades se encuentran prolongación leve del tiempo
de tromboplastina parcial y del tiempo de sangrado. ¿Cuál es el diagnóstico más
probable en esta paciente?
e} Enfermedad de von Willebrand

35. Paciente masculino de una semana de vida, producto de una madre de 42


años de edad. El neonato presenta un fenotipo caracterizado por pliegues
epicánticos, implantación baja de orejas, abertura oral pequeña y pliegue palmar
único. A la exploración cardiovascular se puede percibir un soplo holosistólico en
todo el borde esternal. ¿Cuál de las siguientes anormalidades cromosómicas es la
que con mayor probabilidad causa este cuadro?
d} Mosaicismo

36. ¿Cuál de los siguientes trastornos representa una contraindicación para la


administración de concentrados plaquetarios?
C} Púrpura trombocitopénica trombótica

37. ¿Cuál de los siguientes fármacos se asocia con la vasodilatación de la


circulación esplácnica?
C} Dosis bajas de dopamina (2 a 4 µg/kg/min}

38. ¿Cuál de los siguientes medicamentos es una benzodiacepina de larga


duración de acción?
a} Clorodiazepóxido
39. Paciente masculino de 65 años de edad con diagnóstico de diabetes mellitus
tipo 2 y enfisema con antecedente de tabaquismo intenso. El paciente se presenta
al servicio de urgencias por un cuadro neumónico ocasionado por Streptococcus
pneumoniae. El paciente recibe tratamiento antibiótico adecuado y es egresado
sin eventualidades. Al momento del egreso se le recomienda abandonar el hábito
tabáquico.
¿Cuál de las siguientes medidas también se debe realizar en relación a este
paciente previamente a su egreso?
c} Administrar vacuna contra lainfluenza y elneumococo

40. Paciente masculino de 31 años de edad que se presenta a consulta por tener
una tumoración en el lado derecho del cuerpo, la cual ha aumentado
considerablemente de tamaño desde que apareció hace tres meses. Entre los
antecedentes de importancia, en este paciente se encuentran tabaquismo intenso
desde los 15 años de edad y consumo moderado de alcohol, llegando a la
embriaguez una vez por mes. Se realiza una biopsia de la lesión, la cual revela
una translocación t(14;18). ¿Cuál de los siguientes productos génicos es el que
corresponde a la translocación cromosómica que presenta este paciente?"
a} Sobrexpresión de Bcl2

41. Paciente masculino de 45 años de edad sin ningún antecedente de


enfermedad hepática o digestiva. Durante una exploración física de rutina se
encuentra que tiene una tumoración abdominal en el cuadrante superior derecho y
que existe dolor a la palpación de ésta. Se realiza un ultrasonido el cual revela
hepatomegalia,así como la presencia de múltiples lesiones quísticas hepáticas. Al
interrogatorio el paciente comenta que hace algunas semanas viajó a visitar un
familiar en una granja,y tuvo contacto con perros y ovejas. ¿Cuál de los siguientes
agentes es el que con mayor probabilidad está ocasionando el cuadro de este
paciente?
d} Echinococcus granulosus

42. ¿Cuál es el mecanismo de acción por el que la selegelina puede mejorar la


sintomatología de la enfermedad de Parkinson?
b}Inhibición dela degradación de dopamina porla monoaminooxidasa tipo B

43. ¿Cuál de los siguientes medicamentos es el más adecuado en el caso de


pacientes con enfermedad de Wilson?
a} Penicilamina

44. ¿Cuál de los siguientes medicamentos se asocia con síndrome nefrótico como
efecto adverso?
C} Sales de oro

45. ¿Cuál de los siguientes trastornos se caracteriza por niveles en rangos


normales de hormona luteinizante y testosterona,aumento de la hormona
foliculoestimulante en combinación con un bajo conteo de espermatozoides?
c} Criptorquidia

46. Femenino de 35 años acude a Urgencias debido a un dolor en la fosa lumbar


derecha que se irradia hacia la ingle del mismo lado, pasando por el flanco
derecho. El dolor inició hace algunas horas y no ha cedido con la ingesta de
diclofenaco. La paciente refiere que es la primera vez que le sucede, pero ha
tenido desde hace días molestia al orinar y sensación de vaciado incompleto; se
observa afectada, sudorosa y cambia de postura continuamente. EF: 1.50m;85kg;
TA: 145/85; Temp: 36.5ºC.
46a. ¿Cuál es la principal sospecha diagnóstica?
C} Urolitiasis

46b. ¿cuál es el hallazgo más común en el examen general de orina en pacientes


con litiasis renal?
b} Hematuria

46d. El estándar de oro para el diagnóstico es:


b} TC

46e. Lito más frecuente:


c} Oxalato cálcico
47. Paciente de 46 años con diagnóstico de varios años de esquizofrenia. El
médico tratante decide iniciar un nuevo antipsicótico. Dos semanas después del
cambio de antipsicóticos el paciente se presenta a consulta de urgencia con un
cuadro neumónico grave. La citometría hemática revela reducción del conteo de
neutrófilos, basófilos y eosinófilos. ¿Cuál de los siguientes fármacos es el que con
mayor probabilidad pudo haber causado este trastorno?
b} Clozapina

48. Paciente femenina de 35 años, ama de casa, inicia hace 2 meses con malestar
general,fatiga, artralgia en ambos carpos y articulaciones interfalángicas distales, y
rigidez matutina. Dactilitis en el 2do dedo de la mano derecha. Lesiones:
hiperqueratosis y onicólisis en uñas de las manos de más de un año de evolución.
Dolor a la palpación de la articulación sacroilíaca izquierda.
48a. Señale el diagnóstico más probable con base en la clínica de la paciente:
b} Artritis psoriásica

48b. ¿Cual de los siguientes signos clínicos es característico de la artritis


psoriásica?:
a} Acortamiento de los dedos

48c. Hallazgo radiográfico encontrado en la artritis psoriásica.


d}Imagen de ""lápiz en copa"" en articulacionesinterfalángicas"

48d. El diagnóstico de la artritis psoriásica se realiza con base en:


c} La clínica
48e. Tratamiento indicado en esta paciente para evitar la progresión de la
enfermedad.
a} AntiTNF

49. ¿Cuál de los siguientes enunciados respecto a la recurrencia de la infección


por Clostridium difficile es correcta?
b} Las recurrencias delasinfecciones por C/ostridium difficile se asocian con
complicaciones serias

50. Paciente masculino de 37 años que acude al servicio de urgencias por


presentar dolor súbito tipo cólico en fosa lumbar izquierda, que se irradia a
testículo izquierdo, acompañado de náusea, vómito y diaforesis. A la exploración
física se encuentra afebril, Giordano izquierdo, resto normal. Se solicita radiografía
simple de abdomen en la que no se aprecia litiasis. ¿Cuál es la aseveración
correcta?
c} Le da elalta indicándolelos síntomas de alarma y tratamiento sintomático

51. ¿Cuál de los siguientes movimientos es el que más frecuentemente ocasiona


dolor en pacientes con síndrome del manguito retador?
a} Abducción delbrazo

52. Se presenta a consulta externa paciente del sexo femenino de 20 años de


edad por presentar resequedad y ulceraciones en la piel que cubre las
prominencias óseas, principalmente las rodillas y los codos. Entre los hallazgos de
importancia a la exploración física se pueden observar placas pequeñas grisáceas
en la conjuntiva. ¿Cuál de los siguientes diagnósticos es el más probable en esta
paciente?
b} Deficiencia de vitamina A

53. Paciente femenino de 29 años de edad con antecedente de esquizofrenia.


Había sido internada debido a que sufrió varios episodios en los que decía que
escuchaba voces que la insultaban y le decían qué acciones llevar a cabo. Al
momento del egreso se le prescribió haloperidol.
¿Cuál sintomatología es la más probable que la paciente llegue a presentar?
d} Galactorrea

54. ¿Cuál de los siguientes hallazgos podría estar presente con mayor
probabilidad en pacientes con lesión de la división superior del nervio facial?
a}Incapacidad ipsilateral para fruncirla frente

55. Paciente femenino de 15 años de edad estudiante de bachillerato que se


presenta a consulta. La paciente presenta antecedente de asma leve intermitente
bajo tratamiento adecuado; sin embargo, desde hace cuatro días ha presentado
fiebre, tos, malestar general y mialgias intensas. Al interrogatorio menciona que
varios profesores y estudiantes de su centro de estudios presentan sintomatología
similar.Al revisar su expediente, el médico observa que esta paciente no recibió su
última dosis de inmunoprofilaxis contra la infección del virus de la influenza."
¿Cuál de los siguientes tratamientos es el más adecuado en ella?
b} Tratamiento sintomático basado en antihistamínicos

56. Individuo masculino de 40 años, sin antecedentes heredofamiliares de


importancia, presenta dolor abdominal de inicio súbito que se irradia del flanco
derecho a la región inguinal, persiste aun en reposo; se acompaña de náusea,
sudación y urgencia urinaria. Signos vitales: temperatura = 37 C, frecuencia
respiratoria = 12 respiraciones/min, frecuencia cardiaca = 68 latidos/min, presión
arterial = 130/80 mm Hg. A la exploración física nada relevante. ¿Cuál es el
diagnóstico más probable?
d} Cálculo ureteral derecho

57. Una mujer de 45 años de edad que se presenta a consulta por padecer dolor
de hombro desde hace aproximadamente ocho semanas, el cual es más intenso
cuando acude a sus entrenamientos de tenis. Durante el reposo el dolor suele
desaparecer. A la exploración física el hombro se encuentra doloroso, con
limitación del movimiento activo y pasivo. En las radiografías de la región se
pueden observar calcificaciones por encima del trocánter mayor. Se le indica a la
paciente que lo más probable es que padezca tendinitis calcificada, por lo que
requiere tratamiento durante seis semanas con un cabestrillo, reposo, calor y
antiinflamatorios no esteroideos. Después del periodo de tratamiento la paciente
acude de nuevo a consulta, pero los síntomas persisten. ¿Cuál de las siguientes
opciones describe mejor el siguiente paso en el manejo de esta paciente?
e} Aspiración de la calcificación guiada por ultrasonido

58. ¿Cuál de los siguientes organelos se encuentra ausente en los eritrocitos


maduros, y por esta razón éstos son incapaces de sintetizar el grupo heme de la
hemoglobina, a pesar de que se les inyecte el mRNA correspondiente?"
e} Mitocondria

59. Según la clasificación de fracturas del platillo tibial de Schatzker, ¿cuál de las
siguientes opciones representa la que involucra sólo el platillo tibial lateral sin
presentar depresión?
a} Tipo 1

60. ¿Cuál de las siguientes vitaminas contribuye a la carboxilación de residuos de


glutamina de proteínas en el hígado?
a} Vitamina K

61. Paciente femenino de 94 años, sin antecedentes de interés, salvo asma


extrínseca por epitelio de animales, osteoporosis y cirugía de prolapso uterino con
incontinencia urinaria de esfuerzo residual.¿Cuál de estos elementos no es
indispensable para ser un individuo continente?
d} No ser persona dela tercera edad
62. Paciente masculino de nueve meses de edad que presenta dermatosis
localizada a región genital afectando escroto, periné y región perianal; que está
caracterizada por eritema, erosiones, escama y algunas zonas con costras
serohemáticas y maceración, de 1O días de evolución, caracterizando una
dermatitis del pañal. ¿Qué factores predisponen esta enfermedad?"
a} Contacto prolongado con evacuaciones

63. Paciente femenino de 75 años con diagnóstico de DM2 desde hace 15


años(en tratamiento con metformina e insulina), hipertensa desde los 45 años de
edad (en tratamiento con nifedipino e hidralazina), fumadora desde hace 45 años
(cinco cigarrillos diarios),acude a consulta por sus propios medios (caminando)
debido a que sufrió el día de ayer una caída desde su propia altura, de sentón, en
el baño.Según refiere, la caída la relaciona con una pérdida reciente de fuerza en
las piernas; también se queja de dolor en la región de las nalgas.No presenta
limitaciones del movimiento y comenta que ha perdido altura de manera reciente.
La EF: 1.58m, 65kg; TA: 140/80; Temp: 36.5ºC. A la exploración se registra dolor
en la zona de las nalgas, movilidad y fuerza de los miembros pélvicos sin
alteraciones, reflejo anal y continencia fecal y urinaria conservadas, así como dolor
a la palpación de la columna lumbar.
63a. ¿Cuál de los siguientes estudios sería de mayor utilidad diagnóstica?
d} Radiografías simples de columna vertebral

63b. En el estudio anterior se evidencia disminución de la altura anterior de los


cuerpos vertebrales T2 y T3, además de una xifosis prominente. De acuerdo
conesto, ¿cuálsería su diagnóstico?
b} Fractura vertebral por compresión

63c. Por el tipo de fractura y los antecedentes, usted solicita una densitometría
ósea, la cual se la reportan con un valor de T=-2.8.¿Cuálsería su diagnóstico?
c} Osteoporosis

63d. ¿Cuál es un factor de riesgo modificable para osteoporosis?


b} Tabaquismo

64. Se presenta a la sala de urgencias un paciente masculino de 75 años de edad


que fue encontrado desmayado en el baño por sus familiares. Sus hijas refieren
que el paciente es fumador y que previamente se le habían diagnosticado niveles
elevados de colesterol. A la exploración física el paciente presenta presión arterial
de 180/100 y frecuencia cardiaca de 90 latidos por minuto. Cuando se trata de
interrogar al paciente, éste muestra una expresión de confusión; cuando intenta
comunicarse, su habla es extremadamente fluida pero sin propósitos informativos,
sin patrones gramaticales correctos ni significado. ¿Cuál es el sitio lesionado más
probable en este paciente?
a} Enla mitad posterior de la circunvolución temporal superior y en la parte
adyacente dela circunvolución temporal media izquierda
65. Paciente masculino de 58 años acude a consulta debido a que presenta
molestias en la vista;tiene miopía desde los 15 años y comenta que en las últimas
semanas le cuesta trabajo ver los objetos que están a sus lados e incluso no ve
cuando la gente se le acerca. Entre sus antecedentes, menciona que padece DM2
desde hace 5 años y que hace 1O años le hicieron una paquimetría y le dijeron
que tenía la córnea muy delgada, por lo cual no se operó. EF: 1.70m;85kg; TA:
145/85mm Hg; Temp: 36.5ºC y PIO: 23mm Hg; se observa en el fondo del ojo
hemorragia en astilla del disco óptico, sin datos de microaneurismas ni exudados.
65a. ¿Cuál es la principal sospecha diagnóstica?
c} Glaucoma de ángulo abierto

65b. NO son factores de riesgo para su sospecha diagnóstica:


a} Tabaquismo, raza asiática, hipermetropía"

65c. Las fases de la historia natural de estetrastorno incluyen a todas las


siguientes, excepto:

b}Islote nasal

65d. El tratamiento médico correcto es:


b} Bloqueadores beta e inhibidores dela anhidrasa carbónica

66. Paciente masculino de 59 años de edad que acude a evaluación por padecer
disnea de medianos esfuerzos y dolor torácico. En el expediente clínico no hay
ningún antecedente de importancia para este padecimiento. A la exploración física
los pulsos de las carótidas se encuentran retrasados respecto al ciclo cardiaco, así
como disminuidos en intensidad. El punto de máxima intensidad cardiaco se
muestra hiperdinámico, pero sin desplazamiento. A la auscultación se encuentra
un soplo en crescendo-decrescendo a lo largo del borde esternal izquierdo con
máxima intensidad en telediástole. El componente aórtico del segundo ruido
cardiaco se encuentra ausente. En el electrocardiograma se encuentran datos en
relación a hipertrofia ventricular izquierda. La radiografía de tórax no muestra
incremento del tamaño de la silueta cardiaca y campos pulmonares sin
alteraciones. ¿Cuál de los siguientes estudios es el más adecuado para confirmar
el diagnóstico de este paciente?
c} Ecocardiografía

67. ¿Cuál de las siguientes complicaciones del infarto miocárdico se presenta


como dolor torácico recurrente dos días después del infarto?
d} Pericarditis

68. Con relación a la nefropatía por gota:


d} Con el tratamiento adecuado, se mantiene o mejora la función renal en 80%
delos casos

69. Femenino de 48 años obesa,acude a consulta externa por iniciar con dolor tipo
cólico en hipocondrio derecho, irradiante hacia la espalda, el cual se acompaña de
náusea,anorexia y temperatura de 38.3 ºC, niega ictericia. Antecedntes:
multigesta, con malos hábitos alimentarios. A la EF presenta signo de Murphy
positivo a la maniobra de Pron, palpación profunda dolorosa, rebote positivo.
La entidad patológica responsable del cuadro clínico de esta paciente es:
b} Colecistitis aguda

70. Lactante de cinco meses de edad que recibió al nacimiento, de acuerdo con el
esquema de vacunación 2007, dosis de BCG y hepatitis B. Faltó a la cita de los
dos meses, quedando pendientes las vacunas propias de esta edad. Respecto a
la interrupción en las vacunas de hepatitis B,señale cuál es la conducta más
adecuada en este momento.
c} Administrar la segunda dosis en este momento y ocho semanas después aplicar
la tercera dosis

71. Masculino de 9 meses de edad es traído a consulta por fiebre de 38.7ºC e


irritabilidad de 2 días de evolución. La madre comenta que su bebé se manipula
mucho la oreja derecha. A la exploración física, el paciente se observa irritable, no
cooperador,con signos vitales estables; confirma la presencia de fiebre y realiza
una otoscopia. Observa membrana timpánica eritematosa y abombada, y al fondo
detrás de la membrana percibe niveles sugerentes de derrame purulento. Usted
hace el diagnóstico de otitis media.
71a. ¿Cuáles el organismo que más probablemente esté ocasionando este
cuadro?
b} Streptococcus pneumoniae

71b. De las siguientes opciones, ¿cuál es el mecanismo inicial que produce la


otitis media?
d} Alteración en la función dela trompa de Eustaquio

71. Masculino de 9 meses de edad es traído a consulta por fiebre de 38.7ºC e


irritabilidad de 2 días de evolución. La madre comenta que su bebé se manipula
mucho la oreja derecha. A la exploración física, el paciente se observa irritable, no
cooperador,con signos vitales estables; confirma la presencia de fiebre y realiza
una otoscopia. Observa membrana timpánica eritematosa y abombada, y al fondo
detrás de la membrana percibe niveles sugerentes de derrame purulento. Usted
hace el diagnóstico de otitis media.
71d. ¿Cuál es el tratamiento de elección para este paciente?
c} Amoxicilina 90 mg/kg/día

71e. ¿En qué situación le recomendaría a la madre realizar una miringotomía con
colocación de tubos a su hijo?
b} 3 episodios en 6 meses o 4 en un año (con un episodio enlos últimos 6 meses}

72. Paciente femenino de 88 años de edad es hospitalizada por una caída en su


casa debido a tropezón. No puede caminar o soportar peso sobre su extremidad
inferior derecha. Las radiografías revelan que tiene una fractura desplazada del
cuello del fémur,sin demostrar algún otro hallazgo de importancia. En comparación
con otros pacientes similares con fractura de cadera y desplazamiento
intertrocantérico, ¿cuál de las siguientes condiciones es la que se encuentra en
mayor riesgo de sufrir?
b} Osteonecrosis

73. Paciente femenino de 22 años de edad sufrió una crisis convulsiva en el


servicio de urgencias que le ocasionó luxación del hombro derecho. El residente
de urgencias solicita una interconsulta ortopédica debido a que la paciente exhibe
dolor y limitación grave del rango de movilidad. Antes del episodio convulsivo, no
tenía ninguna alteración en el hombro. A la exploración física,tanto los
movimientos pasivos como activos están limitados, en especial por dolor. La
rotación externa está limitada a menos de 20 grados. ¿Cuál de las siguientes
opciones representa el mejor paso siguiente en el manejo y evaluación de esta
paciente?
a} Radiografía axilar

74. Paciente masculino de seis años de edad con síndrome de Down,acude a


consulta porque la madre refiere conducta hipoacúsica de dos meses de
evolución; niega haber presentado fiebre y otalgia, el diagnóstico más probable es:
b} Otitis media serosa

75. Paciente femenino de 22 años de edad, refiere traumatismo nasal de 3 horas


de evolución, de dirección frontal. Presenta edema moderado de dorso nasal,sin
laterorrinia,con crepitación de huesos propios; en la rinoscopia se observa
abombamiento en región septal anterior bilateral, doloroso a la palpación. El
diagnóstico más probable es:
c} Fractura nasal y hematoma septal

76. Paciente femenino de 71 años que acude a urgencias por náusea y vómito.
Refiere que ha vomitado en cuatro ocasiones, siendo de contenido alimenticio a
simplemente un líquido claro amarillento. En sus antecedentes lo único que refiere
esosteoartritis, tratada con paracetamol de 750 mg. Al IPAS menciona que
últimamente ha requerido tomar más de este medicamento debido a que su
osteoartritis la ha molestado mucho. Cuenta que ha requerido tomarlo en seis
ocasiones en las últimas 24 horas. La hija, que acompaña a la paciente, sin
embargo cree que lo tomó en más ocasiones. Niega fiebre, dolor abdominal,
diarrea o hematoquezia. A la exploración física sus signos vitales incluyen FC
113/min, FR 19/min,T 37.1 C y PA 115/68 mm Hg. No hay anormalidades a la
exploración física. A continuación se muestran estudios de laboratorio:

Examen Resultado"
Leucocitos 8 700/mm;j
Eritrocitos 4 300 000/mm;j
Hb 1 12.0 g/dL
HCT 1 36.3%
VCM 1 75 mm;j
MHC 1 25 pg
CMHC 1 32 g/dL
RDW 13%
Plaquetas 270 000/mm;j
"Glucosa 92 mg/dL
"Creatinina 1.0 mg/dL
BUN 1 11 mg/dL
Albúmina 4.3 mg/dL
Prot. totales 6.1 mg/dL
AST 1 30 Ul/L
ALT 27 Ul/L
"Fosfatasa alcalina, 104 U/L"
Bilirrubina 0.7 mg/dL
¿Cuál de las siguientes aseveraciones es correcta respecto a la intoxicación con
paracetamol?
c} El riesgo de muerte por falla hepática es prácticamente nulo si se administra N-
acetilcisteína en las primeras 8 horas posterior a la sobredosis

77. Paciente masculino de 63 años de edad que acude a consulta por dolor
torácico en varias ocasiones, el cual es prolongado y continuo. El paciente
describe el dolor como una sensación ardorosa,que ocurre más frecuente durante
la noche; a veces el paciente se despierta con disnea después de que ya había
conciliado el sueño. Aunque estos episodios no han ocurrido durante el ejercicio,
el paciente evita cualquier actividad física extenuante como medida preventiva. A
pesar de que en el pasado se le recetó nitroglicerina, el paciente no ha adquirido
el medicamento. En una ocasión, un amigo de él le administró nitroglicerina
durante uno de estos episodios de dolor retroesternal,ocasionando desaparición
del dolor. El día de ayer,tuvo una riña con uno de sus hijos, lo cual le ocasiono
dolor torácico de gran intensidad, el cual mejoró cuando se sentó. ¿Cuál de las
siguientes opciones es con mayor probabilidad el diagnóstico de este paciente?
d} Reflujo esofágico y espasmos

78. Paciente masculino de 29 años, carnicero. Acude a urgencias por presentar


tos persistente sin expectoración de seis meses de evolución, acompañada de
dolor retrosternal relacionado con los accesos de tos. Niega disnea, hemoptisis,
fiebre o afectación del estado general. La exploración física reveló presencia de
adenopatías submandibulares e inguinales, siendo el resto normal. La radiografía
de tórax mostró infiltrados alveolares bilaterales en campos superiores y
adenopatías en hilio izquierdo. La baciloscopia del esputo en tres determinaciones
fue negativa para BAAR, al igual que los cultivos para Mycobacteríum
tuberculosis. El Mantoux también fue negativo. Se realizo una
fibrobroncoscopia,con exploración macroscópica normal y la anatomía patológica
de la biopsia bronquial reveló granulomas no necrosantes compatibles con
sarcoidosis. Se inició tratamiento con corticoesteroides orales en dosis
descendentes, con buena respuesta clínica y radiológica.
En este caso se confundió el diagnóstico de tuberculosis con sarcoidosis. ¿Cuáles
son los otros diagnósticos diferenciales que hay que tener
en cuenta ante la sospecha de tuberculosis?
c} Bronquiectasias, linfoma, vasculitis, infección pulmonar por hongos, cáncer
laríngeo, sarcoidosis

79. Paciente masculino de 60 años originario de una zona endémica de


tuberculosis, proveniente de zona de bajos recursos. Ha trabajo en una fundidora
por varios años. Dentro de sus antecedentes personales patológicos padece
enfermedad pulmonar obstructiva crónica, ha recibido corticosteroides los últimos
seis meses. Ingreso a piso hace una semana evolucionando mal,con
empeoramiento de la falla respiratoria, ha requerido ventilación mecánica asistida
por una posible neumonía.
79a. El tubo endotraqueal se debería enviar para:
d} Tinción de Ziehl-Neelsen

79b. Si la tinción anterior fue positiva para muchos organismos que se tiñen de
rojo, ¿cuál sería el régimen terapéutico adecuado?
a} Rifampicina,isoniazida, vitamina 86, pirazinamida y etambutol

80. Paciente masculino de 35 años con antecedente de uso de drogas


intravenosas acude a urgencias por presentar fiebre y escalofríos de una semana
de evolución y disnea progresiva. Hace dos días presentó parestesias en el brazo
izquierdo con debilidad de inicio súbito que duró aproximadamente 3 h y se
resolvió de manera espontánea. Sus signos vitales son temperatura de 38.5 ºC,
pulso de 120 latidos por minuto, frecuencia respiratoria de 24 respiraciones por
minuto, presión arterial de 116/76 mm Hg. A la exploración física se ausculta soplo
sistólico 3/6 en el borde inferior esternal derecho y campos pulmonares bien
ventilados. Usted diagnostica endocarditis infecciosa y procede a tomar
hemocultivos previo a iniciar tratamiento antibiótico empírico.
¿Cuál es el agente etiológico más común en usuarios de drogas intravenosas y
que debe ser cubierto por el esquema empírico de antibióticos?
b} Staphy/ococcus aureus

81. Paciente masculino de 60 años de edad que acude por disminución de la


agudeza visual central de ojo derecho de cuatro días de evolución, la cual ha sido
lentamente progresiva. No tiene ningún otro síntoma asociado. Nunca ha sido
estudiado para enfermedades sistémicas. A la exploración oftalmológica se
encuentran agudeza visual de 201200 ojo derecho y 20/30 ojo izquierdo,
discromatopsia y defecto pupilar aferente de ojo derecho. La presión intraocular de
ambos ojos es normal. La papila del ojo derecho está hiperémica, tiene edema
difuso y alrededor hay hemorragias en flama y estrechamiento arteriolar, la mácula
es normal. La fundoscopia de ojo izquierdo no muestra alteraciones. La
campimetría muestra un defecto altitudinal inferior del ojo derecho. El campo
visual del ojo izquierdo es normal. El diagnóstico es neuropatía óptica isquémica
anterior no arterítica.
81a. ¿Qué enfermedades sistémicas deben ser descartadas en este paciente, ya
que son factores de riesgo para el desarrollo de la neuropatía óptica isquémica no
arterítica?
d} A y B

81b. El diagnóstico diferencial más importante debido a que se considera una


urgencia oftalmológica es:
b} Neuropatía ópticaisquémica arterítica

82. Paciente masculino de 84 años que acude a consulta por dolor abdominal.
Refiere al dolor de tipo punzante, localizado en el hipogastrio. En su historial
médico hay hipertensión,enfermedad coronaria e hiperlipidemias. Sus
medicamentos incluyen metoprolol,lisinopril, aspirina y simvastatina. Al IPAS
refiere dolor ""de los huesos en las últimas semanas, principalmente de las
extremidades y la espalda. También al interrogatorio menciona que no ha ido a
orinar en 24 horas. A la exploración física sus signos vitales incluyen FC 104/min,
FR 18/min, T 36.7 C y PA 145/80 mm Hg. No hay anormalidades a la exploración
torácica. En el reconocimiento abdominal se palpa una masa en el hipogastrio,
dolorosa altacto, timpánica a la percusión. Hay edema 1+ de extremidades
inferiores. Al tacto rectal se siente próstata agrandada, con una nodulación. Se
realiza una cateterización,y el examen de orina se observa sin anormalidades. A
continuación se muestran estudios de laboratorio:
Examen esultado
Sangre
Glucosa 93 mg/dL
Creatinina 2.1 mg/dL
BUN 40 mg/dL
Albúmina 4.5 mg/dL
Proteínas totales 6.3 mg/dL
Na 143 mEq/L
K 4.8 mEq/L
CI 103 mEq/L
HC03 23 mg/dL

¿Cuál es la causa más probable de la falla renal de este paciente?


c} Nefropatía obstructiva

83. Lo siguiente es cierto sobre la erisipela:


e} Los márgenes de la lesión están bien definidos y sonindurados

84. De los siguientes microorganismos, ¿en cuál NO se ha demostrado una


relación importante con algún tipo de cáncer?
d} Virus herpes simple tipo 1

85. Femenino de 69 años de edad, se presenta al servicio de urgencias por


padecimiento de 6 h de evolución caracterizado por dolor intenso en 2° dedo del
pie izquierdo, hiperestesia y amoratamiento del mismo. Antecedentes: diabetes
mellitus de 15 años de evolución tratada con metformina + glibenclamida,
hipertensión arterial sistémica de 15 años de evolución controlada con losartán y
metoprolol,tabaquismo crónico intenso desde su juventud hasta la actualidad a
razón de 2 cajetillas por día, claudicación intermitente a más de 300 m.
Exploración física: obesa bien hidratada orientada en las tres esferas, abdomen
blando depresible no doloroso con soplo holosistólico en mesogastrio, pulsos
femorales con soplos bilaterales, buena intensidad, pulsos poplíteos, tibiales
anteriores y posteriores disminuidos, en pie izquierdo se observa 2° dedo con
cianosis intensa, hipotermia de todo el dedo e hiperalgesia.
85a. El mecanismo fisiopatológico responsable del cuadro clínico de esta paciente
es:
d} Ateroembolismo distal

85b. La medida terapéutica indicada en esta paciente es:


d} Manejo conservador e iniciar estudio de patología aórtica

85c. El mecanismo fisiopatológico responsable del cuadro clínico de esta paciente


es:
d} Ulceración y embolización de placa de ateroma

85d. Debido a los antecedentes de la paciente, se decide que es necesario


descartar la presencia de aneurisma de laaorta abdominal. ¿Cuál es el estudio de
elección para corroborar este diagnóstico?
c} Angiotomografía

85e. Según los resultados de los estudios realizados, ¿cuál sería el tratamiento
médico de elección para disminuir la claudicación de la paciente?
d} Medidas de higiene arterial + cilostazol

86. Paciente masculino de 69 años de edad que acude al servicio de urgencias


por presentar disnea. Tiene antecedente de tabaquismo intenso desde los 20 años
de edad. Al interrogatorio menciona que ha perdido alrededor de 9 kg de manera
no intencionada en el último mes y que desde hace tres meses ha tenido tos
crónica. A la exploración física la presión arterial es 88/66 mm Hg, la frecuencia
cardiaca es de 109 latidos/minuto. Existe distensión de las venas yugulares. La
auscultación de los campos pulmonares revela disminución de los sonidos
respiratorios en el pulmón derecho. Los ruidos cardiacos se encuentran
notablemente disminuidos. La radiografía de tórax muestra crecimiento de la
silueta cardiaca y una tumoración en el hilio derecho. ¿Cuál de los siguientes
hallazgos electrocardiográficos se esperaría observar en este paciente?
c} Alternancia eléctrica

87. ¿Cuál es la producción normal de cortisol al día?


c} 8 a 25 mg en 24 horas
88. Paciente femenina de 65 años de edad. Nivel socioeconómico bajo.
Antecedente de hipertensión arterial sin tratamiento. Acude a consulta por fatiga y
malestar general de 3 meses de evolución. TA: 140/90, palidez de tegumentos,
edema. Laboratorio reporta BUN elevado, creatinina 2.8 mg/dl, anemia
normocítica-normocrómica. Riñones pequeños por ultrasonido. Filtrado glomerular
por creatinina sérica estimado en 35 mUmin/1.73 m2 .
88a. Según la clasificación KDOQI de la enfermedad renal crónica, ¿en qué
estadio se encuentra la paciente?
b} 111

88b. Señale la causa principal de enfermedad renal crónica:


a} Nefropatía diabética

88c. Señale las alteraciones electrolíticas y metabólicas con las que podría cursar
la paciente:
a} Hiperpotasiemia, acidosis metabólica, hiperfosfatemia, hipocalciemia

88d. ¿Cuál es la principal causa de muerte en la insuficiencia renal crónica?


c} Anomal ías cardiovasculares

88e. ¿Cuál de las siguientes opciones no corresponde a una indicación de


diálisis?: a} Edema

89. Ante un recién nacido pretérmino que cursó con enfermedad de membrana
hialina y requirió de oxigenoterapia y ventilación mecánica;al completar un mes de
vida persiste con dependencia del ventilador,se le solicita una radiografía de tórax.
De acuerdo al diagnóstico sospechado, ¿qué imagen espera encontrar en la
radiografía?"
e} Patrón en esponja

90. A los 1O días de vida un recién nacido presenta caída del cordón umbilical y
posterior a esto, se observa a nivel de la cicatriz umbilical una masa rojiza, blanda
y granular que en ocasiones produce una secreción blanquecina espesa. ¿Cuál es
el manejo más adecuado en este caso?

b} Cauterización con nitrato de plata

91. Masculino de 32 años, llevado a servicio de urgencias presentando: colapso


pulmonar completo con desplazamiento de mediastino, disminución del retorno
venoso e inestabilidad hemodinámica. Los datos clínicos de este paciente
corroboran el diagnóstico de:
d} Neumotórax a tensión

92. Masculino de 22 años presenta síndrome nefrítico, la presencia de anticuerpos


anticitoplásmicos de neutrófilos con patrón citoplasmático (cANCA) positivos, nos
orienta hacia el diagnóstico de:"
a} Granulomatosis de Wegener

93. ¿Cuál de las siguientes neoplasias del sistema nervioso central es la más
frecuente de localización supratentorial en niños?
a} Craneofaringioma

94. Paciente masculino de 5 años con diagnóstico de escarlatina, se indica


tratamiento."
En caso de encontrar una complicación tardía esta es:
c} Fiebre reumática y glomerulonefritis

95. ¿Cuál de los siguientes marcadores tumorales se eleva en los cánceres


ováricos?
a} CA-125

96. Paciente masculino de nueve años de edad que refiere desde hace una
semana malestar faríngeo, así como cefalea y dolor desde hace un día en la
región de la cadera izquierda. En la radiografía de cadera no se observan
anormalidades, sin embargo un aspirado guiado por ultrasonido demuestra la
presencia de un líquido estéril. Inmediatamente después de la aspiración los
síntomas mejoran, a pesar de que marcadores de inflamación se encuentran
elevados (velocidad de sedimentación eritrocitaria de 60 mm durante una hora y
proteína C reactiva de 77 mg/L). Se realiza diagnóstico de artritis viral y es dado
de alta. Tres días después el paciente regresa por dolor en ambos tobillos,
febrícula y eritema en tórax. A la exploración física presenta un soplo sistólico
grado 2/6 en la región de la base. Un ecocardiograma demuesta derrame
pericárdico, únicamente con electrocardiograma normal. En cuanto a estudios de
laboratorio presenta un título de antiestreptolisina O de 1200 unidades Todd y de
antiDNAsa B de 1 920 unidades Todd, así como cultivo faríngeo positivo para
estreptococo hemolítico del grupo"
A.
96a. ¿Cuáles son los criterios mayores para el diagnóstico de fiebre reumática?
c} Carditis, poliartritis, corea, eritema marginado, nódulos subcutáneos"

96b. ¿Cuál es el tratamiento más adecuado en este paciente?


a} Penicilina benzatínica y salicilatos orales en altas dosis

96c. En una evaluación posterior,se determina que el paciente no presenta daño


valvular cardiaco. ¿Cuál es la duración recomendada del tratamiento profiláctico?
b} Diez años o hasta la edad adulta (la que sea de mayor duración}

97. Paciente del sexo femenino de 25 años de edad que acude a consulta médica
porque desde hace tres meses la paciente presenta fatiga, lo cual ha repercutido
en su desempeño académico. Al interrogatorio la paciente describe que ha perdido
aproximadamente 8 kg en dos meses. A la exploración física la paciente presenta
adenomegalias en la región posterior del triá ngulo cervical y en la axila izquierda.
Se realiza una biometría hemática, la cual sólo es relevante por conteo leucocitaria
cercano al rango superior de normalidad. El médico decide realizar una biopsia de
ganglio linfático; los resultados de la biopsia revelan la sobrexpresión de la
proteína antiapoptótica bcl-2. ¿Cuál es la translocación que más frecuentemente
se presenta en este tipo de neoplasia?
d} t(14:18}

98. Femenino de 43 años es ingresada al servicio de Medicina Interna por


cetoacidosis. Posterior a su recuperación metabólica inicia con fiebre, cefalea,
dolor facial,disminución del nivel de conciencia y enrojecimiento nasal con lesión
negruzca en fosa nasal derecha. Antecedentes: portadora de DM tipo 2y HTAS."
La entidad causal de la sintomatología de esta paciente es:
b} Infección por Mucor

99. ¿Cuál de las siguientes opciones es una manifestación de la deficiencia de


cobalamina?
b} Manifestaciones neuropsiquiátricas

1OO. Un paciente masculino de 15 años de edad acude a consulta pediátrica de


seguimiento debido a su padecimiento actual,el cual le produce retraso mental
leve. A la exploración física el paciente presenta un nevo pigmentado de forma
oval en la región lumbar,así como angiofibromas en la frente, los cuales la madre
dice que se han presentado desde el nacimiento. En el expediente se menciona
que en el pasado el paciente tuvo un rabdomioma cardiaco, el cual se encuentra
en regresión. ¿Cuál de las siguientes enfermedades incide con mayor riesgo de
presentar este paciente?
c} Epilepsia
101. Un niño de ocho años de edad tiene el diagnóstico de faringoamigdalitis de
repetición, por lo que se programa para amigdalectomía.
101a. ¿Cuál es una indicación absoluta de la cirugía programada para este
paciente?
a} Amigdalitis aguda que cause crisis convulsivas febriles

101b. ¿Cuál de las siguientes es una complicación temprana después de


amigdalectomia?
a} Sangrado

102. ¿Cuál de las siguientes neoplasias malignas se asocia con policitemia como
síndrome paraneoplásico?
d} Carcinoma de células renales

103. Paciente femenino de 31 años de edad acude dos semanas después de su


primer parto por presentar una tumoración dolorosa en la porción inferior de la
glándula mamaria derecha. La paciente refiere que la leche que produce ha
adquirido una coloración verdosa. A la exploración se encuentra una masa
fluctuante y dolorosa. ¿Cuál es el tratamiento más adecuado en esta paciente?
a} Continuar con alimentación delneonato vía seno materno y administrar
antibióticos

105. Se presenta al departamento de urgencias un lactante masculino de cuatro


meses de edad que sufrió crisis convulsivas hace dos horas. Al interrogatorio los
padres mencionan que el paciente fue producto de un embarazo a término y parto
eutócico, con desarrollo psicomotor normal. La exploración física revela que el
paciente presenta una gran hepatomegalia e hipotonía. Lafrecuencia cardiaca se
mantiene entre 180 y 200 latidos/min,con frecuencia respiratoria de 75
respiraciones/min. Los estudios de laboratorio indican hipoglucemia intensa con
acidosis láctica, hipertrigliceridemia. ¿Cuál es la deficiencia enzimática más
probable en este lactante?
b} Deficiencia de glucosa-6-fosfatasa

106. Paciente masculino de 19 años de edad que se presenta a consulta después


de haber notado la aparición de una úlcera no dolorosa en el pene posterior a una
relación sexual sin protección. ¿Cuál de los siguientes métodos es el mejor para
realizar el diagnóstico del agente etiológico del paciente?
b} Toma de exudado de la úlcera y observación bajo microscopio de campo
oscuro

107. Femenino de 59 años acude a consulta externa refiriendo: plenitud,


estreñimiento, acompañada con resultado de ultrasonido pélvico, imagen quística
en el ovario derecho de 15 por 15 cm. El diagnóstico más probable en esta
paciente es:
a} Cistadenoma seroso

108. Paciente masculino de 18 años de edad que conduce su automóvil en la


madrugada de regreso a casa. Durante el trayecto sufre colisión con otro
automóvil, cuyo conductor manejaba en estado de ebriedad. El paciente refiere
que sólo sufrió un impacto en la cabeza, sin pérdida del estado de alerta y que
tiene leve cefalea. Sin embargo el personal paramédico insiste en llevarlo a un
hospital para evaluación adicional. Una vez en el hospital el paciente relata lo
sucedido cuando de repente se muestra somnoliento y después con pérdida del
estado de alerta.
¿Cuál es el diagnóstico más probable?
e} Hemorragia epidural

109. ¿Cuál es la prevalencia en población general de la artritis reumatoide?


e} 0.5 a 1%

110. ¿Cuál es el mecanismo de acción de las quinolonas?


d}Inhibición dela síntesis del DNA

111. Paciente masculino de 49 años, sin antecedentes heredofamiliares de


importancia, fue sometido a colecistectomía hace cinco años. Tiene un índice
tabáquico de 2, niega alcoholismo. Realiza ejercicio aeróbico tres veces por
semana. Es remitido a consulta por prostatismo moderado, en tratamiento con
fitoterapia. Trae consigo una ecografía abdominal que muestra riñones y vejiga
normales, próstata de 45 g con calcificaciones en su interior.
111a. De las siguientes opciones, cual es la aseveración verdadera:"
b} Haría un tacto rectal

111b. Tras hacerle un tacto rectal, la próstata impresiona de adenomatosa,


volumen 11/IV. Lo siguiente que procedería sería:
b} Solicitar antígeno prostático específico e iniciar tratamiento con alfa-
bloqueadores

111c. En la siguiente consulta,el paciente comenta que ha tenido que acudir al


servicio de urgencias por presentar prostatitis aguda, posterior a esto se le realizó
un antígeno prostático específico, cuyo resultado es de 25 ng/mL. Pregunta: ¿Cuál
sería la conducta a seguir?
d} Mantener el tratamiento antibiótico durante cuatro semanas, así como los alfa-
bloqueadores. Para finalmente repetir elantígeno prostático específico

111d. Cinco semanas después de haber completado el tratamiento adecuado, el


antígeno prostático específico reportado es de 14 ng/mL. Además, el paciente fue
sometido a sondeo vesical por presentar episodio de retención aguda de orina.
¿Qué le sugeriría al paciente como abordaje?
e} Solicitar ecografía transrectal con planimetría y toma de biopsias

111e. Los estudios realizados arrojan los siguientes resultados: próstata de 85 ce


con calcificaciones en su interior,compatible con hiperplasia prostática benigna.
Las biopsias prostáticas se reportan sin evidencia de malignidad.
¿Cuál de los siguientes tratamientos sería el más adecuado?
d} Adenectomía

112. ¿Cuál de las siguientes hormonas gastrointestinales inhibe la contracción de


la vesícula biliar?
a} Somatostatina

113. ¿A partir de qué momento se debe iniciar el tratamiento con FARME


(fármacos modificadores de la enfermedad) en pacientes con artritis reumatoide?
a} Desde elmomento deldiagnóstico

114. Paciente de 59 años de edad que acude al servicio de urgencias por dolor
torácico asociado con disnea y náusea;el paciente tiene dificultad para describir
exactamente el tipo de dolor. Él estaba bajo control por hipertensión y no se le
conoce ningún otro antecedente de importancia. Los signos vitales al momento del
ingreso son: frecuencia cardiaca de 60 latidos/min, presión arterial 140/115 mm
Hg, frecuencia respiratoria de 15 respiraciones por minuto. A la auscultación se
puede detectar un soplo diastólico. La exploración física no se pudo completar
adecuadamente ya que el paciente desarrolla un edema de pulmón agudo, con
aumento en intensidad del soplo diastólico. Se realiza manejo agudo del paciente
y estabilización. Se solicita radiografía de tórax portátil y un ecocardiograma en la
cama del paciente que demuestran la presencia de una raíz aórtica dilatada,
asociada con insuficiencia aórtica moderada. Se realiza una tomografía
computarizada de urgencia la cual confirma la presencia de una disección aórtica
tipo A que involucra la válvula aórtica y ocluye el ostium de la arteria coronaria
derecha.
¿Cuál de los siguientes hallazgos a la exploración física debe crear la sospecha
de disección aórtica aguda?
a} Dolor torácico anterior

115. ¿Cuál de las siguientes arterias se encarga de la irrigación de la mitad


derecha de la curvatura mayor del estómago?
b} Arteria gastroepiploica derecha

116. La amantadina es un fármaco activo contra la influenza tipo A (pero no contra


la influenza tipo B), no obstante, recientemente ha surgido un alto grado de
resistencia a este fármaco. ¿Cuál de los siguientes enunciados se asocia mejor
con el mecanismo de acción de la amantadina?
b}Inhibición dela descapsidación

117. ¿Cuál de los siguientes nervios envía fibras hacia la piel de la región
anteroinferior del abdomen?
c} Nervio iliohipogástrico

118. Paciente de 45 años de edad sin antecedentes de importancia, ingresa al


servicio de urgencias por deterioro neurológico, estado de choque y acidosis
láctica de brecha aniónica aumentada. A la exploración clínica neurológica se
encuentra estado de coma con flacidez y pérdida de los reflejos, pupilas isocóricas
y de lenta respuesta a la luz. La acidosis aniónica continuó empeorando y alcanzó
niveles de ácido láctico de 4.6 mmol/L y brecha osmolar de 11.35, con glucosa,
urea,creatinina y electrólitos normales. Al lavado gástrico se obtiene un material
rojizo. Se solicita un estudio de orina para buscar intencionadamente cristales de
oxalato de calcio. En caso de encontrar cristales de oxalato de calcio, ¿cuál es la
sustancia que más probablemente ingirió este paciente?
c} Etilenglicol

119. Paciente femenino, producto de segunda gesta, 41 semanas de gestación,


obtenido por parto vaginal eutócico, al momento de su nacimiento se observan
datos de dificultad respiratoria; por lo cual se procede a evaluar con la prueba de
Silverman. Indique cuál de los siguientes parámetros no es valorado por esta
evaluación:
a} Coloración

120. Femenino de 75 años acude a consulta externa por referir iniciar con cuadro
diarreico severo de 2 días de evolución. Antecedentes: hipertensión arterial sin
control y manejo médico. Exploración física: deshidratado, oligúrico, adinámico,
refiere visión borrosa. La tensión arterial sistólica es de 80 mmHg. Exámenes de
laboratorio reportan: urea en sangre 440 mg/dL y creatinina 13.5 mg/dL. La
excreción fracciona! de sodio es de 5%. Vd. Cilindros hemáticos en orina."
120a. Los datos clínicos de esta paciente corroboran el diagnóstico:
c} Necrosis tubular aguda

120b. La aparición de cilindros hemáticos en el sedimento urinario en esta


paciente corroboran:
d} Daño glomerular severo

121.Acude paciente femenino de 27 años de edad a consulta de control prenatal


por presentar embarazo de 1O semanas de gestación, no cuenta con
antecedentes de importancia, no toma ningún medicamento y no tiene factor de
riesgo alguno para el embarazo. Según la norma oficial mexicana, ¿cuántas
consultas como mínimo deben tenerse durante el embarazo?"
b} 5

123. Recién nacido de 20 horas de vida, producto de primera gestación, con tipo
sanguíneo 8-, la madre tiene sangre tipo A+. ¿Qué es FALSO en relación con la
incompatibilidad ABO?
b} Nunca ocurre en elprimer embarazo

124. Lactante de dos meses de edad que presenta desde el nacimiento estridor de
forma intermitente, sin taquipnea,aleteo nasal,tiraje subcostal, retracción
subxifoidea. En la exploración física se observa mucosa faríngea de aspecto
rosado, sin hiperemia ni exudados, se realiza laringoscopia directa en la que se
encuentran hallazgos compatibles con laringomalacia (flacidez de la epiglotis con
prolapso sobre la laringe durante la inspiración). De acuerdo al diagnóstico, ¿cuál
es la medida terapéutica más indicada en este caso?"
d} Vigilancia

125. ¿En cuál de las siguientes situaciones se establece con certeza el


diagnóstico de diabetes mellitus?
e} Concentraciones plasmáticas sucesivas de glucosa en ayuno de 147, 165 y
152 mg/dl en una mujer asintomática"

126. Niña de cinco años de edad con una dermatosis diseminada, bilateral y
asimétrica que afecta región axilar e inguinal, constituida por pápulas umbilicadas,
en forma de domo translúcidas, bien delimitadas. ¿A qué familia pertenece el
agente causal de esta dermatosis?
a} Poxvirus

127. ¿A qué se refiere el ""efecto del alba""?


b} Hiperglucemia matutina causada por efectividad reducida de lainsulina en ese
momento
128. Un joven de 17 años de edad es traído a la sala de urgencias con diaforesis,
náusea, vómito y dolor abdominal en hipocondrio derecho. La madre refiere como
antecedentes principales depresión de 2 meses de evolución y el hecho de
encontrarlo en su cuarto junto a un frasco de pastillas no identificadas. En el
interrogatorio revela que tiene 5 h de evolución. Se toman exámenes de
laboratorio reportándose biometría hemática y química sanguínea normal,pero
pruebas de función hepática ligeramente aumentadas. Usted piensa en una
intoxicación."
"128a. Con base en los datos obtenidos en el interrogatorio y los resultados de
laboratorio, ¿cuál es su principal sospecha diagnóstica de intoxicación?
b} Acetaminofén

128b. En un paciente con sospecha de intoxicación de este medicamento, ¿cuál


de los siguientes exámenes de laboratorio se deben realizar?
d} Todos los anteriores

128c. ¿Cuál es la dosis tóxica del medicamento que sospecha es el responsable


de la intoxicación?
d} 140 mg/kg

128d. Según su sospecha clínica, ¿cuál es el tratamiento específico para esta


intoxicación?
c} N-acetil-cisteína

128e. ¿Cuál es la dosis del antídoto que se debe administrar (en este caso vía
intravenosa)?
b} 150 mg/kg en 60 min,después 50 mg/kg en 4 h"

129. La principal e inicial medida terapéutica del estado hiperglucémico


hiperosmolar es:
b} La rehidratación

130. Acude a urgencias paciente femenino de 32 años de edad por dolor


abdominal tipo cólico que inició en fosa ilíaca derecha y ha ido incrementando en
intensidad y frecuencia;se asocia anorexia, astenia y adinamia. Refiere nueve
semanas de amenorrea secundaria. A la exploración física muestra abdomen con
ruidos intestinales disminuidos en frecuencia,abdomen depresible, con
hiperestesia e hiperbaralgesia, doloroso a la palpación en fosa ilíaca
derecha,signo de psoas, obturador y talopercusión negativos. A la exploración
vaginal se observa hemorragia leve, con dolor a la movilización del cuello uterino y
se palpa masa en anexo. Se decide realizar prueba de embarazo y ecografía
transvaginal. ¿Cuál es la localización más frecuente de esta patología?
b} Ampular

131. Masculino de 1O años de edad que es llevado a consulta externa por


presentar exantema maculopapular que comenzó en la cabeza (detrás de las
orejas) y se diseminó de forma cefalocaudal al resto cuerpo. La madre refiere que
el cuadro inició 5 días antes, con fiebre, conjuntivitis, tos, coriza y malestar
general, seguidos de lesiones eritematosas con un centro azul grisáceo en la
mucosa bucal. En la exploración física usted confirma la presencia de un
exantema maculopapular con áreas de confluencia.
"131a. Con base en las características del paciente, ¿cuál es el diagnóstico más
probable?"
d} Sarampión

131b. De acuerdo con su sospecha clínica, ¿cuál es el agente etiológico del


padecimiento de este paciente?
c} Paramyxovirus

131c. En relacióncon su sospecha clínica, ¿qué complicación es posible que se


presente?
d} Complicaciones respiratorias

131d. En relación con su sospecha clínica, ¿cuáles son los hallazgos que
esperaría encontrar en los exámenes complementarios?"
c} Linfopenia

131e. De acuerdo con su sospecha clínica, ¿cuál es el tratamiento indicado para


este paciente?"
c} Paracetamol,protección dela luz fuerte y antibióticos para complicaciones

También podría gustarte